You are on page 1of 728
Mark this question => Question Td : 21677 Question 1 of 30 A 36 year old woman presents with severe, stabbing abdominal pain radiating trom the epigastrium to the back and chest. She is nauseated and her abdomen feels bloated and painfil, the pain is worse lying down, She has tachycardia, a low-srade fever, an absence of bowel sounds, and an exqisitely tender abdomen, She drank excessive amounts of alcohol last night. She has no past medical or funnily history for cardiac disease. A plain radiograph of the abdomen shows no air under the diaphragm, Which of the following process is most likely on-going inthis patient? a) Perforated duodenal uicer ) Myocarcial infarction c) Abdominal aortic aneurysm 4) Acute pancreatitis ©) Renal colic Anower [JERPINENON|) Other User's Exp Question Explanation: Acute pancreatitis is inflammation of the pancreas (end, sometimes, adjacent tissues) caused by the release of activeted pancreatis enzymes. The most common triggers are biliary tract disease and chronic heavy alcohol intake. The condition ranges from rrild (abdominal pain and vorniting) to severe (pancreatic necrosis and a systemic inflammatory process with shock and multiorgan failure) Diagnosis is based on clinical presentation and serum amylase and lipase levels, Treatment is supportive, with IV fluids, analgesics, and fasting. jon Report An Error Mark this question => Question Td : 21677 Question 1 of 30 A 36 year old woman presents with severe, stabbing abdominal pain radiating trom the epigastrium to the back and chest. She is nauseated and her abdomen feels bloated and painfil, the pain is worse lying down, She has tachycardia, a low-srade fever, an absence of bowel sounds, and an exqisitely tender abdomen, She drank excessive amounts of alcohol last night. She has no past medical or funnily history for cardiac disease. A plain radiograph of the abdomen shows no air under the diaphragm, Which of the following process is most likely on-going inthis patient? a) Perforated duodenal uicer ) Myocarcial infarction c) Abdominal aortic aneurysm Y © & Acute pancreatitis ©) Renal colic Anower [JERPINENON|) Other User's Exp Question Explanation: Acute pancreatitis is inflammation of the pancreas (end, sometimes, adjacent tissues) caused by the release of activeted pancreatis enzymes. The most common triggers are biliary tract disease and chronic heavy alcohol intake. The condition ranges from rrild (abdominal pain and vorniting) to severe (pancreatic necrosis and a systemic inflammatory process with shock and multiorgan failure) Diagnosis is based on clinical presentation and serum amylase and lipase levels, Treatment is supportive, with IV fluids, analgesics, and fasting. jon Report An Error ‘Marke this question & => Question Td : 24189 Question 2 of 30 Arman aged 25 years complains of pain in his rectum A perianal abscess is revealed on examination. Which sign is not characteristic ofit? a) Rectal ulcer ') External swelling c) Redness ) 9) Induraiion ¢) Tendemess on rectal examination Answer [Barisnaton) Other User's Explanation Report An Error Question Explanatio ‘An anorectal abscess is a localized collection of pus in the perirectal spaces. Abscesses usually originate in an anal crypt. These abscesses can be very painful; perianal swelling, rednecs, and tendemest are characteristic. Deeper abscesses may be less painful but cause toxic symptoms (eg, fever, chills, malaise). There may be no perianal findings, but digital rectal examination may reveal a tender, fuctuant swelling of the rectal wall. High pelvirectal abscesses may cause lower abdominal paia and fever without rectal syraptcms. Sometimes fever is the orly symptom. Diagnosis i primarily by examination and CT sean or pelvic MRT for deeper abscesses. Treatment is surgical drainage ‘Marke this question & => Question Td : 24189 Question 2 of 30 Arman aged 25 years complains of pain in his rectum A perianal abscess is revealed on examination. Which sign is not characteristic of it? Y © a) Rectal ulcer b) External swelling ) Redes ) 9) Induraiion ¢) Tendemess on rectal examination Answer [Barisnaton) Other User's Explanation Report An Error Question Explanatio ‘An anorectal abscess is a localized collection of pus in the perirectal spaces. Abscesses usually originate in an anal crypt. These abscesses can be very painful; perianal swelling, rednecs, and tendemest are characteristic. Deeper abscesses may be less painful but cause toxic symptoms (eg, fever, chills, malaise). There may be no perianal findings, but digital rectal examination may reveal a tender, fuctuant swelling of the rectal wall. High pelvirectal abscesses may cause lower abdominal paia and fever without rectal syraptcms. Sometimes fever is the orly symptom. Diagnosis i primarily by examination and CT sean or pelvic MRT for deeper abscesses. Treatment is surgical drainage ‘Marke this question & => Question Td : 46514 Question3 of 30 “Which offthe followings not present in initable bowel syndrome (IBS)? a) Recurrent type of nature ) Bloating ©) Bloody darhea 4) Constipation ©) Cramping Anewor (UEIFIRIRAR) otnertsorsExplanation Report An Error Question Explanation: ‘The Rome criteria are standardized symptom based enteria for diagnosing IBS. The Rome criteria require the presence of abdominal pain or discomfort for at least 3 days per-month in the last 3 months along with 22 of the folowing: (1) improvement with defecarion, (2) onset of each episede of discomfort essociated with a change in frequency of defecation, or (3) change in consistency of stool Other symptoms of IBD include abdominal pain or cramping, a bloated feeling, gas (flatulence), diarrhea or constipation, with sometimes alternating bouts of constipation and diarthea and mucus in the stool, ‘Marke this question & => Question Td : 46514 Question3 of 30 “Which offthe followings not present in initable bowel syndrome (IBS)? a) Recurrent type of nature b) Bloating ¥ © o) Bloody ciamhea ) Constipation 2) Cramping Anewor (UEIFIRIRAR) otnertsorsExplanation Report An Error Question Explanation: ‘The Rome criteria are standardized symptom based enteria for diagnosing IBS. The Rome criteria require the presence of abdominal pain or discomfort for at least 3 days per-month in the last 3 months along with 22 of the folowing: (1) improvement with defecarion, (2) onset of each episede of discomfort essociated with a change in frequency of defecation, or (3) change in consistency of stool Other symptoms of IBD include abdominal pain or cramping, a bloated feeling, gas (flatulence), diarrhea or constipation, with sometimes alternating bouts of constipation and diarthea and mucus in the stool, ‘Marke this question & => Question Ta : 46880 Question 4 of 30 A 74 year old man with well-controlled type 2 diabetes mellins is schectiled for an abdominal CT scan with oral and intravenous iodinated contrast, The medication that should be withheld 48 hours before and after the procedure is a) Clyburide (Micronase. DiaBeta) b) Giipizide (Glucotrol) c) Acarbose (Precose) 4) Rosiglitazone (Avandia) ¢) Metformin (Gncophage) Answer | xphnation Other User's Explanation Report An Error Question Explanation: ‘Metformin should be withheld before and after radiographic procedures with contrast, due to its interaction with iodinated contrast materials. This interection may cause impaired renal Binction or lactic acidosis, The other drugs listed do not carry this risk. ‘Marke this question & => Question Ta : 46880 Question 4 of 30 A74 year old man with well-controlled type 2 diabetes mellitus is scheduled for an abdeminal CT scan with oral and intravenous iodinated contrast, The medication that should be withheld 48 hours before and afler the procedure is a) Clyburide (Micronase. DiaBeta) b) Glipizide (Glucotrol) c) Acarbose (Precose) 4) Rosightazone (Aandi) V © &) Metformin (Ghicophage) Answer | xphnation Other User's Explanation Report An Error Question Explanation: ‘Metformin should be withheld before and after radiographic procedures with contrast, due to its interaction with iodinated contrast materials. This interection may cause impaired renal Binction or lactic acidosis, The other drugs listed do not carry this risk. ‘Marke this question & => Question Td : 48963 Question 5S of 30 A.24 year old man develops abdominal pain and diarrhea that is liquid containing mucus and blood three days after coming hack from a camping trip. Stool stucics show Shigella. What is the most appropriate treatment? a) Replacing fluids b) Metronidazole ©) Diphenoxylate ) Loperamide Question Explanatior Shigellosis is an acute infection of the intestine caused by Shigella Symptoms include fever, nausea, vomiting, and diarthea taat is usually bloody. Diagnosis is clinical and confirmed by stool culture. Treatment is supportive, mostly with rehydration, antibiotics (eg, ampicillin or trimethoprim-sulfzmethoxazole) are optional, Antidicrrheal drugs (euch as diphenoxylate or loperemile) may prolong the infection and should not be used. ‘Marke this question & => Question Td : 48963 Question 5S of 30 A.24 year old man develops abdominal pain and diarrhea that is liquid containing mucus and blood three days after coming back from acamping trip. Stool studies show Shigella. What is the most appropriate treatment? ¥ © a) Replacing fluids b) Metronidazole ¢) Diphenoxylate ) Loperamide Question Explanatior Shigellosis is an acute infection of the intestine caused by Shigella Symptoms include fever, nausea, vomiting, and diarthea taat is usually bloody. Diagnosis is clinical and confirmed by stool culture. Treatment is supportive, mostly with rehydration, antibiotics (eg, ampicillin or trimethoprim-sulfzmethoxazole) are optional, Antidicrrheal drugs (euch as diphenoxylate or loperemile) may prolong the infection and should not be used. Mark this question = => Question Id : 51296 Question 6 of 30 A.50 year old man comes to the emergency room with a history of vomiting for three days. His past history reveals thet for approximately 20 years, he has been getting relief from epigastric pein, lasting for 2 to 3 weeks He remembers getting pain reliefby tleing male and antacids. Physival examination showed fillaess in the epigastric area with visible peristalsis, absence of tenderness and normal active bowel sounds. The most likely diagnosis in tis patient is a) Small bowel obstruction +) Gastric outlet obstruction ©) Votilas of the colon 4) Incarcerated umbilical hernia ©) Chelecystits Question Explanation: Intrinsic or extrinsic obstruction of the pyloric chennel or duodemum is the usual pathophysiclogy of gastric outlet obstruction (GOO), ‘Nausea and vomiting are the cardinal symptoms of GOO, Vomiting usually is described as nonbiious, and it charactenstically conlains undigested food particles. In the early stages of obstruction, vomiting may be intermittent and usually occurs within 1 hour of ameal Early satiety and epigastric fulness are common, Weight loss is frequent when the condition approaches chroncity and is ‘most significant in patients with malignant disease, Abdominal pain is not frequent Physical examination often demonstrates the presence of chronic dehyclraticn and malnutition. A dilated stomach may be appreciated as a tympanitic mass in the epicastric area andlor left upper quadrant, Plain abdominal radiographs, contrast upper GI studies (Gastrografin or barium), and CT scans with oral contrast are helpful. Plain raciographs, including the obstruction series (Le., supine abdomen, upnght abdomen, chest posteroanterion), can demonstrate the presence of gastric dilatation and may be helpful in distinguishing the differential dieenosis, Upper endoscopy can help visualize the gastric outlet and may provide a tissue diagnosis when the obstruction is intraluminal ‘Treatment is surgical Mark this question = => Question Id : 51296 Question 6 of 30 A.50 year old man comes to the emergency room with a history of vomiting for three days. His past history reveals thet for approximately 20 years, he has been getting relief from epigastric pein, lasting for 2 to 3 weeks He remembers getting pain reliefby tleing male and antacids. Physival examination showed fillaess in the epigastric area with visible peristalsis, absence of tenderness and normal active bowel sounds. The most likely diagnosis in tis patient is a) Small bowel obstruction © b) Gastric outlet obstruction ©) Volmlas of the cofon 4) Incarcerated umbilical hernia ©) Chelecystits Question Explanation: Intrinsic or extrinsic obstruction of the pyloric chennel or duodemum is the usual pathophysiclogy of gastric outlet obstruction (GOO), ‘Nausea and vomiting are the cardinal symptoms of GOO, Vomiting usually is described as nonbiious, and it charactenstically conlains undigested food particles. In the early stages of obstruction, vomiting may be intermittent and usually occurs within 1 hour of ameal Early satiety and epigastric fulness are common, Weight loss is frequent when the condition approaches chroncity and is ‘most significant in patients with malignant disease, Abdominal pain is not frequent Physical examination often demonstrates the presence of chronic dehyclraticn and malnutition. A dilated stomach may be appreciated as a tympanitic mass in the epicastric area andlor left upper quadrant, Plain abdominal radiographs, contrast upper GI studies (Gastrografin or barium), and CT scans with oral contrast are helpful. Plain raciographs, including the obstruction series (Le., supine abdomen, upnght abdomen, chest posteroanterion), can demonstrate the presence of gastric dilatation and may be helpful in distinguishing the differential dieenosis, Upper endoscopy can help visualize the gastric outlet and may provide a tissue diagnosis when the obstruction is intraluminal ‘Treatment is surgical ‘Marke this question << => Question Td : 51729 Question 7 of 30 4.25 year old woman presents with a § month history of abdominal pain. A physical examination, including pelvic and rectal examinations, is normal, Which component of history would indicate a need for further evaluation? 2) Relief of symptoms with defecation +b) Changes in stool consistency from loose and watery to constipation ©) Passage of mucus with bowel movements 6) Abdominal bloating ©) Worsening symptoms at night Question Explanation: Initable bowel syncrome (BS) is a benign, chronic symptom complex of abered bowel habits and abdominal pain, Iris the most common fictional disorder of the gastrointestinal tract. The presence of nocturnal symptoms is ared flag which should alert the physician to an atemate diagnosis and may require further evaluation. The cther symptome lcted are Rome I and Il criteria for agnosing iritable bowel syndrome. ‘Marke this question << => Question Td : 51729 Question 7 of 30 A.25 year old woman presente with a § month history of abdominal pain. A physical examination, including peivic and rectal examinations, is normal, Which component of history would indicate a need for further evaluation? 2) Relief of symptoms with defecation 'b) Changes in stool consistency from loose and watery to constipation 6) Paccage of mucus with bowel movements 6) Abdominal bloating V¥ © ©) Werscning symptoms at night Question Explanation: Initable bowel syncrome (BS) is a benign, chronic symptom complex of abered bowel habits and abdominal pain, Iris the most common fictional disorder of the gastrointestinal tract. The presence of nocturnal symptoms is ared flag which should alert the physician to an atemate diagnosis and may require further evaluation. The cther symptome lcted are Rome I and Il criteria for agnosing iritable bowel syndrome. Mark this question = => Question Td : 59591. Question 8 of 30 A.66 year old woman presents with evidence of a fecal impaction which you successfully treat. She relates a history of chronic laxative use for most of her adult years. You perform sigmoidoscopy and note that the anal and rectal mucosa contains scattered areas of binish black discoloration. The most likely explanation for the sigmoidoscepic findings is a) Endometiiosis +) Collagenous colitis ©) Melanosis coli 6) Metastatic malignant melanema 6) Arteriovenous malformations Question Explanation: ‘This patient has typical findings of melanosis coli, the term used to describe black or brown discoloration of the mucosa of the colon. Tiresuits from the presence of dark pigment in large mononuclear cells or macrophages in the lamina propria of the mucosa. The coloration is usnally mast intense just inside the anal epkincter and is ighter higher vp in the sigmoid colon. The condition is thought to result from fecal stasis and the use of anthracene cathartic such as cascara sagrada, semma, and dasthron, Ectopic endometrial tissus (endometriosis) most commonly involves the serosal layer ofthose parts of the bowel adjacert to the uterus and fallopian tubes, pparticulanly the rectosigmoid colon. Collagenous colitis does not cause mucosal pigmentary changes. Melanoma rarely metastasizes smubicentrcally to the bowel wall. Maltipie arteriovenous malformations are more coremon in the proximal bowel, and would nat appear as described. Mark this question = => Question Td : 59591. Question 8 of 30 ‘A. 66 year old woman presents with evidence of a fecal impaction which you successfilly teat. She relates a tistory of chronic laxative use for most of her adult years. You perform sigmoidoscopy and note that the anal and rectal rmucosa contains scattered areas of bluish black discoloration. The most likely explanation for the sigmoidoscopic findings is a) Endometriosis +b) Collagenous colitis Y © 0) Melanosis coli Metastatic malignant melancma ¢) Arteriovenous malformations Question Explanation: ‘This patient has typical findings of melanosis coli, the term used to describe black or brown discoloration of the mucosa of the colon. Tiresuits from the presence of dark pigment in large mononuclear cells or macrophages in the lamina propria of the mucosa. The coloration is usnally mast intense just inside the anal epkincter and is ighter higher vp in the sigmoid colon. The condition is thought to result from fecal stasis and the use of anthracene cathartic such as cascara sagrada, semma, and dasthron, Ectopic endometrial tissus (endometriosis) most commonly involves the serosal layer ofthose parts of the bowel adjacert to the uterus and fallopian tubes, pparticulanly the rectosigmoid colon. Collagenous colitis does not cause mucosal pigmentary changes. Melanoma rarely metastasizes smubicentrcally to the bowel wall. Maltipie arteriovenous malformations are more coremon in the proximal bowel, and would nat appear as described. “Mark this question <& => Question Td : 59661 Question 9 of 30 ‘A patient with chronic peptic ulcer disease asks for the best method for curing this disease. Your answer would be which one of the following? a) Helicobacter pylori eradication ) Proton pump inhibitors ©) H2 blockers 4) Nissen fandoplication Anowor [BQRSESREN) osner veers Explanation Report An Eror Question Explanation: HL pylori and NSAIDs disrupt normal rmacosel defense and repair, making the macosa more susceptible to acid. H. pylori Infection is present in 50 to 70% of patients with duodenal ulcers and 30 10 50% of patients with gastric ulcers. IPE. pylon is eradicated, only 10% of patients have recurrence of peptic ulcer disease, compared with 70% recurrence in patients treated with acid suppression alone Symptoms include pain often localized to the epigastrinm and relieved by food or antacids. The pain is described as buming or gmewing, or sometimes as a sensation of hunger. The course is usually chronic and recurrent. Diagnosis of peptic ulcer is suggested by patient history and confirmed by endoscopy (EGD). Treatmert of gastric and duodenal ulcers requires eradication of H. pylori when present, Methods of decreasing acidity inclnde a number of drug: including proten pump inhibitos and H2 blockers. Nissen fundoplication is a surgical treatment for chronic GERD. “Mark this question <& => Question Td : 59661 Question 9 of 30 ‘A patient with chronic peptic ulcer disease asks for the best method for curing this disease. Your answer would be which one of the following? Y © 4) Helicobacter pylori eradication ) Proton pump inhibitors ©) H2 blockers 4) Nissen fandoplication Anowor [BQRSESREN) osner veers Explanation Report An Eror Question Explanation: HL pylori and NSAIDs disrupt normal rmacosel defense and repair, making the macosa more susceptible to acid. H. pylori Infection is present in 50 to 70% of patients with duodenal ulcers and 30 10 50% of patients with gastric ulcers. IPE. pylon is eradicated, only 10% of patients have recurrence of peptic ulcer disease, compared with 70% recurrence in patients treated with acid suppression alone Symptoms include pain often localized to the epigastrinm and relieved by food or antacids. The pain is described as buming or gmewing, or sometimes as a sensation of hunger. The course is usually chronic and recurrent. Diagnosis of peptic ulcer is suggested by patient history and confirmed by endoscopy (EGD). Treatmert of gastric and duodenal ulcers requires eradication of H. pylori when present, Methods of decreasing acidity inclnde a number of drug: including proten pump inhibitos and H2 blockers. Nissen fundoplication is a surgical treatment for chronic GERD. Mark this question & => Question Td : 59772 Question 10 of 30 ‘A female presents with a pan in the right upper quadrant of her anus. The mass coming out is painfil and red, The most appropriate initial management is a) Sitz baths +b) Hot bandages c) Surgery 4) Systemic Antibiotics e) Topical Antibiotics Anewor [REIIRHAN) othe: User's Explanation Repost An vor Question Explanation: “Hemorrhoids are dilated veins of the hemorrhoidal plezns in the lower rectum. Symptoms incinde irritation and bleeding Thrombosed hemorrhoids are painfil, Diagnosis is by inspection or anescopy. Treatments symptomatic or with endoscopic banding, injection, sclerotherapy, or sometimes surgery. Initially, stool softeners and sitz baths are used. Mark this question & => Question Td : 59772 Question 10 of 30 ‘A female presents with a pan in the right upper quadrant of her anus. The mass coming out is painfil and red, The most appropriate initial management is v © a) Sitz baths +b) Hot bandages ) Surgery 4) Systemic Antibiotics ¢) Topical Antibiotics Anewor [REIIRHAN) othe: User's Explanation Repost An vor Question Explanation: “Hemorrhoids are dilated veins of the hemorrhoidal plezns in the lower rectum. Symptoms incinde irritation and bleeding Thrombosed hemorrhoids are painfil, Diagnosis is by inspection or anescopy. Treatments symptomatic or with endoscopic banding, injection, sclerotherapy, or sometimes surgery. Initially, stool softeners and sitz baths are used. ‘Macle this question & => Question Td Question 11 of 30 4.22 year old woman presents with a 3 week history of bloody ciarhea and weight loss. She is febrile, dehydrated and has a distended abdomen with left lower quadrant tenderness. You suspect that she has ulcerative colitis. Which of the following would be not an appropriate initial management? a) Plain abdominal X-ray b) Serum electrolytes c) Stool cultures ad ¢) Barium enema Anewor [RESIRNRNBM) othe: User's Explanation Repost An Enor Question Explanation Ulcerative colitis is 2 chronic inflammatory and ulcerative disease arising in the colonic mucosa, characterized most often by bloody diarthea, Extraintestinal symptoms, particularly arthritis, may occur. Long-term risk of colon cancer is high. Laboratory tests should, be done to screen for anemia, hypoalbuminemia, and electrolyte abnonnalities. Plain x-rays of the abdomen may show mucosal edema, loss ofhaustration, and absence of formed stool in the diseased bowel Barium enema shows similar changes. albeit more clearly, and may also demonstrate ulcerations, but the enema should not be performed during an acute presentation becanse of the risk of inducing perforation, Sigmoidescopy ‘Macle this question & => Question Td Question 11 of 30 4.22 year old woman presents with a 3 week history of bloody ciarhea and weight loss. She is febrile, dehydrated and has a distended abdomen with left lower quadrant tenderness. You suspect that she has ulcerative colitis. Which of the following would be not an appropriate initial management? a) Plain abdominal X-ray b) Serum electrolytes c) Stool cultures 4) Sigmoidoscopy WY © ¢) Barum enema Anewor [RESIRNRNBM) othe: User's Explanation Repost An Enor Question Explanation Ulcerative colitis is 2 chronic inflammatory and ulcerative disease arising in the colonic mucosa, characterized most often by bloody diarthea, Extraintestinal symptoms, particularly arthritis, may occur. Long-term risk of colon cancer is high. Laboratory tests should, be done to screen for anemia, hypoalbuminemia, and electrolyte abnonnalities. Plain x-rays of the abdomen may show mucosal edema, loss ofhaustration, and absence of formed stool in the diseased bowel Barium enema shows similar changes. albeit more clearly, and may also demonstrate ulcerations, but the enema should not be performed during an acute presentation becanse of the risk of inducing perforation, Mark ths queston ==> Question Id : 62510 Question 12 of 30 ‘The type of hemorrhoids that is particularly suitable for treatment by elastic-band lgetion is a) External hemorrhoids with pruritus ani +) External hemorrhoids with ulceration ©) External hemorrhoids with thrombosis 4) Internal hemorrhoids with thrombosis ) Internal hemorrhoids with fecal soiling Question Explanation: Hemorrhoids are dilated veins of the hemochoidal plexus in the lower rectum. Symptoms include inrtation and bleeding. Thrombosed hemorthoics are painful. Diagnosis is by inspection of anoscopy. Treatment is symptomatic or with endoscopic banding, jection sclerotherapy, of sometimes curgery. Elasticinubber band ligation is outpatient trestmont for second-degree (those that bleed with defecation) internal hemorrhoids. Report An Error Mark ths queston ==> Question Id : 62510 Question 12 of 30 ‘The type of hemorthoids that is pauticularly suitable for treatment by elastic-band lgetion is 2) External hemorrhoids with pruritus ani +b) Bernal hemorthoids with ulceration 6) External hemorrhoids with thrombosis Y © 4) Internal hemorrhoids with thrombosis ) Internal hemorrhoids with fecal soiling Question Explanation: Hemorrhoids are dilated veins of the hemochoidal plexus in the lower rectum. Symptoms include inrtation and bleeding. Thrombosed hemorthoics are painful. Diagnosis is by inspection of anoscopy. Treatment is symptomatic or with endoscopic banding, jection sclerotherapy, of sometimes curgery. Elasticinubber band ligation is outpatient trestmont for second-degree (those that bleed with defecation) internal hemorrhoids. Report An Error 31272014 11:16:49 AM Mark this question & => Question Td : 82066 Question 13 of 30 “What is the most sensitive diagnostic method for confirming acute cholecystitis? a) Abdominal CT sean, 6) Abdominal ultrasound ©) Abdominal X-ray. 4) Endoscopic retrograde cholangicpancreatography (ERCP) 8) Radionuclide scan. Answer | Explanation Other User's Explanation Report An Error Question Explanatior A radionuclide scan (e.g, HIDA) is the best method for confirming the diagnosis of acute cholecystitis. The dye is injected intravenously and a positive test is when the gallbladcler fails to he visualized (secondary to inflarnmation preventing uptake of the tracer). The other diagnostic methods are less sensitive 31272014 11:16:49 AM Mark this question & => Question Td : 82066 Question 13 of 30 ‘What is the most sensitive dagnostic method for confirming acute cholecystitis? a) Abdominal CT scan. ‘b) Abdominal ultrasound c) Abdominal X-ray. & Endoscopic retrograde cholangiopancreatography (ERCP). Y © ®) Radionuclide scan. Answer | Explanation Other User's Explanation Report An Error Question Explanatior A radionuclide scan (e.g, HIDA) is the best method for confirming the diagnosis of acute cholecystitis. The dye is injected intravenously and a positive test is when the gallbladcler fails to he visualized (secondary to inflarnmation preventing uptake of the tracer). The other diagnostic methods are less sensitive ‘Mark this question = => Question Id : 95382 Question 14 of 30 A patient has persistent substemal pain despite antacid use, Endoscopy demonstrates inregular erythematous patches several centimeters above the gastraesaphageal janction. Biopsy of one of these lesions demonstrates epithelial changes. The cell type that ‘was mostlikely observed in the involved areas is a) Ciliated columnar epithetinm ) Cuboidal epithetiam c) Keratinizine Squamous epithelium 4) Nonciiated columnar epithetiven ©) Nonkeratinizing Squamous epithelium Answer | Besianation Other User's Explanation Report An Error Question Explanation: ‘The medical condition is Barrett esophagus, in which the nommally nonlkeratinizing squamous epithelium of the esophagus undergoes metaplasia to gastric or intestinal ike epithelum composed of noncilated columnar epithelial cells that alternate in surface and gandular epithelium with goblet cells. Baurett esophagus typically develops in the seiting of chronic gastroesophageal reflux, and signficantly increases the risk of later development of adenocarcinoma of the distal esophagus. Ciiated columnar epithelinm is found in the respiratory tract Cuboidal epithelium is found in the kidney. peritoneal lining, and pleural lining, Keratinising squamous epithelium ic found in slcin. Nonkeratinizing squamous epithelium, in addition to being the normal epithelium of the esophagus, is found in the mouth, nose, and vagina ‘Mark this question = => Question Id : 95382 Question 14 of 30 A patient has persistent substemal pain despite antacid use, Endoscopy demonstrates inregular erythematous patches several centimeters above the gastraesaphageal janction. Biopsy of one of these lesions demonstrates epithelial changes. The cell type that ‘was mostlikely observed in the involved areas is a) Ciliated columnar epithetinm ) Cuboidal epithetiam c) Keratinizine Squamous epithelium Y © @ Nonciiated columnar epithelium ©) Nonkeratinizing Squamous epithelium Answer | Besianation Other User's Explanation Report An Error Question Explanation: ‘The medical condition is Barrett esophagus, in which the nommally nonlkeratinizing squamous epithelium of the esophagus undergoes metaplasia to gastric or intestinal ike epithelum composed of noncilated columnar epithelial cells that alternate in surface and gandular epithelium with goblet cells. Baurett esophagus typically develops in the seiting of chronic gastroesophageal reflux, and signficantly increases the risk of later development of adenocarcinoma of the distal esophagus. Ciiated columnar epithelinm is found in the respiratory tract Cuboidal epithelium is found in the kidney. peritoneal lining, and pleural lining, Keratinising squamous epithelium ic found in slcin. Nonkeratinizing squamous epithelium, in addition to being the normal epithelium of the esophagus, is found in the mouth, nose, and vagina ‘Mark this question & => Question Td : 96410 Question 15 of 30 ‘The appropriate treatment for hemochromatosis is which of the following? a) Venesection ') Transfision ©) Ferrous sulfate 4) Colchicine ¢) Thalidomide Avewor [UEXPIRNSER) otnor Usor Explanation Repert An Error Question Explanatic Removal of excess iron is a major therapeutic modality in hemochromatosis and can be accomplished by venesecticn, Transfusion aad ferrous sulfate would increase the iron load, Colchicine and thalidomide would not be helpful ‘Mark this question & => Question Td : 96410 Question 15 of 30 ‘The appropriate treatment for hemochromatosis is which of the following? Y © a) Venesection ) Transfasion ©) Ferrous sulfate 9) Colchicine ¢) Thalidomide Avewor [UEXPIRNSER) otnor Usor Explanation Repert An Error Question Explanatic Removal of excess iron is a major therapeutic modality in hemochromatosis and can be accomplished by venesecticn, Transfusion aad ferrous sulfate would increase the iron load, Colchicine and thalidomide would not be helpful ‘Mark this question Question Td : 100703 Question 16 of 30 A 32 year old man with a 15 year history of ulcerative colitis develops intermittent cholestatic jaundice. Ultrasonographic examination fails to reveal galltones. Liver hiopay demonstrates a large bile duct ebstruction. Endoscopic retrograde chalangiopancreatography ERCE) would show a) Alternating constriction and dilatation of intrahepatic bile ducts b) Markedly dilated common bile duct containing irregular radiclucent masses ©) Mass at the ampulla of vater 4) Moderately dilated intrahepatic bile ducts and stricture inthe bile duct at the porta hepatic ©) Very Gated bilary tree termineting in a blunt, nipple-like obstruction at the lower end of the common bile duct Answer | Explanation Other User's Explanation Report An Error Question Explanation: ‘The most likely diagnosis is primary sclerosing chelanaits, a disorder with a probable autoimmune component that is associated with ulcerative colitis (wo thirds of primary sclerosing cholangitis patierts have a history of ulcerative colitis). The disease is characterized by inflammation and fibrosis of the intrahepatc and extrahepatic bile ducts, prodacing alternating strictures and diatation of the structures, These changes are seen as "beading’ on endoscopic retrograde cholengopancreatography. Gallstones in the biliary tree produce irregular radiolucent masses. Option C describes the findings associated with carcinoma of the ampulla of Vater Option D describes the findings associated with carcinoma of the extrahepatic bile ducts. Option E describes the findings arsociated with carcinoma of the pancreas, ‘Mark this question <—& => Question Td : 100703 Question 16 of 30 A. 32 year old man with a 15 year history of ulcerative coltis develops intermittent cholestatic jamdice. Ultrasonographic examination fails to reveal gallstones. Liver biopsy demonstrates a large bile duct obstruction. Endoscopic retrograde cholangiopancreatography (ERCP) would show V © a) Alternating constriction and dilatation of intrahepatic bile ducts b) Markedly diated common bile duct containing aregular radiolucent masses c) Mass at the ampulla of vater d) Moderately dilated intrahepatic bile ducts and stricture in the bile duct at the porta hepatic ©) Very dilated biiary tree terminating in a blunt, nipple-like obstruction at the lower end ofthe common bile duct Answer | Explanation Other User's Explanation Report An Error Question Explanation: ‘The most likely diagnosis is primary sclerosing chelanaits, a disorder with a probable autoimmune component that is associated with ulcerative colitis (wo thirds of primary sclerosing cholangitis patierts have a history of ulcerative colitis). The disease is characterized by inflammation and fibrosis of the intrahepatc and extrahepatic bile ducts, prodacing alternating strictures and diatation of the structures, These changes are seen as "beading’ on endoscopic retrograde cholengopancreatography. Gallstones in the biliary tree produce irregular radiolucent masses. Option C describes the findings associated with carcinoma of the ampulla of Vater Option D describes the findings associated with carcinoma of the extrahepatic bile ducts. Option E describes the findings arsociated with carcinoma of the pancreas, ‘Mark this question & => Question Td : 118562 Question 17 of 30 The recommended medical reatment for chronic hepatitis B is which one of the following? a) Liver transplantation b) Steroids c) Interferon alpha ) Antbioties ©) Hepatitis B recombinant vaccine Question Explanation Interferon alpha eliminates serologic markers and improves liver histology and liver enzymes in some patients with chronic active hepatits B. It kas been shownto prolong survival Stopping treatment with this drug results n relapse of the infection. Three million usits subcutaneously three times a week is the treatment of choice. Liver transplantation is a cure, but requires surgical, not medical, irtervention. Steroids are used in treating autoimmune Iupoid hepatits. Antibiotics are used to treat an underlying bacterial infection of the liver, aot vical The hepatitis B vaccine is used to prevent hepatitis B, not to treat it once the infection is already present and has jon Report An Error done considerable damage to the liver. ‘Mark this question & => Question Td : 118562 Question 17 of 30 The recommended medical reatment for chronic hepatitis B is which one of the following? 2) Liver transplantation b) Steroids ¥ © o} Interferon alpha dD) Antibiotics ¢) Hepatitis B recombinant vaccine Question Explanation Interferon alpha eliminates serologic markers and improves liver histology and liver enzymes in some patients with chronic active hepatits B. It kas been shownto prolong survival Stopping treatment with this drug results n relapse of the infection. Three million usits subcutaneously three times a week is the treatment of choice. Liver transplantation is a cure, but requires surgical, not medical, irtervention. Steroids are used in treating autoimmune Iupoid hepatits. Antibiotics are used to treat an underlying bacterial infection of the liver, aot vical The hepatitis B vaccine is used to prevent hepatitis B, not to treat it once the infection is already present and has jon Report An Error done considerable damage to the liver. 3i2/2014 11:17:46 AM Mark this question & => Question Td : 118877 Question 18 of 30 “Which of the following has NOT been incriminated as a cance of hepatic encephalopathy? a) Ammonia. &) Mercaptans. ¢) Octopamine, &) Short chain fatty acids £) Dopamine. Question Explanation: Hepatic encephalopathy is a reversible neuropsychiatric syndrome frequently associated with severe, decompensated liver disease Several toxic metabolites derived ftom metabolism of nitrogenous substrates have been implicated, including ammonia and metaptans Short chain fatty acids, phenol, and false neotransmitters, such as octopamine, have also been incrimnated, Excessive concentrations of gamma aminobutyric acid (GABA), the major inhibitory neurotransmitter, as well as GABA like substances, such as endogenous benzodiazepines, have also been incriminatsd and supported by experimental stacies 3i2/2014 11:17:46 AM Mark this question & => Question Td : 118877 Question 18 of 30 ‘Which of the following has NOT been incriminated as a cause of hepatic encephalopathy? 2) Armonia b) Mercaptans ©) Octopamine ¢) Short chain fatty acids. Y © 2) Dopamine. Question Explanation: Hepatic encephalopathy is a reversible neuropsychiatric syndrome frequently associated with severe, decompensated liver disease Several toxic metabolites derived ftom metabolism of nitrogenous substrates have been implicated, including ammonia and metaptans Short chain fatty acids, phenol, and false neotransmitters, such as octopamine, have also been incrimnated, Excessive concentrations of gamma aminobutyric acid (GABA), the major inhibitory neurotransmitter, as well as GABA like substances, such as endogenous benzodiazepines, have also been incriminatsd and supported by experimental stacies Mark this question ez Question Id: 119510 Question 19 of 30 A77 year old nursing home residen: presents with symptoms of watery Giamthea, dehydration, and malaise. No fecal leukocytes or fever is present, The infectious agent that is most likely responsible for this presentation is 8) Shigella ) Giardia lamblia OE coli ©) Yersinia emerocaliica 8) Strongyloides stercoralis Answer | Bolanation Other User's Explanation Report An Error Question Explanation: Enterotozigenic E. coli secretes an exotoxin that induces secretory diamthea by disrupting intestinal absorption. Clinically, patients present with watery diarrhea but no fever or abdomninal pain. However, certain species of E. coli penetrate the intestinal ining and cause bleeding Giardiasis induces intermittent nausea and pain, and stools tend to be bulky rather than watery Yersinia enterocolitica would cause crampy abdominal pain, fever, and fecal leukocytes, It causes an invasive, exudative bacterial diarrhea Strongylcides siervoralis, a parasite responsible for disease primarily in the tropics, causes diarrhea with vorriting and epigastric pain. Mark this question ez Question Id: 119510 Question 19 of 30 A.77 year old aursing home resident presents wita symptoms of watery ciamhea, dehydration, and malaise. Mo fecal leukocytes or fever is present. The infectious agent thatis most likely responsible for this presentation is 2) Shigella ) Giardia lamblia VO DE colt ©) Yersinia enserocalitica 8) Strongyloides stercoralis Answer | Bolanation Other User's Explanation Report An Error Question Explanation: Enterotozigenic E. coli secretes an exotoxin that induces secretory diamthea by disrupting intestinal absorption. Clinically, patients present with watery diarrhea but no fever or abdomninal pain. However, certain species of E. coli penetrate the intestinal ining and cause bleeding Giardiasis induces intermittent nausea and pain, and stools tend to be bulky rather than watery Yersinia enterocolitica would cause crampy abdominal pain, fever, and fecal leukocytes, It causes an invasive, exudative bacterial diarrhea Strongylcides siervoralis, a parasite responsible for disease primarily in the tropics, causes diarrhea with vorriting and epigastric pain. ‘Marke this question €& => Question Td : 120045 Question 20 of 30 A37 year old woman experiences a greasy. malodorous diarrhea, She works in a daycare and has recently gone camping in westem Pennsylvania. Stool cubures are performed that were heme and leuicocyte negative. Diagnostic feature of giarciasis would be which one ofthe following? a) No infectious organisms in the stool ) Tumbling motile organisms in the stool ©) Postive staining with methylene bine 4) Positive acid-fast staining #) None of the above Question Explanation: Gaardiasis is caused by Giardia lambia. It typically causes a greasy malodorous diarrhea that is both heme and lesleocyte negative The two most commorly associated keywords with Giardia are day care and the Soviet Union, It is diagnosed by the detection of “tumbling motile" organisms in the stool. ‘Marke this question €& => Question Td : 120045 Question 20 of 30 A.37 yeas old woman experiences a greasy, malodorous diatrhea, She works in a daycare and has recently gone camping in westem Pennsylvania. Stool cubures are performed that were heme and leucocyte negative. Diagnostic feature of giardiacis would be which one of the following? a) No infectious organisms in the stool Y © b) Tumbling motile organisms in the stool ©) Postive staining with methylene bine 4) Positive acid-fast staining #) None of the above Question Explanation: Gaardiasis is caused by Giardia lambia. It typically causes a greasy malodorous diarrhea that is both heme and lesleocyte negative The two most commorly associated keywords with Giardia are day care and the Soviet Union, It is diagnosed by the detection of “tumbling motile" organisms in the stool. ‘Marke this question & => Question Ta : 136556 Question 21 of 30, FALSE associations regarding the Zollinger Ellison smidrome is which one of the following? a) Diarrhea results fiom outpouring of large amounts of hydroctoric acid into the proximal duodenum, £) Steatorrhea results from inactivation of pancreatic lipase ©) Uloer disease results from hypergastiinemia ©) Reduced pH within the gut interferes with intrinsic factor mediated vitamin By absorption. ) Renal stones result fom hypoparathpreidism Question Explanation: “There is an association between the MEN I syndrome and the ZE syndrome. Hyperparathyroidism is the most commen component ofthe MENT syndrome seen in persons with the ZE syndrome. The originally described Zollinger Elison (ZE) syncrome consisted of uicer disease associated with markedly increased gastric acid secretion, in association with non beta islet cell pancreatic tumors. Diathea, occuring in approximately 40% of affected persons, results from an outpouring of large amounts ofhydrectloric acid into the proximal duodemum, Steatorthea is due to inactivation of pancreatic lipase by large concentrations of acid in the prostmal stall intestine and from decreases in luminal bile acids. Reduced pH within the gut interferes with intrinsic factor mediated vitamin By absorpticn ‘Marke this question & => Question I Question 21 of 30, FALSE associations regarding the Zollinger Ellison smidrome is which one of the following? a) Diarrhea results fiom outpouring of large amounts of hydroctoric acid into the proximal duodenum, £) Steatorrhea results from inactivation of pancreatic lipase ©) Uloer disease results from hypergastiinemia ©) Reduced pH within the gut interferes with intrinsic factor mediated vitamin By absorption. © ¢) Renal stones result from bypoparathyreidism Question Explanation: “There is an association between the MEN I syndrome and the ZE syndrome. Hyperparathyroidism is the most commen component ofthe MENT syndrome seen in persons with the ZE syndrome. The originally described Zollinger Elison (ZE) syncrome consisted of uicer disease associated with markedly increased gastric acid secretion, in association with non beta islet cell pancreatic tumors. Diathea, occuring in approximately 40% of affected persons, results from an outpouring of large amounts ofhydrectloric acid into the proximal duodemum, Steatorthea is due to inactivation of pancreatic lipase by large concentrations of acid in the prostmal stall intestine and from decreases in luminal bile acids. Reduced pH within the gut interferes with intrinsic factor mediated vitamin By absorpticn Mark this question &= => Question Td : 139133 Question 22 of 30 A42 year old woman has a several year history of progressive abdominal colic and constipation Colonic biopsy stained with H&E shows pinle acellular material deposition in the submucosa and around blood vessels. The acelhilar material exhibits green birefringence when stained with Congo red. The bireftingence is thought to be mosi closely related io which protein properties? a) Ability to bind to oxygen b) Beta pleated sheet secondary structure ©) Electrophoretic mobility 4) Hydroxyprotine content €) Molecular weight Answer | Explanation Other User's Explanation Report An Error Question Explanation: ‘The patient has amyloidosis, which can be presentin the manner illastrated. Originally, it was thought that amyloid represented a single protein. However, as the various settings in which amyloidosis occur were futher explored, it was discovered that a variety of clearly distinct proteins could produce the material Eventually, i: was observed that what these diverse proteins had in common was a beta pleated sheet secondary structure that apparently is reflected in the apple green bireffingence. None of the other properties are thought to contribute to bireftingence when viewed with polarized ight. The ability to bind oxygen is important to hemoglobin. Hlectrophoretic mobility is used to separate serum proteins. Hydrozy proline content is important to collagen structure Molecular weightis a basic protein property. Mark this question &= => Question Td : 139133 Question 22 of 30 A42 year old woman has a several year history of progressive abdominal colic and constipation Colonic biopsy stained with H&E shows pirle acellular material deposition inthe submucora and around blood vessels. The acelular material exhibits green bireftingeace whea stained with Congo red. The bireftingence is thought to be most closely related to which protein properties? a) Ability to bind to oxygen Y © b) Beta pleated sheet secondary structure ©) Electrophoretic mobility 4) Hydroxyproline content €) Molecular weight Answer | Explanation Other User's Explanation Report An Error Question Explanation: ‘The patient has amyloidosis, which can be presentin the manner illastrated. Originally, it was thought that amyloid represented a single protein. However, as the various settings in which amyloidosis occur were futher explored, it was discovered that a variety of clearly distinct proteins could produce the material Eventually, i: was observed that what these diverse proteins had in common was a beta pleated sheet secondary structure that apparently is reflected in the apple green bireffingence. None of the other properties are thought to contribute to bireftingence when viewed with polarized ight. The ability to bind oxygen is important to hemoglobin. Hlectrophoretic mobility is used to separate serum proteins. Hydrozy proline content is important to collagen structure Molecular weightis a basic protein property. Mark this question = => Question Id : 197352 Question 23 of 30 A43 year old female presents with tiredness. Her Hb is 7.8 g/dL, MCV is 72 L, WBC countis 7.6 x 109/L, platelet count is 350 x 109/L, seruin ferritin is Syg/L. She was started on oral iron therapy and a month later her Hb was 8.0 gfdL. The lcely cause of the failure of her haemoglobin to respond to this treatment is a) Coeliac disease ) Folate deficiency ©) Inadequate dosage of'iron 4) Poor compliance with therapy @) Sideroblastic anaemia Question Explanation: ‘The most likely explanation for the failure of an iron deficiency anaemia to respond to iron therapy in a menstruast female is poor compliance. Itis likely that the dose that this patients prescribed would be adequate and ifnot some response would stil be expected There is no evidence of a concomitant folate deficiency as suggested by the blood picture, which would also argue against coeliac disease. Similarly, there is no evidence to cuggest a sideroblastic anaemia where a rsiced MCV and increased ferritin may be expected. Mark this question = => Question Id : 197352 Question 23 of 30 A43 year old female presents with tiredness, Her Hb is 7.8 g/AL, MCV is 72 £L, WBC countis 7.6 x 109/L, platelet count is 350 x 109/L, seruin ferritin is Syg/L. She was started on oral iron therapy and a month later her Hb was 8.0 gfdL. The lcely cause of the flue of her haemoglobin to respond to thic treatment is a) Coeliac disease +) Folate deficiency c) Inadequate dosage of'iron Y © & Poor compliance with therapy ) Sideroblastic anaemia Question Explanation: ‘The most likely explanation for the failure of an iron deficiency anaemia to respond to iron therapy in a menstruast female is poor compliance. Itis likely that the dose that this patients prescribed would be adequate and ifnot some response would stil be expected There is no evidence of a concomitant folate deficiency as suggested by the blood picture, which would also argue against coeliac disease. Similarly, there is no evidence to cuggest a sideroblastic anaemia where a rsiced MCV and increased ferritin may be expected. ‘Marl this question = => Question Td: 197475 Question 24 of 30 A.25 year old female has ingested an unknown quantity of paracetamol tablets 4 hours ago. She presents with nausea, vomiting, anorexia aad right subchondral pain, Which feature cuggests that che shouid be transferred to liver unit? a) ALT 800 untsiL. 'b) Blood glusose 5 mmol c) Heatt glucose 5 mmol/L d) pH7.25 ¢) Systolic BP 100 mmmEig Answer | Bolanation | Other User's Explanation Report An Error Question Explanation: ApH ofless than 7.3 is a poor prognostic factor for this patent The criteria for transfer to a specialist unit are + Encephalopathy © INR 22.0 at <48 of 23.5 at <72h © Serum creatinine > 200 mol/L * Blood pH <73 + Systolic BP <20 mmlig ‘Marl this question = => Question Td: 197475 Question 24 of 30 A.25 year old fernale has ingested an unknown quantity of paracetamol tablets 4 hours ago. She presents with nausea, vorniting, anorexia aad right subchondral pain, Which feature cuggests that che shouid be transferred to liver unit? a) ALT 800 untsiL. 'b) Blood glusose 5 mmol c) Heatt glucose 5 mmol/L Y © dpH7.25 ¢) Systolic BP 100 mmmEig Answer | Bolanation | Other User's Explanation Report An Error Question Explanation: ApH ofless than 7.3 is a poor prognostic factor for this patent The criteria for transfer to a specialist unit are + Encephalopathy © INR 22.0 at <48 of 23.5 at <72h © Serum creatinine > 200 mol/L * Blood pH <73 + Systolic BP <20 mmlig ‘Marke this question €& => Question Td : 197718 Question 25 of 30 4.42 year old previously healthy male is achitted with a sinale haemetemesis after taking 300 mg of aspirin 5 hours ago. His pulse is 120/min, BP of 110/75 mmHg (ying) and 90/60 mmHg (standing). Respiratory and abdominal exam is normal, His hemoglobin is 7 gidL. The likely cause of his haematemesis is 2) Angiodysplasia ) Duodenal ulcer ¢) Gastric cancer 8) Gastric erosions £) Oesophagitis Answer | Explanation Other User's Explanation Report An Error Question Explanation ‘The most likely answer is gastic erosions based upon the fact thatthe incident has occurred after only one dose of Aspirin without any prior history of chronic usage which is associated with oesophagitis and ulcer formation, Malignancy is unlikely because of history of being fit and also the age. ‘Marke this question €& => Question Td : 197718 Question 25 of 30 4.42 year old previously healthy male is achitted with a sinale haemetemesis after taking 300 mg of aspirin 5 hours ago. His pulse is 120/min, BP of 110/75 mmHg (ying) and 90/60 mmHg (standing). Respiratory and abdominal exam is normal, His hemoglobin is 7 gidL. The likely cause of his haematemesis is 2) Angiodysplasia ) Duodenal ulcer ¢) Gastric cancer Y © ¢ Gastric crosions £) Oesophagitis Answer | Explanation Other User's Explanation Report An Error Question Explanation ‘The most likely answer is gastic erosions based upon the fact thatthe incident has occurred after only one dose of Aspirin without any prior history of chronic usage which is associated with oesophagitis and ulcer formation, Malignancy is unlikely because of history of being fit and also the age. ‘Mark this question ==> Question 26 of 30 Question Id : 197728 A.53 year old male presents with general deterioration, He drinks 25 units of alcohol each week and smokes 5 cigarettes daily. Exam reveals jaundice, spider nevi on chest. His temperature is 37.2°C and has hepato-splenomegaly. His bilirubin, ALP and AST are increased. HBsAg is postive, HBeAg is negative and Hepatitis 3 virus DNA is undetectable The likely diagnosis is @) Alcoholic liver disease ) Autoimmmane chronic active hepatitis ©) Carcinoma of the pancreas Chronic hepatitis B infection ©) Chronic hepatitis D (delta) infection Question Explanation: EbsAg is postive, Hepatitis E antigen is negative and the DNA being undetectable suggests that itis Chronic HB infection with low viral replication, ‘Mark this question ==> Question 26 of 30 Question Id : 197728 A.53 year old male presents with general deterioration, He drinks 25 units of alcohol each week and smokes 5 cigarettes daily. Exam reveals jaundice, spider nevi on chest. His temperature is 37.2°C and has hepato-splenomegaly. His bilirubin, ALP and AST are increased. HBsAg is postive, HBeAg is negative and Hepatitis 3 virus DNA is undetectable The likely diagnosis is 2) Alcoholic liver disease +) Autoimmune chronic active hepatitis ¢) Carcinoma of the pancreas Y © & Chronic hepatitis B infection ©) Chronic hepatitis D (delta) infection Question Explanation: EbsAg is postive, Hepatitis E antigen is negative and the DNA being undetectable suggests that itis Chronic HB infection with low viral replication, Mark this question €& => Question Td : 197778 Question 27 of 30 ‘A. 33 year old woman with Crohn's Disease has a history of a right hericolectomy for deo-colonic disease. Since the operation she thas had frequent diarrhoea but no blood in the stools. Her ESR is 10mm (0-20mm/'Ist hour), platelet count is 240) x109/L, and serum CRD is 7 mg/L («10). The best treatment is a) Cholestyramine b) Mesalazine ) Metronidazole 4) Omeprazole ¢) Prednisclone Quostion Explanation: ‘The ESE, CRP and platelet counts are notraised, indicating that this patient's symptoms are not due to active Crohn's. Also the diarrhoea is not bloody which goes against active Crohn‘s colitis. Hence mesalazine or prednisolone would not be effective here Metronidazole is typically given for peri-anal disease. The history inchides a previous right hemicolectomy for ileo-celonic disease. Lose of the terminal ileum frequently leade to bile calt malabsorption and treatment with the bile calt chelator cholestyramine quickly relieves the problem, Mark this question €& => Question Td : 197778 Question 27 of 30 ‘A. 33 year old woman with Crohn's Disease has a history of a right hericolectomy for deo-colonic disease. Since the operation she thas had frequent diarrhoea but no blood in the stools. Her ESR is 10mm (0-20mm/'Ist hour), platelet count is 240) x109/L, and serum CRP is 7 mg/L (10). The best treatment is Y © a) Cholestyramine b) Mesalazine ) Metronidazole 4) Omeprazole ¢) Prednisclone Quostion Explanation: ‘The ESE, CRP and platelet counts are notraised, indicating that this patient's symptoms are not due to active Crohn's. Also the diarrhoea is not bloody which goes against active Crohn‘s colitis. Hence mesalazine or prednisolone would not be effective here Metronidazole is typically given for peri-anal disease. The history inchides a previous right hemicolectomy for ileo-celonic disease. Lose of the terminal ileum frequently leade to bile calt malabsorption and treatment with the bile calt chelator cholestyramine quickly relieves the problem, Mark this question & => Question Td : 199445) Question 28 of 30 ‘A patient presents with haematemesis, Esophagogastroducdenoscopy detects a bleed in the lesser curvature of the stomach. Which atteryis likely to be the cause of the bleeding? 2) Left gastroomental artery ) Pancreaticodiodenal artery «) Right hepatic artery 4) Right gastric artery 6) Splenic artery Anower (EEIFIRRRTNY o1ner users Exton Question Explanation: ‘The pancreaticoduodenal artery supplies mainly the upper and lower duodemm and the head of the pancreas. The gastroomental srteries supply the greater curvature of the stomach, The right gastric artery arises from the hepatic artery or the left hepatic artery, supplies the pylorus and travels along the lesser curvature of the stomach, supplying it, and anastomosing with the left gastric artery. Report An Error Mark this question & => Question Td : 199445) Question 28 of 30 ‘A patient presents with haematemesis, Esophagogastroducdenoscopy detects a bleed in the lesser curvature of the stomach. Which atteryis likely to be the cause of the bleeding? 2) Left gastroomental artery ) Pancreaticodiodenal artery «) Right hepatic artery Y © 4) Right gastric artery 6) Splenic artery Anower (EEIFIRRRTNY o1ner users Exton Question Explanation: ‘The pancreaticoduodenal artery supplies mainly the upper and lower duodemm and the head of the pancreas. The gastroomental srteries supply the greater curvature of the stomach, The right gastric artery arises from the hepatic artery or the left hepatic artery, supplies the pylorus and travels along the lesser curvature of the stomach, supplying it, and anastomosing with the left gastric artery. Report An Error ‘Marke this question & => Question Td : 199455 Question 29 of 30 In apetient with haematemesis an oesophagogastroduodenoscopy detects 2 bleed in the greater curvature of the stomach, Artery that is moct likely to be the cource ofthe bleeding is which one of the folowing? a) Cystic artery +) Right gactric artery c) Right hepate artery ©) Splenic artery ©) Superior pancreaticoduodenal artery Question Explanation: ‘The superior pancreaticoduodenal ertery, a branch of the gestrocucdenal artery, supplies the head of the pancreas and the upper duodenum and anastomoses with the inferior pancreaticoduodenal artery. The right hepatic artery supplies the right lobe of the liver and pert of the candate lobe. The right gastric artery is mainly fommd to supply the pylorus and the lesser curvature of the stemach ‘The splenic artery sends branches to the greater curvature of the Stomach, as well as supplying pancreas and spleen. ‘Marke this question & => Question Td : 199455 Question 29 of 30 In apetient with haematemesis an oesophagogastroduodenoscopy detects 2 bleed in the greater curvature of the stomach, Artery that is moct likely to be the cource ofthe bleeding is which one of the folowing? a) Cystic artery +) Right gactric artery c) Right hepate artery YM © & Splenic artery ©) Superior pancreaticoduodenal artery Question Explanation: ‘The superior pancreaticoduodenal ertery, a branch of the gestrocucdenal artery, supplies the head of the pancreas and the upper duodenum and anastomoses with the inferior pancreaticoduodenal artery. The right hepatic artery supplies the right lobe of the liver and pert of the candate lobe. The right gastric artery is mainly fommd to supply the pylorus and the lesser curvature of the stemach ‘The splenic artery sends branches to the greater curvature of the Stomach, as well as supplying pancreas and spleen. Merle this question & Question Td : 199465 Question 30 of 30 A61 year old female with liver cisthosis has abdominal pain, malaise and nausea. She quit crinking since the diagnosis 8 months ago. On exarn she has moderate ascites and generalized abdominal tenderness. Her Ha is 11.2 gfdL, WBC count is 15 109M, PT is 21 sec, albumin is 28 g/L, and total bilirubin is 56 moliL. Ascitic Quid protein is 26 g/L, ascitic uid anrylese is normal and ascitic Buid ‘WBC count is 500 z109/L. The ikely reason for her current problem is 8) Hepatic vein thrombosis, ) Pancreatic pseudocyst rupture ©) Portal vein thrombosis (PVT) 6) Primary liver cancer ©) Spontaneous bacterial peritonitis Question Explanati ‘The high white cell countin the ascites makes spontaneous bacterial peritontis (SBP) much more likely than Budd-Chiari syndrome (GCS), PVT, hepatocellular carcinoma (HCC) , or a muptured pancreatic , pseudocyst. Abdominal pain is often only mild, or even. absent in SBP, with patients often presenting with otherwise unexplained hepatic decompensation. Merle this question & Question Td : 199465 Question 30 of 30 A61 year old female with liver cisthosis has abdominal pain, malaise and nausea. She quit crinking since the diagnosis 8 months ago. On exarn she has moderate ascites and generalized abdominal tenderness. Her Ha is 11.2 gfdL, WBC count is 15 109M, PT is 21 sec, albumin is 28 g/L, and total bilirubin is 56 moliL. Ascitic Quid protein is 26 g/L, ascitic uid anrylese is normal and ascitic Buid ‘WBC count is 500 z109/L. The ikely reason for her current problem is 8) Hepatic vein thrcmbosis ) Pancreatic pseudocyst rupture 6) Postal vein thrarebesis (PVT) 6) Primary liver cancer Y © ®) Spontaneous bacterial peritonitis Question Explanati ‘The high white cell countin the ascites makes spontaneous bacterial peritontis (SBP) much more likely than Budd-Chiari syndrome (GCS), PVT, hepatocellular carcinoma (HCC) , or a muptured pancreatic , pseudocyst. Abdominal pain is often only mild, or even. absent in SBP, with patients often presenting with otherwise unexplained hepatic decompensation. ‘Marke this question => Question Ta : 24780 Question 1 of 30 A 34-year-old woman with a history of alcoholism and self-neglect, presents with an episode of blocd streaked vomiting attibuted 10 minor Mellory-Weiss tear. She is given an intravenous infusion of 5% dextrose. The following day, her serum potassium concentrations fal to1.9 mEq/L on admission. Whet is the ikcely mechanism for the fallin potassium concentration? a) Cortisol selease in response to siress increasing renal potassium loss ) Decompensated liver failure causing aldosterone secretion ©) Meiabolic acidosis increasing renal potassium excretion 4) Intracellular re-uptake in response to re-feeding with glucose ¢) Potassium levels falling following gastric loss in vomiting Avewor [UERVRNREN) iner veers Explanation Report An Error Question Explanation: This neglected person is being fed with dextrose, which will cause an elevation of circulating insulin to maintain glycemic control, This will consequenlly drive potassium intracellularly so reducing extracellular potassium concentration, ‘Marke this question => Question Ta : 24780 Question 1 of 30 A 34-year-old woman with a history of alcoholism and self-neglect, presents with an episode of blocd streaked vomiting attibuted 10 minor Mellory-Weiss tear. She is given an intravenous infusion of 5% dextrose. The following day, her serum potassium concentrations fal to1.9 mEq/L on admission. Whet is the ikcely mechanism for the fallin potassium concentration? a) Cortisol release in response to siress increasing renal potassium loss +b) Decompensated liver failure causing aldosterone secretion c) Metabolic acidosis increasing renal potassium excretion ¥ © @) Intracellular re-uotalce in response to re-feeding with glucose ¢) Potassium levels falling following gastric loss in vorniting Avewor [UERVRNREN) iner veers Explanation Report An Error Question Explanation: This neglected person is being fed with dextrose, which will cause an elevation of circulating insulin to maintain glycemic control, This will consequenlly drive potassium intracellularly so reducing extracellular potassium concentration, ‘Marke this question & => Question Ta : 28302 Question 2 of 30 A. 53-year-old diabetic has hepatomegaly. Albumin =3 g/dL, Total bilrubin=1.29 mg/dL, ALP=134 UI L, ALT=90 UL, GGT=125 UL and Ferritin= 1450 yg(L. Which of the followingis most suggestive of a diagnosis of Hemockromatosis? 2) Gynecomastia b) Migratory polyarthritis, ©) Chondrocalcinosis 6) Myxedema ©) Chorea Anower [UENSNGREN) incr voers Explanation Report An Error Question Explanation: This man with diabetes has evidence of iver disease with grossly elevated ferritin suggesting a diagnosis of hemochromatosis. A noteinigratory polyarthritis would be suggestive although an cligoarthrtis particular of the hands and hip is more typical Chorea is seen in Wilson's disease. ‘Miyzedema is not a feature of hemochromatosis. Gynecomastiais a feature of liver disease/cirrhosis per se and not just hemochromatosis. However, chondrovalcinosis together with the chronic arthropathy is well recognized in association with hemochromatesis, ‘Marke this question & => Question Ta : 28302 Question 2 of 30 A53-year-old diabetic has hepatomegaly. Albumin =3 g/dL, Total biirubia=1.29 mg/dL, ALP=134 UL, ALT=90 UI L, GGT=125 WL and Ferrtin= 1450 jafL. Which of the following is most suagestive of a diagnosis of Hemochromatosis? 2) Gynecomastia 1b) Migratory polyarthritis ¥ © 0 Chondrocalcinosis ¢) Myxedema ©) Chorea Anower [UENSNGREN) incr voers Explanation Report An Error Question Explanation: This man with diabetes has evidence of iver disease with grossly elevated ferritin suggesting a diagnosis of hemochromatosis. A noteinigratory polyarthritis would be suggestive although an cligoarthrtis particular of the hands and hip is more typical Chorea is seen in Wilson's disease. ‘Miyzedema is not a feature of hemochromatosis. Gynecomastiais a feature of liver disease/cirrhosis per se and not just hemochromatosis. However, chondrovalcinosis together with the chronic arthropathy is well recognized in association with hemochromatesis, ‘Marke this question <= => Question Td : 41527 Question 3 of 30 A 60 year old female comes to the office because of pain in the left lower quadrant of her abdomen for the past 48 hours. The pain is associated with tenesmus, loss of appetite and a sensation of being febrile. History of ritable bowel symptoms is present but she has not taken any medications for it. Viral signs are: temperanure 37.3C (99.2), Pulse 84/imin and regular and respirations 12/min, On physical exemination the abdomen is tender in the left lower quadrant No rebound tencemess is present and bowel sounds are normal Leukocyte count is 10,200 with 71% segmerted neutrophils and 3% band forms. The patient is prescribed tetracycline therapy andis asked to take liquid diet. Three days later she returns because of the worsening of the pain, associeted with chills ‘What is the most appropriate next step? 2) Admit her to the hospital and prepare her for an immediate operation b) Change to ciprofloxacin therepy, orally, and send her home ©) Admit her to the hospital and begin cefoxitin therapy, intravenously 4) Continue the present course of therapy €) Do celonoscopy newer [REIRIRIRHGRY) Other Users Explanation Report An Exes Question Explanatio. ‘The patient described has divertioulosic leading to diverticulitis. You must start antibiotice that cover the gram-positive cocci, gram negative bavill and anaerobes found in the abdomen, Cefoxitin fits the till Immediate surgery ss too aggressive, particularly without even an aitempt ct antbictic therapy. Ciprofloxacin is inappropriate as ithas poor gram-positive coverage and no anaerobic coverage. Continung the present therapy is incorrect because the worsening pain and chills imply infection. Colonoscopy is dangerous in diverticulitis because it can lead to perforation, ‘Marke this question <= => Question Td : 41527 Question 3 of 30 -A.60 year cld female comes to the office because of pain in the left lower quadrant of her abdomen for the past 48 hours. The pein is associated with tenesmus, lose of appetite and a sensation of beng febrile. History of irritable bowel symptoms is present but she has not taken any medications for it. Vital signs are: temperature 37.3C (99.2F), Pulse 84/min and regular and respirations 12min. On pysical examination the abdomen is tender in the left lower quadrant. No rebound tenderness is present and bowel sounds are normal Leukeeyte count is 10,200 with 71% segmented neutrophils and 3% band forms. The patient is prescribed tetracycline therapy and is asked to take liquid diet. Three days later she returns because of the worsening of the pain, associated with chills “What is the most appropriate next step? 2) Admit her to the hospital and prepare her for an immediate operation +b) Change to ciprofloxacin therapy, orally, and send her home YM © ©) Admither to the hospital and begin cefoxitin therapy, intravenously 4) Continue the present course of therapy €) Do colonoscopy newer [REIRIRIRHGRY) Other Users Explanation Report An Exes Question Explanatio. ‘The patient described has divertioulosic leading to diverticulitis. You must start antibiotice that cover the gram-positive cocci, gram negative bavill and anaerobes found in the abdomen, Cefoxitin fits the till Immediate surgery ss too aggressive, particularly without even an aitempt ct antbictic therapy. Ciprofloxacin is inappropriate as ithas poor gram-positive coverage and no anaerobic coverage. Continung the present therapy is incorrect because the worsening pain and chills imply infection. Colonoscopy is dangerous in diverticulitis because it can lead to perforation, “Marke this question & => Question 4 of 30 Question Ta : 48732 An 18 year old boy with a history of fever, fatigue, weight loss, and mild diarrhea of 2 months duration is found te have a palpable macs in the sight lower quadcant cf the abdomen, What is the most Hcely diagnosis? a) Crohn's disease (regional enteritis) +) Ulcerative colitis c) Amebic coltis 4) Divertioultis ©) Lymphoma Question Explanation: ‘When Crohn's disease affects primarily the distal small intestine (regional ententis), a most characterise clinical patter emerges. A young person, usually in the second of third decade, will present with a period of episodic abdominal pain, largely postprandial and, offen periumbilical, occasionally with low-grade fever and mild diarhea. Anorexia, navsea, and voiting may also be present, Weight Joss is fequent, Some patients may be aware of tendemess in the rightlower quadrant and even ofa palpable mass in that region, “Marke this question & => Question Td : 48732 Question 4 of 30 An 18 year old boy with a history of fever, fatigue, weidht loss, and mild diarrhea of 2 months duration is found to have apalpable macs in the sight lower quadcant cf the abdomen, What is the most Hcely diagnosis? Y © a) Crohn's disease (regional enteritis) +) Ulcerative colitis c) Amebic coltis 4) Divertioultis ©) Lymphoma Question Explanation: ‘When Crohn's disease affects primarily the distal small intestine (regional ententis), a most characterise clinical patter emerges. A young person, usually in the second of third decade, will present with a period of episodic abdominal pain, largely postprandial and, offen periumbilical, occasionally with low-grade fever and mild diarhea. Anorexia, navsea, and voiting may also be present, Weight Joss is fequent, Some patients may be aware of tendemess in the rightlower quadrant and even ofa palpable mass in that region, ‘Marle this question ez Question Tad : 51286 Question 5 of 30 “What is the best method to cure 2 patient who has a chronic peptic ulcer disease? a) Hebcobacter pylori eradication ) Proton pump inhibitors ©) H2 blockers 4) Nissen fundoplication Avewor (JENSEN) other UsorsExplanation Repost An Error Question Explanation: E_ pylori and NSAIDs disrupt normal mucosal defense and repair, making the mucesa more susceptible to acid. HL. pylori infection is present in 50 to 70% of patients with duodenal ulcers and 30 to 50% of patients with gastric ulcers. IPE, Pylori is eradicated, only 10% of patients have recurrence of peptic ulcer disease, compared with 70% recurrence in patients treated with acid suppression alone Symptoms inchde pain oftea localized to the epigasirium and relieved by food or antacids. The pain is described as buning or giawing, or sometimes as a sensation of hunger. The course is usually chronic and recurrent. Diagnosis of peptic ulcer is suggested by patient history and confirmed by endoscopy (EGD). Treatment of gastric and duodenal ulcers requires eradication of H. pylori when present. Methods of decreasing acidity include a number of drugs including proton pump inhibitors and H2 blockers. Nissen fimdoplication is a surgical treatment for chronic GERD, ‘Marle this question ez Question Tad : 51286 Question 5 of 30 “What i the best method to cure « patient who has a chronic peptic ulcer disease? WV © a) Hekcobacter pylori eradication +) Proton purep inhibitors ©) H2 blockers 4) Nissen fundoplication Avewor (JENSEN) other UsorsExplanation Repost An Error Question Explanation: E_ pylori and NSAIDs disrupt normal mucosal defense and repair, making the mucesa more susceptible to acid. HL. pylori infection is present in 50 to 70% of patients with duodenal ulcers and 30 to 50% of patients with gastric ulcers. IPE, Pylori is eradicated, only 10% of patients have recurrence of peptic ulcer disease, compared with 70% recurrence in patients treated with acid suppression alone Symptoms inchde pain oftea localized to the epigasirium and relieved by food or antacids. The pain is described as buning or giawing, or sometimes as a sensation of hunger. The course is usually chronic and recurrent. Diagnosis of peptic ulcer is suggested by patient history and confirmed by endoscopy (EGD). Treatment of gastric and duodenal ulcers requires eradication of H. pylori when present. Methods of decreasing acidity include a number of drugs including proton pump inhibitors and H2 blockers. Nissen fimdoplication is a surgical treatment for chronic GERD, Mark this question & => Question Td : 54269 Question 6 of 30 A 52 year old male presents to the emergency room with severe epigasitic pain, low-grade fever, tachycardia, and mild hypotension, ‘The patient has 2 history of moderate to heavy social drinking, The resident suspects acute pancreatitis. The single mos: important laboratory finding to confirm the diagnosis of pancreatitis is a) Hypetlipidernia b) Hyperbilinubinemia ¢) Elevated serum phospholipase A &) Elevated serum amylase ¢) Elevated serum alkaline phosphatase Question Explanation: Acute pancreatitis is inflammation of the pancreas (and, sometimes, adjacent tissues) caused by the release of activated pancreatic enzymes. The most common triggers are biliary tract disease and chronic heavy alcohol intake. The condition ranges from mild (abdominal pain and vomiting) to severe (pancreatic necrosis and a systemic inflammatory process with shock and mubiorgan failure), Diagnosis is based on clinical presentation and serum amylase and lipase levels. Treatment is supportive, with TV fluids, analgesics, snd fasting Mark this question & => Question Td : 54269 Question 6 of 30 A 52 year old male presents to the emergency room with severe epigasitic pain, low-grade fever, tachycardia, and mild hypotension, ‘The patient has 2 history of moderate to heavy social drinking, The resident suspects acute pancreatitis. The single mos: important laboratory finding to confirm the diagnosis of pancreatitis is a) Hypetlipidernia b) Hyperbilinubinemia ¢) Elevated serum phospholipase A Y © & Elevated serum amylase ¢) Elevated serum alkaline phosphatase Question Explanation: Acute pancreatitis is inflammation of the pancreas (and, sometimes, adjacent tissues) caused by the release of activated pancreatic enzymes. The most common triggers are biliary tract disease and chronic heavy alcohol intake. The condition ranges from mild (abdominal pain and vomiting) to severe (pancreatic necrosis and a systemic inflammatory process with shock and mubiorgan failure), Diagnosis is based on clinical presentation and serum amylase and lipase levels. Treatment is supportive, with TV fluids, analgesics, snd fasting ‘Mark this question e& => Question Td : 55502 Question 7 of 30 ATS year old complains of pain with defection, and loose stools over the last several months. His past medical history includes prostate cancer treated with radiation therapy 5 years ago, hypertension, and osteoarthritis. Medications incinde hydrochlorothiazide (HydreDIURIL), a Beta-blocker, and acetaminophen. On colonoscopy, ne polyps or cancer are found, but the rectal and sigmoid areas show pallor with iiabilty and telangisctasias. The most likely diagnosis is 2) Farnilial angiodysplasia +b) Osler Weber Rendu syndrome ) Late-onset ulcerative colitis a) @) Sensitivity to acetaminophen breakdown products Answer | Boplanation Other User's Explanation Report An Etror Question Explanation: Chronic radiation proctitis develops months to years after radiation and is characterized by pain with defecation, diarrhea, and sometimes rectal bleeding, On colonoscopy, the mucosa is pale and friable with telangiectares which are sometimes large, multiple, and serpiginous. Radiation proctitis ‘Mark this question e& => Question Td : 55502 Question 7 of 30 ATS year old complains of pain with defection, and loose stools over the last several months. His past medical history includes prostate cancer treated with radiation therapy 5 years ago, hypertension, and osteoarthritis. Medications incinde hydrochlorothiazide (HydreDIURIL), a Beta-blocker, and acetaminophen. On colonoscopy, ne polyps or cancer are found, but the rectal and sigmoid areas show pallor with iiabilty and telangisctasias. The most likely diagnosis is 2) Farnilial angiodysplasia +b) Osler Weber Rendu syndrome ) Late-onset ulcerative colitis Y¥ © d) Radiation prociitis ¢) Sensitivity to acetaminophen breakdown products Answer | Boplanation Other User's Explanation Report An Etror Question Explanation: Chronic radiation proctitis develops months to years after radiation and is characterized by pain with defecation, diarrhea, and sometimes rectal bleeding, On colonoscopy, the mucosa is pale and friable with telangiectares which are sometimes large, multiple, and serpiginous. ‘Marke this question & => Question Ta : 79955 Question 8 of 30 A 26-year-old female presents with altered mental status and spider angiomas on exam. She was found to have cirrhosis of the liver oftunknown etology. She was not an alcoholic and she did not take drugs. What is the most likely diagnosis? 2) Systemic lupus erythematosus £) Sarcoidosis ©) Wilson's disease 6) Schizophrenia ©) Hemochromatosis Question Explanation: “Wilson's disease is an autosomal recessive disorder that causes accumulaton of copper in the iver, brain, and other organs. Clinical sign: first appear anywhere between the ages oF 2 and 50, and most frequently chiring adclescenc=, Accumulation of copper in the liver often progresses jo cirthosis. Accumulation in the brain offen leads to moter disturbances (tremors, dysphagia, dystonia, poor coordination, etc.) or inappropriate behavior resembling schizophrenia or manic-depression, This disease should be suspected in any patient under 40 years with neurologic changes, chronic active hepatitis, and cirrhosis oP unknown etology. Patients with SLE usually do not present intielly with liver disease. Renal disease is more common. CNS changes can occur because of cerebritis. Sarcoidosis, is a granulomatous disease of unknown etiology thet can affect any orgen system, Patients with schizophrenia may present with psychotic changes, but cathosis is not common, Hemochromatosis is an iron storage disorder with accumulation of excess iron in the heaxt, iver, and pititary gland, Bronze skin pigmentation is common ‘Marke this question & => Question Ta : 79955 Question 8 of 30 A 26-year-old female presents with altered mental status and spider angiomas on exam. She was found to have cirrhosis of the liver oftunknown etology. She was not an alcoholic and she did not take drugs. What is the most likely diagnosis? 2) Systemic lupus erythematosus £) Sarcoidosis Y © ©) Wilson’s disease 6) Schizophrenia ©) Hemochromatosis Question Explanation: “Wilson's disease is an autosomal recessive disorder that causes accumulaton of copper in the iver, brain, and other organs. Clinical sign: first appear anywhere between the ages oF 2 and 50, and most frequently chiring adclescenc=, Accumulation of copper in the liver often progresses jo cirthosis. Accumulation in the brain offen leads to moter disturbances (tremors, dysphagia, dystonia, poor coordination, etc.) or inappropriate behavior resembling schizophrenia or manic-depression, This disease should be suspected in any patient under 40 years with neurologic changes, chronic active hepatitis, and cirrhosis oP unknown etology. Patients with SLE usually do not present intielly with liver disease. Renal disease is more common. CNS changes can occur because of cerebritis. Sarcoidosis, is a granulomatous disease of unknown etiology thet can affect any orgen system, Patients with schizophrenia may present with psychotic changes, but cathosis is not common, Hemochromatosis is an iron storage disorder with accumulation of excess iron in the heaxt, iver, and pititary gland, Bronze skin pigmentation is common ‘Mark this question & => Question Id : 79979 Question 9 of 30 A 43-year-old smoker and alcoholic has severe epigastric pain, diarrhea, and hematemesis. His father also had similar symptoms Cienetidine did not reliave hirm, Milk and food de not relieve these symptoms. Diagnese 2) Duodenal ulcer +) Zollinger Hlicon syndrome ) Pancreatitis 4) Appendicitis 6) Coloric obstruction Answer | Brptanaton | Other User's Explanation Report An Error Question Explanation: Zollinger-Elison syndrome is caused by extremely elevated acid levels in the stomach, causing ulcers in unusual places in the gastric tmicosa. An elevated gastrin level is associated with this smekome and the usual antacids and H2 blockers do not usnelly relieve the symptoms. It can bs hereditary and also associated with steatomhea, Omeprazole or surgical resections are the oaly options for treatment, Duodenal ulcers are usually relieved by milk, food, antacids, and H2 blockers. Pancreatitis would not cause acute diarrhea unless it was chronic pancreatitis associated with malabsorption Diffuse abdominal pain is more common The amylase and lipase level would be elevated, Appendicitis would not be hereditary and the pain would be located in the sight lower quadrant. In obstruction, there would be no diahea or passing of atus. The pain would be in the lower quadrants. ‘Mark this question & => Question Id : 79979 Question 9 of 30 A 43-year-old smoker and alcoholic has severe epigastric pain, diarrhea, and hematemesis. His father also had similar symptoms Cienetidine did not reliave hirm, Milk and food de not relieve these symptoms. Diagnese 2) Duodenal ulcer J © b) Zollinger Ellison syndrome ) Pancreatitis 4) Appendicitis 6) Coloric obstruction Answer | Brptanaton | Other User's Explanation Report An Error Question Explanation: Zollinger-Elison syndrome is caused by extremely elevated acid levels in the stomach, causing ulcers in unusual places in the gastric tmicosa. An elevated gastrin level is associated with this smekome and the usual antacids and H2 blockers do not usnelly relieve the symptoms. It can bs hereditary and also associated with steatomhea, Omeprazole or surgical resections are the oaly options for treatment, Duodenal ulcers are usually relieved by milk, food, antacids, and H2 blockers. Pancreatitis would not cause acute diarrhea unless it was chronic pancreatitis associated with malabsorption Diffuse abdominal pain is more common The amylase and lipase level would be elevated, Appendicitis would not be hereditary and the pain would be located in the sight lower quadrant. In obstruction, there would be no diahea or passing of atus. The pain would be in the lower quadrants. Mark this question & => Question Td : 80287 Question 10 of 30 A.46-year cld has hepatocellular carcinoma, He was premovsly diagnosed with allergic bronchopulmonary eosinopiulia when he also hhad wheezing and a CXR revealed pulmonary infiltrates. Corticosteroids were affective contrast to anti-fungal agents. What agent is responsible? @) Candida Albicans 6) Histoplasma capsulatum. ©) Coccidiodes imrnitis ® Aspergillus Javus ©) Pneumocystis jirovecti Question Explanation: Aspergillus flavus is a cause of allergic bronchopuimonery aspergillosis. The treetment of aspergillosis does not depend on antifungal agents because the symptoms arise as a result of aberrant immune responses to the fungus rather than the fimgal agent cansing syreptoms itself Therefore, steroids are typically employed for treatment. Aspergilhis flavus releases an aflatoxin that increases the sk for development of hepatocellular carcinoma Mark this question & => Question Td : 80287 Question 10 of 30 A.46-year old has hepatocelilar carcinoma, He was premously diagnosed with allergic bronchopulmonary eosinophilia when he also had wheezing and a CXR revealed pulmonary infiltrates. Corticosteroids were affective in contrast to anti-fungal agents. What agent is responsible? a) Candida Albicans +) Hictoplacma capsulatum ©) Coccidiodes iramitis V © & Aspergillus lavus ©) Faeumocystis jrovecti Question Explanation: Aspergillus flavus is a cause of allergic bronchopuimonery aspergillosis. The treetment of aspergillosis does not depend on antifungal agents because the symptoms arise as a result of aberrant immune responses to the fungus rather than the fimgal agent cansing syreptoms itself Therefore, steroids are typically employed for treatment. Aspergilhis flavus releases an aflatoxin that increases the sk for development of hepatocellular carcinoma ‘Mark this question & => Question Td : 85045 Question 11 of 30 A.27 year old female gives bitth to a baby at term. The newborn infant has the most common type of tracheoesophagealfisila (type ). When snd how will this developmental anomaly mast likely be discovered? a) Immedictely, with the baby’s first breaths ') On the frst day, with hie Bret meal ©) Affer 1 week, with persistent weight loss 4) Benween 1 and 3 years, with exercise intolerance «) Never, itis usually asymptomatic Answer | bxplanation Other User's Explanation Report An Error Question Explanation: ‘The most common type of trache oesophageal fisula involves a proximal esophagus ending in a blind loop, and a distal esophagus that originates in the tracheoesophageal fismala. With the first meal, mik collects in the proximal esophagus and spills over into the trachea when the epigiotis opens with breathing. The infant wil quite cuddenly develop recpiratory distress and cyanosis due to aspirated ail. ‘Tracheoesopkageal fistula usually becomes apparent when the child aspirates. The oral secretions generally do not produce respiratory distress immediately postpartam, although the child may appear "drooly." Immediate respiratory distress (choice A) indicates a major respiratory tract cbstruction or cardiovascular defect. TTracheoesopkageal fistula would almost certainly be detected as a consequence of dyspnea while eating, well before weight loss and failure to thrive would develop (choice C). Exercise intolerance (choice D) ina child is most suggestive of cardiac insufficiency and may reflect a congenital cardiac malformation. Trachesesophageal fistula heccmes apparent with esting, nat with exercise Trachsorsophageal fistula is always symptematic (choice E) ‘Mark this question & => Question Td : 85045 Question 11 of 30 A.27 year old female gives bitth to a baby at term. The newborn infant has the most common type of tracheoesophagealfisila (type ). When snd how will this developmental anomaly mast likely be discovered? a) Immedictely, with the baby’s first breaths Y © d) On the first day, with his first meal ©) Affer 1 week, with persistent weight loss 4) Benween 1 and 3 years, with exercise intolerance «) Never, itis usually asymptomatic Answer | bxplanation Other User's Explanation Report An Error Question Explanation: ‘The most common type of trache oesophageal fisula involves a proximal esophagus ending in a blind loop, and a distal esophagus that originates in the tracheoesophageal fismala. With the first meal, mik collects in the proximal esophagus and spills over into the trachea when the epigiotis opens with breathing. The infant wil quite cuddenly develop recpiratory distress and cyanosis due to aspirated ail. ‘Tracheoesopkageal fistula usually becomes apparent when the child aspirates. The oral secretions generally do not produce respiratory distress immediately postpartam, although the child may appear "drooly." Immediate respiratory distress (choice A) indicates a major respiratory tract cbstruction or cardiovascular defect. TTracheoesopkageal fistula would almost certainly be detected as a consequence of dyspnea while eating, well before weight loss and failure to thrive would develop (choice C). Exercise intolerance (choice D) ina child is most suggestive of cardiac insufficiency and may reflect a congenital cardiac malformation. Trachesesophageal fistula heccmes apparent with esting, nat with exercise Trachsorsophageal fistula is always symptematic (choice E) 3:2/2014 11:24:40 AM € www interface.edu.pk/i) x \\_ CL www.interface.edupk/med dutd=14759 apps Ellcoogle A settings [Signin Sou wlohe dul eo ually Of cio ge igzall JS [Free Hotmall exams /test-analy #38 Gi other bookmar ‘Mare this question e& => Question Ta : 86183 Question 12 of 30 A patient's segment of jejunum had a nodalar lesion located in the subimucosa, Histologic examination revealed that the tumor is composed of uniform round cells arranged in trabeculae, with 2 "salt and pepper" chromatin pattern Electron microscopic studies reveal secretory granules, and immunchistochemical stains axe positive for serotonin Which of the following parameters correlates best with the metastatic potential of this tumor? a) Architectural pattern b) Cell pleomorphisn ©) Chromogranin positivity 4) Hormonal production ©) Site and size ) Maltiphicity g) Neuron-specific enclase positivity Question Explanation: ‘The tumor described is a carcinoid, This neoplasm derives from the neuroendocrine cells ofthe intestinal epithelium and often retains the capacity to produce hormonal substances, such as serotonin, gastrin, somatostatin, eto. The architectural arrangement is variable, although trabecular and insular pattems are the most common, Carcincid tumors may be found throughout the intestinal canal, as well asin the bronchial tree and other organs. Location and size are the most important predictors of their metastatic potential. Carcinoids ofthe appendix and cecum (the most frequent) only rarely metastasize to the liver. Carcinoids of the stomach, ileum, and colon have a high metastatic propensity, especially if they are larger than 2.cm in main diameter Neither architectural pattern nor cell pleomorphism is a good predictor of the malignant potential of carcinoid tumors. In general, this rule applies to most endocrine ‘tumors, in which histologic features and biologic behavior do not often correlate with one another. Ih cther words, a tumor that shows relatively bland cells may undergo metastatic spread while tumors with more anaplastic features may behave in a benign fashion. Positive immanchistechemical reactivity for chromogranin and neuron specific enolase are helpful in establishing that a tumor ic a carcinoid but do not confer information about prognosis. Hormonal production is cormmon in carcinoids but does not often lead 10 a clinical syndrome nor does it correlate with metastate potential, Multiplicty is relatively frequent in intestinal carcincids. Some series have reported muiple carcinoids in up to 35% of patients. Total Questions ke ke Ki bs IG ie | SRE BARBERBE XxXxXXXXKXKXKXKXKXKXKXKXKKXKXKKKKKKKKKKKKM 3:2/2014 11:24:40 AM € www interface.edu.pk/i) x \\_ CL www.interface.edupk/med dutd=14759 apps Ellcoogle A settings [Signin Sou wlohe dul eo ually Of cio ge igzall JS [Free Hotmall exams /test-analy #38 Gi other bookmar ‘Mare this question e& => Question Ta : 86183 Question 12 of 30 A patient's segment of jejunum had a nodular lesion located in the submucosa, Histologic examination revealed that the tamor is composed of uniform round cells arranged in trabeculae, with 2 "salt and pepper" chromatin pattem Blectron microscopic studies reveal secretory granules, and immunohistochemical stains are positive for serotonin, Which of the following parameters correlates best with the metastetic potential ofthis tumor? a) Architectural pattern b) Cell pleomorphism ©) Chremograrin positivity 4) Hormonal production JY © 6) Site and size £) Multiplicity 2) Nenron-specific enclase positivity Question Explanation: ‘The tumor described is a carcinoid, This neoplasm derives from the neuroendocrine cells ofthe intestinal epithelium and often retains the capacity to produce hormonal substances, such as serotonin, gastrin, somatostatin, eto. The architectural arrangement is variable, although trabecular and insular pattems are the most common, Carcincid tumors may be found throughout the intestinal canal, as well asin the bronchial tree and other organs. Location and size are the most important predictors of their metastatic potential. Carcinoids ofthe appendix and cecum (the most frequent) only rarely metastasize to the liver. Carcinoids of the stomach, ileum, and colon have a high metastatic propensity, especially if they are larger than 2.cm in main diameter Neither architectural pattern nor cell pleomorphism is a good predictor of the malignant potential of carcinoid tumors. In general, this rule applies to most endocrine ‘tumors, in which histologic features and biologic behavior do not often correlate with one another. Ih cther words, a tumor that shows relatively bland cells may undergo metastatic spread while tumors with more anaplastic features may behave in a benign fashion. Positive immanchistechemical reactivity for chromogranin and neuron specific enolase are helpful in establishing that a tumor ic a carcinoid but do not confer information about prognosis. Hormonal production is cormmon in carcinoids but does not often lead 10 a clinical syndrome nor does it correlate with metastate potential, Multiplicty is relatively frequent in intestinal carcincids. Some series have reported muiple carcinoids in up to 35% of patients. Total Questions ke ke Ki bs IG ie | SRE BARBERBE XxXxXXXXKXKXKXKXKXKXKXKXKKXKXKKKKKKKKKKKKM Marke this question & => Question Ta : 86834 Question 13 of 30 A 36 year old man with prolonged history of alcohol abuse presents to his physician asking for help to step drinking According to him he has not feit well over the last couple of months and has lost around 30 Ib. He also smokes one pack of cigarettes/day and admits to the occasional use of heroin for which he has spent time in jal. Bilateral lower extremity edema is present on physical examination Blood studies reveal anotmocytic anemia, high red blood cell distribution width, and low serum albumin, Further evaluation reveals increased levels of carcinoembryonic antigen. The most likely diagnosis is a) Bronchogenic carcinoma ) Colon cancer ) Gastric cancer 4) Hepatocellular carcinoma €) Pancreatic cancer Question Explanation: ‘The patient's history reveals roultiple risk factors for various diseases some of which overlap. Tumor markers although helpful in the diagnosis of cancer, especially when the primary is in doubt, are of particular use in monitoring response to treatment Carcinoembryonic antigen can be elevated in both pancreate and colenic neoplasms. The patients mise of microcytic and macracytic anemia (ron and B12ffolate deficiency, respectively) indicate that the neoplasia is more licely originating from the cclon. Bronchogenic carcinomas are not associated with elevated levels of CEA. Bombesin is a tumor marker associated with gastric carcinotnas. Alpha-fetoprotein (AFP) is a normal fetal protein that can be produced in the adutt by hepatocellar carcinomas and nonseminomatous germ-cell tumors of the testes. CEA is associated with pancreatic cancers, but as explained above, itis alese lcely siaggnosis: Lesions in the pancreatic head can also lead to a conjugated hyperbilirubinemia, Marke this question & => Question Ta : 86834 Question 13 of 30 A 36 year old man with prolonged history of alcohol abuse presents to his physician asking for help to step drinking According to hirn he has not fot well over the last couple of months and has lost around 30 Ib. Hie also smokes one packe of cigarettes/day and admits to the occasional use of heroin for which he has spent time in jal. Bilateral lower extremity edema is present on physical examination Blood studies reveal anormocytic anemia, high red blood cell distribution width, and low serum albumin, Further evauation reveals increased levels of carcinaembryemic antigen. The most lkely diagnosis is 2) Bronchogenic carcinoma Y © b) Colon cancer ©) Gastric cancer 4) Hepatocellular carcinoraa €) Pancreatic cancer Question Explanation: ‘The patient's history reveals roultiple risk factors for various diseases some of which overlap. Tumor markers although helpful in the diagnosis of cancer, especially when the primary is in doubt, are of particular use in monitoring response to treatment Carcinoembryonic antigen can be elevated in both pancreate and colenic neoplasms. The patients mise of microcytic and macracytic anemia (ron and B12ffolate deficiency, respectively) indicate that the neoplasia is more licely originating from the cclon. Bronchogenic carcinomas are not associated with elevated levels of CEA. Bombesin is a tumor marker associated with gastric carcinotnas. Alpha-fetoprotein (AFP) is a normal fetal protein that can be produced in the adutt by hepatocellar carcinomas and nonseminomatous germ-cell tumors of the testes. CEA is associated with pancreatic cancers, but as explained above, itis alese lcely siaggnosis: Lesions in the pancreatic head can also lead to a conjugated hyperbilirubinemia, ‘Merle this question <—c=> Question 14 of 30 A 24 year old man has the gradual onset of intermittent dicrrhea which, over year, progresses to severe diarrhea, alternating with constipation, rectal bleeding, and passage of mucus. The abdomen is tender over the colon on examination, Stool examination does not show patasites. Colonoscopy demonstrates inflammation limited to the rectum, with no higher lesions. ‘The disease that would most likely be seen in a close relative of the patient is a) Celiac disease ) Crohn disease ©) Hirschsprung disease ©) Tropical spme. €) Whipple disease Question Explanation: ‘The presentationis classic for ulceraiive colitis. Family members have an increased incidence of both ulcerative colts and Crotn disease, supporting the idea that these two diseases are actually different ends ofthe same spectrum. In contrast to Crohn disease, in which the lesions may be patchy and involve the distal ileum and even the esophagus. in ulcerative colitis the lesions involve the rectum and may extend continuously proximally for varying distances up to the cecum and very distal end of the eum. Colice diseace is a small intestinal cisease related to giuten Intolerance. Flattening of vill, elongeted crypts and marked inflammation in the larrina propria are aoted histologically. Hirschsprung disease is a congenital cause of severe constipation and megacolon due to a lack of gangtion cells in the distal colon. Tropical sprue clinically resembles celiac disease but may be related to infection ‘Whipple disease is an intestinal diarrheal disease that has bea shown to be due to a bacterial infection, Ulcerative colts is lmited to the colon and is the case described in the question stem. Itis one of two inflammatory bowel diseases (IBDs) characterized by inflammation of the Intestinal tract, probably due to an immune reaction, The other IBD is Crohn disease, which affects the whole lenath of the gnstrointestinal tract with characteristic skep lesions. Remember that IBD hac a clear farilal tendency linked ta a genetic predisposition to the disease Report An Error ‘Merle this question <—c=> Question 14 of 30 A 24 year old man has the gradual onset of intermittent diarrhea which, over year, progresses to severe diarhea, alternating with constipation, rectal bleeding, and passage of mucus. The adomenis tender over the colon on examination. Stool examination does not show parasites. Colonoscopy demonstrates inflammation limited to the rectum, with no higher lesions. ‘The disease that would most likely be seen in a close relative of the patient is a) Celiac disease ¥ © b) Crohn disease ©) Hirschsprung disease © Tropical spme e) Whipple disease Question Explanation: ‘The presentationis classic for ulceraiive colitis. Family members have an increased incidence of both ulcerative colts and Crotn disease, supporting the idea that these two diseases are actually different ends ofthe same spectrum. In contrast to Crohn disease, in which the lesions may be patchy and involve the distal ileum and even the esophagus. in ulcerative colitis the lesions involve the rectum and may extend continuously proximally for varying distances up to the cecum and very distal end of the eum. Colice diseace is a small intestinal cisease related to giuten Intolerance. Flattening of vill, elongeted crypts and marked inflammation in the larrina propria are aoted histologically. Hirschsprung disease is a congenital cause of severe constipation and megacolon due to a lack of gangtion cells in the distal colon. Tropical sprue clinically resembles celiac disease but may be related to infection ‘Whipple disease is an intestinal diarrheal disease that has bea shown to be due to a bacterial infection, Ulcerative colts is lmited to the colon and is the case described in the question stem. Itis one of two inflammatory bowel diseases (IBDs) characterized by inflammation of the Intestinal tract, probably due to an immune reaction, The other IBD is Crohn disease, which affects the whole lenath of the gnstrointestinal tract with characteristic skep lesions. Remember that IBD hac a clear farilal tendency linked ta a genetic predisposition to the disease Report An Error ‘Mark this question & => Question Td : 90254 Question 15 of 30 A.44 year old woman presents with a fever of 102.5°F, nausea, vomiting, and unrelenting right upper quadrant pain Muephy’s sian is positive and WBC is 20,000. What is the most litely diagnosis? a) Cholelithiasis +) Acute cholecystitis, ©) Acute appendicitis, 4) Diverticulitis 2) Diverticulosis Question Explanation The age of the patient fever. nausea, and unrelenting right upper quadrant pain with this WBC and Murphy's sign (increased subhepatic tenderness during a deep breath) are all consistent with acute cholecystitis. Chololithiasis is often asymptomatic, and when symptoms occur they are not usually associated with 2 fever, elevated WBC, or Murphy's sign. The pain usually resolves spontaneously, as opposed to the unrelenting pain of chelecystitis. The pain in appendicitis is located in the right lower quadrant and usvally occurs in younger patients. The pain of divesticulis is localized to the leftlower quadrant. Diverticulosis is not associated with the signs ofllness listed in the question. ‘Mark this question & => Question Td : 90254 Question 15 of 30 A 44 year old woman presents with a fever of 102.5°F, nausea, vomiting, and unrelenting right upper quadrant pain Murphy's sian is positive and WEC is 20,000, What is the most likely diagnosis? a) Cholelithiasis WV © ») Acute cholecystitis ©) Acute appendicitis. ) Diventicutits #) Diverticulosis Question Explanation The age of the patient fever. nausea, and unrelenting right upper quadrant pain with this WBC and Murphy's sign (increased subhepatic tenderness during a deep breath) are all consistent with acute cholecystitis. Chololithiasis is often asymptomatic, and when symptoms occur they are not usually associated with 2 fever, elevated WBC, or Murphy's sign. The pain usually resolves spontaneously, as opposed to the unrelenting pain of chelecystitis. The pain in appendicitis is located in the right lower quadrant and usvally occurs in younger patients. The pain of divesticulis is localized to the leftlower quadrant. Diverticulosis is not associated with the signs ofllness listed in the question. Mark this question & => Question Id : 93649 Question 16 of 30 A man presents to a physician because of troubling heartburn and dificuty swallowing, Esophageal motilty studies demonstrate a near complete absence of smooth muscle peristalsis and lower esophageal sphincter tone. Mo mass lesions are noted and the esophagus is not dilated. The finding that would also be present is 2) Anemia ) Atrophic alosstis ©) Hourgaass-shaped stomach on barium swallow ¢) Massively dilated coloa ©) Thick skin Question Explanation: Mear complete absence of muscle tone ard peristalsis is characteristic of involvement of the esophagus with scleroderma, which causes replacement of muscle by dease connective tissue. Similar changes in the dermris cause the skin to be thickened The thickened, shiny skin of the hands may cause them to resemble claws Anemia and atrophic glossitis are associated with esophageal webs in Plummer Vinson syndrome, An hourglass shaped stomach within the thoracic cavity is a feature of a siding hictal hernia. A maccively dilated esophagus (megaezophagus) can be caused by Chagas cicease, a trypanosomal disease that can also cause massive dilation of the colon Mark this question & => Question Id : 93649 Question 16 of 30 A man presents to a physician because of troubling heartburn and dificuty swallowing, Esophageal motilty studies demonstrate a near complete absence of smooth muscle peristalsis and lower esophageal sphincter tone. Mo mass lesions are noted and the esophagus is not dilated. The finding that would also be present is 2) Anemia ) Atrophic alosstis ©) Hourgaass-shaped stomach on barium swallow ¢) Massively dilated coloa Y © ©) Thick skin Question Explanation: Mear complete absence of muscle tone ard peristalsis is characteristic of involvement of the esophagus with scleroderma, which causes replacement of muscle by dease connective tissue. Similar changes in the dermris cause the skin to be thickened The thickened, shiny skin of the hands may cause them to resemble claws Anemia and atrophic glossitis are associated with esophageal webs in Plummer Vinson syndrome, An hourglass shaped stomach within the thoracic cavity is a feature of a siding hictal hernia. A maccively dilated esophagus (megaezophagus) can be caused by Chagas cicease, a trypanosomal disease that can also cause massive dilation of the colon ‘Marke this question & => Question Ta : 94322 Question 17 of 30 A 66 year old man presents with a chief complaint that solid food gets stuck in the middle of his chest He lost I 1kg weight over the lest 3 months. The most ilkely diagnosis is a) Esophagitis ) Lower esophageal ring ©) Esophageal carcinema 3) Cerebrovascular accidents €) Myocardial infarction Answer | Explanation Other User's Explanation Report An Error Question Explanation: ‘The most common malignant esophageal tumor is squamous cell carcinoma, followed by adenocarcinoma Symptoms are progressive dysphagia end weight loss. Diagnosis is by endoscopy. followed by CT and endoscopic ultrasound. for staging Treatmert varies with stage and generaly includes surgery with or without chemotherapy and radiation. Long term survivals poor except for those with local disease. ‘Marke this question & => Question Ta : 94322 Question 17 of 30 A 66 year old man presents with a chief complaint that solid food gets stuck in the middle of his chest He lost I 1kg weight over the lest 3 months. The most ilkely diagnosis is a) Esophagitis ) Lower esophageal ring Y © 0) Esophageal carcinoma 3) Cerebrovascular accidents €) Myocardial infarction Answer | Explanation Other User's Explanation Report An Error Question Explanation: ‘The most common malignant esophageal tumor is squamous cell carcinoma, followed by adenocarcinoma Symptoms are progressive dysphagia end weight loss. Diagnosis is by endoscopy. followed by CT and endoscopic ultrasound. for staging Treatmert varies with stage and generaly includes surgery with or without chemotherapy and radiation. Long term survivals poor except for those with local disease. Mark this question ez Question Id : 95311 Question 18 of 30 A.33 year old female presents with complaints of buming substemal chest pain of several months duration, The pain is exacerbated by large meals, cigarettes, and caffeine, Her symptoms are worse when she lies on her back, especially when sleeping at aight. Antacids often improwe her symptoms. This woman is at risk for developing which of the following conditions? 2) Cardiac ischemia +) Columnar metaplasia of the distal esophagus ©) Esophageal web 4) Leiomyoma of the esophagus €) Mallory-Weiss tears in the esophagus newer [UBIRNBWEN) otnerueorsExplanation Report An Error Question Explanation: This woman likely suffers from reflux esophagitis, a condition in which the lower esophageal sphincter (LES) does nat adequately prevent acidic gastric contents from refusing backing to the distal esophogus. Most commonly there is a defect in the LES mechenism itself, in additon to secondary causes such as pregnancy (due to increased abdominal pressure) and some medications (Gnticholinergics, beta 2 agonists, theophylline, nitrates and calcium channel blockers). Symptoms can mimic cardiac chest pain and must be carefully evaluated. Complications of refx esophaaitis include esophageal strictures, ulcerations, laryngitis. pulmonary spiration, and Barrett esophagus (colimnar metaplasia of the distal esophagus). Barrett esophagus is considered a premalignant statz, with roughly a 30 fold increase in the incidence of esophageal adenocarcinoma. The normal squamous epithelium of the esophagus transforms into columnar epithelium similar to gastric epithelium as a result of recurrent reflux of acidic gastric contents -Anginal pain, signaling cardiac ischemia is generally not burning in nature and is not relieved by antacids. Esophageal webs tend to involve the upper esophagus and have been associated in some patients with iron deficiency anemia Leiomyoma is a benign smooth: muscle tumor that can occur in the esophagus but is unrelated to acid reflux, Mallory- Weiss tears are tears of epithelia of the proximal stomach or distal esophagus as aresub of retching (seen in anerexics and alcoholics). Mark this question ez Question Id : 95311 Question 18 of 30 4.33 year old female presents with complaints of burning substemal chest pain of several months duration. The painis exacerbated by large meals, cigareties, and caffeine. Her symptoms are worse whea she lies on ker back, especially when sleeping at aight. “Antacids offen improve her symptoms. This woman is at risk for developing which of the following conditions? a) Cardiac ischemia Y © b) Columnar metaplasia of the distal esophagus ©) Esophageal web 4) Leiomyoma of the esephagas ©) Mallory-Weiss tears in the esophagus newer [UBIRNBWEN) otnerueorsExplanation Report An Error Question Explanation: This woman likely suffers from reflux esophagitis, a condition in which the lower esophageal sphincter (LES) does nat adequately prevent acidic gastric contents from refusing backing to the distal esophogus. Most commonly there is a defect in the LES mechenism itself, in additon to secondary causes such as pregnancy (due to increased abdominal pressure) and some medications (Gnticholinergics, beta 2 agonists, theophylline, nitrates and calcium channel blockers). Symptoms can mimic cardiac chest pain and must be carefully evaluated. Complications of refx esophaaitis include esophageal strictures, ulcerations, laryngitis. pulmonary spiration, and Barrett esophagus (colimnar metaplasia of the distal esophagus). Barrett esophagus is considered a premalignant statz, with roughly a 30 fold increase in the incidence of esophageal adenocarcinoma. The normal squamous epithelium of the esophagus transforms into columnar epithelium similar to gastric epithelium as a result of recurrent reflux of acidic gastric contents -Anginal pain, signaling cardiac ischemia is generally not burning in nature and is not relieved by antacids. Esophageal webs tend to involve the upper esophagus and have been associated in some patients with iron deficiency anemia Leiomyoma is a benign smooth: muscle tumor that can occur in the esophagus but is unrelated to acid reflux, Mallory- Weiss tears are tears of epithelia of the proximal stomach or distal esophagus as aresub of retching (seen in anerexics and alcoholics). ‘Maré this question = => Question Td : 98292 Question 19 of 30 A.30 year old woman presents with fatigue. Blood analysis demonstrates macro ovalocytes and hypersegmented aewroplils, with serum vitamin B12 levels being 89 mg/ml. LEAST licely to be a cause of her condition is a) Infection by diphyllobothrium lau ) Crohns disease c) Pemicious anemia 4) Viral hepatitis ©) Gastrectomy Anowor [SERBIAN ote: sors explanation Report An Evror Question Explanation: Vitamin B12 is obtained in virtually every food It is bound to intrinsic factor, a cabstance that is released fom parietal cells in the stomach. Vitamin B 12's ultimate site of absorption isin the eum. Factors that disrupt this can be harmful io proper amounts of vitamin B 12 being absorbed, a necessary step for the formation of nomnal red blood cells. Diphyllobotfrium Latur or the fish tapeworm interferes with vitamin B12 in the gut. Crohn's disease reduces ileal absorption of vitamin B 12, while gastrectomy and pemicious anemia interfere with binding of vitamin B12 to intracis factor. Pernicious anemia is the most common cause of vitamin B12 deficient anemia ‘Maré this question = => Question Td : 98292 Question 19 of 30 A.30 year old woman presents with fatigue, Blood analysis demonstrates macro ovalocytes and hypersegmented aeureplils, with serum vitamin B12 levels being 89 mg/ml. LEAST licely to be a cause of her condition is a) Infection by diphyllob othrium latum ) Crohns disease c) Pernicious anemia Y © d) Viral hepatitis c) Gastrectomy Anowor [SERBIAN ote: sors explanation Report An Evror Question Explanation: Vitamin B12 is obtained in virtually every food It is bound to intrinsic factor, a cabstance that is released fom parietal cells in the stomach. Vitamin B 12's ultimate site of absorption isin the eum. Factors that disrupt this can be harmful io proper amounts of vitamin B 12 being absorbed, a necessary step for the formation of nomnal red blood cells. Diphyllobotfrium Latur or the fish tapeworm interferes with vitamin B12 in the gut. Crohn's disease reduces ileal absorption of vitamin B 12, while gastrectomy and pemicious anemia interfere with binding of vitamin B12 to intracis factor. Pernicious anemia is the most common cause of vitamin B12 deficient anemia 3i2/2014 11:26:37 AM ‘Mark this question & => Question Id: 112729 Question 20 of 30 ‘True statement regarding hepatitis C is which one of the folowing? 2) Normal ALT transaminase levels are conarnon in hepatitis C infection +) Hepatitis C is transmitted via the fecal oral route ©) Most people infected with chronic hepaitis C are symptomatic. 4) Treatment with recombinant interferon alfa-23 showed rapid decreases in ALT levels end reduction of viral RIVA after a siz mcath course of treatment three times weeldy ¢) Chroric HCV infection may be a progressive disease associated with a decreased risk of death from liver Ealure or hepatocellular carcinoma Question Explanation: Interferon alpha-2b improves liver histology and decreases viral RNA levels so thet there has been alower rate of cirrhosis and hhepatecellular carcinoma, Interferon treatment should nct be administered to paiients with autoimmune hepattis or decompensated cirrhosis. Elevated iver enzymes, especially ALT, are elevated in chronic hepatiis C. The degree of elevation of the enzymes and the severity of symptoms do not corvelate. Hepatitis Cis transmitted mainly by needle injection ot blood transfusions. Most people infected with chronic hepatitis C! infection are asymptomatic and are informed by their physicians because of abnormal iver tests. An. increased likelhood of death from liver failure or liver cancer occurs with chronic non treated hepatitis C infection, 3i2/2014 11:26:37 AM ‘Mark this question & => Question Id: 112729 Question 20 of 30 ‘True statement regarding hepatitis C is which one of the folowing? 2) Normal ALT transaminase levels are conarnon in hepatitis C infection +) Hepatitis C is transmitted via the fecal oral route ©) Most people infected with chronic hepaitis C are symptomatic. Y © 4) Treatment with recombinant interferon alfa-2B showed rapid decreases in ALT levels and reduction of viral RNA after a siz mcath course of treatment three times weeldy ¢) Chroric HCV infection may be a progressive disease associated with a decreased risk of death from liver Ealure or hepatocellular carcinoma Question Explanation: Interferon alpha-2b improves liver histology and decreases viral RNA levels so thet there has been alower rate of cirrhosis and hhepatecellular carcinoma, Interferon treatment should nct be administered to paiients with autoimmune hepattis or decompensated cirrhosis. Elevated iver enzymes, especially ALT, are elevated in chronic hepatiis C. The degree of elevation of the enzymes and the severity of symptoms do not corvelate. Hepatitis Cis transmitted mainly by needle injection ot blood transfusions. Most people infected with chronic hepatitis C! infection are asymptomatic and are informed by their physicians because of abnormal iver tests. An. increased likelhood of death from liver failure or liver cancer occurs with chronic non treated hepatitis C infection, ‘Mark this question & => Question Td : 120190 Question 21 of 30 The disorder of ths esophagus that will present with dysphagia to beth liquids and solids is which one of the following? a) Esophageal carcinoma b) Esophageal we c) Esophageal stricture 4) Achatasia ¢) None of the above Question Explanation: Dysphagia to both liquids and solids implies esophageal motility disorder ike achalasia, scleroderma, or diffuse esophageal spasm. The other choices cause obstruction and, hence, present initially with dysphagia to solids only. ‘Mark this question & => Question Td : 120190 Question 21 of 30 ‘Tae disorder of the esophagus that will present with dysphagia to both liquids and solids is which one of the fellowing? a) Esophageal carcinoma ) Esophageal web ) Esophageal stricture V © 4) Achalasia ¢) None of the above Question Explanation: Dysphagia to both liquids and solids implies esophageal motility disorder ike achalasia, scleroderma, or diffuse esophageal spasm. The other choices cause obstruction and, hence, present initially with dysphagia to solids only. c Mark this question & => Question Td : 136650 Question 22 of 30 ‘True statement regarding alcoholic iver disease is which one of the following? 2) The type of alcohol consumed is directly related to the degree of ver damage. +) A defect in acetaldehyde metabolism produces liver fibrosis c) For the same relative alcohol consumption, women develop milder forms of alcoholic liver disease than men 4) Petivenular deposition of collagen may be the initial manifestation of significant ongoing fibrosis, ©) enough alcchol is consumed, cirthosis will develop in more than 90% of persons. Question Explanation: Alcohol consumption is the leading cause of cirrhosis both in the United States and Europe. The amourt and duration of alcohol consumption, rather than the type of alcchel consumed, are directly related to the degree ofiver damage produced. No metabolic defect in alcohol metabolisia has been identified in cirshotic patients or their family members which suggests a predisposition to fibrosis or cirrhosis, For the same reletive alcohol consumption, women develop more severe forms of alcoholic liver disease than men. Estimated consumption of 80 grams of ethanol daily for 20 years or 160 grams daily for 10 years have been reported to be associated with cirrhosis. However, only 10-15% of alcoholics are thought to develop cirrhosis of the liver. Perivenular deposition of collagen is thought to be one of the earliest indicators of eventual cirthosis, Mark this question & => Question Td : 136650 Question 22 of 30 ‘True statement regarding alcoholic iver disease is which one of the following? 2) The type of alcohol consumed is directly related to the degree of ver damage. +) A defect in acetaldehyde metabolism produces liver fibrosis c) For the same relative alcohol consumption, women develop milder forms of alcoholic liver disease than men Y © 6) Petivenular deposition of collagen may be the initial manifestation of significant ongoing fibrosis ©) enough alcchol is consumed, cirthosis will develop in more than 90% of persons. Question Explanation: Alcohol consumption is the leading cause of cirrhosis both in the United States and Europe. The amourt and duration of alcohol consumption, rather than the type of alcchel consumed, are directly related to the degree ofiver damage produced. No metabolic defect in alcohol metabolisia has been identified in cirshotic patients or their family members which suggests a predisposition to fibrosis or cirrhosis, For the same reletive alcohol consumption, women develop more severe forms of alcoholic liver disease than men. Estimated consumption of 80 grams of ethanol daily for 20 years or 160 grams daily for 10 years have been reported to be associated with cirrhosis. However, only 10-15% of alcoholics are thought to develop cirrhosis of the liver. Perivenular deposition of collagen is thought to be one of the earliest indicators of eventual cirthosis, Mark thie question <& => Question Td : 139041 Question 23 of 30 A 42 year old woman with pelcythemia Vera develops progressive severe ascites and tender hepatomegaly over a period of several months. LETs are near normal. The likely diagnosis would be established by a) Endoscopic retrograde cholangiopancreatography (EPCP) ) Hepatic venography ©) Serum alpha fetoprotein 4) Serum cervloplestain ) Senam iton stadies Question Explanation: The clnical presentation is most consistent with Budd Chiari syndrome (hepatic vein obstruction, usually de to thrombosis) which may occur as a complication of thrombogenic and mycloproligcrative disorders, including polycythemia vera. The presentation illustrated is the most common; alternative presentations include fukninant iver failure and cases in which intractable abdominal pain is the most prominent intial finding The best method listed to establish the diagnosis of Budd Chiari syncrome is hepatic venography to demonstrate the occlusion of the hepatic vencus system. Liver biopsy to provide evidence of centrilobular congestion and sinusoidal dilation (in the absence of right-sided heart failure) is definitive but moze invasive. Endoscopic retrograde cholangiopancreatography is most usefil in demonstrating lesions of the biliary tree, Serum alpha fetoprotein is a marker for hepatocellular carcinoma, Cemloplasmin levels are altered in Wilson disease. in which citthosis and brain damage occur secondary to abnormalities in the metabolism of copper Serum iron studies are usefil when considering hemochromatosis as a cause of cirthosis Mark thie question <& => Question Td : 139041 Question 23 of 30 A 42 year eld woman with poleythemia Vera develops progressive severe ascites and tender hepatomegaly over a periad of several months, LFTs are near normal. The likely diagnosis would be established by a) Endoscopic retrograde cholangiopancreatography (EPCP) YW © b) Hepatic venography ©) Serum alpha fetoprotein 4) Serum ceruloplasmin ) Senam iton stadies Question Explanation: The clnical presentation is most consistent with Budd Chiari syndrome (hepatic vein obstruction, usually de to thrombosis) which may occur as a complication of thrombogenic and mycloproligcrative disorders, including polycythemia vera. The presentation illustrated is the most common; alternative presentations include fukninant iver failure and cases in which intractable abdominal pain is the most prominent intial finding The best method listed to establish the diagnosis of Budd Chiari syncrome is hepatic venography to demonstrate the occlusion of the hepatic vencus system. Liver biopsy to provide evidence of centrilobular congestion and sinusoidal dilation (in the absence of right-sided heart failure) is definitive but moze invasive. Endoscopic retrograde cholangiopancreatography is most usefil in demonstrating lesions of the biliary tree, Serum alpha fetoprotein is a marker for hepatocellular carcinoma, Cemloplasmin levels are altered in Wilson disease. in which citthosis and brain damage occur secondary to abnormalities in the metabolism of copper Serum iron studies are usefil when considering hemochromatosis as a cause of cirthosis Mark this question €_c=> Question 24 of 30 A 59-year-old woman has advanced colorectal cancer. Despite aggressive surgery and adjuvant chemotherapy. the disease hes spread throughout her pelvis and eroded into her left acetabulum. Her pain control plan involves transitionng her to a long-acting, sustained-release forma of fentanyl, Which of the common clinically significant side effect she is likely to experience? a) Addiction ») Nausea and vorniting ¢) Constipation &) Respiratory depression €) Sedation Answer (Explanation Other User's Explanation Report An Error Question Explanation: “Although all of the isted answers are known side effects of narcotics, the most common side effect is constipation. It occurs in nearly all patients on opioids and is dose-dependent, meening that constipation is worse in patients requising the most treatment, As this patent's level of nercotic is increased fiom oxycodone to the more powerful fentanyl, she is at rsk for worsening constipation, which needs to be treated aggressively and proactively. Addiction in patients who have legitimate pain concems is extremely rare, Although physical tolerance and dependence (defined by withdrawal symptoms ifthe drug is abruptiy stopped) are common, psychological and behavioral dependence are rare, occurringin less than 0.1% of patients ‘Nausea and vomiting are not as common as constipation but are sill fequent. Up to one third of patients started on opioids’ suffer nausea, which usually resolves in the first 72 hours, Ifthe problem persists, itis sometimes resolved with switching the class of opioid or with the administration of common attiemetics, such as prochlorperazine. Respiratory depression is most common in patients who have underlying lung disease o: those already predisposed to respiratory problems, such as patients who have large, malignant pleural effasions. Eventually this may become problem for this paiient, The most common side effect, however, is constipation Sedation like nausea is usually present when a patient first starts an opicid, Tolerance to the sedative effects of opioids develops quickly. This patient who has been taking opioids before starting fentanyl, is unlikely to suffer hew sedation unless her dose of medication is dramatically increased. Mark this question €_c=> Question 24 of 30 A 59-year-old woman has advanced colorectal cancer. Despite aggressive surgery and adjuvant chemotherapy. the disease hes spread throughout her pelvis and eroded into her left acetabulum. Her pain control plan involves transitionng her to a long-acting, sustained-release forma of fentanyl, Which of the common clinically significant side effect she is likely to experience? a) Addiction ») Nausea and vorniting Y¥ © ¢) Constipation &) Respiratory depression €) Sedation Answer (Explanation Other User's Explanation Report An Error Question Explanation: “Although all of the isted answers are known side effects of narcotics, the most common side effect is constipation. It occurs in nearly all patients on opioids and is dose-dependent, meening that constipation is worse in patients requising the most treatment, As this patent's level of nercotic is increased fiom oxycodone to the more powerful fentanyl, she is at rsk for worsening constipation, which needs to be treated aggressively and proactively. Addiction in patients who have legitimate pain concems is extremely rare, Although physical tolerance and dependence (defined by withdrawal symptoms ifthe drug is abruptiy stopped) are common, psychological and behavioral dependence are rare, occurringin less than 0.1% of patients ‘Nausea and vomiting are not as common as constipation but are sill fequent. Up to one third of patients started on opioids’ suffer nausea, which usually resolves in the first 72 hours, Ifthe problem persists, itis sometimes resolved with switching the class of opioid or with the administration of common attiemetics, such as prochlorperazine. Respiratory depression is most common in patients who have underlying lung disease o: those already predisposed to respiratory problems, such as patients who have large, malignant pleural effasions. Eventually this may become problem for this paiient, The most common side effect, however, is constipation Sedation like nausea is usually present when a patient first starts an opicid, Tolerance to the sedative effects of opioids develops quickly. This patient who has been taking opioids before starting fentanyl, is unlikely to suffer hew sedation unless her dose of medication is dramatically increased. Mark this question xz Question Id : 197239 Question 25 of 30 A44 year old woman is diagnosed with a duodenal ulcer. The most sensitive testfor detecting current infection with Helicobacter pylotiis which one of the following? 2) A gasiric findal biopsy b) Cuture of a gasttic biopsy ) The (13C) urea breath test 4) The presence of Helicobacter pylori serum ansibodies €) The Urease test on gastric biopsy Question Explanation: ‘The gold standard for diagnosis of H. puloni remains culture of a gastric biopsy. Yet this testis only 72% sensitive. However, the rapid Urease test on a biopsy is 80-95% sensitive and 95-100% specific. Histology is 80-90% sensitive and 95% specific. However, the urease breath test is approximately 95% sensitive and 98-10% specific. Therefore the most specific and clinically epplicable would be the urease breath test. The presence of [eG antibodies to H. pylori could indicate previous infection. A gastric anual biopsy can give false negative results following PPT treatment, n Report An Error Mark this question xz Question Id : 197239 Question 25 of 30 A.44 year old womanis diagnosed with a duodenal ulcer. The most sensitive test for detecting current infection with Helicobacter pylori is which one of the following? 2) A gastric findal biopsy bb) Cuiture of a gastic biopsy S © c) The (13C) urea breath test 4) The presence of Helicobacter pylori serum antibodies €) The Urease test on gastric biopsy Question Explanation: ‘The gold standard for diagnosis of H. puloni remains culture of a gastric biopsy. Yet this testis only 72% sensitive. However, the rapid Urease test on a biopsy is 80-95% sensitive and 95-100% specific. Histology is 80-90% sensitive and 95% specific. However, the urease breath test is approximately 95% sensitive and 98-10% specific. Therefore the most specific and clinically epplicable would be the urease breath test. The presence of [eG antibodies to H. pylori could indicate previous infection. A gastric anual biopsy can give false negative results following PPT treatment, n Report An Error ‘Mark this question ¢=> Question 26 of 30 Question Td : 199277 Amale aged 68 years has alcoholic cithosis complicated by mild ascites. Which feature is lcely to be present inthis pationt? ) Increased serum sodium ) Increased vascular resistance «) Reduced renin concentrations 4) Reduced winary potassium excretion 2) Reduced urinary sodium excretion Anower (UBINRIIEY) oer veers Explanation Report An Enor Question Explanation: Remember that he har secondary hyperaldosteronism which results in Sodium retertion with consequent potassium loss. There is decreased vascular resistance, increased plasma volume and serum sodium, ‘Mark this question ¢=> Question 26 of 30 Question Td : 199277 Amale aged 68 years has alcoholic cithosis complicated by mild ascites. Which feature is lcely to be present inthis pationt? ) Increased serum sodium ) Increased vascular resistance «) Reduced renin concentrations 4) Reduced winary potassium excretion JY © ¢) Reduced urinary sodium excretion Anower (UBINREIEN oer veers Explanation Report An Eno Question Explanation: Remember that he har secondary hyperaldosteronism which results in Sodium retertion with consequent potassium loss. There is decreased vascular resistance, increased plasma volume and serum sodium, Merk this question & => Question Ta : 199435 Question 27 of 30 FALSE statement regarding a patient with ascites due to liver cirrhosis is which one of the fellowing? 2) Spontancous bacterial peritonitis is a recognized feature ') The usual source of the ascitic uid is mainly from the exudation from the sucface ofthe liver ¢) Hepatic intrasinusoidal pressure is clevated 6) Urinary sodium concentration is usually less than 10mmo\/. ¢) Cardiac output is often elevated Anewor (UEQNEWAR) Otnor UeorsExplanation Report An Error Question Explanatiot Hepatocellular falure is associated: with hyperdynamic cisculaton a systemic vasodiatation, with increased vascular capacitance ‘Most patients have sodium and water retention. Merk this question & => Question Ta : 199435 Question 27 of 30 FALSE statement regarding a patient with ascites due to liver cirrhosis is which one of the fellowing? 2) Spontancous bacterial peritonitis is a recognized feature Y © ¥) The usual source of the ascitic fhid is mainly ftom the exudation from the surface of the liver ¢) Hepatic intrasinusoidal pressure is clevated 6) Urinary sodium concentration is usually less than 10mmo\/. ¢) Cardiac output is often elevated Anewor (UEQNEWAR) Otnor UeorsExplanation Report An Error Question Explanatiot Hepatocellular falure is associated: with hyperdynamic cisculaton a systemic vasodiatation, with increased vascular capacitance ‘Most patients have sodium and water retention. Mark this question ez Question Id 159643 Question 28 of 30 A.76 year old man is passing large volumes of watery diarrhea, 3 liters a day, with no blood. It has been present for 5 montis and is gradually becoming more frequent. It often wakes him at night with the urge to defecate. Investigations are normal. Stool microscopy and culture are normal, and Clostridium ciffcile toxin is negative. Sigmoidoscopy shows normal large bowel He may benefit from which treatment? a) Gluten free diet 1b) High fare diet ©) Low residue ciet ©} Oral Cholestyrarine ©) Oral prednisolone toons EEE 08- se Question Explanation: Tn the absence of infection and with this typical history in an elderly individual the diagnosis is licely to be microscopic colts. “Although coeliac disease is a possibility, this is unlikely, given the patent's age, and the presentation, Microscopic colitis can only be Question Id : 19541 Question 1 of 30 A 25-year-old woman pregnant lady with a long history of ulcerative colitis takes mesalazine, She also smokes 15 cigarettes daily. She now presents with a deterioration of symptoms with six bloody stools per day. Which of the following statements is correct? a) Initiating an elemental diet predisposes to fetal malnutrition b) Mesalazine therapy should be withdrawn ©) Azathioprine would be contraindicated ©) Steroid therapy is contraindicated 8) Termination of the pregnancy is advised Question Explanation: Azathioprine should not generally be started in pregnancy, Wel controlled vicerative colits is more important for the baby from a nutritional point of view. _An elemental diet simply contains pre-digested food and would nat lead to fetal malnutrition The safety of the 5-arrinosalicylic acid (5-ASA) drugs in pregnancy is best supported by the data on Salazopyrine which have been available for the longest. jon Report An Error ‘Mark this question => Question Id : 19541 Question 1 of 30 A 25-year-old woman pregnant lady with along history of ulcerative colitis takes messlazine. She also smokes 15 cigarettes daily. ‘She now presents with a deterioration of symptoms with six bloody stools per day. Which of the following statements is correct? a) Initiating an elemental diet predisposes to fetal malnutrition +b) Mesalazine therapy should be withdrawn ¥ © ©) Azathioprine would be contraindicated oD) Steroid therapy is contrandicated ¢) Termination of the pregnancy is advised Question Explanation: Azathioprine should not generally be started in pregnancy, Wel controlled vicerative colits is more important for the baby from a nutritional point of view. _An elemental diet simply contains pre-digested food and would nat lead to fetal malnutrition The safety of the 5-arrinosalicylic acid (5-ASA) drugs in pregnancy is best supported by the data on Salazopyrine which have been available for the longest. jon Report An Error Mark this question —& => Question Td : 50579 Question 2 of 30 A.45 year old woman with jaundice presents with nausea and vomiting, enorezia and depression, She is on rifampin, INH and pyridoxine. Physical exam shows hepatomegaly and abdominal tenderness. Labs reveal her SGOT (AST) and SGPT (ALT) are greatly increased, alkaline phosphatase is slightly increased, The most ikely diagnosis is a) Drug indaced hepatitis ) Viral hepatitis ©) Acute pancreatitis 4) Liver cancer Question Explanation: INE stands for isoriazid, But INH could also stand for Injures Neurons and Hepatocytes’ Because of the risk of hepatitis, patients taking INH should have their blood liver tests monitored monthly and should notiy their physicians immediately if symptoms or signs of hepatitis arise These symptoms and signs include unexplained loss of appetite, nausea, vomiting, dark urine, yellow skin or a yellowish tinge to the whites of the eyes, persistent fatigue, weakness or fever of greater than 3 days duration, or abdominal tendemess or discomfort, especialy in the nght upper part of the abdomen. Damage to nerves (peripheral neuropathy) may occur with INE and cause numbness and tingling of the hans or feet. Other rare side effects of the nervous system inchide encephalopathy (nflammation of the brain), optic neuritis (in lammation of the nerve coming from the eye), airophy (degeneration) of the nerve coming fom the eye, seizures, impaired memory, psychosis. Pyridoxine (vitamin B6), taken daily, decreases the risk of neural side effects Mark this question —& => Question Td : 50579 Question 2 of 30 A.45 year old woman with jaundice presents with nausea and vomiting, enorezia and depression, She is on rifampin, INH and pyridoxine. Physical exam shows hepatomegaly and abdominal tenderness. Labs reveal her SGOT (AST) and SGPT (ALT) are greatly increased, alkaline phosphatase is slightly increased, The most ikely diagnosis is Y © a) Drug indaced hepatitis ) Viral hepatitis ©) Acute pancreatitis 4) Liver cancer Question Explanation: INE stands for isoriazid, But INH could also stand for Injures Neurons and Hepatocytes’ Because of the risk of hepatitis, patients taking INH should have their blood liver tests monitored monthly and should notiy their physicians immediately if symptoms or signs of hepatitis arise These symptoms and signs include unexplained loss of appetite, nausea, vomiting, dark urine, yellow skin or a yellowish tinge to the whites of the eyes, persistent fatigue, weakness or fever of greater than 3 days duration, or abdominal tendemess or discomfort, especialy in the nght upper part of the abdomen. Damage to nerves (peripheral neuropathy) may occur with INE and cause numbness and tingling of the hans or feet. Other rare side effects of the nervous system inchide encephalopathy (nflammation of the brain), optic neuritis (in lammation of the nerve coming from the eye), airophy (degeneration) of the nerve coming fom the eye, seizures, impaired memory, psychosis. Pyridoxine (vitamin B6), taken daily, decreases the risk of neural side effects ‘Marke this question & => Question Td : 54741 Question 3 of 30 66 year old obese man complains of strong and sudden mid abdominal pain radiating to bis let flank. Physical exam does not show any findings. The most likely diagnosis is a) Acute pancreatitis 4) Cholecystitis ©) Ureteral stone ©) Mesenteric ischemia 6) Rupture of abdominal aorta Question Explanation: Acute mesenteric ischemia is interruption of intestinal blood flow by embolism, thrombosis, or a low-flow state. It leads to mediator release, inflammation, and ubimately infarction, Abdominal pain is out of proportion to physical findings The eatly hallmar of mesenteric ischemia is severe pain but minimal physical findings. The abdomen remains soft, with litle or no tendemess, Mild tachycardia may be present. Early diagnosis is dificult, but angiography and exploratory laparotomy have the most sensitivity, other imaging modalties often become positive only late in the disease. Treatment is by embolectomy, revascularization of viable segments, or resection, sometimes vasodilator therapy is successful. Mortality is high, Report An Error ‘Marke this question & => Question Td : 54741 Question 3 of 30 -A.66 year old abese man complains af strong end sudden mid abdominal pain radiating to his left flank. Physical exam does not show aay findings. The mest likely diagnosis is a) Acute pancreatitis £) Cholecystitis ©) Ureteral stone V © & Mesenteric ischemia 6) Rupture of abdominal aorta Question Explanation: Acute mesenteric ischemia is interruption of intestinal blood flow by embolism, thrombosis, or a low-flow state. It leads to mediator release, inflammation, and ubimately infarction, Abdominal pain is out of proportion to physical findings The eatly hallmar of mesenteric ischemia is severe pain but minimal physical findings. The abdomen remains soft, with litle or no tendemess, Mild tachycardia may be present. Early diagnosis is dificult, but angiography and exploratory laparotomy have the most sensitivity, other imaging modalties often become positive only late in the disease. Treatment is by embolectomy, revascularization of viable segments, or resection, sometimes vasodilator therapy is successful. Mortality is high, Report An Error ‘Marke this question e& => Question Td : 55492 Question 4 of 30 ‘When considering therapy for the obese patients, one should know that the major component of caloric expendinare is a) Dietary thermogenesis (the energy required for metabolism of food) ) The basal metabolic rate (the amount of energy required to maintain metabolic homeostasis) ©) Physical activity 6) The negative feedbacle response of growth hormone and Cortisol to the ingestion of food Anewor [UEXVIRISER) otner usar Exptan Question Explanation: “When caloric intake is greater than energy expenditure, obesity results, The major components of caloric expenciture are the basal metabolic rate, distary thermogenesis, and physicel activity. The basal metabolic rate (the amount of energy required to maintain metabolic, homeostasis) accounts for 60%-70% of caloric expenciture. Dietary thermogenesis (the energy required for digestion and metabolism of food) accounts for 54-10% of caloric expenditure and depends upon the amount and type of food eaten and, to an extent, on the degree of obesity. Physical activity accounts for about 25%-35% of caloric expenditare in the average person Report An Error ‘Marke this question e& => Question Td : 55492 Question 4 of 30 ‘When considering therapy for the obese patients, one should know that the major component of caloric expendinare is a) Dietary thermogenesis (the energy required for metabolism of food) Y © b) The basal metabolic rate (the amount of energy required to maintain metabolic homeostasis) ©) Physical activity 6) The negative feedbacle response of growth hormone and Cortisol to the ingestion of food Anewor [UEXVIRISER) otner usar Exptan Question Explanation: “When caloric intake is greater than energy expenditure, obesity results, The major components of caloric expenciture are the basal metabolic rate, distary thermogenesis, and physicel activity. The basal metabolic rate (the amount of energy required to maintain metabolic, homeostasis) accounts for 60%-70% of caloric expenciture. Dietary thermogenesis (the energy required for digestion and metabolism of food) accounts for 54-10% of caloric expenditure and depends upon the amount and type of food eaten and, to an extent, on the degree of obesity. Physical activity accounts for about 25%-35% of caloric expenditare in the average person Report An Error Mark this question = => Question Td : 61698 Question 5 of 30 A. 62 year old man is admitted to she hospital because of acute pancreatitis Laboratory studies show: Amslase=1,000 U/L Hematocrit42% Caleiun=8.4 mg/dl WBC=14000/nm? BUNS mgidl Results of serum liver chemistry profile are normal, After 4 hours of uid therapy and observation, which of the following lab study ‘would indicate poor prognosis? a) Serum alanine aminatransferase (ALT) of 106 U/L ) Serum amylase of 2,000 U/L ©) Serum bilibin of 4 2 mala. ) Serum glucose of 200 mg/dL, 8) Serum calcium of 6.6 maslL. Question Explanatios Of the laboratory abnormalities (elevated ALT amylase, bilirubin, and glucose) the one that is most suggestive of severe pancreatic irflammation, and even necrosis, is hypocalcemia. This is presumably due to the results of sapondiication of calcium by released fatty acids. Mark this question = => Question Td : 61698 Question 5 of 30 A. 62 year old man is admitted to she hospital because of acute pancreatitis Laboratory studies show: Amslase=1,000 U/L Hematocrit42% Caleiun=8.4 mg/dl WBC=14000/nm? BUNS mgidl Results of serum liver chemistry profile are normal, After 4 hours of uid therapy and observation, which of the following lab study would indicate poor prognosis? a) Serum alanine arrinotransferase (ALT) of 106 U/l. b) Serum amylase of 2,000 U/L ) Serum bilirubin of 4.2 maid. d) Serum glucose of 200 mg/dL V © &) Serum calcium of 6.6 mafdL, Question Explanatios Of the laboratory abnormalities (elevated ALT amylase, bilirubin, and glucose) the one that is most suggestive of severe pancreatic irflammation, and even necrosis, is hypocalcemia. This is presumably due to the results of sapondiication of calcium by released fatty acids. ‘Mark this question & => Question Id : 61758 Question 6 of 30 ‘True statement about celiac diseases (gluten sensitive enteropathy) in adults is 4) Ikie mors common among blacks +) Symptoms are limited to gastrointestinal complaints ) Type 2 diabetics aro at increased sisk for the disease 4) Serum antibody tests are sensitive and specific ) Colonoscopy with mucosal biopsy is required to make the diagnosis newer [UEIRERIBNY Other Users Explanation Report An Exes Question Explanation: Celiac disease is thought to be greatly underdiagnosed in Canada. Antibody tests indicate that the prevalence is approximately 1:250 among adult Canadians of European ancestry. Approximaicly 7% oftype 1 diabeies have celiac disease. A number of other autoimmune syndromes have been associated with celiac disease, including thyroid disease and rheumatoid arthritis. There is no reported association with type 2 diabetes. Gestointestinal involvement may manifest as dierthea, constipation, or other symptoms of malabsorption, such as bloating, flats, or belching Fatigue, depression, fbromyalgia like symptoms, aphthous stomatitis, bone pain, cyspepsia, gastrocsophegeal refux, and other nonspecific symptoms may be present and can make the diagnosis quite challenging ‘Detmnatitis herpetiformis is seen in 10% of patients with celiac disease. Serum antibody testing, especialy Ie antiendomysial antibody, is highly sensitive and specific and readily available at a cost of about $100 to 200. Definitive diagnosis generally requires esophagogastroduodenoscopy with a biopey of the distal duodenum to detect characteristic villous fattening, ‘Mark this question & => Question Id : 61758 Question 6 of 30 ‘True statement about celiac diseases (gluten sensitive eréeropathy) in adults is a) Ikis more common among blacks +b) Symptoms are limited te gastrointestinal complaints ©) Type 2 diabetics are at increased tisk for the disease Y © 4) Serum antibody tests are sensitive and specific 2) Colonoscopy with mucosal biopsy is required to malze the diagnosis newer [UEIRERIBNY Other Users Explanation Report An Exes Question Explanation: Celiac disease is thought to be greatly underdiagnosed in Canada. Antibody tests indicate that the prevalence is approximately 1:250 among adult Canadians of European ancestry. Approximaicly 7% oftype 1 diabeies have celiac disease. A number of other autoimmune syndromes have been associated with celiac disease, including thyroid disease and rheumatoid arthritis. There is no reported association with type 2 diabetes. Gestointestinal involvement may manifest as dierthea, constipation, or other symptoms of malabsorption, such as bloating, flats, or belching Fatigue, depression, fbromyalgia like symptoms, aphthous stomatitis, bone pain, cyspepsia, gastrocsophegeal refux, and other nonspecific symptoms may be present and can make the diagnosis quite challenging ‘Detmnatitis herpetiformis is seen in 10% of patients with celiac disease. Serum antibody testing, especialy Ie antiendomysial antibody, is highly sensitive and specific and readily available at a cost of about $100 to 200. Definitive diagnosis generally requires esophagogastroduodenoscopy with a biopey of the distal duodenum to detect characteristic villous fattening, ‘Marke this question & => Question Ti : 62058 Question 7 of 30 A44 year old man presents with dark skin anda palpable liver His father died of citthosis. The most likely diagmosis is a) Wilson diseases, 6) Hemochromatosis ©) Diabetes mellitus 4) Hepatic cirrhosis Question Explanatio: “Hemochromatosis in an inherited disorder characterized by excessive iron accumulation cansing tissue damage, Symptoms do nat develop unti orgen damage often irreversible, develops Symptoms include fetigne, hepetomegaly, and bronze skin pigmentation, loss oflibido, arthalgies, and manifestations of cirshosis, diabetes, or cardiomyopathy. 90% of patients will present with excessive skin pigmentation Diagnostic testing will reveal that the serum iron is increased ( 300 mg/L, Phlebotomy is the simplest method of excess iron removal in most cases. The other choices would not typically present with dark skin pigmentation, ‘Marke this question & => Question Ti : 62058 Question 7 of 30 A.A4 year old man preserts with dari skin and a palpable liver. His father died of cirrhosis. Tae mostlikely diagnosis is a) Wilson diseases WV © b) Hemochromatosis ©) Diabetes mellitus 4) Hepatic cirrhosis Question Explanatio: “Hemochromatosis in an inherited disorder characterized by excessive iron accumulation cansing tissue damage, Symptoms do nat develop unti orgen damage often irreversible, develops Symptoms include fetigne, hepetomegaly, and bronze skin pigmentation, loss oflibido, arthalgies, and manifestations of cirshosis, diabetes, or cardiomyopathy. 90% of patients will present with excessive skin pigmentation Diagnostic testing will reveal that the serum iron is increased ( 300 mg/L, Phlebotomy is the simplest method of excess iron removal in most cases. The other choices would not typically present with dark skin pigmentation, ‘Marke this question & => Question Ta : 62760 Question 8 of 30 Prolonged vorniting is usuelly associated with what electrolyte abnormality? a) Hypochloremic hypokalemic metabolic acidosis, ) Hypochloremic hypokalemic metabolic alkalosis, ©) Hyperchloremic metabolic acidosis ) Fyperkalemia Question Explanation: During prolonged vomiting, the patient will vomit out protors (H), potassinm ions (K) and chloride ions (Ci). This will eventually lead to low potassium level, low chloride level and a shift toward alkalosis (since protons contribute to acid stats) ‘Marke this question & => Question Ta : 62760 Question 8 of 30 Prolonged vomiting is usually associated with what electrolyte abnomnalty? a) Hypochloremic hypokalemic metabolic acidosis ¥ © b) Fypochloremic hypolalemic metabolic alkalosis c) Hyperchloremic metabolic acidosis ) Hyperkalemia Question Explanation: During prolonged vomiting, the patient will vomit out protors (H), potassinm ions (K) and chloride ions (Ci). This will eventually lead to low potassium level, low chloride level and a shift toward alkalosis (since protons contribute to acid stats) Mark this question & => Question Td : 63778 Question 9 of 30 ‘Which of the followingis not a feanure of necrotizing enterocolitis? a) Pheumatosis intestinalis b) Peritoneal calcification ©) Portal venous gas ¢) Pneumoperitonium Anewor [UEIRNSHEN) Other UsorsExplanation Report An Error Question Explanation Necrotizing enterocolitis (NEC) is an acquired disease, primanly of preterm or sick neonates, characterized by mucosal or even deeper intectinal necrosic. Sigs and eymptorns include feeding intolerance, lethargy, temperature instability, ilens, bloating, bilious emesis, hermatockezia, reducing substances in the stool, apnea, and sometimes signs of sepsis. Diagnosis is clinical and is confirmed by imaging studies. Screening the stools of enterally fed premature infants for occult blood or reducing substances may help diagnose NEC early. Barly X-rays may be nonsperific and reveal only ileus. However, a fixed, dilated intestinal loop that does not change on repeated x-rays indicates NEC. X-ray signs diagnostic of NEC are pnenmetosis intestinalis and portal vein gas. Pneumoperitonewmn indicates bowel perforation and en urgent need fer surgery. Treatment is primerily supportive end inshudes nasogastric suction, parenteral fics, TPN, antibiotics, isolation in cases of infection, and, often, surgery. Mark this question & => Question Td : 63778 Question 9 of 30 “Which ofthe following is not a feanure of necrotizing enterocolitis? a) Pheumatosis intestinalis © ») Peritoneal calcification ©) Fortal venous gas ¢) Pneumoperitonium Anewor [UEIRNSHEN) Other UsorsExplanation Report An Error Question Explanation Necrotizing enterocolitis (NEC) is an acquired disease, primanly of preterm or sick neonates, characterized by mucosal or even deeper intectinal necrosic. Sigs and eymptorns include feeding intolerance, lethargy, temperature instability, ilens, bloating, bilious emesis, hermatockezia, reducing substances in the stool, apnea, and sometimes signs of sepsis. Diagnosis is clinical and is confirmed by imaging studies. Screening the stools of enterally fed premature infants for occult blood or reducing substances may help diagnose NEC early. Barly X-rays may be nonsperific and reveal only ileus. However, a fixed, dilated intestinal loop that does not change on repeated x-rays indicates NEC. X-ray signs diagnostic of NEC are pnenmetosis intestinalis and portal vein gas. Pneumoperitonewmn indicates bowel perforation and en urgent need fer surgery. Treatment is primerily supportive end inshudes nasogastric suction, parenteral fics, TPN, antibiotics, isolation in cases of infection, and, often, surgery. ‘Marle this question & => Question Td : 81381 Question 10 of 30 A 65 year old hypertensive fernale has chroaic constipation, She presents with metabolic acidesis and hypokalenia. The most ely cause of her present condition is a) Diuretic ingestion +) Magnesium depletion ©) Vomiting 6) Treatment with aminoglycosides ©) Laxative abuse newer [FEIRIRIRNBN) ote: User's expan Question Explanation: Laxative abuse can canse metabolic acidosis and hypokalemia, Diuretic ingestion (A) would produce a metabolic allealosis in association with hypokalemia, as would vomiting (C). Magnesium depletion (B) would resui in hypokalemia in the presence of a normal acid-base status, as would treatment with aminoglycosides (D). Laxative abuse causes constipation jon Report An Error ‘Marle this question & => Question Td : 81381 Question 10 of 30 A 65 year old hypertensive fernale has chroaic constipation, She presents with metabolic acidesis and hypokalenia. The most ely cause of her present condition is a) Diuretic ingestion +) Magnesium depletion ©) Vomiting 6) Treatment with aminoglycosides oY © ©) Laxative abuse newer [FEIRIRIRNBN) ote: User's expan Question Explanation: Laxative abuse can canse metabolic acidosis and hypokalemia, Diuretic ingestion (A) would produce a metabolic allealosis in association with hypokalemia, as would vomiting (C). Magnesium depletion (B) would resui in hypokalemia in the presence of a normal acid-base status, as would treatment with aminoglycosides (D). Laxative abuse causes constipation jon Report An Error Mark this question & => Question Td : 85809 Question 11 of 30 A.48 year old male presents to the emergency room with sudden onset of severe upper abdominal pan with vomiting. The pain is lccated in the epigastrium with radiation ta the back. History af chronic epigastric pain is net presert. The mast coremonly cacoustered predisposing factors for this patient's condltion is a) Alcohol use and gallstones ') Helicobacter pylori infection and excess gastric acid secretion, c) Hepatitis B infection and iron overload ) Hypertension and cystic medical necrosis ©) Obesity and high serum cholesterol £ Pacumonia end trauma a) Stress and cigarette use Answer [Bonet Other User's Explanation Report An Error Question Explanation: ‘The clinical scenario is typical of acute pancreatitis. The overwhelmingly most imp ortant contributing factors for developmert of acute pancreatitis are gallstones (particularly small ones) and alcohol abuse. The diagnosis can be confirmed by demonstrating markedly elevated serum amylase aad lipase. Helccbacter pylori infection and excess gastric acid seoretion are predisposing factors for peptic ulcer disease of the stomach and duodenun, Ulcers usually produce chronic epigastric pain, Hepatitis B infection and iron overload predispose for citrhosis. Hypertension and cystic medial necross are predsposng factors for aortic dissection, which can cause sharp thoracic and upper abdominal pain. The pain charactetistically" moves" over a period of hours, Predisposing factors for muyocerdial infurction include obesity, high serura cholesterol rece, aad cigarette cmolting. The pain ic charactorstically thoracio with referral to the left shoulder. Pneumonia and trauma are predisposirg factors for pleurits which can cause sharp lower thoracic pai. Mark this question & => Question Td : 85809 Question 11 of 30 A.48 year old male presents to the emergency room with sudden onset of severe upper abdominal pan with vomiting. The pain is lccated in the epigastrium with radiation ta the back. History af chronic epigastric pain is net presert. The mast coremonly cacoustered predisposing factors for this pationt's condltion is Y © a) Alcohol use and gallstones ') Helicobacter pylori infection and excess gastric acid secretion, c) Hepatitis B infection and iron overload ) Hypertension and cystic medical necrosis ©) Obesity and high serum cholesterol £ Pacumonia end trauma a) Stress and cigarette use Answer [Bonet Other User's Explanation Report An Error Question Explanation: ‘The clinical scenario is typical of acute pancreatitis. The overwhelmingly most imp ortant contributing factors for developmert of acute pancreatitis are gallstones (particularly small ones) and alcohol abuse. The diagnosis can be confirmed by demonstrating markedly elevated serum amylase aad lipase. Helccbacter pylori infection and excess gastric acid seoretion are predisposing factors for peptic ulcer disease of the stomach and duodenun, Ulcers usually produce chronic epigastric pain, Hepatitis B infection and iron overload predispose for citrhosis. Hypertension and cystic medial necross are predsposng factors for aortic dissection, which can cause sharp thoracic and upper abdominal pain. The pain charactetistically" moves" over a period of hours, Predisposing factors for muyocerdial infurction include obesity, high serura cholesterol rece, aad cigarette cmolting. The pain ic charactorstically thoracio with referral to the left shoulder. Pneumonia and trauma are predisposirg factors for pleurits which can cause sharp lower thoracic pai. 3i2/2014 5:21:21 PM J % vow interface scu.ck/i x \ BM évigeSIVl sioll aie € CD www. interface edu.pk/medical-exams;test-analysis php 2utid=14759. Apps [Google ° Settings [Signin Zo. bbe dul Eo cedll dine Ol ..co yc fosoll JS [) Free Hotmail leleT #8 ther bookmar ‘Marie this question = => (Question Ti : 85829 Question 12 of 30 A.69 year old man is found to have occult blood in the stool during a periocic health maintenance examination Colonoscopy seveals a mass in the sigmoid colon, He undergoes surgery for resection of the mass which is shown in the photoaraph, The most siandicant tisk factor associated with the incidence of this disease is Q reer sical Eason a) Cigarette smoking ) Excessive alcohol intake ©) Excessive consumption of vitamins A, C, and E 4d) Helicobacter pylori infection ¢) Reduced aber content in the dist newer [ERAN omer sors Explanation Report An Error Question Explanation: ‘A diet with low Biber content has been shown to be portively correlated with increased incislence of colen cancer. The mechanism of this pathogenic actioa is sill unclear, although itis hyp othesized that prolonged transit of fecal material may expose the intestinal epithelium to carcinogenic compounds. Diets with a high content of refined foods and meat (typical ofindnstriaized nations) are associated with an increased risk of developing colon cancer. Cigarette smoking is strongly associated with lung cancer. However, a weaker associetion with colon cancer has been observed as well, Smoking also predisposes to cancer of the kidneys, urinary bladder, and uterine cervix, Excessive alcohol intake is associated vwith increased risk of esophageal squamous cell carcinoma and gastric adenocarvinoma, but not colon cancer Excessive consumption of vitamins A, C, and E is not a rske factor for colon cancer. On the contrary, administration of these vitamine may protectizom colon carcinogenesis, Clinical smdies are underway 10 define the role of these antioxidants ia preventing cancer of the colon, Helicobacter pylori infection has been shown to increase the risk of gastric adenocarcinoma, gestrc lymphoma, and chronic eee Total Questions ho ko FI ke BREBBREBERBEBBRHB GE & xxXxXXXKXKXKXKKKXKXKXKKXKKXKKKKKKKKKKKKK 3i2/2014 5:21:21 PM J % vow interface scu.ck/i x \ BM évigeSIVl sioll aie € CD www. interface edu.pk/medical-exams;test-analysis php 2utid=14759. Apps [Google ° Settings [Signin Zo. bbe dul Eo cedll dine Ol ..co yc fosoll JS [) Free Hotmail leleT #8 ther bookmar ‘Marie this question = => (Question Ti : 85829 Question 12 of 30 A.69 year old man is found to have occult blood in the stool during a periocic health maintenance examination Colonoscopy seveals a mass in the sigmoid colon, He undergoes surgery for resection of the mass which is shown in the photoaraph, The most siandicant tisk factor associated with the incidence of this disease is Q reer sical Eason a) Cigarette smoking 'b) Excessive alcohol insake ©) Excessive consumption of vitamins A, C, and E 4) Helicobacter pylori infection JY © ©) Reduced Gber content in the dist newer [ERAN omer sors Explanation Report An Error Question Explanation: ‘A diet with low Biber content has been shown to be portively correlated with increased incislence of colen cancer. The mechanism of this pathogenic actioa is sill unclear, although itis hyp othesized that prolonged transit of fecal material may expose the intestinal epithelium to carcinogenic compounds. Diets with a high content of refined foods and meat (typical ofindnstriaized nations) are associated with an increased risk of developing colon cancer. Cigarette smoking is strongly associated with lung cancer. However, a weaker associetion with colon cancer has been observed as well, Smoking also predisposes to cancer of the kidneys, urinary bladder, and uterine cervix, Excessive alcohol intake is associated vwith increased risk of esophageal squamous cell carcinoma and gastric adenocarvinoma, but not colon cancer Excessive consumption of vitamins A, C, and E is not a rske factor for colon cancer. On the contrary, administration of these vitamine may protectizom colon carcinogenesis, Clinical smdies are underway 10 define the role of these antioxidants ia preventing cancer of the colon, Helicobacter pylori infection has been shown to increase the risk of gastric adenocarcinoma, gestrc lymphoma, and chronic eee Total Questions ho ko FI ke BREBBREBERBEBBRHB GE & xxXxXXXKXKXKXKKKXKXKXKKXKKXKKKKKKKKKKKKK 3:2/2014 5:22:28 PM. € wovw.interfaceedu.ck/i says5)Yl aan by CL www. interface.edu.pk/medi ysis. pho ?utd=14759 apps El.coogle A settings [)Sgnin Sou wlohe duly Xam oo wsdl dips Ofte ge sgsoll JS) Free Hotmall * 8 Gi other bookmar ‘Matic this question & => Question Id : 87794 Question 13 of 30 A 24 year old snudent comes to the emergency room complaining of right upper quadrant abdominal pain for several hows. He admits to drinking excessive amounts of alcchol 3 days ago. Over the course of the last few days he recalls that he ingested the contents of an entire bottle of acetaminophen, He is afebrile, with norreal vital signs and sight tandemess in the right upper quadrant. The most likely set of liver finction enzymes inthis patient would be a) Aspartate aminotransferase (AST) = High, Alanine anzinotransferase (ALT) =High, Alkaline phosphatase = High, Asnylase=Normal bb) Aspartate aminotransferase (AST) = High, Alanine aminotransferase (ALT) =High, Alkaline phosphatase = Nommal, Amslase=Normal ©) Aspartate aminotransferase (AST) = High, Alenine aminotransferase (ALT) Question Id : 87794 Question 13 of 30 A 24 year old snudent comes to the emergency room complaining of right upper quadrant abdominal pain for several hows. He admits to drinking excessive amounts of alcchol 3 days ago. Over the course of the last few days he recalls that he ingested the contents of an entire bottle of acetaminophen, He is afebrile, with norreal vital signs and sight tandemess in the right upper quadrant. The most likely set of liver finction enzymes inthis patient would be a) Aspartate aminotransferase (AST) = High, Alanine anzinotransferase (ALT) =High, Alkaline phosphatase = High, Asnylase=Normal Y © b) Aspartate aminotransferase (AST) = High, Alanine aminotransferase (ALT) High, Allcaline phosphatase = Normal, Amslase=Normal ©) Aspartate aminotransferase (AST) = High, Alenine aminotransferase (ALT) Question Td : 88118 Question 14 of 30 A 24 year old man presents to a rheumatologist with complaints of joint pain involving the large joints of the legs. He also complains ofa persistent itchy rash that consists of red vesicles on the extensor surface of his forearms. Further questioning inckcates that exacerbations in the joint pain are frequently accompanied by diarrhea ‘The gastrointestinal disease that is most likely to be implicated as the cause of the patient's joint and dermetologic problems is a) Amebic coltis ) Celiac disease ©) Chronic appendicitis 4) Diverticulosis, €) Pseudomembranous colitis Answer | Explanation Other User's Explanation Report An Error Question Explanation: Several gastrointestinal diseases are associated with rheumatologic complaints. The most frequent of these are the chronic inflammatory bowel diseases, ulcerative colitis, and Crohn disease, which can be associated with sacroilits (related to HILA-B27) or lower limb arthritis. Other gastrointestinal diseases associated with arthropathy include bypass surgery, Whipple disease, Behcet syndrome and celiac disease (seen in this patient), Dermatitis herpetiformis the name given to the pruritic papulovesicular skin lesions often involving the extensor surfaces of the extrerrities, trank, buttocks, scalp, and neck found in association with celiac disease, Amebic colitis is caused by ingestion of infectious cysts (typically from Entamoeba histolytica). Symptoms include abdominal fpain charthea malaise weight loss. Cecal amebiasis can resemble acute appendicitis. Chronic appendicitis may be asymptomatic or cause poorly defined abdominal pain. Diverticulosis is usually a disease of older adults. Itis often asymptomatic unless inflammation supervenes. Pseudemembrancus colitis is a severe form of diasrhea usually seen in the setting of prior antbiotic use. The causative organistn is almost always Clostridium difficile Marke this question & => Question Td : 88118 Question 14 of 30 A.24 yecr eld man presents to a chenmatologist with complaints of joint pain iewolving the large jeints of the legs. He also complains of a persistent itchy rash that consists of red vesicles on the extensor surface of his forearms. Further questioning indicates that exacerbations in the joint pain are frequently accompanied by diathea. The gastrointestinal disease that is most likely to be implicated as the cause of the patient's joint and dermetologic problems is 2) Amebic colitis Y © b) Celiac disease ©) Chronic appendicitis 4) Diverticulosis «) Pseudomembranous colitis Answer | Explanation Other User's Explanation Report An Error Question Explanation: Several gastrointestinal diseases are associated with rheumatologic complaints. The most frequent of these are the chronic inflammatory bowel diseases, ulcerative colitis, and Crohn disease, which can be associated with sacroilits (related to HILA-B27) or lower limb arthritis. Other gastrointestinal diseases associated with arthropathy include bypass surgery, Whipple disease, Behcet syndrome and celiac disease (seen in this patient), Dermatitis herpetiformis the name given to the pruritic papulovesicular skin lesions often involving the extensor surfaces of the extrerrities, trank, buttocks, scalp, and neck found in association with celiac disease, Amebic colitis is caused by ingestion of infectious cysts (typically from Entamoeba histolytica). Symptoms include abdominal fpain charthea malaise weight loss. Cecal amebiasis can resemble acute appendicitis. Chronic appendicitis may be asymptomatic or cause poorly defined abdominal pain. Diverticulosis is usually a disease of older adults. Itis often asymptomatic unless inflammation supervenes. Pseudemembrancus colitis is a severe form of diasrhea usually seen in the setting of prior antbiotic use. The causative organistn is almost always Clostridium difficile ‘Marle this question & => Question Ta : 89799 Question 15 of 30 The gastrointestinal symptoms NOT associated with malabsorption is a) Flats ) Constipation ©) Diffase abdorninal pain ©) Greasy, bulky, malodorous stools ©) Weight loss despite a good appetite Anewor [UEXIRIBHER) otnorUsor'sExptan Question Explanation: Diarrhea occurs # there is either impaired absorption or increased secretion of water and electrolytes. Flatus results from bacterial fermentation of unabsorbed carbohydrate. Greasy bulky malodorous stools may result from increased fat in the stool. Diffuse abdotninal pain resuits from inflammation of infiltration of tissue or from inflamination or distention of the bowel, which may occur in lymphoma, pancreatic insufficiency, or regional enteritis. Weight loss in the presence of a good appetite results from loss of calories due to malabsorption of protein, fat, and cabohydrates, jon Report An Error ‘Marle this question & => Question Ta : 89799 Question 15 of 30 ‘The gastrointestinal syraptoms NOT asccoiated with malabcorption is a) Flatus Y © & Constipation ©) Diffuse abdominal pain ©) Greasy, bulky, malodorous stools 6) Weight loss despite a good appetite Anewor [UEXIRIBHER) otnorUsor'sExptan Question Explanation: Diarrhea occurs # there is either impaired absorption or increased secretion of water and electrolytes. Flatus results from bacterial fermentation of unabsorbed carbohydrate. Greasy bulky malodorous stools may result from increased fat in the stool. Diffuse abdotninal pain resuits from inflammation of infiltration of tissue or from inflamination or distention of the bowel, which may occur in lymphoma, pancreatic insufficiency, or regional enteritis. Weight loss in the presence of a good appetite results from loss of calories due to malabsorption of protein, fat, and cabohydrates, jon Report An Error ‘Marke this question & => Question Ta : 91983 Question 16 of 30 Correct statement regarding the development of appendicitis in pregnancy is a) It occurs rarely. +) It caly occurs curing the third trimester. ©) It should be managed with intravenous antbioties. 4) Surgery should be delayed until afer delivery. 6) Surgery should be done prompty. Answer ( Expionaion | Other User's Explanation Report An Error Question Explanation: Appendicitis in the pregnant female should be treated promptly by surgical resection to avoid complications to both mother and fetus. Appendicitis is as common in pregnant women as itis in nonpregnant women (A). Tt occurs equaly throughout pregnancy (B). Antibiotics are not adequate management of appendicitis (C) ‘Marke this question & => Question Ta : 91983 Question 16 of 30 Correct statement regarding the development of appendicitis in pregnancy is a) It occurs rarely. +) It caly occurs curing the third trimester. ©) It should be managed with intravenous antbioties. 4) Surgery should be delayed until afer delivery. Y © 6) Surgery should be done prompty. Answer ( Expionaion | Other User's Explanation Report An Error Question Explanation: Appendicitis in the pregnant female should be treated promptly by surgical resection to avoid complications to both mother and fetus. Appendicitis is as common in pregnant women as itis in nonpregnant women (A). Tt occurs equaly throughout pregnancy (B). Antibiotics are not adequate management of appendicitis (C) ‘Marke this question <= => Question Td : 100277 Question 17 of 30 A SM year old male has chronic watery diarrhea, weight loss, and hypokalemia Large pancreatic mass is found by computerized axial tomography. Elevated levels of which of the following would be revealed on serum analysis? a) Catecholarrines b) Vasoactive intestinal peptide c) Gastrin, 6) Insulin ©) Glucagon Answer [BEINBREN) othe: User's Explanation Repost An Exor Question Explanati A -ViPoma, usually a large pancreatic tumor that secretes excessive quantities of vasoactive intestinal peptide, is the most likely cause of this syndrome. Elevated serum catecholanines, gastrin, insulin, or glucagon would not preseat this way. ‘Marke this question <= => Question Td : 100277 Question 17 of 30 AS year old male has chronic watery diarvhea, weight loss, and hypokalemia Large pancreatic mass is found by computerized axial tomography. Hlevated levels of which of the following would be revealed on serum analysis? a) Catecholarrines o © b) Vasoactive intestinal peptide. c) Gastrin, 6) Insulin, 6) Glucagon, Answer [BEINBREN) othe: User's Explanation Repost An Exor Question Explanati A -ViPoma, usually a large pancreatic tumor that secretes excessive quantities of vasoactive intestinal peptide, is the most likely cause of this syndrome. Elevated serum catecholanines, gastrin, insulin, or glucagon would not preseat this way. 3i2/2014 5:23:55 PM % wwrw.interface.edu.ck/i: > \ BM) soysSIMl ae lay ie CD www. interface.edu.pk/medical-exams test-analysis php 2utid=14759. EE apps Google Asettngs [Sgnin Zo. whe dulw Cu gouedlliae Gf..ei0 G2 isiall JS [ Free Hotmall #8 ther bookmar ‘Mark this question ez Question Td : 100795 Question 18 of 30 ‘A52 year old alcoholic man is found te have elevated liver enzymes. A liver biopsy show the features revealed in the Photomerograrh. Ifthe patent stops from further crnkng. this condton wil most Heely evolve into wthich of he Flowing? FO Mee) Sip OS, ES ice oe , cS * st DS wat ed ad a) Acute hepattis 'b) Chronis hepatitis c) Complete regression 4) Hyperplastic nodules e) Malignant degeneration Answer (Bolanation | Other User's Explanation Report An Error Question Explanation: The picture shows a classic case of iver steatosis or faty change which results from accurmilation of fat within hepatocytes. Alcchal is one of the most frequent causes of liver steatosis, Atleastin part, the mechanism of alcohol induced steatosis is due to acotaldehyde fermation, which disrupts microtubules and increases the NADH/NAD+ ratio. Fatty change of the maacrovesicular type is entirely reversible # the noxious stimulus (i this case, excessive alcohol intake) is removed. Acute hepatitis results from acute ingults to the liver. Alcoholic hepatitis is characterized by hepatocyte necrosis intracytoplasmic hyaline bodies (Mallory bodes), and neutyophilic infiltration. Usually, alcoholic hepatitis develope after a bout of heavy alcohol consurption Fatty change maybe superimposed on histologic signs of hepatitis but steatosis docs not evolve into hepatitis if alcohol intake is discontinued. Chronic hepatitis may result fora long term heavy alcohol consumption and is characterized by chronic inflammation of portal spaces and progressive fibrosis. Steatosis is usually associated with chronic hepatitis due to alcchel. Hyperplastic nodules represent a morphologic component: of cirrhosis. Foci of hepatooyte hyperplasia develop ac a compencatory mecharism to continous loss of hepatocytes. In this case where only farty change and no Abrosis is seen no regenerative nodules would need to form, The licelihood of malignant degeneration is increased in alcoholic cirrhosis as in any other pathogenetic form of cirthosis However fatty change by iteelfis not a harbinger or precursor of hepatocellular carcinoma. Total Questions ho ko HL I te I RY Bile fo fe be bs be Bbw SBBREREBRBEI xxXxxXxXKXKXXKKXKKXKKXKKXKKXKKKKKKKKKKKKK 3i2/2014 5:23:55 PM % wwrw.interface.edu.ck/i: > \ BM) soysSIMl ae lay ie CD www. interface.edu.pk/medical-exams test-analysis php 2utid=14759. EE apps Google Asettngs [Sgnin Zo. whe dulw Cu gouedlliae Gf..ei0 G2 isiall JS [ Free Hotmall #8 ther bookmar ‘Mark this question ez Question Td : 100795 Question 18 of 30 ‘A52 year old alcoholic man is found te have elevated liver enzymes. A liver biopsy show the features revealed in the Photomerograrh. Ifthe patent stops from further crnkng. this condton wil most Heely evolve into wthich of he Flowing? FO Mee) Sip OS, ES ice oe , cS * st DS wat ed ad a) Acute hepattis 'b) Chronic hepatitis Y © c) Complete regression d) Hyperplastic nodules ¢) Malignant degeneration Answer (Bolanation | Other User's Explanation Report An Error Question Explanation: The picture shows a classic case of iver steatosis or faty change which results from accurmilation of fat within hepatocytes. Alcchal is one of the most frequent causes of liver steatosis, Atleastin part, the mechanism of alcohol induced steatosis is due to acotaldehyde fermation, which disrupts microtubules and increases the NADH/NAD+ ratio. Fatty change of the maacrovesicular type is entirely reversible # the noxious stimulus (i this case, excessive alcohol intake) is removed. Acute hepatitis results from acute ingults to the liver. Alcoholic hepatitis is characterized by hepatocyte necrosis intracytoplasmic hyaline bodies (Mallory bodes), and neutyophilic infiltration. Usually, alcoholic hepatitis develope after a bout of heavy alcohol consurption Fatty change maybe superimposed on histologic signs of hepatitis but steatosis docs not evolve into hepatitis if alcohol intake is discontinued. Chronic hepatitis may result fora long term heavy alcohol consumption and is characterized by chronic inflammation of portal spaces and progressive fibrosis. Steatosis is usually associated with chronic hepatitis due to alcchel. Hyperplastic nodules represent a morphologic component: of cirrhosis. Foci of hepatooyte hyperplasia develop ac a compencatory mecharism to continous loss of hepatocytes. In this case where only farty change and no Abrosis is seen no regenerative nodules would need to form, The licelihood of malignant degeneration is increased in alcoholic cirrhosis as in any other pathogenetic form of cirthosis However fatty change by iteelfis not a harbinger or precursor of hepatocellular carcinoma. Total Questions ho ko HL I te I RY Bile fo fe be bs be Bbw SBBREREBRBEI xxXxxXxXKXKXXKKXKKXKKXKKXKKXKKKKKKKKKKKKK Mark this question & => Question Td ; 101402 Question 19 of 30 A.37 year old male reports a mild intermittent jaundice without other associated symptoms for the past several years. His liver finction test are normal except for a total bilirubin of 1.3 mg/dL. (N 0 3-1.0)) and an indirect or unconjugated bilirubin of 1.0 mg/dl. QV0.2-08), His CBCis normal, His past medical and surgical history is unremarkable. Findings are simlar on repeatlaboratory testing, Which of the following is the most lly cause of these fincings? 8) Hepatitis C b) Wilson's disease ©) Sickle cell anemia ) Drug toxicity ©) Gilbert's syndrome Question Explanation: Gilbert's syndrome is the most common inherited disorder of bilirubin metabolism, In patients with a normal CBC and liver function tests, except for recurrent mildly elevated total and unconjugated hyperbilirubinemia, the most likely diagnosis is A Gilbert's syndrome, Fasting, heavy physical exertion, sicide cell anetnia, and drug toxicity can also cause hyperbiliubinenia, Report An Error Mark this question & => Question Td ; 101402 Question 19 of 30 A.37 year old male seporis a nild intermittent jaundice without other associated symptoms for the past several years. His liver finction test are normal except for a total bilimbin of 1.3 mg/dL (N 0 3-1.) and an indirect o unconjugated bilinahin of 1.0 ragfdl. (QV 0.2-08), His CBC is normal. His past medical and surgical history is unremerkable. Findings are sinilar on repeatlaboratery testing, Which of the following is the most likely cause of these fincings? 8) Hepatitis C ) Wilson's disease 6) Sickle cell anemia 6) Drug toxicity Y © ®) Gilbert's syndrome Question Explanation: Gilbert's syndrome is the most common inherited disorder of bilirubin metabolism, In patients with a normal CBC and liver function tests, except for recurrent mildly elevated total and unconjugated hyperbilirubinemia, the most likely diagnosis is A Gilbert's syndrome, Fasting, heavy physical exertion, sicide cell anetnia, and drug toxicity can also cause hyperbiliubinenia, Report An Error Mark this question <& => Question Td : 101512 Question 20 of 30 A74 year old mele is admitted to hospital with left lower quadrant abdominal pain, nausea, alow grade fever, and left lower quadrant tendemess. He is diagnosed as acute diverticuliis. Which of the following is not an appropriate trestment option? @) Broad spectrum antibiotics £) Immediate laparctomy ©) Intravenous fluids 3) Meperidine @) Nasogasiric suction Question Explanation: ‘Divertcuits is inflammation of a diverticulum, which can result in phlegmon of the bowel wall, peritonitis, perforation, fstala, or abscess. The primary symptom is abdominal pain Diagnosis is by CT scan, Patients with more severe ssmptems (pain, fever, marked leukocytosis) should be hospitalized, as should patients taking prednisone (who are at higher sisk for perforation and general peritonitis). Treatment is bed rest, nothing by mouth, TV fluids, and TV antibiotics. About 80% of patients can be treated successfully without surgery. An abscess may respond to percutaneous drainage. (CT guided). Ifresponse is satisfactory, the patient remans hospitalized until symptoms are relieved and a soft diet is resumed A barium enema is performed 2 2 wk after symptoms have resolved, Surgery is required for patents with free perforation or general peritonitis and for patients with severe symptoms that do not respond io nonsurgical treatment within 48 hours Mark this question <& => Question Td : 101512 Question 20 of 30 A74 year old mele is admitted to hospital with left lower quadrant abdominal pain, nausea, alow grade fever, and left lower quadrant tenderness. He is diagnosed as acute civerticultis. Which of the following is not an appropriate treatment option? 2) Broad spectrum antbiotics Y © b) Immediate laparotomy ©) Intravenous fuids 4) Meperidine 6) Nacogasivic suction Question Explanation: ‘Divertcuits is inflammation of a diverticulum, which can result in phlegmon of the bowel wall, peritonitis, perforation, fstala, or abscess. The primary symptom is abdominal pain Diagnosis is by CT scan, Patients with more severe ssmptems (pain, fever, marked leukocytosis) should be hospitalized, as should patients taking prednisone (who are at higher sisk for perforation and general peritonitis). Treatment is bed rest, nothing by mouth, TV fluids, and TV antibiotics. About 80% of patients can be treated successfully without surgery. An abscess may respond to percutaneous drainage. (CT guided). Ifresponse is satisfactory, the patient remans hospitalized until symptoms are relieved and a soft diet is resumed A barium enema is performed 2 2 wk after symptoms have resolved, Surgery is required for patents with free perforation or general peritonitis and for patients with severe symptoms that do not respond io nonsurgical treatment within 48 hours ‘Mark this question & => Question Td : 102951 Question 21 of 30 ‘Upper endoscopy end biopsy ofa 45 year cld man with subacute epigastric pain, weight loss, and ealy satiety shows helicobacter pylori infection and gastric carcinoma, FALSE statement regarding H. pylori and gastric cancer is a) Helicobacter pylori causes chronic inflammation that leads to alvophie gastritis, ia which altered mucosal cells replace nonmal ones +) Helicebacter pylori has been linked to mucoca-accosiated lymphoid tiesue lymphoma (MALT), ) The gastric cancer associated with H. pylon infection is usually anaplaste signet ring cancer, not well differentiated intestinal adenocarcinoma. 4) Gastric adenocarcinoma is most prevalent in Brazil, Korea, China, and Japan. ¢) Heicebecter pylori reduces the antioxidant vitamin C in the gastric mucosa, leading to the formation of nitresamnines in the stomach, Question Explanation: The gastric cancer associated with Helicobacter pylori is usually a well differentiated intestinal adenocarcinoma, nota diffised aneplastis signet-ring cancer. H. pylori colonizes the mucous layer ofthe gastric epithelium, primarily the antrum, which can alter the nonmal mucosal cells and thus leacl t> malignancy. MALT type lymphomas are found in the antrum, and investigators have found B cell infiltration ofthe epithelium and mucosal lyiaphoid folicles. In the United States, where H. pylon is increasingly less common, the tate of gastric cancer has correspondlinely decined steadily since 1930. Tn countries such as Brafil, Japan, and China, where H. pylorlinfocts over halfthe population at a young age, the pravalence of gastric cancer is very high, lack of vitamin C (en antioxidant) leads to formation of free-radicals and nitrosamines which are carcinogenic. H. pylori reduces vitamin C in the gastric mucosa. ‘Mark this question & => Question Td : 102951 Question 21 of 30 ‘Upper endoscopy and biopsy of a 45 yeer cld man with subacute epigestrc pain, weight loss, and eauly satiety shows helicobacter pylori infection and gastric carcinoma, FALSE statement regarding H. pylori and gastic cancer is a) Heicobecter pylori causes chronic inflammation that leads to atrophic gaswitis, in which altered mucosal cells replace normal ones b) Helicobacter pylori has been linked to mucosa-associated lymphoid tisrue lymphoma (MALT), Y © c) The gastric cancer associated with H. pylori infection is usually anaplastc signet ring cancer, not well diffsrentiated intestinal adenocarcinoma d) Gastric adenocarcinoma is most prevalent in Brazil, Korea, China, and Japan. €) Heicobecter pyloti reduces the antioxidant vitamin C in the gastric mucosa, leading to the formation of nitrosamnines in the stomach. Question Explanation: The gastric cancer associated with Helicobacter pylori is usually a well differentiated intestinal adenocarcinoma, nota diffised aneplastis signet-ring cancer. H. pylori colonizes the mucous layer ofthe gastric epithelium, primarily the antrum, which can alter the nonmal mucosal cells and thus leacl t> malignancy. MALT type lymphomas are found in the antrum, and investigators have found B cell infiltration ofthe epithelium and mucosal lyiaphoid folicles. In the United States, where H. pylon is increasingly less common, the tate of gastric cancer has correspondlinely decined steadily since 1930. Tn countries such as Brafil, Japan, and China, where H. pylorlinfocts over halfthe population at a young age, the pravalence of gastric cancer is very high, lack of vitamin C (en antioxidant) leads to formation of free-radicals and nitrosamines which are carcinogenic. H. pylori reduces vitamin C in the gastric mucosa. ‘Mark this question & => 103095 Question 22 of 30 ‘Whats the approximate incidence of significant gastrointestinal bleeding as a result of diverticulosis? a) 90%. 6) 25% c) 10%. 50%, 2) 75%. Question Explanation: As many as 25% of neividuals with diveriiculosis develop significant gastrointestinal bleeding at some poi ‘Mark this question & => 103095 Question 22 of 30 ‘What is the approximate incidence of significart gastrointestinal bleeding as a result of diverticulosis? a) 90%, Y © 25% c) 10%, 4) 50% ©) 75%, Question Explanation: As many as 25% of neividuals with diveriiculosis develop significant gastrointestinal bleeding at some poi Merk this question = => Question Ta : 114301 Question 23 of 30 ‘Treatment for hypericalemia that causes a true excretion of potassium is a) Calcium chloride +) Sodium polystyrene sulfonate (potassium binding resin) ©) Bicarbonate infission 6) Insulin and glucose infusion ®) Albuterol Answer | Paplanation Other User's Explanation Report An Error Question Explanation: Sodium polystyrene sulfonate (potassium binding resin) binds potassium in the GTtract and causes excretion of potassium from the body. Celcinm stabilizes the myocardial membrane. Bicarbonste, insulin, and albuterol cause only a potassium shit fom extracellular +o intracelular Merk this question = => Question Ta : 114301 Question 23 of 30 ‘Treatment for hypercalemia that causes a true excretion of potassium is a) Calcium chloride ¥ © +) Sodium polystyrene sulfonate (potassium binding resin) c) Eicarbonate infusion 6) Insulin and glucose infusion ®) Albuterol Answer | Paplanation Other User's Explanation Report An Error Question Explanation: Sodium polystyrene sulfonate (potassium binding resin) binds potassium in the GTtract and causes excretion of potassium from the body. Celcinm stabilizes the myocardial membrane. Bicarbonste, insulin, and albuterol cause only a potassium shit fom extracellular +o intracelular Mark this question e& => Question Td : 197219 Question 24 of 30 A-male aged 43 years presents with weightloss and watery diarrhoea Labs reveal hypokalaemia with a pancreatic mess. A diagnosis of VIPoma would be tupported by which one of the following? a) Achlorhydiia ») Hypoalyeacmia ©) Increased pancreatic polypeptide a) Migeatory erythema ©) Pellagra Answer | Explanation Other User's Explanation Report An Error Question Explanation: “Achlochydria is classically associated with VIPoma together with profuse diarthoza, ahypokalaemic acidosis and hyperglycaemia. Migratory erythema is associated with a ghicagonama Although raised pancreatic polypeptide is seen with a VIPoma it ie uusual end is more commenly associated with its own syndrome, Pellagra is associated with the carcinoid syndrome. Mark this question e& => Question Td : 197219 Question 24 of 30 A-male aged 43 years presents with weightloss and watery diarrhoea Labs reveal hypokalaemia with a pancreatic mess. A diagnosis of VIPorma would be tupported by which one of the following? Y © a) Achlorhydria ») Hypoalyeacmia ©) Increased pancreatic polypeptide a) Migeatory erythema ©) Pellagra Answer | Explanation Other User's Explanation Report An Error Question Explanation: “Achlochydria is classically associated with VIPoma together with profuse diarthoza, ahypokalaemic acidosis and hyperglycaemia. Migratory erythema is associated with a ghicagonama Although raised pancreatic polypeptide is seen with a VIPoma it ie uusual end is more commenly associated with its own syndrome, Pellagra is associated with the carcinoid syndrome. 3122014 7% weve interface ec k/i % e 1 PM Bh aseiSiyl soil sb, * \_\ CD www.interface.edu,pk/medical-exarns test-analysis,php ?utid=14759 apps Ellcoogle A settings [Signin Sou wo dul ce anal Bays ap go jiyiall JS) Free Hotinall #38 G other bookmar ‘Marie this question & => Question Td : 197465 Question 25 of 30 Avnew diagnostic test for malabsorption has been analysed and the results are as follows, [Diseasel[present test result yes Ino re foo for [ve fo2__ fos Applying this test to a case of chronic diatrhoea fiom a patient group where the prevalence of malabsorption is known to be 20% (probability = 0.2), The probability of 2 patient having malabsorption if they have a positive testis a) 0.16 b) 0.24 ©) 048 4) 0.64 2) 08 Answer (UEIFIRIMIY ores Users Explanation Repow Annas Question Explanation: This is tough but the College are putting more and more Evidence Based Medicine Questions inta the exam. This question tests understanding of pretest and posttest odds, ikelihood ratios, sensitivity and speciicity. The calculation is as follows Sensitivity = 0.91 (0.9 + 0.2)=0.818 Specificity = 0.8 / (0.1+ 0.8) = 0.889 Likelihood ratio for a positive test CR+) = 0.818/ (1 - 0889) =7.2 Pre-test odds = 0.2/ (1 -0.2)= 0.25 Post-test odds = pre-test odds KLR+ Post-test probebilty = 1.8/(18+ 1 Disease present test result yes no -+ve true positive (A) false positive @) -ve false negative (C) true negative (D) Sensitivity (how march a test is positive in disease) =A (A +C) Specificity (how much a test is negative in health) =D / (B +D) Positive Predictive Value = A /(A +B) Negative Predictive value =D | (C+D) Pre-test odds = the odds ofhaving the disease before you da the test (e.g. your rule-of-thumb guestimate or the prevalence of the disease the population or based on clinical findings etc.) Post-test odds = the odds of having the cisease after you did the CGS Systematic Error = (A+B) / (A + C) = good statisti for -1) breaking the ice at a party of epidemiologsts, 2) confusing your fellow SpRS meetings Likelihood Rato (LR) + (the ratio of the chance af having 2 -+ve test the disease is present ta the chance af having a positive test ifthe disease ic absent) = senstivity / (1 - specificity) LR- = (1 - sensitivity) / specificity Likelihood Ratios are good for 25K7.2=18 0.64 # Directly calculating post-test adds Total Questions bi HbR BSE Is bie em ee SBEBAERS&EBBE xxxKXKXKKXKKKKXKKKKKKKKKKKKKKKKKKK 3122014 7% weve interface ec k/i % e 1 PM Bh aseiSiyl soil sb, * \_\ CD www.interface.edu,pk/medical-exarns test-analysis,php ?utid=14759 apps Ellcoogle A settings [Signin Sou wo dul ce anal Bays ap go jiyiall JS) Free Hotinall #38 G other bookmar ‘Marie this question & => Question Td : 197465 Question 25 of 30 Avnew diagnostic test for malabsorption has been analysed and the results are as follows, [Diseasel[present test resutt|yes —_|[no [tre 0.9 0.1. [ve fo2 ios Applying this test to a case of chronic diarrhoea fiom a patient group where the prevalence of malabsorption is known to be 20% (probability = 0.2). The probability of a patient having malabsorpiioa if they have a positive test is a) 0.16 b) 0.24 °c) 0.48 vO d064 208 Answer (UEIFIRIMIY ores Users Explanation Repow Annas Question Explanation: This is tough but the College are putting more and more Evidence Based Medicine Questions inta the exam. This question tests understanding of pretest and posttest odds, ikelihood ratios, sensitivity and speciicity. The calculation is as follows Sensitivity = 0.91 (0.9 + 0.2)=0.818 Specificity = 0.8 / (0.1+ 0.8) = 0.889 Likelihood ratio for a positive test CR+) = 0.818/ (1 - 0889) =7.2 Pre-test odds = 0.2/ (1 -0.2)= 0.25 Post-test odds = pre-test odds KLR+ Post-test probebilty = 1.8/(18+ 1 Disease present test result yes no -+ve true positive (A) false positive @) -ve false negative (C) true negative (D) Sensitivity (how march a test is positive in disease) =A (A +C) Specificity (how much a test is negative in health) =D / (B +D) Positive Predictive Value = A /(A +B) Negative Predictive value =D | (C+D) Pre-test odds = the odds ofhaving the disease before you da the test (e.g. your rule-of-thumb guestimate or the prevalence of the disease the population or based on clinical findings etc.) Post-test odds = the odds of having the cisease after you did the CGS Systematic Error = (A+B) / (A + C) = good statisti for -1) breaking the ice at a party of epidemiologsts, 2) confusing your fellow SpRS meetings Likelihood Rato (LR) + (the ratio of the chance af having 2 -+ve test the disease is present ta the chance af having a positive test ifthe disease ic absent) = senstivity / (1 - specificity) LR- = (1 - sensitivity) / specificity Likelihood Ratios are good for 25K7.2=18 0.64 # Directly calculating post-test adds Total Questions bi HbR BSE Is bie em ee SBEBAERS&EBBE xxxKXKXKKXKKKKXKKKKKKKKKKKKKKKKKKK Marl this question = => Question Td : 197698 Question 26 of 30 A 34 year old obese Afrocarribean woman has mild jaundice, She is teetoteler and her BMI is 30kg /in2. Her bifrubin is 25 wmoVL, AST is 140 UML (1-31), ALT is 155 U/L (5-35), and random blood ghicose is 11.2 mmol/L, Hepatitis B and C screening is negative and anti-nuclear antibodies titre is 1: 16. Ultrasound abdomen reveals hyperechogeric hepatic parenchyma, Liver biopsy reveals, lesions suggestive of alcoholic iver disease. The lcely diagnosis is @) Alcoholic liver disease ) Autoimmune hepatitis ©) Non-dlecholic steatohepatitis 6) Primary biliary cinthosis ©) Viral hepatitis Answer | Explanation Other User's Explanation Report An Error Question Explanati This is a case of non-alccholic steatohepatitis, the diagnosis of which is made only by histology of liver biopsy which shows lesions suggestive of ethanol intake in a patient known to consume less than 40g of alcohol per week. The diagnosis is supported by the presence of obesity, hyperglycaemia and hyperechogenic hepatic parenchyma. In alcholic hepatitis, AST to ALT ratio is moro than 1 Marl this question = => Question Td : 197698 Question 26 of 30 A.34 year old obese Affocarrisean woman has mild jaundice. She is teetotaler and her BMI is 30kg /m2. Her biirubin is 25 umolL, ASTis 140 UIL (1-31), ALT is 155 UML 6-35), and random blood glucose is 11.2 mmol/L. Hepaiitis B and C screening is negative aad anti-nuclear antbodies titre is 1: 16. Ultrasound abdemen reveals hyperechogenic hepatic parenchyma. Liver biopsy reveals lesions suggestive of alcoholic iver disease. The licely diagnosis is 2) Alcoholic liver disease b) Autoimmune hepatitis ¥ © ©) Non-decholic steatohepatitis © Primary biliary cirrhosis ©) Viral hepatiis Answer | Explanation Other User's Explanation Report An Error Question Explanati This is a case of non-alccholic steatohepatitis, the diagnosis of which is made only by histology of liver biopsy which shows lesions suggestive of ethanol intake in a patient known to consume less than 40g of alcohol per week. The diagnosis is supported by the presence of obesity, hyperglycaemia and hyperechogenic hepatic parenchyma. In alcholic hepatitis, AST to ALT ratio is moro than 1 ‘Mark this question = => Question Id : 199383 Question 27 of 30 A. woman aged 28 years develops abdominal pein, jaundice and ascites worsening over a week. She dkinks 10 units of alcohol each week and takes the oral contraceptive pill, Which finding would make a diagnosis ofhepatic vein thrombosis Badd Chiari syndrome [BCS] most lkely? 2) Acute liver Faure +) Alenine aminetrancferase (ALT) of 345 UIL (5-35) ©) Ankle oedema 4) Ascites fuid protein of 38 afl 6) Tender enlarged liver Question Explanation: The most common causes of an acute severe liver injury in a young woman ace: * Viruses (including hepatitis A virus [LAV], hepatitis B virus [HBV]) + Drugs (particularly paracetamol overdose) * Autoimmue hepatitis and + Hepatic vein thrombosis (often precipitated by pregnancy or oral contraceptive pill [OCP] use) ‘The presence of liver failure, ankle oedema, and an exudative ascites do not help differentiate between these etiologies. The ALT of 345 is moderately elevated and compatible with BCS. With viral or drug related hepatitis the peak ALT is usually much higher than this; the ALT may already be on the way down she has had symptcms for a week. Tender hepetomegaly is one of the hallmarks of ECS. In acute severe viral, autoimune or drug / toxin related liver disease the necrotic liver decreases in size. ‘Mark this question = => Question Id : 199383 Question 27 of 30 A. woman aged 28 years develops abdominal pain, jaundice and ascites worsening over a week. She drinks 10 units of alzohol each week and takes the oral contraceptive pill, Which finding would make a diagnosis ofhepatic vein thrombosis Badd Chiari syndrome [BCS] most likely? 2) Acute liver failure b) Alanine aminotransferase (ALT) of 345 UIL (5-35) ) Ankle oedema 4) Ascites fluid protein of 38 g/L Y © @) Tender enlarged tiver Question Explanation: The most common causes of an acute severe liver injury in a young woman ace: * Viruses (including hepatitis A virus [LAV], hepatitis B virus [HBV]) + Drugs (particularly paracetamol overdose) * Autoimmue hepatitis and + Hepatic vein thrombosis (often precipitated by pregnancy or oral contraceptive pill [OCP] use) ‘The presence of liver failure, ankle oedema, and an exudative ascites do not help differentiate between these etiologies. The ALT of 345 is moderately elevated and compatible with BCS. With viral or drug related hepatitis the peak ALT is usually much higher than this; the ALT may already be on the way down she has had symptcms for a week. Tender hepetomegaly is one of the hallmarks of ECS. In acute severe viral, autoimune or drug / toxin related liver disease the necrotic liver decreases in size. Marke this question >> Question Td : 212738 Question 28 of 30 ‘4.43 year old man has intermittent episodes of sewere, crushing chest pain that extends to the back and the jaw and last from a few seconds to several minutes. Many times the pein is accompanied by dysphagia and triggered by the ingestion of cortain foods. ECG znd carciac enzymes have been negative. Barium swalow shows disorganized peristalic contractions. Manometry shows that approximately one half of wet swallows produce repetitive sirmultaneous esophageal sphincter has normal pressures and exhibits oral relazation, The most likely diagnosis is a) Achalasia of the esophagus ') Cancer of the lower esophagus ©) Diffuse esophageal spasm 4) Nuteracker esophagus 6) Zenker diverticutum Question Explanation: ‘The clinical, radiologic, and manometric criteria of diffise esophageal spasm are all described in the vignette. Achalasia (A) typically thas clinical progression and regurgitation of undigested food Ta the manometris shadies, there are no effective peristaltic contractions and there is increased resting pressure in the lower sphincter, which falls to celax during swellowing The dysphagia of esophageal cancer (B) is typically progressive, irom solids to liquids. The barium swallow would be diagnostic. Nutcracker esophagus (D) is very similar to diffuse esophageal spasm. On manometry, however, there is mean distal esophageal peristaltic amplitade of more than 180 mmm He, inclading an elevated baseline pressure in the lower sphincter. Zenker diverticulum (E) produces regurgitation of uncigested food and symptoms referable to the upper esophagus. The barium swallow would be diagnostic. Report An Error Marke this question >> Question Td : 212738 Question 28 of 30 ‘4.43 year old man has intermittent episodes of sewere, crushing chest pain that extends to the back and the jaw and last from a few seconds to several minutes. Many times the pein is accompanied by dysphagia and triggered by the ingestion of cortain foods. ECG znd carciac enzymes have been negative. Barium swalow shows disorganized peristalic contractions. Manometry shows that approximately one half of wet swallows produce repetitive sirmultaneous esophageal sphincter has normal pressures and exhibits oral relazation, The most likely diagnosis is a) Achalasia of the esophagus ') Cancer of the lower esophagus Y © 0) Diffse esophageal spasm 4) Nuteracker esophagus 6) Zenker diverticutum Question Explanation: ‘The clinical, radiologic, and manometric criteria of diffise esophageal spasm are all described in the vignette. Achalasia (A) typically thas clinical progression and regurgitation of undigested food Ta the manometris shadies, there are no effective peristaltic contractions and there is increased resting pressure in the lower sphincter, which falls to celax during swellowing The dysphagia of esophageal cancer (B) is typically progressive, irom solids to liquids. The barium swallow would be diagnostic. Nutcracker esophagus (D) is very similar to diffuse esophageal spasm. On manometry, however, there is mean distal esophageal peristaltic amplitade of more than 180 mmm He, inclading an elevated baseline pressure in the lower sphincter. Zenker diverticulum (E) produces regurgitation of uncigested food and symptoms referable to the upper esophagus. The barium swallow would be diagnostic. Report An Error ‘Mark this question e& => Question Td : 213822 Question 29 of 30 A 67-year-old woman is concerned about her risks for breast, ovarian, and colon cancer. Breast and pelvic examination is unremarkable. A mammogram is normal, Findings from 2 barium enema are shown, Which of the following is the most common presenting symptom of this condition? a) Anenia ») Blumer shelf «) Obstruction 4) Virchow node 2) Weight loss Question Explanation: ‘The barium enema demonstrates an irregular lesion in the sigmoid colon that is highly suspicious for adenocarcinoma, Cancers arising in the lef and sigmoid colon are characterized by rectal bleeding, obstructive symptoms, and possibly constipation alternating with diarthea Stools may have a narrow caliber Although anemia, Blumer shelf (inetastasis to the pelvic cul-de-sac). Virchow node (palpable lymph node in the left supraclavicular space) and weight loss can be associated with cancers of the left and sigmoid colon, they are more commonly associated with right-sided colon cancer. ‘Mark this question e& => Question Td : 213822 Question 29 of 30 A 67-year-old woman is concerned about her risks for breast, ovarian, and colon cancer. Breast and pelvic examination is unremarkable. A mammogram is normal, Findings from 2 barium enema are shown, Which of the following is the most common presenting symptom of this condition? a) Anenia ») Blumer shelf v © ©) Obstruction & Virchow node 2) Weight loss Question Explanation: ‘The barium enema demonstrates an irregular lesion in the sigmoid colon that is highly suspicious for adenocarcinoma, Cancers arising in the lef and sigmoid colon are characterized by rectal bleeding, obstructive symptoms, and possibly constipation alternating with diarthea Stools may have a narrow caliber Although anemia, Blumer shelf (inetastasis to the pelvic cul-de-sac). Virchow node (palpable lymph node in the left supraclavicular space) and weight loss can be associated with cancers of the left and sigmoid colon, they are more commonly associated with right-sided colon cancer. ‘Marke this question & Question Ta : 215988 Question 30 of 30 A 67 year old man has epigastric and substemel chest pain, Sirilar episodes are present in the past On each ED wisithis cardiac evaluations are negative and he is discharged. He has been treated for gastroesophageal redlux disease (GERD) over the past 10 years with ranitidine and antacids, which only intermittently relieve his symptoms. The next best step at this sims is 8) Barum swallows b) Endoscopy only ©) Endoscopy with biopsies 6) Omeprazole ©) pH mentoring Anewor (UBIVIRIRAR) otner sors Explanation Report An Error Question Explanation: “Any patient who has alarm symptoms should receive endoscopy These include age greater than 45 years, dysphasialodynephaga, and bleeding. This pationt hes « history oflong-stending GERD with persistent symptoms. Barrett esophagus is a premalignant condition that occurs secondary to damage of the lower esophagus in longstanding acid refux disease Endoscopy and biopsies should therefore be performed to assess the possibilty of dysplastic changes. Barium swallow (4) is 2 radiographic stady in which, the patient swallows barium and images are taken as the dye travels down the esophagus and into the stomach. This stucy helps determine where the gastroesophageal junction lies, butt does not allow attissue diagnosis ofpossible dysplasia, Visualization itself @)is not adequate, because the tissue must be sampled as described. IFthis patient were younger and without a history of alarm syinptoms, a tial of PPI. such as omeprazole (D) might be appropriate. Once a tissue diagnosis is confirmed, further management strategies can be determined. Monitoring of the esophageal pH may be done to confirm the diagnosis of gastroesophageal reflux. GERD may ultimately lead to Barreti esophagus, but an abnormal pH is of litle value at this tine (€). ‘Marke this question & Question Ta : 215988 Question 30 of 30 A 67 year old man has epigastric and substemal chest pain. Sinilar episodes are present in the past. On each ED wait his cardiac evaluations are negetive and he is discharged. He hac been treated for gastroesophageal redux disease (GERD) over the past 10 yeats with ranitidine and antacids, which only intermittently relieve bis symptoms. The nes best step at this time is 8) Barium swallows +b) Endoscopy only JY © 0) Endoscopy with biopsies 6) Omeprazole ©) pH monitoring Anewor (UBIVIRIRAR) otner sors Explanation Report An Error Question Explanation: “Any patient who has alarm symptoms should receive endoscopy These include age greater than 45 years, dysphasialodynephaga, and bleeding. This pationt hes « history oflong-stending GERD with persistent symptoms. Barrett esophagus is a premalignant condition that occurs secondary to damage of the lower esophagus in longstanding acid refux disease Endoscopy and biopsies should therefore be performed to assess the possibilty of dysplastic changes. Barium swallow (4) is 2 radiographic stady in which, the patient swallows barium and images are taken as the dye travels down the esophagus and into the stomach. This stucy helps determine where the gastroesophageal junction lies, butt does not allow attissue diagnosis ofpossible dysplasia, Visualization itself @)is not adequate, because the tissue must be sampled as described. IFthis patient were younger and without a history of alarm syinptoms, a tial of PPI. such as omeprazole (D) might be appropriate. Once a tissue diagnosis is confirmed, further management strategies can be determined. Monitoring of the esophageal pH may be done to confirm the diagnosis of gastroesophageal reflux. GERD may ultimately lead to Barreti esophagus, but an abnormal pH is of litle value at this tine (€). ‘Mark this question => Question Ti : 23894 Question 1 of 30 Following are the causes of major lower gastrointestinal hemorrhage except a) Inflammatory bowel disease b) Hemomthoids ©) Angiodysplasia 4) Aottoenterie fistula ¢) Colon cancer Avever [RESRIRIRHBA) other Users Explanation Report An Error Question Explanatios “When lower gastrointestinal bleeding occurs in a patient, hefshe can presert with melena, hematacheria and a positive fecal occult blood test. A blood test may show anemia “The causes of acute lower Gl bleeding include divertculosis, angiodysplasia, colon cancer, colitis (including infectious, ischemic, or radiation-induced forms). inflammatory bowel disease. polyps, Meckel diverticulum, and aottoenteric fistula, Hemorrhoids are probably the most common cause oflower GI bleeding, but they usually de not pose difficulties in the diagnosis and they rarely cause massive bleeding, ‘Mark this question => Question Ti : 23894 Question 1 of 30 Following are the causes of major lower gastrointestinal hemorrhage except a) Inflammatory bowel disease Y © b) Hemorrhoids ©) Angiodysplasia 4) Aottoenterie fistuia ¢) Colon cancer Avever [RESRIRIRHBA) other Users Explanation Report An Error Question Explanatios “When lower gastrointestinal bleeding occurs in a patient, hefshe can presert with melena, hematacheria and a positive fecal occult blood test. A blood test may show anemia “The causes of acute lower Gl bleeding include divertculosis, angiodysplasia, colon cancer, colitis (including infectious, ischemic, or radiation-induced forms). inflammatory bowel disease. polyps, Meckel diverticulum, and aottoenteric fistula, Hemorrhoids are probably the most common cause oflower GI bleeding, but they usually de not pose difficulties in the diagnosis and they rarely cause massive bleeding, ‘Marle this question & => Question Ta : 30503 Question 2 of 30 A woman was referred with iron deficiency anemia. She was 5d year old and suffering from rheumatoid arthritis. Endoscopy showed several superficial antral erosions, with small bowel biopsy showing mild villous blunting, apopotic bodies, occesional eosinophils and mild increase in chronic inflammatory cells. Colonoscopy was reported as normal Which of the followings the most likely cause of these findings? a) Celiac disease ) Crohn's disease c) Tropical Sprue d) Whopple’s disease e) NSAID therapy Question Explanatio: ‘The salient features in this patient's case revolve around the fact that she has rheumatoid arthritis (hence the requirement for NSAIDs) the iron deficiency anemia and superficial ulceration on endoscopy with features indicative of inflammation are due to chronic NSAID suse, Celice disease is aesociated with villous atrophy and lymphocyte infltration. There is no cuggestion on the biopey of lymphocyte infiltration which argues against lymphoma or celiac. ‘Marle this question & => Question Ta : 30503 Question 2 of 30 A worsan was referred with iron deficiency anemia. She was 54 yeer old and suffering from rheumatoid arthritis. Endoscopy showed several superficial antral erosions, with small bowel biopsy showing mild villous blunting, epepetic bodies, occasional eosinophils and mild increase in chronic inflammatory cells. Colonoscopy was reported as normal. Which ofthe followings the most liely cause of these findings? a) Celiac disease ) Crohn's disease c) Tropical Spme d) Whopple’s disease Y¥ © e) NSAD therapy Question Explanatio: ‘The salient features in this patient's case revolve around the fact that she has rheumatoid arthritis (hence the requirement for NSAIDs) the iron deficiency anemia and superficial ulceration on endoscopy with features indicative of inflammation are due to chronic NSAID suse, Celice disease is aesociated with villous atrophy and lymphocyte infltration. There is no cuggestion on the biopey of lymphocyte infiltration which argues against lymphoma or celiac. ‘Marle this question & => Question Td : 46626 Question3 of 30 “Which of the following is most likely to be associated with a recurrent painfid swelling of a Submancibular salivary gland? a) Virus infection ) Sarzoidosis ©) Mined rumner ) Caloutas e) AIDS Question Explanation: Sialolithiasis is the result of tiny stones, called sialoliths, forming inside the salivary glands. Symptoms of sialolithiasis may include swollen gands around the neck, mouth pain, mouth tendemess and redness, swelling around the mouth, neck pain or swelling Risk factors for developing sialolithiasis include: dehydration, which thickens the saliva, not eating a lot, which decreases the demand for saliva and medications that decrease saliva production, incliding certain antihistamines, blood pressure drugs and psychiatric medications. Ifthe stones cause a blockage within the salivary glands, i can lead to a painful, infection called sialadenitis. ‘Marle this question & => Question Td : 46626 Question3 of 30 “Which of the following is mast ikely to be associated with a recurrent painfil swelling of a Submancibular salivary gand? a) Virus infection ') Sarcoidosis c) Mixed tumor YO A Caleuas e) AIDS Question Explanation: Sialolithiasis is the result of tiny stones, called sialoliths, forming inside the salivary glands. Symptoms of sialolithiasis may include swollen gands around the neck, mouth pain, mouth tendemess and redness, swelling around the mouth, neck pain or swelling Risk factors for developing sialolithiasis include: dehydration, which thickens the saliva, not eating a lot, which decreases the demand for saliva and medications that decrease saliva production, incliding certain antihistamines, blood pressure drugs and psychiatric medications. Ifthe stones cause a blockage within the salivary glands, i can lead to a painful, infection called sialadenitis. 32/2014 5:28:45 PM J % wow interface.ecu.ck/n x \ EM aiousSIyl doll ley x z € > C BG www.interface edu nk /medical-examns/test-analysis pho 2utd=14759 * 8 apps Flcoogle Asenngs [sgn Zo. woke aul Cou geuedilésye Of..eio ge ieiall Js [) Freetotmal > CJ other hookrar Total Questions ‘Marie this question = => (Question Ti Question 4 of 30 A.45 year old woman presents with left lower quadrant pain with non-bloody stool. Tendemess is present over the area. She is afebrile and her rectal examinaton is normal. Diverticulum is seen in a bariura enema study. The most appropriate intervention is me, “ramen clon » | lg srt 3 Diverticular Disease a) Try high fer ciet b) Give antibiotic prophylanis for weeks €) Colonoscopy Ske eHR BRIE bE 4) Surgery Question Explanation: A diverticulum (plural: diverticula) is medical teem for an outpouching of a hollow (or a fi filled) structure in the body, ike an intestine. The development of diverticula is caused by spasms of the muscular layer ofthe intestine. The resulting pressure that these spasms exert on the intestinal wall causes a part of the wall to bulge ata point of weakness. Diverticulosis can cause painfil cramps, diarrhea or cther bowel movement disturbances, and blood in the stool. The aurrow opening of a diverticulum cen bleed, sometimes hheavly, into the intestine and out through the rectum, Stoal that is trapped in a diversiculum may cause not only bleeding but also inflammation and infection, resulting n diverticulitis. The dagnosis is confirmed by barium enema x-ray study or by colonoscopy. However, ifthe person has severe abdominal rain, computed tomography (CT) is performed instead, so as not to supture the inflamed intestine. The goal of treatment is usually to reduce intestinal spasms, whick is best achieved by meintaining a high Sber dict (which consists of vegetables, fruits, and whole grains) and drinking plenty of fuids. An increased bulk in the large intestine reduces spasms, which in turn decreases the pressure on the walls ofthe large intestine. Since this patient has no fever or signs of infection Le a BRBSBRRBBERBE BEER EBBEBM RR e hee ee 32/2014 5:28:45 PM J % wow interface.ecu.ck/n x \ EM aiousSIyl doll ley x z € > C BG www.interface edu nk /medical-examns/test-analysis pho 2utd=14759 * 8 apps Flcoogle Asenngs [sgn Zo. woke aul Cou geuedilésye Of..eio ge ieiall Js [) Freetotmal > CJ other hookrar Total Questions ‘Marie this question = => (Question Ti Question 4 of 30 A.45 year old woman presents with left lower quadrant pain with non-bloody stool. Tendemess is present over the area. She is afebrile and her rectal examinaton is normal. Diverticulum is seen in a bariura enema study. The most appropriate intervention is me, “ramen clon » | lg srt 3 Diverticular Disease J © a)Try high Sher det +b) Give antibioiic prophylaxis for weeks €) Colonoscopy Ske eHR BRIE bE 4) Surgery Question Explanation: A diverticulum (plural: diverticula) is medical teem for an outpouching of a hollow (or a fi filled) structure in the body, ike an intestine. The development of diverticula is caused by spasms of the muscular layer ofthe intestine. The resulting pressure that these spasms exert on the intestinal wall causes a part of the wall to bulge ata point of weakness. Diverticulosis can cause painfil cramps, diarrhea or cther bowel movement disturbances, and blood in the stool. The aurrow opening of a diverticulum cen bleed, sometimes hheavly, into the intestine and out through the rectum, Stoal that is trapped in a diversiculum may cause not only bleeding but also inflammation and infection, resulting n diverticulitis. The dagnosis is confirmed by barium enema x-ray study or by colonoscopy. However, ifthe person has severe abdominal rain, computed tomography (CT) is performed instead, so as not to supture the inflamed intestine. The goal of treatment is usually to reduce intestinal spasms, whick is best achieved by meintaining a high Sber dict (which consists of vegetables, fruits, and whole grains) and drinking plenty of fuids. An increased bulk in the large intestine reduces spasms, which in turn decreases the pressure on the walls ofthe large intestine. Since this patient has no fever or signs of infection Le a BRBSBRRBBERBE BEER EBBEBM RR e hee ee Merk this question = => Question Ta : 62550 Question 5 of 30 A.65 year cld female complains of sight upper quadrant pain, She is febrile. Physical exam shows slight abdominal tenderness. The appropriate investigation to confiem the diagnosis is a) Biopsy ) Bari study ©) Peritoneal Aspiration ©) Rectosiemoidoscopy ©) Ultrasound Anower [JEQUBNBHEN) incr User's Explanation Report An Error Question Explanatio CChelelitiacis is the presence of one or more calculi (allstoner) in the gollbladder. 20% of people > 65 years have gallstones, and most disorders of the extrahepatic biliary tract arise ftom gallstones. Gallstones may be asymptomatic or cause biliary colic. Pain may occur in the right upper quadrant but is often poorly localized or occurs elsewhere in the abdomen, particularly among diabetics and the elderly. Mild right uper quadrant or epigastric tenderness may be presert, but peritoneal findings are absent, and laboratory tests ace aommal Diagnosis is usually based on ultrasound. If cholelithiasis causes symptoms or complications, cholecystectomy becomes necessary. Merk this question = => Question Ta : 62550 Question 5 of 30 A.65 year cld female complains of sight upper quadrant pain, She is febrile. Physical exam shows slight abdominal tenderness. The appropriate investigation to confirm the diagnosis is @) Biopsy &) Barium study: ©) Peritoneal Aspiration ©) Rectosiameidoscopy ¥ © ©) Ulnasound Anower [JEQUBIEEN) incr User's Explanation Report An Error Question Explanatio CChelelitiacis is the presence of one or more calculi (allstoner) in the gollbladder. 20% of people > 65 years have gallstones, and most disorders of the extrahepatic biliary tract arise ftom gallstones. Gallstones may be asymptomatic or cause biliary colic. Pain may occur in the right upper quadrant but is often poorly localized or occurs elsewhere in the abdomen, particularly among diabetics and the elderly. Mild right uper quadrant or epigastric tenderness may be presert, but peritoneal findings are absent, and laboratory tests ace aommal Diagnosis is usually based on ultrasound. If cholelithiasis causes symptoms or complications, cholecystectomy becomes necessary. ‘Mark this question & => Question Td : 62750 Question 6 of 30 A 32 year old woman with known HIV presents with a 3 month history of watery diarthea, severe weakness, and a 22 pound weight loss. Multiple stool tests far bacteria, ova, and parasites are repeatedly negative. A colonascapy is normal. What is the most Heely cause of her diarrhea? a) Cryptosporidiosis +) Cytomegalovins infection c) Entamoeba histolytica Enterctoxigenic Escherichia ccli ) Shigella dysenteriae Question Explanation: ‘This patient has a typical small bowel type cairhea, seen in HIV with watery camhea, weakness, and weight loss. The most common etiology for this smndrome is cryptosporiciosis infection of the stnall intestine, where the spores cen be seen on the tips of the wii on biopsy This organism can be demonstrated with special culture media, Cther organisms in the same family, such as microsporidia aad Isospora bell, produce identical syndromes, ‘Mark this question & => Question Td : 62750 Question 6 of 30 A 32 year old woman with known HIV presents with a 3 month history of watery diarthea, severe weakness, and a 22 pound weight loss. Multiple stool tests far bacteria, ova, and parasites are repeatedly negative. A colonascapy is normal. What is the most Heely cause of her diarrhea? Y © a) Cryptosporidiosis +) Cytomegalovins infection c) Entamoeba histolytica Enterctoxigenic Escherichia ccli ) Shigella dysenteriae Question Explanation: ‘This patient has a typical small bowel type cairhea, seen in HIV with watery camhea, weakness, and weight loss. The most common etiology for this smndrome is cryptosporiciosis infection of the stnall intestine, where the spores cen be seen on the tips of the wii on biopsy This organism can be demonstrated with special culture media, Cther organisms in the same family, such as microsporidia aad Isospora bell, produce identical syndromes, ‘Mark this question = => Question Ti : 63748 Question 7 of 30 ‘Which of the following is not caused by exposure to lead? a) Anemia 6) Paresis o) Liver cirrhosis 6) Abdomixal pain ©) Porphprinuria Anewor [UEIVIRNSHAN) otnerUsors Exptan Question Explanation: Chronic lead poisoning in children may canse mental retardation, seimre disorders, aggressive behavior disorders, developmental regression, chronic abdominal pain, and ancmia Cirrhosis ofthe lireris not associated with lead Report An Error ‘Mark this question = => Question Ti : 63748 Question 7 of 30 ‘Which of the following is not caused by exposure to lead? 8) Aneenia 6) Paresis SM © o&) Liver cirthosis 6) Abdomixal pain ©) Porplayrimuria Anewor [UEIVIRNSHAN) otnerUsors Exptan Question Explanation: Chronic lead poisoning in children may canse mental retardation, seimre disorders, aggressive behavior disorders, developmental regression, chronic abdominal pain, and ancmia Cirrhosis ofthe lireris not associated with lead Report An Error ‘Mark this question <= => Question Td : 72478 Question 8 of 30 A 26-year-old female has spontaneous bacterial peritonitis. Which one ofthe following infectous agents is LEAST likely to produce spontaneoue bacterial peritonitic? a) Escherichia col, 'b) Staphylococeus Aurous c) Streptococcus pneumoniae ) Streptococcus viridans €) Group D streptococcus Answer (Barisnenon) Other User's Explanation Report An Error Question Explanatio: Spontaneous bacterial pertonitis, or the onset of bacterial peritonitis in the absence of cther underlying causes, is most often caused bby Escherichia col. Other common cases inchude streptococcal species such as group D Streptococcus, Streptococcus Pacumoniae, and Streptococcus viridians. Staphylococcus aursus has act been implicated in spontancous bacterial peritonitis ‘Mark this question <= => Question Td : 72478 Question 8 of 30 A 26-year-old female has spontaneous bacterial peritonitis. Which one ofthe following infectous agents is LEAST likely to produce spontaneoue bacterial peritonitic? a) Escherichia col, Y © b) Staphylococcus Aureus c) Streptococcus pneumoniae ) Streptococcus viridans €) Group D streptococcus Answer (Barisnenon) Other User's Explanation Report An Error Question Explanatio: Spontaneous bacterial pertonitis, or the onset of bacterial peritonitis in the absence of cther underlying causes, is most often caused bby Escherichia col. Other common cases inchude streptococcal species such as group D Streptococcus, Streptococcus Pacumoniae, and Streptococcus viridians. Staphylococcus aursus has act been implicated in spontancous bacterial peritonitis ‘Marke this question & => Question Td : 74311 Question 9 of 30 All of the following are true of technetium 99m-labeled sulfar colloid liver scans EXCEPT a) The technetium 99m-labeled sulfur celloid is taken up by the reticuloendothelial cells in the liver. b) A "cold!" arcais produced by any disease process that replaces Kupffer cells. ©) Hepatic vein obstruction may resut in preferential uptake by the caudate lobe of the liver 6) Lesions greater than 0.5 cin are ezsily detected. ©) There may be preferential uptake by the bone marrow in persons with postal hypertension. newer [UENSIHEN) ner Users Explanation Report An Error Question Explanation: ‘Technetiun 99rn-labeled culfir colloid is the most commonly used radiepharmacentical for anatomic evaluation of the liver. It is talcen up by the reticulcendothelial (Kupffer) cells in the liver. Any disease process thet replaces these cells, such as timors, produces 2 "cold" spot onthe scan. Lesions are reliably detected if they are greater than 2 cm in diameter. Hepatic vein obstruction as seen in the Bucd-Chiasi syndrome may resuit in preferential uptake of technetium by the candate lobe, Preferential uptake by the bone marrow and spleen may occur in persons with portal hypertension, ‘Marke this question & => Question Td : 74311 Question 9 of 30 All ofthe following are true of technetium 99m labeled sulfse colloid fiver scans EXCEPT 2) The technetium 99m-labeled sulfur cclloidis taken up by the reticuloendothelial cells in the liver. £) A ‘cold" arcais produced by any disease process that replaces Kupffer cells. ©) Hepatic vein obstruction may resut in preferential uptake by the caudate lobe of the liver © 4) Lesions greater than 0.5 cm are easily detected 6) There may he preferential uptake by the bone marrow in persons with portal hypertension newer [UENSIHEN) ner Users Explanation Report An Error Question Explanation: ‘Technetiun 99rn-labeled culfir colloid is the most commonly used radiepharmacentical for anatomic evaluation of the liver. It is talcen up by the reticulcendothelial (Kupffer) cells in the liver. Any disease process thet replaces these cells, such as timors, produces 2 "cold" spot onthe scan. Lesions are reliably detected if they are greater than 2 cm in diameter. Hepatic vein obstruction as seen in the Bucd-Chiasi syndrome may resuit in preferential uptake of technetium by the candate lobe, Preferential uptake by the bone marrow and spleen may occur in persons with portal hypertension, ‘Marke this question & => Question Td : 83572 Question 10 of 30 A 70 year old fermale complains of constipation and abdominal pain. Over a period of 48 hours, her symptoms worsen and she is transported to the hospital for laparoscopic evaluation ofan acute abdomen. At laparoscopy, aloop of bowel is noted to be twisted and darkly hemorrhagic. The most likely location for the bowel loop is, a) Appendix and cecum: ) Ascending colon Descending colon Sigmoid colon 9 a ¢) Transverse colon Question Explanatio The twisted bowel loop is a volmihss, which is dangerous because the twist may cat offblood supply, leading to bowel infarction ‘Typically, an elderly patient develops an "acute abdomen end is found at laparoscopy or laparotomy to have a volvulus thet is usually in the poorly supported sigmoid colon. Most other parts of the colon (choice A, B, C) are hold in place by the posterior peritoneal membranes, and the transvesse colon (choice E) is stretched so that it cannot twist In contrast, intussusception tends to occur in babies and young children or in patierts vwith a mass lesion that gets dragged by peuistalsis into the adjacent section of large or small intestixe. ‘Marke this question & => Question Td : 83572 Question 10 of 30 A 70 year old fermale complains of constipation and abdominal pain. Over a period of 48 hours, her symptoms worsen and she is transported to the hospital for laparoscopic evaluation ofan acute abdomen. At laparoscopy, aloop of bowel is noted to be twisted and darkly hemorrhagic. The most likely location for the bowel loop is, a) Appendix and cecum: ) Ascending colon Descending colon Sigmoid colon ¢) You ¢) Transverse colon Question Explanatio The twisted bowel loop is a volmihss, which is dangerous because the twist may cat offblood supply, leading to bowel infarction ‘Typically, an elderly patient develops an "acute abdomen end is found at laparoscopy or laparotomy to have a volvulus thet is usually in the poorly supported sigmoid colon. Most other parts of the colon (choice A, B, C) are hold in place by the posterior peritoneal membranes, and the transvesse colon (choice E) is stretched so that it cannot twist In contrast, intussusception tends to occur in babies and young children or in patierts vwith a mass lesion that gets dragged by peuistalsis into the adjacent section of large or small intestixe. ‘Marke this question << => Question Ta : 84667 Question 11 of 30 4.25 year old female gives hirth to an apparently normal male. He begns feeding well by the second day, then at 6 months, suddenly develops colicky abdominal pain and vomiting, Ibis determined that he has gasirointestinal obstruction, The most iicely cause of this presentaton is 2) Adhesions ) Congenital pyloric stenosis ©) Hirschsprung disease &) Intussusception ©) Volvulus due to matroration Question Explanation: ‘All of the concitions listed can cause gastrointestinal obstruction, but the clinical presentation is most suggestive of innussusception, Tntussusception is the most common cause of gastrointestinal obstruction in children 3 months to 6 years (peak age is between 5 and 10 months), Tn intussusception there is telescoping cf one bowel segmentinto another, more distal segment. Tt occurs due to the pace support offtred ty the bowel sections thin mesentery. Intussusception produces intestinal obstruction, and it may produce bowel ischemia or infarction by trapping mesenteric vessels along with the affected segment. In some cases, the mntussusception maybe reduced by diagnostic barium enema “Adhesions (choice A) can cause bowel obsiruction following surgery or inflammatory bowel diseases Congenital pyloric stenosis (choice 3) typically presents as projectile vomiting in a 3 to 4 week-old baby. Hirschsprung disease (choice C), caused by absence of ganalion cels in the distal bowel, is usually diagnosed in the first few days of lie when there is a failure to pass meconim. Volrulas due to malrotation (choice E) in the newbcm presents as bowel obstmction ‘with emesis, which maybe bilious With continued obstruction, compromised blood flow may produce signs of bowel ischemia, ‘Marke this question << => Question Ta : 84667 Question 11 of 30 4.25 year old female gives hirth to an apparently normal male. He begns feeding well by the second day, then at 6 months, suddenly develops colicky abdominal pain and vomiting, Ibis determined that he has gasirointestinal obstruction, The most iicely cause of this presentaton is 2) Adhesions ) Congenital pyloric stenosis ©) Hirschsprung disease Y © 4) Intussusception ©) Volvulus due to matroration Question Explanation: ‘All of the concitions listed can cause gastrointestinal obstruction, but the clinical presentation is most suggestive of innussusception, Tntussusception is the most common cause of gastrointestinal obstruction in children 3 months to 6 years (peak age is between 5 and 10 months), Tn intussusception there is telescoping cf one bowel segmentinto another, more distal segment. Tt occurs due to the pace support offtred ty the bowel sections thin mesentery. Intussusception produces intestinal obstruction, and it may produce bowel ischemia or infarction by trapping mesenteric vessels along with the affected segment. In some cases, the mntussusception maybe reduced by diagnostic barium enema “Adhesions (choice A) can cause bowel obsiruction following surgery or inflammatory bowel diseases Congenital pyloric stenosis (choice 3) typically presents as projectile vomiting in a 3 to 4 week-old baby. Hirschsprung disease (choice C), caused by absence of ganalion cels in the distal bowel, is usually diagnosed in the first few days of lie when there is a failure to pass meconim. Volrulas due to malrotation (choice E) in the newbcm presents as bowel obstmction ‘with emesis, which maybe bilious With continued obstruction, compromised blood flow may produce signs of bowel ischemia, Mark this question = => Question Id : 85013, Question 12 of 30 A.56 year old anemic woman is found have a vitamin B12 level of $5 pmol/L (normal is> 150 pmol/L). Antibodies to intrinsic factor ate identified. Levels of all other essential vitamins are within normal limits. The most likely associated condition is which of the following? a) Angiodysplasia ) Atrophic gastitis c) Dietary vitamin B12 deficiency 4) Duodenal ulcer €) Menetrier disease Ulcerative coltis Question Explanation: An important cause of vitamin B12 deficiency is pemicious anemia, an autoimmune disease associated with atrophic gastritis. In atrophic gastritis, the gastric epithelia undergoes intestinal metaplasia, thereby replacing gestric chief and parietal cells with geblet cells. The metaplastic epithelinm produces insufficient intrinsic Factor to bind the dietary vitamin B12. Megaloblastic anemia develops Report An Error over a number of years “Angiodysplasia (Choice A) is a disease of intermitent lower intestinal bleeding due to rupnure of mucosal and submucosal venules and. cagilleries under normal colonic wall tension, Angiodysplasia may produce iron deficiency anemia. Ih dietary vitamin B12 deficiency (choice C) antibodies to intrinsic factor would not be present. Duodenel ulcers (choice D) may also produce iron deficiency anemia due to chronic blood loss. They are caused by damage to the duodenal epithelium by gastric acids and are associated with cirthosis, COPD, chronic renal failure, and hypesparaihyroidism, ‘Menettier disease (choice E) is also known as hypertrophic gestropathy. Itis an sdiopathic condition typitied by thickened gastric mugae and hyperplastic mucosa, producing a thickened gastric wall, hypochlorhydria, epigastric distress, and protein losing gastroenterapathy. [Tleerative colitis (choice F) is an inflammatory bowel disease almost entirely restricted to the large intestine. Tt usually causes intestinal distress and diarshea, and it may produce a malabsorption syndrome that includes vitamin B12 deficiency. Intrinsic factor secretion is normal in ulcerative colitis Mark this question = => Question Id : 85013, Question 12 of 30 A.56 year old anemic woman is found have a vitamin B12 level of $5 pmol/L (normal is> 150 pmol/L). Antibodies to intrinsic factor ate identified. Levels of all other essential vitamins are within normal limits. The most likely associated condition is which of the following? a) Angiodysplasia Y © b) trophic zastitis c) Dietary vitamin B12 deficiency 4) Duodenal ulcer €) Menetrier disease Ulcerative coltis Question Explanation: An important cause of vitamin B12 deficiency is pemicious anemia, an autoimmune disease associated with atrophic gastritis. In atrophic gastritis, the gastric epithelia undergoes intestinal metaplasia, thereby replacing gestric chief and parietal cells with geblet cells. The metaplastic epithelinm produces insufficient intrinsic Factor to bind the dietary vitamin B12. Megaloblastic anemia develops Report An Error over a number of years “Angiodysplasia (Choice A) is a disease of intermitent lower intestinal bleeding due to rupnure of mucosal and submucosal venules and. cagilleries under normal colonic wall tension, Angiodysplasia may produce iron deficiency anemia. Ih dietary vitamin B12 deficiency (choice C) antibodies to intrinsic factor would not be present. Duodenel ulcers (choice D) may also produce iron deficiency anemia due to chronic blood loss. They are caused by damage to the duodenal epithelium by gastric acids and are associated with cirthosis, COPD, chronic renal failure, and hypesparaihyroidism, ‘Menettier disease (choice E) is also known as hypertrophic gestropathy. Itis an sdiopathic condition typitied by thickened gastric mugae and hyperplastic mucosa, producing a thickened gastric wall, hypochlorhydria, epigastric distress, and protein losing gastroenterapathy. [Tleerative colitis (choice F) is an inflammatory bowel disease almost entirely restricted to the large intestine. Tt usually causes intestinal distress and diarshea, and it may produce a malabsorption syndrome that includes vitamin B12 deficiency. Intrinsic factor secretion is normal in ulcerative colitis Merk this question & => Question Td : 87583 Question 13 of 30 A 65 year old man complains of mild weight loss, constipation, fatigue, and shortness of breath. He also has cough and a mild fever. Blood studies indicate iron deficiency anemia, A stool guaiac testis positive. Chest X-ray reveals a moderate sized pleural effision is noted and subsequently tapped. Analysis of the fluid reveals increased lactate dehycrogenase levels and carcinoembryonic antigen ‘The most likely diagosis is a) Bronchioloalveolar carcinoma b) Empyema secondary to bacterial pneumonia ©) Left-sided heart failure 4) Mesothelioma ©) Metastatic colon cancer Question Explanation: Several sgns should alert the clinician to the possibilty of both colon cancer and #s metastatic nature, Whenever anernia is encountered in a middie aged or elderly patient, a neoplasm of the colon must be considered. In this case, the patient has occult blood in the stool The change in bowel habits is also of concem. Stool in the ascending colon is more liquid in neturc and therefore is notimpeded by an intraluminal mass. In the descending colon, a mass can impede amore solid stocl, leading to the characteristic "pencil stool." The metastatic nature of the cancer can be assessed by the high lactate dehydrogenase levels in the effusion, but more importantly by elevated levels of the tumor maser. Primary lung cancer may pretent with a malignant effusion; however, CEA would not be elevated and such a primary cancer would not explain the patient's intestinal symptoms, Inflammatory and infectious conditions could lead to increased LDH but not CEA. In left sided heert failure the effusion is 2 transudate that does not have increased levels of either LDH or CEA. A primary neoplasm of the pleura may lead to a malignart effusion; however, CEA is not elevated in mesothelioma, Merk this question & => Question Td : 87583 Question 13 of 30 A 65 year old man complains of mild weight loss, constipation, fatigue, and shortness of breath. He also has cough and a mild fever. Blood studies indicate iron deficiency anemia. A. stool guaiac testis positive. Chest X-ray reveals a moderate sized pleural effusion is noted and subsequently tapped. Analysis of the faid reveals increased lactate dehycrogenase levels and carcinoembryonic antigen. ‘The most likely diagosis is ) Bronchioloalveolar carcinoma b) Empyema secondary to bacterial pneumonia ©) Left-sided heart failure 4) Mesothelioma © &) Metastatic colon cancer Question Explanation: Several sgns should alert the clinician to the possibilty of both colon cancer and #s metastatic nature, Whenever anernia is encountered in a middie aged or elderly patient, a neoplasm of the colon must be considered. In this case, the patient has occult blood in the stool The change in bowel habits is also of concem. Stool in the ascending colon is more liquid in neturc and therefore is notimpeded by an intraluminal mass. In the descending colon, a mass can impede amore solid stocl, leading to the characteristic "pencil stool." The metastatic nature of the cancer can be assessed by the high lactate dehydrogenase levels in the effusion, but more importantly by elevated levels of the tumor maser. Primary lung cancer may pretent with a malignant effusion; however, CEA would not be elevated and such a primary cancer would not explain the patient's intestinal symptoms, Inflammatory and infectious conditions could lead to increased LDH but not CEA. In left sided heert failure the effusion is 2 transudate that does not have increased levels of either LDH or CEA. A primary neoplasm of the pleura may lead to a malignart effusion; however, CEA is not elevated in mesothelioma, Mark this question = => Question Td : 92034 Question 14 of 30 A 35 year old alcoholic male presents with severe abdominal pain, nausea and vomiting. He feels slightly better lying in the fetal postion on his let side. On examination, his bowel sounds were diminished and his abdomen was diffusely tender. Cullen's sign was peecent. Lab data reveals elevated lipase and amylaze levele in the cerum. Which of the following is not associated with poor prognosis of this disordes? 2) Lactate debydrogenase level of 114 ') Hypocalcemia, ©) Hypoxenia 4) A fallin the hematoseit greater than 10% 6) Ascites Question Explanation: Lactate dehycrogenase above 200 is associated with a poor prognosis according to Ransen’s criteria for pancreatitis. Hypocalcemia (@) is associated with a poor prognosis. Saponification of the pancreatic enzymes and calcium deposits occur. An oxygen concentration less than 60 mm Hg (C) is associated with a poor prognosis. The patient may need to be intubated Blood loss with a decrease in hematocrit (D) is associated with a poor prognosis. Third spacing and ascites fluid (B) ic associated with decrease in perfusion and a poor prognosis, Mark this question = => Question Td : 92034 Question 14 of 30 A 35 year old alcoholic male presents with severe abdominal pain, nausea and vomiting. He feels slightly better lying in the fetal postion on his let side. On examination, his bowel sounds were diminished and his abdomen was diffusely tender. Cullen's sign was peecent. Lab data reveals elevated lipase and amylaze levele in the cerum. Which of the following is not associated with poor prognosis of his disordes? Y © 2) Lactate debydrogenase level of 114 ') Hypocalcemia, ©) Hypoxenia 4) A fallin the hematoseit greater than 10% 6) Ascites Question Explanation: Lactate dehycrogenase above 200 is associated with a poor prognosis according to Ransen’s criteria for pancreatitis. Hypocalcemia (@) is associated with a poor prognosis. Saponification of the pancreatic enzymes and calcium deposits occur. An oxygen concentration less than 60 mm Hg (C) is associated with a poor prognosis. The patient may need to be intubated Blood loss with a decrease in hematocrit (D) is associated with a poor prognosis. Third spacing and ascites fluid (B) ic associated with decrease in perfusion and a poor prognosis, Mark this question & => Question 15 of 30 High cietary ingestion of which one of the following is related to development of gesttic carcinoma? a) Benzene ) Nitrate ¢) Arsenic & Alcohol ) Fiber Answer | Boplanation Other User's Explanation Report An Error Question Explanation: ‘A. correlation has been found between the ingestion of high concentrations of nitrates, which can be converted to carcinogenic nitrites, end gastric cancer. Benzene exposure appears to be related to some hematologic malignancies, but not to gastric cancer. Arsenic is not associated with gastric cancer. Alcohol may be associated with a slightly increased rick: for gastric cancer, butts relationship ie much less than nitrates. Fiber is thought to be beneficial in preventing colon cancer and is not related to gastric cancer. Mark this question & => Question 15 of 30 High cietary ingestion of which one of the following is related to development of gestric carcinoma? ) Benzene Y © 6) Nitrate ¢) Arsenic 4) Alcokol ) Fiber Answer | Boplanation Other User's Explanation Report An Error Question Explanation: ‘A. correlation has been found between the ingestion of high concentrations of nitrates, which can be converted to carcinogenic nitrites, end gastric cancer. Benzene exposure appears to be related to some hematologic malignancies, but not to gastric cancer. Arsenic is not associated with gastric cancer. Alcohol may be associated with a slightly increased rick: for gastric cancer, butts relationship ie much less than nitrates. Fiber is thought to be beneficial in preventing colon cancer and is not related to gastric cancer. ‘Marke this question & => Question Td : 93854 Question 16 of 30 £55 year old male with a history of glomerulonephrtis is diagnosed with renal faitore. He subsequently complains of heartburn and nausee, and states thathe has been vomiting each meming for the last few days. The type of gastritis that would most licely be found in his patient is which of the following? 8) Acute gastritis ) Chronic antral gastritis ©) Chronic findal gstitis 6) Hypertrophic gastritis ©) Lymphocytic gastritis Anewor (UEXVIRISHAR) omer sors Explanation Report An Error Question Explanation: “Acute gastritis, characterized by patches of erythematous mucora, scmetimes with petechiae and ulceration, can he seen as a complication of a variety of other condtions (alsohol use, use of aspisin and other NSAIDs, smoking, shock, steroid use, and emia), hich usually have in common disruption of the mucosal banter ofthe stomach, Chronic antral gastritis is associeted with. Helicobacter pylori. Chronic fiindal gastritis is the type associated with pemicious anemia, Hypertrophic gastrtis (Menetrier disease) is an idiopathic condition characterized by markedly enlarged mucosal folds. Lymphocytic gastritis is thought to be a gastric manifestation of celiac sprue. ‘Marke this question & => Question Td : 93854 Question 16 of 30 £55 year old male with a history of glomerulonephrtis is diagnosed with renal faitore. He subsequently complains of heartburn and nausee, and states thathe has been vomiting each meming for the last few days. The type of gastritis that would most licely be found in his patient is which of the following? JV © a@) Acute gastritis +) Chronic antral gastritis ©) Chronic fimdal gashitie 6) Hypertrophic gastritis ©) Lymphocytic gactrtis Anewor (UEXVIRISHAR) omer sors Explanation Report An Error Question Explanation: “Acute gastritis, characterized by patches of erythematous mucora, scmetimes with petechiae and ulceration, can he seen as a complication of a variety of other condtions (alsohol use, use of aspisin and other NSAIDs, smoking, shock, steroid use, and emia), hich usually have in common disruption of the mucosal banter ofthe stomach, Chronic antral gastritis is associeted with. Helicobacter pylori. Chronic fiindal gastritis is the type associated with pemicious anemia, Hypertrophic gastrtis (Menetrier disease) is an idiopathic condition characterized by markedly enlarged mucosal folds. Lymphocytic gastritis is thought to be a gastric manifestation of celiac sprue. ‘Marke this question & => Question Td : 100785 Question 17 of 30 A patient develops persistent Macrocytic anemia. The patient is experiencing symmetric paresthesias of the hands and feet Which of the following organ’s cancer is most strongly associated with this patient's condition? a) Colon ) Duodenum c) Esophagus Teun 8) Stomach Question Explanatior Megaloblastic anemia with neuropathy suggests vitamin B12 deficiency, which in tim suggests pemicions anemia. In this condition, an autoimmune attack on gastric panetal cells leads to atrophic gestritis with deficient synthesis of the intrinsic factor nesded for vitamin B12 absorption, Vitamin B 12 is required for DIVA synthesis in blood cell precursors; deficiency produces megaloblasiic anemia, Chronic atrophic gastritis predisposes an individual to the development of gastric carcinoma Predisposing factors for colon cancer include familial polyposis syndromes, inflammatory bowel disease, isolated adenomatous polyps, and diet. Duodenal andileal cancers are uncommon. Predisposing factors for esophageal cancer include Barrett esophagus and Plummer Vinson syndrome. ‘Marke this question & => Question Td : 100785 Question 17 of 30 A patient develops persistent Macrocytic anemia. The patient is experiencing symmetric paresthesias of the hands and feet Which of the following organ’s cancer is most strongly associated with this patient's condition? a) Colon ) Duodenum c) Esophagus Teun ¥ © 2) Stomach Question Explanatior Megaloblastic anemia with neuropathy suggests vitamin B12 deficiency, which in tim suggests pemicions anemia. In this condition, an autoimmune attack on gastric panetal cells leads to atrophic gestritis with deficient synthesis of the intrinsic factor nesded for vitamin B12 absorption, Vitamin B 12 is required for DIVA synthesis in blood cell precursors; deficiency produces megaloblasiic anemia, Chronic atrophic gastritis predisposes an individual to the development of gastric carcinoma Predisposing factors for colon cancer include familial polyposis syndromes, inflammatory bowel disease, isolated adenomatous polyps, and diet. Duodenal andileal cancers are uncommon. Predisposing factors for esophageal cancer include Barrett esophagus and Plummer Vinson syndrome. Mark this question = => Question Id : 104934 Question 18 of 30 A.43 yeer cld man with a history of chronic alcoholintake presents with diarrhea, weightloss, and malnutrition, He has bulky, greasy, excessively malodorous stools that float, Qualitative stool fat showed mmerous fat globules per high powered field, The seram carotene level was decreased. The most likely diagnosis is 2) Pemicious anemia 1b) Crohn's disease ©) Celiac sprue 4) Fat malabsorption secondary to pancreatic insufficiency 6) Infectious gastroenteritis Question Explanation: Chronic pancreatic insufficiency secondary to chrotic alcohol use is one of the most common causes of fat malabsorption. The secretin test and betiromide unnery excretion test can diagnose this condltion, Abdominal X-rays would show pancreatic calcifications. Pemicious anemia results in vitamin B12 malabsorption in the ileum This can resuttin severe neurologic impaiment if the condltion is not corrected, Crokn's disease can also result in fat malabsorption, but tke patient would have chronic, reucous ciasthea initially Alcoholism does not cause Crohn's disease. Celiac sprue is a disease of unknown etiology characterized by malabsorption resubing fiom a gliten-indaced damage to the cifferentiated villus epithelia cells ofthe small intestine. In this patient, he trad no known intake of gluten or wheat that would result in steatorrheal stool. Infectious gastroenteritis would usually present initially with fever and atdeminal pain. It can also cause melabsception by bactorial overgrowth ia the intestines. Absorption of Vitamia A (carotene) will be decreased as itis a fat soluble vitamin resulting in decreased levels Mark this question = => Question Id : 104934 Question 18 of 30 A.43 yeer cld man with a history of chronic alcoholintake presents with diarrhea, weightloss, and malnutrition, He has bulky, greasy, excessively malodorous stools that float. Qualitative stool fat showed mamerous fat globules per high powered field. The seramn carotene level was decreased. The most itely diagnosis is 2) Pemicious anemia ) Crohn's diceace ©) Celiac sprue Y © 4) Fat malabsorption secondary to pancreatic insufficiency. 6) Infectious gastroenteritis Question Explanation: Chronic pancreatic insufficiency secondary to chrotic alcohol use is one of the most common causes of fat malabsorption. The secretin test and betiromide unnery excretion test can diagnose this condltion, Abdominal X-rays would show pancreatic calcifications. Pemicious anemia results in vitamin B12 malabsorption in the ileum This can resuttin severe neurologic impaiment if the condltion is not corrected, Crokn's disease can also result in fat malabsorption, but tke patient would have chronic, reucous ciasthea initially Alcoholism does not cause Crohn's disease. Celiac sprue is a disease of unknown etiology characterized by malabsorption resubing fiom a gliten-indaced damage to the cifferentiated villus epithelia cells ofthe small intestine. In this patient, he trad no known intake of gluten or wheat that would result in steatorrheal stool. Infectious gastroenteritis would usually present initially with fever and atdeminal pain. It can also cause melabsception by bactorial overgrowth ia the intestines. Absorption of Vitamia A (carotene) will be decreased as itis a fat soluble vitamin resulting in decreased levels ‘Mark thie question & => Question Td : 107627 Question 19 of 30 4.22 year old man while working in Aftica complains that his urine is amber, his stools gray, and he has a yellow tnge to his skin and eyes. He has fatigue and anorexia, His liver finction tests were elevated. Ths recommended treatmeat inthis patient is a) IV antibiotics ) Immunoglobulins c) Rest 6) Interferon 8) Steroids Question Explanation: This patient has hepatitis A, which is an RNA virus transmitted via the fecel-oral route, This patients in an endemic area to ge this isease. Tho only remedy is to encourage increases in liquid intalee and rest. The symptoms will eventually go away. There is now a vaccination for hepatitis A. IV antibiotics would not be needed in this viral infection. There is no evidence of a bacterial infection, Immunoglobulins may be given in acute hepatitis B, but only ifthe dose was given within a few days after symptom onset Alpha interferon is the deug of choice in shronic hepatitis B. Thic patient chows no evidence of chroricity. Steroids would be contraindicated and would actually cause the hepatitis to become more fulninant. ‘Mark thie question & => Question Td : 107627 Question 19 of 30 (A.22 year old man while working in Aftica complains that his urine is amber, his stool is gray, and he has a yellow tinge to his skin and eyes. He has fatigue and anorexia. His liver function tests were elevated. The recommended treatment in this patient is a) IV antbioties bv) Immunoglobulins YO ORest ¢) Interferon. 6) Steroids Question Explanation: This patient has hepatitis A, which is an RNA virus transmitted via the fecel-oral route, This patients in an endemic area to ge this isease. Tho only remedy is to encourage increases in liquid intalee and rest. The symptoms will eventually go away. There is now a vaccination for hepatitis A. IV antibiotics would not be needed in this viral infection. There is no evidence of a bacterial infection, Immunoglobulins may be given in acute hepatitis B, but only ifthe dose was given within a few days after symptom onset Alpha interferon is the deug of choice in shronic hepatitis B. Thic patient chows no evidence of chroricity. Steroids would be contraindicated and would actually cause the hepatitis to become more fulninant. 3i2/2014 5: 225 PM. Mark this question —& => Question Td : 120200 Question 20 of 30 “What is the most common stnall bowel malignancy? @) Lymphoma, ) Carcinoid ©) Adenocarcinoma 4) Leiomyosarcoma, ¢) Pheochromocytoma. Answer | Explanation Other User's Explanation Report An Error Question Explanatio Metastatic adenocarcinoma causes approximately 75% of small bowel cancers. Primary small bowel cancers include: adenocarcinoma, carcinaid, and lymphoma Pheochromacytamas are umually of the adrenal medulla, Leiomyosarcomais a rare ‘malignant smooth muscle tuner, generally originating in the utsrus or relzoperitoneal space. 3i2/2014 5: 225 PM. Mark this question —& => Question Td : 120200 Question 20 of 30 ‘What is the most common small bowel malignancy? 2) Lymaphoma ) Carcinoid Y © 0) Adenocarcinema. 4) Leiomyecarcoma ¢) Pheochromocytoma. Answer | Explanation Other User's Explanation Report An Error Question Explanatio Metastatic adenocarcinoma causes approximately 75% of small bowel cancers. Primary small bowel cancers include: adenocarcinoma, carcinaid, and lymphoma Pheochromacytamas are umually of the adrenal medulla, Leiomyosarcomais a rare ‘malignant smooth muscle tuner, generally originating in the utsrus or relzoperitoneal space. ‘Mark this question & => Question Td : 136482 Question 21 of 30 ‘Which one of the following clinical sign may NOT be caused by malabsorption? a) Purpura b) Kerophthalmia ©) Tetany ) Hypertension. ¢) Follicular hyperkeratosis. Question Explanation: Dehydration and hypotension may result from malabsorption of water and electrolytes. Dermatologic findings in malabsorption inchide follicular hypericeratosis and dermatitis secondary to deficiency of vitamin A. zinc, and other essential fatty acids. Purpura may result ftom vitamin K. deficiency. Malebsorption of calcium and magnesinm are associated with tetany. Xerophthalmia and night blindness are associated with vitamin A deficiency. ‘Mark this question & => Question Td : 136482 Question 21 of 30 “Which one of the following clinical sign may NOT be caused by malabsorption? a) Purpura +b) Xetophthalmia. o) Tetany. Y¥ © & Hypertension. #) Follicular hyperkeratosis Question Explanation: Dehydration and hypotension may result from malabsorption of water and electrolytes. Dermatologic findings in malabsorption inchide follicular hypericeratosis and dermatitis secondary to deficiency of vitamin A. zinc, and other essential fatty acids. Purpura may result ftom vitamin K. deficiency. Malebsorption of calcium and magnesinm are associated with tetany. Xerophthalmia and night blindness are associated with vitamin A deficiency. '3i2/2014 5:32:55 PM Mark this question & => Question Id : 142056 Question 22 of 30 A.23 year cld febrile student presents with fatigue and difficulty swallowing, Examination shows eiudative tonsils and cervical Iymphocytosis, with about 20% cf the lymphocytes exhibiting atypical features, ac well as mild thrambceytaperia, Serum sample causes the agglutination of sheep red blood sells. What addtional finding is most likely to be present? 2) Antibodies thet aggiutinate autologous erythrocytes at4°C +) Antibodies thet agglutinate cardiclipin ©) Antibodies that aggiutinate Proteus OX strain 6) Antibodies to Epstein Barr nuclear antigens ©) Antibodies to the Paul Bunnell antigen Question Explanation: ‘The syndrome reoresented by this cinical vignette is infectious mononucleosis. Epstein Barr virus (EBV) is the cause of heterophile postive infectious mononucleosis, cytomegalovirus is responsible for heterophile negative mononucleosis. Diagnosis is made by the finding of heterophile entibodies (antibodies that agglutinaie animal erythrocytes by binding to the Paul Bunnell antigen) and by a complete blood count, which may reflect up to 70% lymphocytosis. Antibodies that agelutinate autologous erythrocytes at 4°C are also known as cold agglutinins They are detected in 65% of patients with Mycoplasma pneumoniae, but are not bound in mononucleosis, Antibodies that agghutinate cardiolipin are the basis of the VDEL (Venereal Disease Research Lab) test for syphilis but are not found in mononucleosis. Antibodies that agalutinate Proteus OX strain would be detected with the Weil-Felix reaction which is used to, diagnose rickettsial infections Antibodies to Epstein Basr nuclear antigen are not typically elevated in the acute phase of mononucleosis. These antibodies tend to rise after acute disease and may reflect an association between the viras and an observed malignancy. Report An Error '3i2/2014 5:32:55 PM Mark this question & => Question Id : 142056 Question 22 of 30 A. 23 year old febrile student presents with fatigue and dificulty ewallowing, Examination shows exudative tonsllts and cervical lymphocytosis, with about 30% of the lymphocytes exhibiting atypical features, as well as mild thrombocytopenia. Serum sample causes the agglutination of sheep red blood cells. What additional finding is most likely to be present? 2) Antibodies thet aggiutinate autologous erythrocytes at4°C ‘b) Antibodies that aggiutinate cardiolipin ©) Antibodies that aggiutinate Proteus OX strain ¢) Antibodies to Epstein Barr auclear antigens Y © ©) Antibodies to the Paul Bunnell antigen. Question Explanation: ‘The syndrome reoresented by this cinical vignette is infectious mononucleosis. Epstein Barr virus (EBV) is the cause of heterophile postive infectious mononucleosis, cytomegalovirus is responsible for heterophile negative mononucleosis. Diagnosis is made by the finding of heterophile entibodies (antibodies that agglutinaie animal erythrocytes by binding to the Paul Bunnell antigen) and by a complete blood count, which may reflect up to 70% lymphocytosis. Antibodies that agelutinate autologous erythrocytes at 4°C are also known as cold agglutinins They are detected in 65% of patients with Mycoplasma pneumoniae, but are not bound in mononucleosis, Antibodies that agghutinate cardiolipin are the basis of the VDEL (Venereal Disease Research Lab) test for syphilis but are not found in mononucleosis. Antibodies that agalutinate Proteus OX strain would be detected with the Weil-Felix reaction which is used to, diagnose rickettsial infections Antibodies to Epstein Basr nuclear antigen are not typically elevated in the acute phase of mononucleosis. These antibodies tend to rise after acute disease and may reflect an association between the viras and an observed malignancy. Report An Error ‘Marle this question & => Question Td : 161175 Question 23 of 30 A.57 year old mae is brought with severe abdominal pain that began shorty after taking a drug for GERD. On exam he almost has total bowel obstruction secondary to the presence of several small bowel tumors. Which dmg was likely taken? a) Antacids ) Famotidine c) Lansoprazole 6) Metoclopramide ©) Sucralfate Question Explanation: Medications, liquds, and food products should be judiciously ingested by individuals with any degree of bowel cbsiruction, because of the risk of worsening the obstruction or causing perforation of the mtestinal tract As a general rule, the administration of any product that stimulates the intestinal tract is contrandicated in patients with bowel obstruction. These products can actualy promote an intestinal tract perforation secondary to the buildup of pressure behind the obstruction, Since metoclopramide is a prokinetic agent that stimulates the intestinal tract, the use of this agent is absolutely contraindicated in patients with a bowel obstruction, The use of antacids may be acceptable for these patients. However, certain antacids can cause constipation and may worsen the degree of obstruction, Agents that suppress acid secretion, such es famotidine and lansoprazole are considered safe in patients with bowel cbstruction. Sucralfate is generally not used in patients with a partial bowel obstruction since it can lead to the development of constipation ‘Marle this question & => Question Td : 161175 Question 23 of 30 A.57 year old mae is brought with severe abdominal pain that began shorty after taking a drug for GERD. On exam he almost has total bowel obstruction secondary to the presence of several small bowel tumors. Which dmg was likely taken? a) Antacids ) Famotidine c) Lansoprazole Y © 8 Metoclopramide ©) Sucralfare Question Explanation: Medications, liquds, and food products should be judiciously ingested by individuals with any degree of bowel cbsiruction, because of the risk of worsening the obstruction or causing perforation of the mtestinal tract As a general rule, the administration of any product that stimulates the intestinal tract is contrandicated in patients with bowel obstruction. These products can actualy promote an intestinal tract perforation secondary to the buildup of pressure behind the obstruction, Since metoclopramide is a prokinetic agent that stimulates the intestinal tract, the use of this agent is absolutely contraindicated in patients with a bowel obstruction, The use of antacids may be acceptable for these patients. However, certain antacids can cause constipation and may worsen the degree of obstruction, Agents that suppress acid secretion, such es famotidine and lansoprazole are considered safe in patients with bowel cbstruction. Sucralfate is generally not used in patients with a partial bowel obstruction since it can lead to the development of constipation Marle this question & => Question Td : 169904 Question 24 of 30 A. 60 yese eld male has left leg pain and weight lees. On exer, unilateral erythema, swelling, and warmth are noted over the left leg. On ultrasound day his lef leg is no longer painful but his right leg is swollen, Ubrasound shows a deep venous thrombosis in the right leg, but no clot inthe left leg. What underlying condition is the tkely cause of lis presentation? 2) Diverticulitis ) Pancreatic carcinoma ¢) Progressive systemic sclerosis &) Rheumatoid arthritis ©) Schistosomiasis Question Explanation: TTrousseau’s syndrome is the association of migratory thrombophlebitis (lot formation and dissolution secondary to tumor secretion of platelet aggregating factors and procoagulants) with cancer, traditionally pancreatic cancer. Weight loss and abdominal pain are also associated with this malignancy. Troussean diagnosed his own fatal pancreatic cancer this way. Lung cancer, colon cancer, and gastric cancer may also present with these symptoms. None ofthe other disorders listed is associated with migratory thrombophlebitis, n Report An Error Marle this question & => Question Td : 169904 Question 24 of 30 A. 60 yese eld male has left leg pain and weight lees. On exer, unilateral erythema, swelling, and warmth are noted over the left leg. On ultrasound day his lef leg is no longer painful but his right leg is swollen, Ubrasound shows a deep venous thrombosis in the right leg, but no clot inthe left leg. What underlying condition is the tkely cause of lis presentation? 2) Diverticulitis Y © 6) Pancreatic carcinoma ¢) Progressive systemic sclerosis &) Rheumatoid arthritis ©) Schistosomiasis Question Explanation: TTrousseau’s syndrome is the association of migratory thrombophlebitis (lot formation and dissolution secondary to tumor secretion of platelet aggregating factors and procoagulants) with cancer, traditionally pancreatic cancer. Weight loss and abdominal pain are also associated with this malignancy. Troussean diagnosed his own fatal pancreatic cancer this way. Lung cancer, colon cancer, and gastric cancer may also present with these symptoms. None ofthe other disorders listed is associated with migratory thrombophlebitis, n Report An Error Mark this question &= => Question Tad : 197260 Question 25 of 30 A31 year old Caucasian male has a six month history of weightloss, abdominal pain, and diarioea. On examination you note finger chnbbing The LEAST likely diagnosis is which one of the following? 2) Crohn’s disease ) Tleerative colitis ©) Cotliae disease 4) Whipple's disease 6) lec-caecal TB Question Explanatio Ieo-caecal TB is the only condition mentioned not associated with clubbing and would be very rare in a young Caucasian in the UK. Mark this question &= => Question Tad : 197260 Question 25 of 30 A31 year old Caucasian male has a six month history of weightloss, abdominal pain, and diarioea. On examination you note finger chnbbing The LEAST likely diagnosis is which one of the following? 2) Crohn’s disease ) Tleerative colitis ©) Cotliae disease 4) Whipple's disease © 6) leo-caecal TB Question Explanatio Ieo-caecal TB is the only condition mentioned not associated with clubbing and would be very rare in a young Caucasian in the UK. Mark this question = => Question Td : 197435 | Question 26 of 30 A. group construction workers presented with diarrhoea, Bushing, sweating and ahot mouth. They ate iunchin the staff canteen and fallill irutes after that. They admitted that they ate tuna ash and wine. The likely cause of food poisoring is which one of the following? a) Clostridium perfiingens ) Heavy metal ©) Mushroom 4) Scrombotenin €) Staphylococexs aureus Question Explanation: Sctombotoxin food poisoningis caused by the ingestion of foods that contan high levels of histamine and possbly other vasoactive amines and compounds. Histamine and other amines are formed by the growth of certain bacteria and the subsequent action of their decarboxylase enzymes on histidine and other amino acids in food, either during the production of a product such as Swiss cheese or by spoilage of foods such as fishery procucts, particularly tuna or mahi, Incubation period is 10-60 minutes. Report An Error Mark this question = => Question Td : 197435 | Question 26 of 30 A. group construction workers presented with diarrhoea, Bushing, sweating and ahot mouth. They ate iunchin the staff canteen and fallill irutes after that. They admitted that they ate tuna ash and wine. The likely cause of food poisoring is which one of the following? a) Clostridium perfiingens ) Heavy metal ©) Mushroom Y © 4) Scrombotonin «) Staphylococes aureus Question Explanation: Sctombotoxin food poisoningis caused by the ingestion of foods that contan high levels of histamine and possbly other vasoactive amines and compounds. Histamine and other amines are formed by the growth of certain bacteria and the subsequent action of their decarboxylase enzymes on histidine and other amino acids in food, either during the production of a product such as Swiss cheese or by spoilage of foods such as fishery procucts, particularly tuna or mahi, Incubation period is 10-60 minutes. Report An Error Mark this question & => Question Td : 197645 Question 27 of 30 A 54 year old woman has lethergy. diarrhea together with joint pain anc intermittent fever. These symptoms have developed over the 6 months during which time she has lost 6 keg in weight. Supraclavicular lymphadenopathy is noted. The most llcely diagnosis is which one ofthe following? a) Bacillary dysentery £) Campylobacter infection ©) Corliar disease 6) Giardliasis ©) Whipple's disease Anewor [UEIFIRIRHAN) otner usar Exptan Question Explanation: “Whipple’s disease is cauced by Tropheryma whippeli and eymaptoms include Report An Error * chronic diarrhoea aerate + pyrexia + lymphadenopathy. Diagnosis is by microscopy of jejunal biopsy specimen which shows macrophages with p-amninosalicylic acid (PAS) postive granules, Treatments co-trimoxazole Bacillarary dysentry and Campylebarter jejuni infection are characterised by bloody diarrhoea and are not chronic. Coeliac disease and giardiasis have no lymph involvement. Mark this question & => Question Td : 197645 Question 27 of 30 A 54 year old woman has lethergy. diarrhea together with joint pain anc intermittent fever. These symptoms have developed over the 6 months during whish time the has lost 6 kg in weight. Supraclavicular lymphadenopathy is noted, The moct licely diegnosis ic which one of te following? a) Bacillary dysentery +) Campylobacter infection ©) Coeliac disease 6) Giardiasis S © ©) Whpple‘s disease Anewor [UEIFIRIRHAN) otner usar Exptan Question Explanation: “Whipple’s disease is cauced by Tropheryma whippeli and eymaptoms include Report An Error * chronic diarrhoea aerate + pyrexia + lymphadenopathy. Diagnosis is by microscopy of jejunal biopsy specimen which shows macrophages with p-amninosalicylic acid (PAS) postive granules, Treatments co-trimoxazole Bacillarary dysentry and Campylebarter jejuni infection are characterised by bloody diarrhoea and are not chronic. Coeliac disease and giardiasis have no lymph involvement. Merk this question & => Question Td : 197798 Question 28 of 30 A 37 year old male has a three day history of bloody diarrhea, He was apyresial and mildly icteric. His Hb is 10.5 g/dL. WBC count is 19 2109/L, platelet count is 70 x109/L, serum urea is 12.5 mmol/L (2.5-7.5), Serum AST is 90 U/L (1-31) and PTis 12s (11.5. 15.5), Peripheral film shows fragmented red cells. The most licely cause of his ilmess is which one of the folowing? a) Escherichia col 0157 colts £) Ischaemic colic ©) Leptospirosis, 6) Salmonella enterocolitis @) Ulcerative colitis Question Explanation: The combination of bloody diarrhoea, haemolytic anaemia, thrombocytopaenia but normal clotting, and renal impairment suggests hacmolytic-uracmic syndrome, This is associated with E coli 0157 toxin most commonly. Merk this question & => Question Td : 197798 Question 28 of 30 A.37 year old male has a three day history of bloody diarrhea, He was apyrexial and mildly icteric, His Hb is 10.5 gidL., WBC count is 19 2109/1, platelet count is 70 x109/L, serum urea is 12.5 mmel/L (2.5-7.5), Serum AST is 90 U/L (1-31) and PT is 12s (11.5- 15.5). Peripheral film shows fragmented red cells. The most likely cause of his illness is which one of the folowing? Y © a) Escherichia coi 0157 cottis, ) Ischaemic colitic ©) Leptospirosis ) Salmonella enterocolitis 2) Ulcerative colitis Question Explanation: The combination of bloody diarrhoea, haemolytic anaemia, thrombocytopaenia but normal clotting, and renal impairment suggests hacmolytic-uracmic syndrome, This is associated with E coli 0157 toxin most commonly. Mark this question & => Question Td : 1995 Question 29 of 30 A 23 year old man presents 1 week after returing from a 6 month visit to Pakistan He has fever, rigors and headache. On exam he is febrile (38°C) with BP of 115/65 mmHg, and pulse of 10min. His abdomen is tender in the right upper quadrart. Hh is 11.0 gL, WBC count is 15.5 x109/L with acutrophils 13.5 x109L and platelet couat is 350 x109/L. Blood film is negative for melavial parasites. ALP, AST and CRP ate elevated, CCR shows small right pleural effusion, Which investigation would be of diagnostic value? a) Hopatiis E serology 6) Sigmoidoscopy ©) Stool microscopy for ova, cysts and parasites 4) Typhoid serology ¢) Ultrasound scan of the abdomen Avewor [UEIPISNSHIR) otnorUsore Explanation Report An Error Question Explanatio: The presentation is not consistent with hepatitis E infection Typhoid serology is unreliable. The differential diagnosis is mainly pyogenic or amoebic liver abscess. Pyogenic abscesses present with -swngng pyrexia ~ neatroplulia and - high inflamenstory macleers. Right sided pleural effusions are common and blood cukures are often positive ‘The presentation of amoebic liver abscess (ALA) is very similar Most patients do not have bowel syiaptoms at any time and amoebic cysts are found in stool in less than 50% of proven cases of ALA. Serology is the mainstay of diagnosis, Ultrasound scan would confirm most moderate-sized to large liver abscesses and could guide a diagnostic aspiration. Small lesions are dest demonstrated by computerized tomograpiy (CT) or magnetic resonance imaging (MRD). Mark this question & => Question Td : 1995 Question 29 of 30 A 23 year old man presents 1 week after returning fiom a 6 month visit to Pakistan, He has fever, rigors and headache. On exam he is febrile (38°C) with BP of 115/65 mmHg, and pulse of 100/min. His abdomen is tender in the right upper quadrart. Hb is 11.0 gL, WBC count is 15.5:x109/L with acutrophils 13.5 109/L and platelet coat is 350 x109/L. Blood film is negative for malavial parasites. ALP, AST and CRP are elevated. CAR shows small right pleural effusion. Which investigation would be of diagnostic value? a) Hepatitis E serology +) Sigmoidoscopy 6) Stool microscopy for eva, cysts and parasites 4) Typhoid serology Y © ¢) Ultrasound scan of the abdomen. Avewor [UEIPISNSHIR) otnorUsore Explanation Report An Error Question Explanatio: The presentation is not consistent with hepatitis E infection Typhoid serology is unreliable. The differential diagnosis is mainly pyogenic or amoebic liver abscess. Pyogenic abscesses present with -swngng pyrexia ~ neatroplulia and - high inflamenstory macleers. Right sided pleural effusions are common and blood cukures are often positive ‘The presentation of amoebic liver abscess (ALA) is very similar Most patients do not have bowel syiaptoms at any time and amoebic cysts are found in stool in less than 50% of proven cases of ALA. Serology is the mainstay of diagnosis, Ultrasound scan would confirm most moderate-sized to large liver abscesses and could guide a diagnostic aspiration. Small lesions are dest demonstrated by computerized tomograpiy (CT) or magnetic resonance imaging (MRD). 32/2014 5:34:53 PM. € ‘yew interface acu.ck/ x \ BE) aiqysSYl gall dls CL www. interface edu.pk/med nalysis pho ?utid=14759 #38 ingestion, Since there is no history of fish consumption in this patient along with the fact that his syrmptoms have been occursing for the last few months, this diagnosis is unicely. Apps EJ coogle A settings [) Sgnin g Con yo ell Bye OF .tio yt oral! IS Free Hotmail >» (© other bookmar ‘Mark this question <& Question Td: 217251] Total Questions 1 x Question 30 of 30 a. ae A S6-year-old man hae chronic cecretery diarvhea and episodes of duching for the last few months. He also reports 10 to 12 foul- 3.x smelling stools in a day. Stool cultures and a stool test are negetive. During Dushing, he appears dyspneic with wheezing. What nowis 4 the most appropriate test to confirm the diagnosis? 5 Xx 8) 24-hour urine for 5- hydroseyindoleacetic acid (5-HIAA) 2g se 'b) 24-hour rine for catecholamine and metanephrine 7x ©) ACTH stimulation test 2 x 4) Dexamethasone stimulation test > x ©) FSH and LH levels, 1 x £) No laboratory tesis are required since scombroid poisoningiis a clinical ckagnosis ux 2) Serum catecholamines 2 x Answer (Borat) Other User's Explanation —_ Report An Error as We Question Explanation: oc This patient has symptoms (cutaneous Flushing, darthea, and bronchospasm causing wheezing and dyspaea) to suggest carcinoid ao, ae syndrome. Tn patients with carcinoid symdeome there is altered metabolism of tryptophan. Normally, only about 1% of the tryptophan 16 is converted to serotonin, but in patients with carcinoid syndrome, this figure may be close to 70%. Patients with cercinoid syndrome 17x therefore have excessive production of serotonin, Serotonin is metabolzed to 5-tydroxyindoleacetis acid (S-HIAA) andeimnated = 55 via the urine, Measuring urinary 5-HIAA is diagnostic of carcinoid syndrome. IF this test is equivocal, measurement of serum aS serotonin is the best available biochemical marker for diagnosing carcinoid syndrome. yg x A.24 hour urine for catecholamine and metenephrins is used for detecting pheschremocytoms, which can present with episodes of x ‘Wheezing and diasthea is in this patient but will also present with headaches, palpitations, sweating, tachycardia, paroxysmal dh 3S hhypertension, etc. This patient's history lacks the majority of symptoms for pheochromocytoma, plus his blood pressure is nonmal 2» xX during an episode, pointing toward carcinoid syndrome as the most likely diagnosis Ss ‘The ACTH stimulation tect end dexamethasone suppression test are used for the diagnosis of adroral insufficiency and Cushirg 2 syndrome, respectively. x ESH and LH levels, as well as serum catecholamines, are not helpfi in the diagnosis of carcinoid syndrome 2 We To perform no laboratory studies because scombroid poisoning is clinical diagnosis is incorrect. Scombroid poisoningis related the = 296 X, consumption of certain types of marine fch (tune, mahimahi, mackerel, and bluefith). The cymaptoms occur within 15 minutes of eee x x + 28 23 a 32/2014 5:34:53 PM. € ‘yew interface acu.ck/ x \ BE) aiqysSYl gall dls CL www. interface edu.pk/med nalysis pho ?utid=14759 #38 ingestion, Since there is no history of fish consumption in this patient along with the fact that his syrmptoms have been occursing for the last few months, this diagnosis is unicely. Apps EJ coogle A settings [) Sgnin g Con yo ell Bye OF .tio yt oral! IS Free Hotmail >» (© other bookmar ‘Mark this question <& Question Td: 217251] Total Questions 1 x Question 30 of 30 a. ae A S6-year-old man hae chronic cecretery diarvhea and episodes of duching for the last few months. He also reports 10 to 12 foul- 3.x smelling stools in a day. Stool cultures and a stool test are negetive. During Dushing, he appears dyspneic with wheezing. What nowis 4 the most appropriate test to confirm the diagnosis? 5 Xx V © 8) 24-hour urine for 5- hydroseyindoleacetic acid (5-HIAA) 6 x 'b) 24-hour rine for catecholamine and metanephrine 7x ©) ACTH stimulation test 2 x 4) Dexamethasone stimulation test > x ©) FSH and LH levels, 1 x £) No laboratory tesis are required since scombroid poisoningiis a clinical ckagnosis ux 2) Serum catecholamines 2 x Answer (Borat) Other User's Explanation —_ Report An Error as We Question Explanation: oc This patient has symptoms (cutaneous Flushing, darthea, and bronchospasm causing wheezing and dyspaea) to suggest carcinoid ao, ae syndrome. Tn patients with carcinoid symdeome there is altered metabolism of tryptophan. Normally, only about 1% of the tryptophan 16 is converted to serotonin, but in patients with carcinoid syndrome, this figure may be close to 70%. Patients with cercinoid syndrome 17x therefore have excessive production of serotonin, Serotonin is metabolzed to 5-tydroxyindoleacetis acid (S-HIAA) andeimnated = 55 via the urine, Measuring urinary 5-HIAA is diagnostic of carcinoid syndrome. IF this test is equivocal, measurement of serum aS serotonin is the best available biochemical marker for diagnosing carcinoid syndrome. yg x A.24 hour urine for catecholamine and metenephrins is used for detecting pheschremocytoms, which can present with episodes of x ‘Wheezing and diasthea is in this patient but will also present with headaches, palpitations, sweating, tachycardia, paroxysmal dh 3S hhypertension, etc. This patient's history lacks the majority of symptoms for pheochromocytoma, plus his blood pressure is nonmal 2» xX during an episode, pointing toward carcinoid syndrome as the most likely diagnosis Ss ‘The ACTH stimulation tect end dexamethasone suppression test are used for the diagnosis of adroral insufficiency and Cushirg 2 syndrome, respectively. x ESH and LH levels, as well as serum catecholamines, are not helpfi in the diagnosis of carcinoid syndrome 2 We To perform no laboratory studies because scombroid poisoning is clinical diagnosis is incorrect. Scombroid poisoningis related the = 296 X, consumption of certain types of marine fch (tune, mahimahi, mackerel, and bluefith). The cymaptoms occur within 15 minutes of eee x x + 28 23 a Mark this question => Question Td : 21500 Question 1 of 30 4 54 year old white male smoker suffered from gastroesophageal refus symptoms unrslicved by intensive medical therapy with PPT A recent biopsy performed during upper endoscopy idemified Berren’s esophagus. Which one of the following is true about this, condition? 2) E will regress after antireflux surgery +) It will regress following esophageal dilation c) E will regress after Heliobacter pylori treatment 4) Ibis associated with increased risk of squamous cell carcinoma ©) Tis associated with an increased risk of adenocarcinoma Question Explanation: Barrett's esophagus is an acquired intestinal metaplasia of the distal esophagus associated with longstanding gastroesophageal acid reflux, although a quarter of patients with Barrett's esophagus have no refhus syraptems. It is more common in white and Hispanic men over 50 with loagstanding severe refiuz symptoms, and posable risle factors include obesity and tobacco use. Tis a sisk factor for adenocercinoma of the esophagus, with a rate of about one case in every 200 patients with Barrett's esophagus per year. ‘Treatment is drected at reducing redux, thus reducing symptoms. Neither medical nor surgical treatment has been shown to reduce the carcinoma risk. One reasonable screening suggestion is to perform esophagoduodenoscopy in all men over 50 with gastroesophageal refhiz disease (GERD), but these recommendatione are baced only on expert opinion (level C' evidence), and no outcomes-based guidelines are available. Report An Error Mark this question => Question Td : 21500 Question 1 of 30 4 54 year old white male smoker suffered from gastroesophageal refus symptoms unrslicved by intensive medical therapy with PPT A recent biopsy performed during upper endoscopy idemified Berren’s esophagus. Which one of the following is true about this, condition? 2) E will regress after antireflux surgery +) It will regress following esophageal dilation c) E will regress after Heliobacter pylori treatment 4) Ibis associated with increased risk of squamous cell carcinoma Y © 28) Kis associated with an increased risk of adenocarcinoma Question Explanation: Barrett's esophagus is an acquired intestinal metaplasia of the distal esophagus associated with longstanding gastroesophageal acid reflux, although a quarter of patients with Barrett's esophagus have no refhus syraptems. It is more common in white and Hispanic men over 50 with loagstanding severe refiuz symptoms, and posable risle factors include obesity and tobacco use. Tis a sisk factor for adenocercinoma of the esophagus, with a rate of about one case in every 200 patients with Barrett's esophagus per year. ‘Treatment is drected at reducing redux, thus reducing symptoms. Neither medical nor surgical treatment has been shown to reduce the carcinoma risk. One reasonable screening suggestion is to perform esophagoduodenoscopy in all men over 50 with gastroesophageal refhiz disease (GERD), but these recommendatione are baced only on expert opinion (level C' evidence), and no outcomes-based guidelines are available. Report An Error Marke this question << => Question Ta Question2 of 30 All of the following are true regarding pancreatic pseudocysts, except: a) Ibis caused by dact disruption ') Clinically suspected if persisting pain > 2.weeks following diagnosis of acute pancreatitis, €) Surgical intervention is typically delayed to allow pseudocyst to mature & Lacks tre epithelium €) Mejority are weated surgically Question Explanation: Tn chronic pancreatitis, a CT scan show calcifications and other pancreatic abnommnalities (eg, pseudocyst or dilated dusts) but still may be normal early in the disease. Surgical treatment may be effective for pain relief A pancreatic pseudocyst, which may cause chronic pain, can be decompressedinto a nearby structure to which itfirmly adheres (eg. the stomachs or into a defuunctionalized loop of jejummn) However, surgical approaches should be reserved for patients who have stopped using alcohol and who can manage diabetes that may be intensified by pancreatic resection. Marke this question << => Question Ta Question2 of 30 All of the following are true regarding pancreatic pseudocysts, except: a) Ibis caused by dact disruption ') Clinically suspected if persisting pain > 2.weeks following diagnosis of acute pancreatitis, €) Surgical intervention is typically delayed to allow pseudocyst to mature & Lacks tre epithelium Y © 6) Majority are weated surgically Question Explanation: Tn chronic pancreatitis, a CT scan show calcifications and other pancreatic abnommnalities (eg, pseudocyst or dilated dusts) but still may be normal early in the disease. Surgical treatment may be effective for pain relief A pancreatic pseudocyst, which may cause chronic pain, can be decompressedinto a nearby structure to which itfirmly adheres (eg. the stomachs or into a defuunctionalized loop of jejummn) However, surgical approaches should be reserved for patients who have stopped using alcohol and who can manage diabetes that may be intensified by pancreatic resection. ‘Marke this question => Question Ti : 48913 Question 3 of 30 AT] year old men complains of fatigue and a 10 kg weight loss for 3 months Physical exemination reveals jaundice without abdominal tendemess or organ enlargement. The moxt llcely diagnosis ic which of the following? a) Cholecystitis &) Pancreatitis c) Pancreatic carcinoma ) Hemochromatosis ©) Hepatic vein thrombosis Question Explanation: Tn pancreatic cancer, symptoms include weight loss, abdominal pain, and jaundice. Syinptoms occur late; by diagnosis, 90% of rratients have locally advanced tumors that have involved retroperitoneal structures, spread to regional lymph nodes, or-metastasized to the liver or lang. Most patients have severs upper abdominal pain, which usually radictes to the back. The pain may be relicved by bending forward, Weight loss is common, Adenocarsinomas of the head of the pancreas produce obstructive jauncice (often causing pruritus) in 80 to 90% of patients Diagnosis is by CT. Treatment is surgical resection and adjuvant chemotherapy and radiation therapy. Prognosis is poor because siseast is often advanced atthe time of diagnosis. Report An Error ‘Marke this question => Question Ti : 48913 Question 3 of 30 AT] year old men complains of fatigue and a 10 kg weight loss for 3 months Physical exemination reveals jaundice without abdominal tendemess or organ enlargement. The moxt llcely diagnosis ic which of the following? a) Cholecystitis &) Pancreatitis Y © 0) Pancreatic carcinoma ) Hemochromatosis ©) Hepatic vein thrombosis Question Explanation: Tn pancreatic cancer, symptoms include weight loss, abdominal pain, and jaundice. Syinptoms occur late; by diagnosis, 90% of rratients have locally advanced tumors that have involved retroperitoneal structures, spread to regional lymph nodes, or-metastasized to the liver or lang. Most patients have severs upper abdominal pain, which usually radictes to the back. The pain may be relicved by bending forward, Weight loss is common, Adenocarsinomas of the head of the pancreas produce obstructive jauncice (often causing pruritus) in 80 to 90% of patients Diagnosis is by CT. Treatment is surgical resection and adjuvant chemotherapy and radiation therapy. Prognosis is poor because siseast is often advanced atthe time of diagnosis. Report An Error 3i2/2014 5:36:25 PM Mark this question = => Question Ta : 49033 Question 4 of 30 Jn the management of patients with moderately severe uicerative colitis, which drugs or group of drugs is contraindicated? a) Antibiotics ) Hydrophilic bulking agents ¢) Corticosteroids 2) Oral Suifasalazine €) Nercotic antidiarrheal agents Question Explanation: Uloerative colitis (UC) is a chronic disease in which the large intestine becomes inflamed and ulcerated (pitted or eroded), leading to fare-ups (bouts or attaces) of bloody diarrhea, abdominal cramps, and fever. The long-term risk of colon cancers increased. Patients taking antidierrheal drugs must be closely monitored to avoid preciptating toxic megacolon. Toxic megacolon can be spontaneous in people with particularly severe UC. In some cases it can result from the overuse of certain drugs including narcotics, drugs used for pain relief, anticholinergics, drugs used for depression, anxiety, and nervousness: and antidiarsheals, such as loperamide Report An Error 3i2/2014 5:36:25 PM Mark this question = => Question Ta : 49033 Question 4 of 30 Jn the management of patients with moderately severe uicerative colitis, which drugs or group of drugs is contraindicated? a) Antibiotics ) Hydrophilic bulking agents ¢) Corticosteroids 2) Oral Suifasalazine Y © ¢) Netcotic antidiarrheal agents Question Explanation: Uloerative colitis (UC) is a chronic disease in which the large intestine becomes inflamed and ulcerated (pitted or eroded), leading to fare-ups (bouts or attaces) of bloody diarrhea, abdominal cramps, and fever. The long-term risk of colon cancers increased. Patients taking antidierrheal drugs must be closely monitored to avoid preciptating toxic megacolon. Toxic megacolon can be spontaneous in people with particularly severe UC. In some cases it can result from the overuse of certain drugs including narcotics, drugs used for pain relief, anticholinergics, drugs used for depression, anxiety, and nervousness: and antidiarsheals, such as loperamide Report An Error ‘Marke this question & => Question Td : 50689 Question 5 of 30 An uncommon infections cause of acute diarrhea is a) Escherichia coh ) Shigella ©) Norwalk virus 9) Vibsio cholerae ©) Helicobacter pylori Question Explanation: -Actte diarrhea can be caused by Escherichia coli, shizella, norwalk virus, vibrio cholerae. H. pyloriis a common gastric pathogen that causes gastritis, peptic ulcer disease, gastric adenocarcinoma, and low-grade gastric lymphoma ‘Marke this question & => Question Td : 50689 Question 5 of 30 An uncommon infections cause of acute diarrhea is a) Escherichia col b) Shigella ©) Norwalk virus @ Vibrio cholerae ¥ © ©) Helicobacter pylori Question Explanation: -Actte diarrhea can be caused by Escherichia coli, shizella, norwalk virus, vibrio cholerae. H. pyloriis a common gastric pathogen that causes gastritis, peptic ulcer disease, gastric adenocarcinoma, and low-grade gastric lymphoma ‘Mark this question & => Question Td : 54229 Question 6 of 30 ‘The medication that works well to discourage people with alcohol abuse from drinkingis a) Abily ) Disulfirara ) Fluoxetine 4) Clozapine Question Explanation: In the treatment of alcohol abuse, a rchabilitation program is often the best approach. Rehabilitation programs combine medical supervision and psychotherapy, including one-on-one and group therapy. Alcoholics Anonymous (AA) has benefited alcoholics more than any other approach. The patient must find an AA group in which he is comfortable, AA provides the patient with nondrinking friends who are always available and a nondrinking environment in which to socialize Drug therapy should be used in combination with psychotherapy. Disulfiram interferes with the metabolism of acetaldehyde (an intermediary product in the oxidation of alcohol) so thet acetaldehyde accumulates. Drinking alcohol within 12 hours of taking disuifiram produces facial flushing in 5 to 15 minutes, then intense vasodilation of the face and neck with suffusion of the conjunctivae, throbbing headache, tachycardia, hyperpnea, and sweating, With high doses of alcchol, nausea and vomiting may follow in 30 to 60 minutes and may leed to hypotension, dizziness, ond sometimes fainting and collapse. The reaction can last up to 3 hours. Few patients risk ingesting alcohol while taking disuifiram because of the intense discomfort. ‘Mark this question & => Question Td : 54229 Question 6 of 30 ‘The medication that works well to discourage people with alcohol abuse from drinkingis a) Abily Y © b) Disulfiram ) Fluoxetine 4) Clozapine Question Explanation: In the treatment of alcohol abuse, a rchabilitation program is often the best approach. Rehabilitation programs combine medical supervision and psychotherapy, including one-on-one and group therapy. Alcoholics Anonymous (AA) has benefited alcoholics more than any other approach. The patient must find an AA group in which he is comfortable, AA provides the patient with nondrinking friends who are always available and a nondrinking environment in which to socialize Drug therapy should be used in combination with psychotherapy. Disulfiram interferes with the metabolism of acetaldehyde (an intermediary product in the oxidation of alcohol) so thet acetaldehyde accumulates. Drinking alcohol within 12 hours of taking disuifiram produces facial flushing in 5 to 15 minutes, then intense vasodilation of the face and neck with suffusion of the conjunctivae, throbbing headache, tachycardia, hyperpnea, and sweating, With high doses of alcchol, nausea and vomiting may follow in 30 to 60 minutes and may leed to hypotension, dizziness, ond sometimes fainting and collapse. The reaction can last up to 3 hours. Few patients risk ingesting alcohol while taking disuifiram because of the intense discomfort. ‘Marke this question & => Question Ti : 62350 Question 7 of 30 Tn managing a patent with an acute GT bleed, the most sensitive indicator of decreased blood volume is a) Heart rate ) Venous distension ©) Orthostatic blood pressure. 4) Amount of hematemesis or melena e) Hematociit. Answer | Bplanation Other User's Explanation Report An Error Question Explanatior Orthostatic fallin blood pressure greater than 10 mmHg signifies a 20% or greater blood less and is the most sexsitve indicator of intravascular volume, Tachycardia accompanies hypotension, but alone itis an unreliable sign. Hematemesis or melena amounts may bbe misleading, Hematocrit can be umveliable as a fall in hematocrit following blood loss takes equilibration time. ‘Marke this question & => Question Ti : 62350 Question 7 of 30 Th managing a patient with an acute GT bleed, the most sensitive indicator of decreased blood volume is a) Heart rate. b) Venous distension. Y © c) Orthostatic blood pressure. d) Amount of hematemesis or melena. e) Hematocsit, Answer | Bplanation Other User's Explanation Report An Error Question Explanatior Orthostatic fallin blood pressure greater than 10 mmHg signifies a 20% or greater blood less and is the most sexsitve indicator of intravascular volume, Tachycardia accompanies hypotension, but alone itis an unreliable sign. Hematemesis or melena amounts may bbe misleading, Hematocrit can be umveliable as a fall in hematocrit following blood loss takes equilibration time. ‘Mark this question = => Question Ti : 62600 Question 8 of 30 AAT year old man complains of fatigue and shortness of breath Peripheral blood smear shows macrocytosis. Labs are as follows: Hematocsit: 32% (Normal 42%-52%) Hemoglobin 103 g/l. (Normal 140-174 gil) Serum vitarrin B12 level: 66pmoV/L (Normal 150-750 pmol/L) ‘Serum folate level: 16nmol/L (Wormal 4-22 nmo¥L) ‘The most unlikely cause is 8) Colonic diverticulitis b) Vegetarians ©) Regional enteritis 6) Pancreattis 8) Fish tapeworm infection Answer | Bavianation | Other User's Explanation Report An Error Question Explanation: ‘This patient has vitamin B12 deficiency related macrocytic anemia, Pemnicious anemiais a chronic ilness caused by impaired absorption of vitamin B12 because of @ lack of intrinsic factor (IF) in gastric secretions, Causes incinde inadequate dietary intake Cie, vegetariaa diet), chronic pancreatitic, tapeworm infestation with Diphylleb cthrium latum occur: from eating pootly cooked lake fish, surgical loss of the ileum or diseases such as tropical sprue, regional enteritis and ulcerative colitis ‘Mark this question = => Question Ti : 62600 Question 8 of 30 A.47 year old man complains of fatigue and shortness of breath, Peripheral blood smear shows macrocytosis. Labs are as follows Hematocrit: 32% (Normal 42%-52%) Femaglobin: 103 g/l (Nermsal 140-174 gf) Soruma vitarrin B 12 level: 66pemoVL (Normal 150-750 penol/L) Serum folate level 16nmel/L (Tonal 4-22 nmol/L) ‘The most unlikely cause is Y © a) Colonic diverticulitis 'b) Vegetarianism 6) Regional enteritis 6) Pancreattis 8) Fish tapeworm infection Answer | Bavianation | Other User's Explanation Report An Error Question Explanation: ‘This patient has vitamin B12 deficiency related macrocytic anemia, Pemnicious anemiais a chronic ilness caused by impaired absorption of vitamin B12 because of @ lack of intrinsic factor (IF) in gastric secretions, Causes incinde inadequate dietary intake Cie, vegetariaa diet), chronic pancreatitic, tapeworm infestation with Diphylleb cthrium latum occur: from eating pootly cooked lake fish, surgical loss of the ileum or diseases such as tropical sprue, regional enteritis and ulcerative colitis Mark this question = => Question Td : 63738 Question 9 of 30 A 22 year old college student presents with intermittent chest pain for about 6 months. He states that the pain can occur et any time, ‘out often occurs late in the day. The pain can be sharp, dall, or aching and may last for hours. He is physically active and is carrying a full academic load. The pain is not related to exercise and does not restrict his activities. His father and petemal uncle had myocardial infarchons within the past year Physical exemination is normal, akhough the patient appears restless and has a heart rate of 100/nin. The most likely diagnosis is a) Generalized anxisty disorder ) Reflux esophagitis ©) Transient infarction & Costochondriis Question Explanation: Tncompetence of the lower esophageal sphincter allows reflux of gastric contents into the esophagus, causing burning pain. Prolonged reflux may lead to esophagitis, stricture, and rarely metaplasia or malignancy. The most prominent symptom of gastro esophageal reflux disease (GERD) is hearibum, with or without regurgitation of gastric contents into the mouth. Infants present with vomiting, inntabilty, cnorezia, and sometimes symptoms of chronic aspiration. Both aduls and infants with chronic aspiration may have cough, hoarseness, or wheezing Diagnosis is clnical, sometimes with endoscopy, with or without acid testing. Treatment involves lifestyle modification, acid suppression using proton pump inhibitors, and sometimes surgical repair. Mark this question = => Question Td : 63738 Question 9 of 30 A 22 year old college student presents with intermittent chest pain for about 6 months. He states that the pain can occur et any time, ‘out often occurs late in the day. The pain can be sharp, dall, or aching and may last for hours. He is physically active and is carrying a full academic load. The pain is not related to exercise and does not restrict his activities. His father and petemal uncle had myocardial infarchons within the past year Physical exemination is normal, akhough the patient appears restless and has a heart rate of 100/nin. The most likely diagnosis is a) Generalized anxisty disorder ¥ © b) Reflux esophagitis ©) Transient infarction & Costochondriis Question Explanation: Tncompetence of the lower esophageal sphincter allows reflux of gastric contents into the esophagus, causing burning pain. Prolonged reflux may lead to esophagitis, stricture, and rarely metaplasia or malignancy. The most prominent symptom of gastro esophageal reflux disease (GERD) is hearibum, with or without regurgitation of gastric contents into the mouth. Infants present with vomiting, inntabilty, cnorezia, and sometimes symptoms of chronic aspiration. Both aduls and infants with chronic aspiration may have cough, hoarseness, or wheezing Diagnosis is clnical, sometimes with endoscopy, with or without acid testing. Treatment involves lifestyle modification, acid suppression using proton pump inhibitors, and sometimes surgical repair. Mark this question —& => Question Td : 64560 Question 10 of 30 A 36 year old man returns for follow of an abnormal liver chemistry profile ordered 3 weeks ago during aroutine examination. At that time, bis physical examination was nonmal, but he had a serum AST concentration of 72 U/L His Serum bilimbin and alkaline phosphatase concentrations were normal. History incindes an epicode of hepatic A at age 22 years. He has no history of transfisions or intravencus drug use. He drinks two to three bears daily. Today's follow-up test results show: Anti-HAV Positive Ant-HBs Negative EBsAg Positive EBeag Positive ‘The most appropriate next step in management is 2) Begin interferon-alla therapy ) Begin corticosteroid therapy ©) Have him cease all alcohol consumption and retest him in 2 months 4) Order hepatitis B virus DIVA polymerase study €) Schedule lirer biopsy Anower [UBINSERWEH) otner veers Explanation Report An Error Question Explanation: ‘The indication for extensive testing in this patient was the abnormal liver enzymes. The results of this testing reveals a positive HAV antibody which is indicative of prior exposure to the hepatitis A vimas. Hepatitis B vias, a DIVA virus, is transmitted through parenteral ot mucosal exposure to infectious blood or body fiuids and can lead to chronic as well as acute infection, citrhosis, and primary hepatocellular carcincma, Orly approximately 10% of all acute hepatitis B infections progress to chronic infection in adubs ‘The presence of a positive HBsAg (surface antigen to hepatitis B) and HBsAg (core antigen) indicates acute, active hepatitis B infection. After infection and 1 to 6 weeks before symptoms occur HBsAg appears. The relevent question consems the actual effect ‘on liver enzymes. For this reasons, the patient should refrain from-alcohol consumption, which also elevates liver enzymes. Mark this question —& => Question Td : 64560 Question 10 of 30 A 36 year old man returns for follow of an abnormal liver chemistry profile ordered 3 weeks ago during aroutine examination. At that time, bis physical examination was nonmal, but he had a serum AST concentration of 72 U/L His Serum bilimbin and alkaline phosphatase concentrations were normal. History incindes an epicode of hepatic A at age 22 years. He has no history of transfisions or intravencus drug use. He drinks two to three bears daily. Today's follow-up test results show: Anti-HAV Positive Ant-HBs Negative EBsAg Positive EBeag Positive ‘The most appropriate next step in management is 2) Begin interferon-alla therapy ) Begin corticosteroid therapy ¥ © ©) Have him cease all alcohol consumption and retest him in 2 months 4) Order hepatitis B virus DIVA polymerase study €) Schedule lirer biopsy Anower [UBINSERWEH) otner veers Explanation Report An Error Question Explanation: ‘The indication for extensive testing in this patient was the abnormal liver enzymes. The results of this testing reveals a positive HAV antibody which is indicative of prior exposure to the hepatitis A vimas. Hepatitis B vias, a DIVA virus, is transmitted through parenteral ot mucosal exposure to infectious blood or body fiuids and can lead to chronic as well as acute infection, citrhosis, and primary hepatocellular carcincma, Orly approximately 10% of all acute hepatitis B infections progress to chronic infection in adubs ‘The presence of a positive HBsAg (surface antigen to hepatitis B) and HBsAg (core antigen) indicates acute, active hepatitis B infection. After infection and 1 to 6 weeks before symptoms occur HBsAg appears. The relevent question consems the actual effect ‘on liver enzymes. For this reasons, the patient should refrain from-alcohol consumption, which also elevates liver enzymes. ‘Mark this question = => Question Td : 75908 Question 11 of 30 All ofthe following are intrahepatic causes of portal hypertension EXCEPT a) Alcoholic liver disease ) Hemochromatosis, ©) Schistosomiasis ) Budd-Chiasi syndrome. ©) Wilson's disease £) Peliosis Hepatits Question Explanation Budd-Chiatt smidrome is a posthepatic cause of postal hypertension. Ibis produced by increased resistance to flow in the hepatic veins or inferior vena cava above the liver. The other causes are all intrahepatic causes of portal hypertension, Peliosis Hepatis ic an uacormmon vascular condition characterized by randomly distributed maultiple blood-filled cavities throughout the liver. ‘Mark this question = => Question Td : 75908 Question 11 of 30 All of the folowing are intrahepatic causes of portal hypertension EXCEET a) Alcoholic liver disease. ) Hemochromatosis, c) Schistosomiasis. ¥ © &) Budd-Chiari syndrome. e) Wilson's disease 8 Peliosis Hepatits Question Explanation Budd-Chiatt smidrome is a posthepatic cause of postal hypertension. Ibis produced by increased resistance to flow in the hepatic veins or inferior vena cava above the liver. The other causes are all intrahepatic causes of portal hypertension, Peliosis Hepatis ic an uacormmon vascular condition characterized by randomly distributed maultiple blood-filled cavities throughout the liver. Mark this question = => Question Td : 81452 Question 12 of 30 ‘Which of the followingis NOT related with pigmented gallstone formation? @) Hemolytic anemia +) Cirrhosis ©) Biliary strictures 4) Parasitic biliary infestation ©) Oral contraceptives Question Exp Pigmented gallstones are composed primarily of calcium bilrubinate, Stones resut from precipitation of uacongugated, insoluble tilirabin with calcium to forma calcium bilirubinete. Black pigment stones result primarily from increased heme turnover. leading to increased biliary secretion of unconjugated bilisubin. Conditions predisposing to this include hemolytic anemias and hypersplenism associated with cirrhosis (B). Brown pigment stones contain calcium soaps of fatty acids in addition to calcium biiiubinate. These stones are produced in chronically infected bile. Conditions predisposing to such stones include biliary stricture and, especially in “Asia, parasitic biliary infestation [(C) and (D)]. Cral contraceptives are associated with decreased bile calt secretion and cholesterol gall stone formation ©). Mark this question = => Question Td : 81452 Question 12 of 30 ‘Which of the followingis NOT related with pigmented gallstone formation? 2) Hemolytic anemia +) Cirrhosis ©) Biliary strictures 4) Parasitic bikary infestation Y © ©) Oral contraceptives Question Exp Pigmented gallstones are composed primarily of calcium bilrubinate, Stones resut from precipitation of uacongugated, insoluble tilirabin with calcium to forma calcium bilirubinete. Black pigment stones result primarily from increased heme turnover. leading to increased biliary secretion of unconjugated bilisubin. Conditions predisposing to this include hemolytic anemias and hypersplenism associated with cirrhosis (B). Brown pigment stones contain calcium soaps of fatty acids in addition to calcium biiiubinate. These stones are produced in chronically infected bile. Conditions predisposing to such stones include biliary stricture and, especially in “Asia, parasitic biliary infestation [(C) and (D)]. Cral contraceptives are associated with decreased bile calt secretion and cholesterol gall stone formation ©). Mark this question = => Question Id : 81498 Question 13 of 30 A.45 year old man presents to the ER with several episodes of severe vomiting. Investigations demonstrate a normal hemoglobin concentration, a normal mezn copuscular hemoglobin concentration, and a hypochromic anemia. History reveals that the patient has consumed alcohol heavily for the past 20 years. Physical examination reveals dyephagia and baitle nail beds. Tae most likely cauce of the patient's anemia is a) Peptic ulceration ) Gastric carcinoma ) Hiatal hemia ) Malory-Weiss syndrome ©) Celiac sprue Question Explanation: Mallory-Weiss syndrome, a disease of alcoholics, often results in iron deficiency anemia in the later stages, a finding consistent with the hypochrotric anemia. This disease may often present with hemoptysis, an easy explanation for the anemia in this petient. Peptic ulceration (A), gastric carcinoma (B), hiatal hemia (C), and celiac cprue (), while all are causes of ron. deficiency anemia, will not present with vomiting Mark this question = => Question Id : 81498 Question 13 of 30 A.45 year old man presents to the ER with several episodes of severe vomiting. Investigations demonstrate a normal hemoglobin concentration, a normel mean corpuscular hemoglobin concentration, and a hypochromic anentia, History reveals that the patient has consumed alcohol heavily for the past 20 years. Physical examination reveals dyephagia and baitle nail beds. Tae most likely cauce of the patient's anemia is a) Peptic ulceration ) Gastric carcinoma ) Hiatal hemia V © A Malory-Weiss syndrome ©) Celiac sprue Question Explanation: Mallory-Weiss syndrome, a disease of alcoholics, often results in iron deficiency anemia in the later stages, a finding consistent with the hypochrotric anemia. This disease may often present with hemoptysis, an easy explanation for the anemia in this petient. Peptic ulceration (A), gastric carcinoma (B), hiatal hemia (C), and celiac cprue (), while all are causes of ron. deficiency anemia, will not present with vomiting ‘Marle this question & => Question Ta : 84953 Question 14 of 30 A patient complains of nausea, vorriting, and diarrhea He notes that his symptoms improve markedly when bread and pasta are removed from his diet, Mariced histologic changes would most likely be seen at which of the folowing sites in this patient's disease? 9) Distal large bowel b) Distal stall bowel 6) Entire large bowel 6) Entire small bowel ©) Proximal small bowel Anower [JEQUBNBEN) Oiner User's Explanation Report An Error Question Explanation: ‘The patient has celiac disease which is apparently an acquized hypersensitivity to the gluten (such as gliadin) in wheat. Unllke tropical sprue (which maybe related to enterotoagenic E. coil infection), which involves the entire small bowel, celiac sprue is usually limited to the proximal small bowel This may occur because the giuten antigens have not yet been digested at this point in their joumey through the bowel. A gluten five diet usually restores the small bowel nmcosa ‘Marle this question & => Question Ta : 84953 Question 14 of 30 A patient complaine of nausea, voriting, and diarrhea He notes that hie eymnptomc improve maceedly when bread and pasta are removed from his diet. Marked histologic changes would most lkely be seen at which of the folowing sites inthis patient's disease? 2) Distal large bowel +) Distal small bowel 6) Entire large bowel ©) Entire small bowel WV © ©) Proximal small bowel Anower [JEQNENEN) otner User's Explanation Report An Error Question Explanation: ‘The patient has celiac disease which is apparently an acquized hypersensitivity to the gluten (such as gliadin) in wheat. Unllke tropical sprue (which maybe related to enterotoagenic E. coil infection), which involves the entire small bowel, celiac sprue is usually limited to the proximal small bowel This may occur because the giuten antigens have not yet been digested at this point in their joumey through the bowel. A gluten five diet usually restores the small bowel nmcosa '3i2/2014 5:33:37 PM ‘Mark this question & => Question Id : 89749 Question 15 of 30 ‘The ciinical sign that is described by a periumbilical hematoma is a) Grey-Turner’s sign ) Homan’s sign c) Murphy's sign ) Collen’s sign 2) Psoas sign Answer | Besionaion Other User's Explanation Report An Error Question Explanation: Pancreatitis can produce both a pesiumbilical hematoma (Cullen's sign) and a flank ecchymoses (Grey Tumer's sign). Homan’s sign sefers to pain on dorsflexion in the presence of a calf deep venous thrombosis, butis not very sensitive or specific. Abdominal pain under the right costal margin during simmitaneous palpation and inspiration is Murphy's sign, which can be found in acute cholecystitis The psoas sign is when right leg extension produces abdominal pain, and is found in acute appendicitis (the appendix overlies the psoas muscle). Crohn's disease, a subset of inflammatory bowel disease, is not commonly associated with any of these clinical signs. '3i2/2014 5:33:37 PM ‘Mark this question & => Question Id : 89749 Question 15 of 30 ‘The clinical sign that is described by a periumbilical hematoma is a) Grey-Turner’s sign ») Homan’s sign c) Murphy's sign ¥ © A Cullen's sign 2) Psoas sign Answer | Besionaion Other User's Explanation Report An Error Question Explanation: Pancreatitis can produce both a pesiumbilical hematoma (Cullen's sign) and a flank ecchymoses (Grey Tumer's sign). Homan’s sign sefers to pain on dorsflexion in the presence of a calf deep venous thrombosis, butis not very sensitive or specific. Abdominal pain under the right costal margin during simmitaneous palpation and inspiration is Murphy's sign, which can be found in acute cholecystitis The psoas sign is when right leg extension produces abdominal pain, and is found in acute appendicitis (the appendix overlies the psoas muscle). Crohn's disease, a subset of inflammatory bowel disease, is not commonly associated with any of these clinical signs. Mark this question == Question Id : 101562 Question 16 of 30 Out of the following, which is associated with biliary colic? 2) Rebound tenderness ») Epigastric pain 6) Jamadice 4) Murphy sign Question Explanation: “About one third of patients with gallstones develop biliary colic or other complications. Colic pain is precipitated by spasm of a dilated cystic duct that is obstructed by gallstones. Attacks of biliary colic are more common at night, possibly because the gallbladder shifts to a horizontal position, facilitating the entry of stones into the cystic duct. Biliary colic is characterized by the sudden onset of intense right upper abdominal pain thet may radiate to the shoulder Sweating and vomiting are common. The pain tends to be steady and lasts up to three hours. Residual cbdominal tendemess may occur. Murphy sign, which is inspiratory arrest ‘with palpation of right upper quadrants seen in cholecystitis. Mark this question == Question Id : 101562 Question 16 of 30 Out of the following, which is associated with biliary colic? 2) Rebound tenderness Y © ») Epigastric pain 6) Jamadice 4) Murphy sign Question Explanation: “About one third of patients with gallstones develop biliary colic or other complications. Colic pain is precipitated by spasm of a dilated cystic duct that is obstructed by gallstones. Attacks of biliary colic are more common at night, possibly because the gallbladder shifts to a horizontal position, facilitating the entry of stones into the cystic duct. Biliary colic is characterized by the sudden onset of intense right upper abdominal pain thet may radiate to the shoulder Sweating and vomiting are common. The pain tends to be steady and lasts up to three hours. Residual cbdominal tendemess may occur. Murphy sign, which is inspiratory arrest ‘with palpation of right upper quadrants seen in cholecystitis. Mark this question & => Question Ta : 105006 Question 17 of 30 Costicosteraid treatment is appropriate in which of the following clinical scenario? a) Patient with chronic active hepatitis and fivefold elevation in transaminases ) Patient with HBsAg-positive chronic active hepatitis ) Patient with drug-induced hepatitis 4) Non A,non B chronic hepatitis ) Acute hepatitis Answer (Explanation) Other User's Explanation Report An Error (Question Explanation: Coriticosteroids can lead to cramatic clinical improvement in these patients. In patients with HBsAg-positve chronic active hepattis, steroids may lead to immunosuppression secondary to enhanced viral replication. Drug-induced hepatitis will not respond to steroids and non A, non B hepatitis wil usually resolve spoataneously and, thus, steroids ae aot indicated, Mark this question & => Question Ta : 105006 Question 17 of 30 Corticosteraid treatment is appropriate in which of the following clinical scenario? Y © a) Patient with chronic active hepatitis and fivefold elevation n transaminases ') Patient with HBsAg-positive chronic active hepatite ©) Patient with drug-induced hepatitis a) Non A, non B chronic hepatitis ) Acute hepatitis Answer (Explanation) Other User's Explanation Report An Error (Question Explanation: Coriticosteroids can lead to cramatic clinical improvement in these patients. In patients with HBsAg-positve chronic active hepattis, steroids may lead to immunosuppression secondary to enhanced viral replication. Drug-induced hepatitis will not respond to steroids and non A, non B hepatitis wil usually resolve spoataneously and, thus, steroids ae aot indicated, ‘Mark this question Out of the following, which statement is correct? «=> Question 18 of 30 4) Ipecac should be administered to children who are comatose. +) Ipecac should be administered to children who have ingested strong acids ©) Ipecac should aot be administered te children who have ingested lye 4) Ipecac can be safely given to children under the age of six ©) Ipecacis almost always necessary for the treatment of poisoning in adulis Anower [UBIPANSHIR) otnerUsor® Explanation Report An Eixor Question Explanation: Question Td : 118857 Ipecac should act be administered to children (or adults) who have ingested le, or other stroag bases or acids, since vomiting will only increase the exposure of the esophagus to the risk of bums. Ipecac should never be administered to comatose patients, since aspiration may occur, As indicated above, ipecac should not be administered to those who have ingested strong acids. Ipecac should not be givento children under the age of si, since it can result in arpiration Ipecac is not usually necessary for the treatment of poisoning in acts, ‘Mark this question Out of the following, which statement is correct? «=> Question 18 of 30 4) Ipecac should be administered to children who are comatose. +) Ipecac should be administered to children who have ingested strong acids Y © of Ipecac should aot be administered to children who have ingested lye 4) Ipecac can be safely given to children under the age of six ©) Ipecacis almost always necessary for the treatment of poisoning in adulis Anower [UBIPANSHIR) otnerUsor® Explanation Report An Eixor Question Explanation: Question Td : 118857 Ipecac should act be administered to children (or adults) who have ingested le, or other stroag bases or acids, since vomiting will only increase the exposure of the esophagus to the risk of bums. Ipecac should never be administered to comatose patients, since aspiration may occur, As indicated above, ipecac should not be administered to those who have ingested strong acids. Ipecac should not be givento children under the age of si, since it can result in arpiration Ipecac is not usually necessary for the treatment of poisoning in acts, ‘Marke this question & => Question Td : 131437 Question 19 of 30 A woman aged 67 years presents with upper right quadrant abdominal pain, Fever and jaundice exist. The most likely diagnosis is a) Gastritis ) Peptic ulcer ©) Cholangitis 4d) Hemochromatosis e) Hepatic cinthosis Answer { Biptanation | Other User's Explanation Report An Error Question Explanatio1 Chelarsitis is a disease of stones, most often in the common bile duct. The Charcot's triad of paia, fever, and jaundice represents the hallmark picture of cholangitis. The other choices-gastrtis, peptic ulcer, hemochromatosis, and hepatic cisrhosis-do not typically present in this way. ‘Marke this question & => Question Td : 131437 Question 19 of 30 A woman aged 67 years presents with upper night quadrant abdominal pain Fever and jaundice exist. The most likely diagnosis is a) Gastritis b) Peptic ulcer Y © c) Cholangitis d) Hemochromatosis ¢) Hepatic cirthosis Answer { Biptanation | Other User's Explanation Report An Error Question Explanatio1 Chelarsitis is a disease of stones, most often in the common bile duct. The Charcot's triad of paia, fever, and jaundice represents the hallmark picture of cholangitis. The other choices-gastrtis, peptic ulcer, hemochromatosis, and hepatic cisrhosis-do not typically present in this way. 3i2/2014 5: 243 PM ‘Marke this question & => Question Td : 133348 Question 20 of 30 Ti bicarbonate ic used te treat which complication ofliver faihure? @) Metabolic acidosis &) Encephalopathy c) Hyponatremia 8) Hypokalemia ©) Respiratory alkalosis, Question Explanation: ‘Treatment of iver failure consists of supportive therapy and treatment ofits complications. Water restriction is effective management of the hyponatremia that resulis rom impaired fice water clearance. IV bicarbonate is useful in the management of metabolic acidosis resuiting from lactic acidosis. Protem restriction and gut cleansing are the principles of management of encephalopathy. Hypokalemia is treated with potassium supplementetion. Respiratory allalosis does not require treatment. Cerebral edema may be treated with mannitol 3i2/2014 5: 243 PM ‘Marke this question & => Question Td : 133348 Question 20 of 30 TV bicarbonate is used to treat which complication of liver faiure? Y © 2) Metabolec acidosis +b) Encephalopathy ©) Hyponatremia ¢) Hypokalemia ©) Respiratory alkalosis Question Explanation: ‘Treatment of iver failure consists of supportive therapy and treatment ofits complications. Water restriction is effective management of the hyponatremia that resulis rom impaired fice water clearance. IV bicarbonate is useful in the management of metabolic acidosis resuiting from lactic acidosis. Protem restriction and gut cleansing are the principles of management of encephalopathy. Hypokalemia is treated with potassium supplementetion. Respiratory allalosis does not require treatment. Cerebral edema may be treated with mannitol Mark this question & => Question Te Question 21 of 30 Fresh frozen plasma is used in management of which complication of liver falare? a) Hemorrhage ») Hypokalemia ©) Respiratory allealosis, 4) Cerebral edema ¢) Azotemia Anowor [UBIARIRIEN) omer veors Explanation Report An Error Question Explanation: ‘Treatment of liver failure consists of supportive therapy and treetment ofits complications. Vitamin K is used to help control hemorrhage dae to decreased synthesis of clotting factors. Correcting the hypovoletria is all thet is usually necessary in the imanagement of azctemia. Hypokalemia is treated with potassium supplementation Respiratory alkalosis does not require treatment Cerebral edema may be treated with mannitol. Mark this question & => Question Te Question 21 of 30 Fresh frozen plasma is used in management of which complication of iver falare? ¥ © a) Hemorrhage ») Hypokalemia ©) Respiratory allealosis, 4) Cerebral edema ¢) Azotemia Anowor [UBIARIRIEN) omer veors Explanation Report An Error Question Explanation: ‘Treatment of liver failure consists of supportive therapy and treetment ofits complications. Vitamin K is used to help control hemorrhage dae to decreased synthesis of clotting factors. Correcting the hypovoletria is all thet is usually necessary in the imanagement of azctemia. Hypokalemia is treated with potassium supplementation Respiratory alkalosis does not require treatment Cerebral edema may be treated with mannitol. Mark this question €e& => Question Td : 139051 Question 22 of 30 A person presents with recurrent episodic diarrhea, triggered by eating too much or drinking alcohol. According to his wife "he tums as redac a beet, starts wheering, and looks just ghastly’ during these episodes. A lang macs ic seen on Chest X-ray. The most likely cause of his symptoms is a) Carcinoid tumor +) Primary tuberculosis c) Recurrent tuberculosis ) Squamous cell carcinoma ¢) Superior vena cava syndrome Answer | Explanation Other User's Explanation Report An Error Question Explanation: “This patient has carcinoid syndrome, in which vasoactive mediators secreted by carcinoid tumors (Iypically ofthe img or small bowel) produce recurrent diarrhea, fushing. wheering, and carcinoid heart disease. Although meny substances may be produced by these tamors, serotonin is one of the more important mediators produced, probably accounting for the observed diarrhea, Primary and recurrent tuberculosis can be uncommorly associated with finger clubbing, ot hypertrophic osteopathy, but not wih carcinoid syndrome, Squamous cell carcinoma can cause hypercalcemia by producing circulating hormones (humoral hypercalcemia of malignancy). Superior vena cava syndrome results from extrinsic compression of the superior vena cava by a tumor Dysoneais the most common symptom, followed by extremity swellng, facial swelling, and cough. Ithas no relationship to eating or drinking alcohol, and does not cause diarrhea. Mark this question €e& => Question Td : 139051 Question 22 of 30 A person presents with recurrent episodic diarrhea, triggered by eating too much or drinking alcohol. According to his wife "he tums as redac a beet, starts wheering, and looks just ghastly’ during these episodes. A lang macs ic seen on Chest X-ray. The most likely cause of his symptoms is Y © a) Carcinoid tumor +) Primary tuberculosis c) Recurrent tuberculosis ) Squamous cell carcinoma ¢) Superior vena cava syndrome Answer | Explanation Other User's Explanation Report An Error Question Explanation: “This patient has carcinoid syndrome, in which vasoactive mediators secreted by carcinoid tumors (Iypically ofthe img or small bowel) produce recurrent diarrhea, fushing. wheering, and carcinoid heart disease. Although meny substances may be produced by these tamors, serotonin is one of the more important mediators produced, probably accounting for the observed diarrhea, Primary and recurrent tuberculosis can be uncommorly associated with finger clubbing, ot hypertrophic osteopathy, but not wih carcinoid syndrome, Squamous cell carcinoma can cause hypercalcemia by producing circulating hormones (humoral hypercalcemia of malignancy). Superior vena cava syndrome results from extrinsic compression of the superior vena cava by a tumor Dysoneais the most common symptom, followed by extremity swellng, facial swelling, and cough. Ithas no relationship to eating or drinking alcohol, and does not cause diarrhea. ‘Marke this question & => Question Td : 197209 Question 23 of 30 A51 year old ex footballer has a long history of alcohol misuse. He now presents with epigastric pain. Out of the following, which one suggests a diagnosis of peptic ulceration rather than chronic pancreatitis? 2) Back pain b) Exacerbation with alcohol ©) Loose stool ¢) Releved by food ©) Weight loss Question Explanation: Relief with food suggests peptic (and specifically) duodenal ulceration. Ibis ikely thet food would precipitate the pain of chronic pancreatitis. Loose stool is suggestive of pancreattisimalab sorption. Pain referred to the back occurs in both situations and hence not suggestive. Weight loss can occur in bath gastric ulcers and pancreatitis and not very suggestive. Alechal may well exacerbate beth types of pain ‘Marke this question & => Question Td : 197209 Question 23 of 30 A51 year old ex footballer has a long history of alcohol misuse. He now presents with epigastric pain. Out of the following, which one suggests a diagnosis of peptic ulceration rather than chronic pancreatitis? 2) Back pain b) Exacerbation with alcohol ©) Loose stool Y © 6) Releved by food ©) Weight loss Question Explanation: Relief with food suggests peptic (and specifically) duodenal ulceration. Ibis ikely thet food would precipitate the pain of chronic pancreatitis. Loose stool is suggestive of pancreattisimalab sorption. Pain referred to the back occurs in both situations and hence not suggestive. Weight loss can occur in bath gastric ulcers and pancreatitis and not very suggestive. Alechal may well exacerbate beth types of pain “Mark this question €c:> Question Id : 197342 Question 24 of 30 A.62 year old male with known alcohol retated cirshosis presented with ascites, abdominal tendemess and peripheral oedema A diagnostic tap revealed a neutrophil count of 400/mm3 (<25Cmm3). Which one would provide most immediate benefit? a) Fiuid rectriction and ano added calt diet +b) Intravenous antibiotics ©) Oral spironclactone ) 4) Therapeutic Paracentesis ¢) Trans-Tugular intrahepatic porto-systemic shunt Answer | Booanation) Other User's Explanation Report An Error Question Explanatioy This man has spontaneous bacterial peritonitis (SBP). Appropriate treatment is IV antibiotics. He is likely to have a decreased intravascular volume and require IV albumin as vohime expansion. Fluid restriction, diaretics, or large volume paracentesis are likely to cause firher hypovelaemnia and precipitate renal failure, There ie no stated indication for a TIPS, indications are: churetic resistant ascites, intractable portal hypertensive bleeding and hepeto-renal failure “Mark this question €c:> Question Id : 197342 Question 24 of 30 A.62 year old male with known alcohol retated cirshosis presented with ascites, abdominal tendemess and peripheral oedema A diagnostic tap revealed a neutrophil count of 400/mm3 (<25Cmm3). Which one would provide most immediate benefit? a) Fiuid rectriction and ano added calt diet Y¥ © b) Intravenous antibiotics ©) Oral spironclactone ) 4) Therapeutic Paracentesis ¢) Trans-Tugular intrahepatic porto-systemic shunt Answer | Booanation) Other User's Explanation Report An Error Question Explanatioy This man has spontaneous bacterial peritonitis (SBP). Appropriate treatment is IV antibiotics. He is likely to have a decreased intravascular volume and require IV albumin as vohime expansion. Fluid restriction, diaretics, or large volume paracentesis are likely to cause firher hypovelaemnia and precipitate renal failure, There ie no stated indication for a TIPS, indications are: churetic resistant ascites, intractable portal hypertensive bleeding and hepeto-renal failure 3i2/2014 5:40:41 PM ‘Mark this question e& => Question Td : 197748 Question 25 of 30 A 53 year old woman hes a history of worsening dysphagia over many years. Recently there had been episodes ofl defined central chest discomfort and noctumal cough. The most likely diagnosis is which one of the following? a) Achalasia b) Barrett's oesophagus ©) Motor neurone disease 4) Oesophageal carcinoma ©) Pharyngeal pouch Anower (REIRRRHBRY) Other Users Explanation — Report An E01 Question Explanation: “Achalasia presents most offenin the 3rd-Sth decade, Symptoms usual develop years before the patient preserts. Vague chest discomfortis common, 30% have a nocvarnal cough due to aspiration of esophageal contents, Barrett's esophagus does not cause dysphagia. MND causes dysphagia due to problems with chewing and initiating a swallow and would not cause chest discomfort Esophageal carcincma is very unlicely due to the curation of symptoms (years). A pharyngeal pouch usually presents in the 6th-Tth decade with regurgitation and would not cause chest discomfert, 3i2/2014 5:40:41 PM ‘Mark this question e& => Question Td : 197748 Question 25 of 30 A 53 year old woman hes a history of worsening dysphagia over many years. Recently there had been episodes ofl defined central chest discomfort and noctumal cough. The most licely diagnosis is which one of the following? Y © a) Achalasia b) Barrett's oesophagus ©) Motor neurone disease 4) Oesophageal carcinoma ©) Pharyngeal pouch Anower (REIRRRHBRY) Other Users Explanation — Report An E01 Question Explanation: “Achalasia presents most offenin the 3rd-Sth decade, Symptoms usual develop years before the patient preserts. Vague chest discomfortis common, 30% have a nocvarnal cough due to aspiration of esophageal contents, Barrett's esophagus does not cause dysphagia. MND causes dysphagia due to problems with chewing and initiating a swallow and would not cause chest discomfort Esophageal carcincma is very unlicely due to the curation of symptoms (years). A pharyngeal pouch usually presents in the 6th-Tth decade with regurgitation and would not cause chest discomfert, ‘Mark this question —& => Question Id : 197808 Question 26 of 30 ATI year old woman has ahistory of pancteatitis and persistent dierthea She also has osteoporosis and a history of deep vein thrombosis. Which drug would become less effective after she starts taking Cholestyramine to relieve intolerable itching? a) Aspirin ) Folic Acid ) Thiamine ) Vitamin D e) Wasfasin Anewor [UEIRISHAR) otnerusorsExplanation Report An Evor Question Explanatio Choestyramine is an anion exchange resin, and wil interfere with the absorption of fat-sohible vitamins. Thus vitamin D absorption will be reduced, making treatment with this drug less effective when given along with cholestyramine Chclestyramine may enhance or reduce the anticoagulant effect of warfarin, ‘Mark this question —& => Question Id : 197808 Question 26 of 30 A 71 year old woman has a history of pancreatitis and persistent diarrhea She also has osteoporosis and a history of deep vein thrombosis, Which drug would become less effective after she starts taking Cholestyramine to releve intolerable itcking? a) Aspirin ') Folic Acid ¢) Thiamine V © 4) Vitamin D e) Warfarin Anewor [UEIRISHAR) otnerusorsExplanation Report An Evor Question Explanatio Choestyramine is an anion exchange resin, and wil interfere with the absorption of fat-sohible vitamins. Thus vitamin D absorption will be reduced, making treatment with this drug less effective when given along with cholestyramine Chclestyramine may enhance or reduce the anticoagulant effect of warfarin, ‘Mark this question ea => Question Td : 197924 Question 27 of 30 ATI year oldman has pruritus and 2 kg weight loss of 2 weeks duration He is jaundiced and has not drunk any alcohol for at least ight years. A month ago he completed a course of co-amoxiclav for sinusitis and is also taking ibuprofen for ostecarthritis. His bilimbi, AST and ALP are elevated. Abdominal ubrasound reveals gallstones but no biliary duct dilatation. The most ikely cause of tis jaundice is 8) Cholengjo-carcinoma ) Co-amoxiclar ©) Hepatitis B infection 4) Hepatitis © infection #) Ibuprofen Question Explanation: Co-amoxiclav (Augmentin) is notorious for causing drug induced jaundice, often with a mixed hepatiticcholestatic picture, A four week delay in symptoms and sigas is not unusval Fluclozacillin ie another common culprit, The patient mnst be warned that this could reoccur fhe is gven co-amoxiclav again. ‘Mark this question ea => Question Td : 197924 Question 27 of 30 ATI year oldman has pruritus and 2 kg weight loss of 2 weeks duration He is jaundiced and has not drunk any alcohol for at least ight years. A month ago he completed a course of co-amoxiclav for sinusitis and is also taking ibuprofen for ostecarthritis. His bilimbi, AST and ALP are elevated. Abdominal ubrasound reveals gallstones but no biliary duct dilatation. The most ikely cause of tis jaundice is 8) Cholengjo-carcinoma Y © d) Co-amoxiclar ©) Hepatitis B infection 4) Hepatitis © infection ®) Ibuprofen, Question Explanation: Co-amoxiclav (Augmentin) is notorious for causing drug induced jaundice, often with a mixed hepatiticcholestatic picture, A four week delay in symptoms and sigas is not unusval Fluclozacillin ie another common culprit, The patient mnst be warned that this could reoccur fhe is gven co-amoxiclav again. ‘Marke this question & => Question Ta : 199633 Question 28 of 30 A 25 year old woman had ulcerative colitis (JC) for seven years and was prescribed mesalazine 15 g per day She smoked 20 cigarettes per day and was 10 weeks pregnant. She has worsening of symptoms with six bloody stools per day. Which statement is correct? 8) Azathioprine is relatively contraindicated +) Initiation of an elemental diet risk fetal malnutrition ©) Oral corticosteroids are contraindicated 6) Oral mesalanine therapy should be withdrawn ¢) Termination of the pregnancy is advisable Anewor (UBIVIRIRAN) otner sors Explanation Report An Error Question Explanation: The effect of pregnancy on TIC is variable Oral corticosteroids and mesalazine are not contraindicated. In general, the health of a mother with UC is the best predictor of the outcome of the pregnancy. Hence dmg treatment s preferred to leaving active disease untreated ‘The BNE states that azathioprine should not generally be started during pregency, but its only relatively contraindicated. However, azathioprine is concentrated in breastfeeding. Tn the context of pregnancy, an elemental dict does not risk matcrnal and/or fetal malnutrition. ‘Marke this question & => Question Ta : 199633 Question 28 of 30 A 25 year old woman had ulcerative colitis (JC) for seven years and was prescribed mesalazine 15 g per day She smoked 20 cigarettes per day and was 10 weeks pregnant. She has worsering of symptoms with six bloody stools per day. Which statemont is correct? J © a) Azathioprine is relatively contraindicated +) Laitition of an elemental diet risk fetal malnutrition 6) Oral corticosteroids are contraindicated 6) Oral mesalazine therapy should be withdrawn 6) Termination ofthe pregnancy is advisable Anewor (UBIVIRIRAN) otner sors Explanation Report An Error Question Explanation: The effect of pregnancy on TIC is variable Oral corticosteroids and mesalazine are not contraindicated. In general, the health of a mother with UC is the best predictor of the outcome of the pregnancy. Hence dmg treatment s preferred to leaving active disease untreated ‘The BNE states that azathioprine should not generally be started during pregency, but its only relatively contraindicated. However, azathioprine is concentrated in breastfeeding. Tn the context of pregnancy, an elemental dict does not risk matcrnal and/or fetal malnutrition. ‘Mark this question = => Question Td : 217141 Question 29 of 30 An 82-year-old bedridden woman has severe constipation with no bowel moves in the past 5 days and from today, she has lower abdominal discomfort and distention. She takes hydrochlarathiazide and acetaminophen with codeine for severe arthritic pain in both hips. The rectal vault is fled with hard stool. Which of the following is the most appropriate nest step in management? a) Barium enema ) Colonoscopy ©) Flexible sigmoidoscopy ) Manual disimpaction 2) Passage of a nacogasiric tube Answer | Boplanaion Other User's Explanation Report An Error Question Explanation: This elderly woman who has been bedridden and on a narcotic analgesic has become severely constipated, This multifactorial condition is caused by her immatilization, bowel hypcmatilty secondary ta the codeine, and possible mild debydation frorn her dluetic. The treatment is manual disimpection, which wil promptly "open the floodgates" and yield immediate relief for the paticmy. No other imaging studies ie., barium enema colonoscopy, or exible sigmoidoscopy are necessary at present. The patient should be disimpacted to promptly resolve her abdominal distention and discomfort. Passage of a nasogastric tube would he indicated ifthe patient showed signs of upper gastrointestinal bleeding, bat é is not be appropriate for this woman with obvious constipation. ‘Mark this question = => Question Td : 217141 Question 29 of 30 An 82-year-old bedridden woman has severe constipation with no bowel moves in the past 5 days and from today, she has lower abdominal discomfort and distention. She takes hydrochlarathiazide and acetaminophen with codeine for severe arthritic pain in both hips. The rectal vault is fled with hard stool. Which of the following is the most appropriate nest step in management? a) Barium enema ) Colonoscopy ©) Flexible sigmoidoscopy Y © d) Manwal disimpaction 2) Passage of a nacogasiric tube Answer | Boplanaion Other User's Explanation Report An Error Question Explanation: This elderly woman who has been bedridden and on a narcotic analgesic has become severely constipated, This multifactorial condition is caused by her immatilization, bowel hypcmatilty secondary ta the codeine, and possible mild debydation frorn her dluetic. The treatment is manual disimpection, which wil promptly "open the floodgates" and yield immediate relief for the paticmy. No other imaging studies ie., barium enema colonoscopy, or exible sigmoidoscopy are necessary at present. The patient should be disimpacted to promptly resolve her abdominal distention and discomfort. Passage of a nasogastric tube would he indicated ifthe patient showed signs of upper gastrointestinal bleeding, bat é is not be appropriate for this woman with obvious constipation. Merle this question & Question Td : 217262 Question 30 of 30 A.10-year ald (clo of previous uncomplicated Ml) has a 1-week history of crampy lower abdominal pain and bloody diarrhea Symptoms began with mid postprandial abdominal cramping followed by diarthea, which became bloody after 2 days. She hes moderate teademecs to palpation of the leftlower quadrant. A restal examination reveals bloody stool and no masses. Which of the followings the most likely diagnosis? a) Arteriovenous malformation +) Diverticulitis 6) Diverticulosis 9) Ischemic colitis 6) Ulcerative coltis Question Explanation: This elderly woman with a history of coronary artery disease has now developed colitc symptorss, as demonstrated by the left lower quadrant pain and bloody darchea. Weight loss from fear of pain associated with esting is also typical. This is typically due to occlusion, vasospasm, andfor hypoperfusion of the mesenteric vasculature resulting in ischemic colts. Diminished blood flow leads to ‘bowel wall jchemia The most commen cause of iechemic colitis ic desreaced bowel perfusion due to low cardiac output in patients with cardiac disease os prolonged shock. Patients experience abdominal pain localized to the left side ofthe abdomen, along with tenderness and bloody diarthea. The clagnosis is suspected clinically and generally confirmed with CT scanning and/or endoscopic studies, Arteriovencus malformation produces painless bleeding in the elderly most commonly located in the cecum and often, but not always, associated with aortic stenosis Diverticuitis does present with postprandial crampy let lower quackant abdominal pain and fever, but does net produce bloody diarhea. Often, these patients have symptoms of constipation. Diverticulosis also presexts with postprandial crampy left lower quadrant pain and bright red blood per rectum or may be syinptornatic, but does not present with a diartheal illness Uleerative colis typically presents in a subacute or chroric fachicn in younger patient population. Although it can occur in this patient ‘s age group, itrarely preseats as an acutz ilness Merle this question & Question Td : 217262 Question 30 of 30 A.10-year ald (clo of previous uncomplicated Ml) has a 1-week history of crampy lower abdominal pain and bloody diarrhea Symptoms began with mid postprandial abdominal cramping followed by diarthea, which became bloody after 2 days. She hes moderate teademecs to palpation of the leftlower quadrant. A restal examination reveals bloody stool and no masses. Which of the followings the most likely diagnosis? a) Arteriovenous malformation +) Diverticulitis ©) Diverticulosis Y @ O Ischemic colitis 6) Ulcerative coltis Question Explanation: This elderly woman with a history of coronary artery disease has now developed colitc symptorss, as demonstrated by the left lower quadrant pain and bloody darchea. Weight loss from fear of pain associated with esting is also typical. This is typically due to occlusion, vasospasm, andfor hypoperfusion of the mesenteric vasculature resulting in ischemic colts. Diminished blood flow leads to ‘bowel wall jchemia The most commen cause of iechemic colitis ic desreaced bowel perfusion due to low cardiac output in patients with cardiac disease os prolonged shock. Patients experience abdominal pain localized to the left side ofthe abdomen, along with tenderness and bloody diarthea. The clagnosis is suspected clinically and generally confirmed with CT scanning and/or endoscopic studies, Arteriovencus malformation produces painless bleeding in the elderly most commonly located in the cecum and often, but not always, associated with aortic stenosis Diverticuitis does present with postprandial crampy let lower quackant abdominal pain and fever, but does net produce bloody diarhea. Often, these patients have symptoms of constipation. Diverticulosis also presexts with postprandial crampy left lower quadrant pain and bright red blood per rectum or may be syinptornatic, but does not present with a diartheal illness Uleerative colis typically presents in a subacute or chroric fachicn in younger patient population. Although it can occur in this patient ‘s age group, itrarely preseats as an acutz ilness ‘Mark this question => Question Td : 28954 Question 1 of 30 A.67-year-old man with known aortic valvular disease is admitted with deteriorating dyspnea He has Hemogobin= MCV=70L. g/dL and [Upper gastrointestinal tract endoscopy formal [Duodenal biopsy [Normal “What investigation is useful? a) Barium enema b) Colonoscopy ©) CT abdomen 4 ¢) Small bowel enema Answer | Popanation Other User's Explanation Report An Error Question Explanation: Tn the older age group investigation of the lower gastrointestinal (GI) tract is vital to exchide a lower GT malignancy. CT scans do not demonstrate colonic pathology and colonoscopy which is stil the gold standard. Angiography is only helpful if the patient is bleeding briskly at the time of the examination, ‘Mesenteric angiography ‘Mark this question => Question Td : 28954 Question 1 of 30 A.67-year-old man with known aortic valvular disease is admitted with deteriorating dyspnea He has Hemogobin= MCV=70L. g/dL and [Upper gastrointestinal tract endoscopy formal [Duodenal biopsy [Normal “What investigation is useful? a) Barium enema Y © b) Colonoscopy ©) CT abdomen 4 ¢) Small bowel enema Answer | Popanation Other User's Explanation Report An Error Question Explanation: Tn the older age group investigation of the lower gastrointestinal (GI) tract is vital to exchide a lower GT malignancy. CT scans do not demonstrate colonic pathology and colonoscopy which is stil the gold standard. Angiography is only helpful if the patient is bleeding briskly at the time of the examination, ‘Mesenteric angiography Mark this question —& => Question Td : 48823 Question 2 of 30 A patient who has chronic ulcer asks for the best method for caring this disease. What would you answer? 8) Proton punep inhbitors b) H2 blockers 6) Helcobacter pylori eradication 6) Nissen fundoplication Question Explanati E_ pylori and NSAIDs disrupt normal mucosal defense and repair, making the mucosa more susceptible to acid. H. pylori infection is present in 50 to 70% of patents with duodenal ulcers and 30 to 50% of patients with gastric ulcers. IFH. pylon is eradicated, only 10% of patients have recurrence ofpeptic ulcer disease, compared with 70% recurrence in patients treated with acid suppression alone Symptoms inckds pain oftea localized to the epigastrium and relieved by food or antacids. The pain is described as buming or gnawing, or sometimes as a sensation of hunger. The couse is usually chroric and recurrent. Diagnosis of peptic ulcer is suggested by patient history and cotfirmed by endoscopy (EGD). Treatment of gastric and duodenal ulcers requires eradication of H. pylori when present Methods of decreasing avidity include a number of drugs including proton punzp inubitos and H2 blockers. Nissen fundoplication is a surgical treatment for chronic GERD. Mark this question —& => Question Td : 48823 Question 2 of 30 A patient who has chronic ulcer asks for the best method for caring this disease. What would you answer? 8) Proton punep inhbitors b) H2 blockers SY © 6) Helicobacter pylori eradication 6) Nissen fundoplication Question Explanati E_ pylori and NSAIDs disrupt normal mucosal defense and repair, making the mucosa more susceptible to acid. H. pylori infection is present in 50 to 70% of patents with duodenal ulcers and 30 to 50% of patients with gastric ulcers. IFH. pylon is eradicated, only 10% of patients have recurrence ofpeptic ulcer disease, compared with 70% recurrence in patients treated with acid suppression alone Symptoms inckds pain oftea localized to the epigastrium and relieved by food or antacids. The pain is described as buming or gnawing, or sometimes as a sensation of hunger. The couse is usually chroric and recurrent. Diagnosis of peptic ulcer is suggested by patient history and cotfirmed by endoscopy (EGD). Treatment of gastric and duodenal ulcers requires eradication of H. pylori when present Methods of decreasing avidity include a number of drugs including proton punzp inubitos and H2 blockers. Nissen fundoplication is a surgical treatment for chronic GERD. Mark this question & => Question Td : 51549 Question 3 of 30 A.34 year old male with epigastric pain and positive serology for H. pylori was treated with triple therapy. Several months later, he ‘was asymptomatic but his serology was positive. The best management of this patient is 2) Repeat the triple therapy +) Perform endoscopy ©) Repeat serology in three months 4) Reassure and counsel ©) Test his spouse for H. pylori Anower [UBQNERER) otter Users Explanation Report An Error Question Explanation: HL. pylosiis a common gastric pathogen that causes gastritis, peptio ulcer disease, gasiric adenocarcinoma, and low grade gastiic igmphoma. Infection may be arymptomatic or result in varying degrees of dyspepsia. Diagnosis is by urea breath test anc testing of endoscopic biopsy samples. Laboratory and office-based serologc assays for antibodies to H. pylorihave sensitivity and specificity of > 85% and are considered the nonirwasive tests of choice for initial documentation of H. pylori infection. However, because qualitative assays remein positive for up to 3 years after succeseful treatment and because quantitative axtibody levels do net dectine significantly for 6 to 12 months after treatment, serologic assays are not usually used to assess cure. Mark this question & => Question Td : 51549 Question 3 of 30 -A.34 year old male with epigastric pain and positive serology for H. pylori was treated with triple therapy. Several months later, he ‘was asymptomatic but his serology was positive. The best management of this patient is 2) Repeat the tiple therapy +) Perform endoscopy c) Repeat serclogy in three months Y © 6) Reassure and counsel ¢) Test his spouse for H. pylon Anower [UBQNERER) otter Users Explanation Report An Error Question Explanation: HL. pylosiis a common gastric pathogen that causes gastritis, peptio ulcer disease, gasiric adenocarcinoma, and low grade gastiic igmphoma. Infection may be arymptomatic or result in varying degrees of dyspepsia. Diagnosis is by urea breath test anc testing of endoscopic biopsy samples. Laboratory and office-based serologc assays for antibodies to H. pylorihave sensitivity and specificity of > 85% and are considered the nonirwasive tests of choice for initial documentation of H. pylori infection. However, because qualitative assays remein positive for up to 3 years after succeseful treatment and because quantitative axtibody levels do net dectine significantly for 6 to 12 months after treatment, serologic assays are not usually used to assess cure. '3i2/2014 6:22:20 PM ‘Marke this question & => Question Td : 545 Question 4 of 30 A.35 year old letter carrier has ulcerative esophagitis and has lost 30 Ib in 3 months. He is emaciated and appears il. Pulse is 98 beats/min, temperature 37.8°C, respiratosy rate 24/hnin, and blood pressure 95/6OmmETg. His dentition is poor, and there is evidence of oral thrush. Some posterior cervical and anillary nodes are palpable. The heart, lung, and abdominal examinations are normal. As you contin to investigate, you take amore detaled history. The finding that is most likely to be related to the patient's problemis a) Easy bruisabiliy ) Intravenous drug use c) A family history of esophageal cancer 4) Chest pain relieved by nitroglycerin @) Recent travel to Russia Question Explanation: A young man with weight loss, oral thrash, lymphadenopathy, and ulcerative esophagitis is lcely to have HIN infection Tntravermous drug use is responsible for over a quarter of HIV infections in Canada, Esophageal disease develops in more than half of all patients with advanced infection during the course of their illuess. The most common pathogens causing esophageal ulceration in HIV-positive patients include Candida, herpes simplex: vims, and cytomegalovirus. Identifying the causative agent through culture or tissue sampling is ixeportant for providing prompt and specific therapy. '3i2/2014 6:22:20 PM ‘Marke this question & => Question Td : 545 Question 4 of 30 A.35 year old letter carrier has ulberative esophagitis and hee loct 30 Ib in 3 monthe. He is emaciated and appears il. Pulte ie 98 beats/min, temperature 37.8°C, respiratory rate 24/min, and biood pressure 95/60mmnEg. His dentition is poor, and there is evidence of oral thrush. Some posterior cervical and axillary nodes are palpable. The heart, hing, and abdominal examinations are normal. As you coatinue to investigate, you take amore detaled history. The Binding that is most licely to be zelated to the patient's problem is a) Easy bruisability ¥ © b) Intravencus drug use c) A family history of esophageal cancer 4) Chest pain relieved by nitroglycerin @) Recent travel to Russia Question Explanation: A young man with weight loss, oral thrash, lymphadenopathy, and ulcerative esophagitis is lcely to have HIN infection Tntravermous drug use is responsible for over a quarter of HIV infections in Canada, Esophageal disease develops in more than half of all patients with advanced infection during the course of their illuess. The most common pathogens causing esophageal ulceration in HIV-positive patients include Candida, herpes simplex: vims, and cytomegalovirus. Identifying the causative agent through culture or tissue sampling is ixeportant for providing prompt and specific therapy. ‘Marle this question & => Question Td : 54711 Question 5 of 30 4.44 year old woman presents with pruritus and progressive jaundice, with increased liver function test and total bilirubin. The next appropriate investigation to be performed is 8) Ulrasound ) Serum lead level ©) ERCE 4) Anticmitochondtrial antibocies Answer [UEIRIRERHBRTY) Other Users Explanation Repost An Exar Question Explanation: Primary biliary cithosis (PBC) is inflammation with progressive scarring of the bile ducts in the liver. Evertually, the ducts are blocked, the liver becomes scarred, and liver failure develops. PBC is most common among women aged 35 to 70, although i can occur in men and women of any age. An autoimmune reaction is the llkely cause of primary biliary cirrhosis. Itchiness, fatigue, a dry mouth and eyes, and jaundice ars common A blood test to measure anti-mitochondrial antibodies is highly accurate for the disgrosis. ‘Treatment focuses on relieving symptoms, slowing liver damage, and treating complications. ‘Marle this question & => Question Td : 54711 Question 5 of 30 A 44 year old woman presents with pruritus end progressive jaundice, with increased liver function test and total biliubin, The next appropriate investigation to be performed is 2) Ultrasound ) Serum lead level ©) ERCF SY © & Anti-mitochondrial antibodies Answer [FEIIRERHBRTY) Other Users Explanation Repost An Feat Question Explanation: Primary biliary cithosis (PBC) is inflammation with progressive scarring of the bile ducts in the liver. Evertually, the ducts are blocked, the liver becomes scarred, and liver failure develops. PBC is most common among women aged 35 to 70, although i can occur in men and women of any age. An autoimmune reaction is the llkely cause of primary biliary cirrhosis. Itchiness, fatigue, a dry mouth and eyes, and jaundice ars common A blood test to measure anti-mitochondrial antibodies is highly accurate for the disgrosis. ‘Treatment focuses on relieving symptoms, slowing liver damage, and treating complications. Mark this question & => Question Td : 63637 Question 6 of 30 ‘Total parenteral muttition is appropriate for patents a) Who cannot ewallow because ofan esophageal motlity problem +) With poorly functioning gastrointestinal tracts who cannot tolerate enteral feeding ©) Who rafise to eat 4) In whom maintenance nutrition is desited for a short period folowing recovery from surgery Question Explanation Total parenteral munition (IPL\) is indicated for patients with poorly functioning gastrointestinal tracts who cannot tolerate other ‘means of mutritional support and for those with high caloric requirements that cannot otherwise be met. Patients who cannot swallow because of an esophageal motility problem and those who are resistant to feeding can be managed with tbe feedings. Peripheral aimentation, which provides fewer calories then TPM or liquid tube feedinge, would be more appropriate over the short term in patients recovering from surgery. Mark this question & => Question Td : 63637 Question 6 of 30 ‘Total parenteral nnttion is appropriate for patents a) Who cannot swallow because of an esophageal motility problem Y © b) With poorly functioning gastrointestinal tracts who cannot tolerate enteral feeding c) Who refse to eat d) In whom maintenance nutrition is desired for a short period following recovery from surgery Question Explanation Total parenteral munition (IPL\) is indicated for patients with poorly functioning gastrointestinal tracts who cannot tolerate other ‘means of mutritional support and for those with high caloric requirements that cannot otherwise be met. Patients who cannot swallow because of an esophageal motility problem and those who are resistant to feeding can be managed with tbe feedings. Peripheral aimentation, which provides fewer calories then TPM or liquid tube feedinge, would be more appropriate over the short term in patients recovering from surgery. Mark this question = => Question Id : 64038 Question 7 of 30 A patient presents with weightloss, easy bruising and greasy smelly stools. A dermatitis herpetiformis rash is also noted on his arms. A dagnosis of celiac sprve is suspected. The most appropriate method for diagnosis is 2) EGD £) Bowel biopsy ©) Colonoscopy 4) CT ofabdomen Question Explanation: Celiac sprue is an immunologically mediated disease in genetically suscepibie individuals caused by intolerance to gluten, resulting in ‘mucosal inflarrmation, which causes malabsorption, Symptoms usuelly include diarrhea and abdominal discomfort. Celiac sprue is ahereditary disorder caused by sensitivity to the gliadin fraction of gluten, a protein found in wheat: simlar proteins occur in rye and barley. The inflammatory resporse produces characteristic mucosal vilous atrophy in the small bowel. In adults, fassitude, weakness, and anorexia are most common. Mild and intermittent diarrhea is sometimes the presenting symptom, Steatorrhea ranges stom mild to severe (7 to 50 g fat/day). Some patients have weight loss, rarely enough to become underweight Anemia, alosstis, angular stomatitis, and aphthous ulcers are usually seen in these patients. About 10% have dermatitis herpetiformis, {an intensely pruritic papulovesicular rash that is symmetrically distributed over the extensor areas of the elbows, knees, buttacks, shoulders, and scalp. This rash can be induced by ahigh gluten dict. Diagnosis is by small bowel biopsies demonstrating characteristic ‘though not specific pathologic changes of villous atrophy thet improve with a strict giuten fee diet Mark this question = => Question Id : 64038 Question 7 of 30 A patient presents with weightloss, easy bruising and greasy smelly stools. A dermatitis herpetiformis rash is also noted on his arms. A dagnosis of celiac sprve is suspected. The most appropriate method for diagnosis is 2) EGD Y © ¥) Bowel biopsy ©) Colonoscopy 4) CT ofabdomen Question Explanation: Celiac sprue is an immunologically mediated disease in genetically suscepibie individuals caused by intolerance to gluten, resulting in ‘mucosal inflarrmation, which causes malabsorption, Symptoms usuelly include diarrhea and abdominal discomfort. Celiac sprue is ahereditary disorder caused by sensitivity to the gliadin fraction of gluten, a protein found in wheat: simlar proteins occur in rye and barley. The inflammatory resporse produces characteristic mucosal vilous atrophy in the small bowel. In adults, fassitude, weakness, and anorexia are most common. Mild and intermittent diarrhea is sometimes the presenting symptom, Steatorrhea ranges stom mild to severe (7 to 50 g fat/day). Some patients have weight loss, rarely enough to become underweight Anemia, alosstis, angular stomatitis, and aphthous ulcers are usually seen in these patients. About 10% have dermatitis herpetiformis, {an intensely pruritic papulovesicular rash that is symmetrically distributed over the extensor areas of the elbows, knees, buttacks, shoulders, and scalp. This rash can be induced by ahigh gluten dict. Diagnosis is by small bowel biopsies demonstrating characteristic ‘though not specific pathologic changes of villous atrophy thet improve with a strict giuten fee diet Mark this question = => Question Ta : 74036 Question § of 30 An immigrant from Santo Domingo hes severe myalgias, diarrhea, and fever. He had a marked eosinophilia and his Creatinine phosphokinase level was elevated. There were na changes in his dietary habits. Wat is the treatment of choice inthis patient? 2) Discontinue L-tryptophan ) Thrombolytic therapy ©) Vigorous intravenous hydration 4) Thiahendazole 6) Steroids Question Explanatio ‘This patient has tichinosis from ingestion of infected pork meat. The meat contains encysted larvae of Trichinella spiralis. Seven days alter ingestion, the patient can have pulmonary and abdominal symptoms. Serologic testing is usefil and treatment is with thiabendazole. L-traptophan has been in the news for causing marked eosinophilia and myalgias. The CPK level would not be clevated. Thrombolytic therapy would be indicated ifthis pationt has an acute myocardial infarction within four to six hours of presettation. He does not present wits chest pain, and the elevated CPX level is fom skeletal muscle involvement, Intravenous hydration would be indicated in thabdomyolysis, which involves a very high CPK level and skeletal muscle breakdown with release of myoglobin, Eosinophilia would not be associated with thabdemyolysis. Steroids can sometimes cause dissemination of tichinosis infection, but t can be uced in severe inflammatory involvement of the central nervous system and myocar, This petient presents ‘with gastrointestinal symptoms Mark this question = => Question Ta : 74036 Question § of 30 An immigrant from Santo Domingo hes severe myalgias, diarrhea, and fever. He had a marked eosinophilia and his Creatinine phosphokinase level was elevated. There were na changes in his dietary habits. Wat is the treatment of choice inthis patient? 2) Discontinue L-tryptophan ) Thrombolytic therapy ©) Vigorous intravenous hydration JV © @ Thiabendazole 6) Steroids Question Explanatio ‘This patient has tichinosis from ingestion of infected pork meat. The meat contains encysted larvae of Trichinella spiralis. Seven days alter ingestion, the patient can have pulmonary and abdominal symptoms. Serologic testing is usefil and treatment is with thiabendazole. L-traptophan has been in the news for causing marked eosinophilia and myalgias. The CPK level would not be clevated. Thrombolytic therapy would be indicated ifthis pationt has an acute myocardial infarction within four to six hours of presettation. He does not present wits chest pain, and the elevated CPX level is fom skeletal muscle involvement, Intravenous hydration would be indicated in thabdomyolysis, which involves a very high CPK level and skeletal muscle breakdown with release of myoglobin, Eosinophilia would not be associated with thabdemyolysis. Steroids can sometimes cause dissemination of tichinosis infection, but t can be uced in severe inflammatory involvement of the central nervous system and myocar, This petient presents ‘with gastrointestinal symptoms Mark this question —& => Question Td : 87133 Question 9 of 30 A 37 year old patient consults a physician because of bloating and increased abdominal size. Exploratory laporatomy is performed, which reveals loculated masses of semi translucent mucinous material produced by 2 mucinous cystadenoma, The most likely sites for a prinary tumor in this patient are? a) Colon or spleen ') Liver or pancreas ©) Tung ot bladder ) Ovary ot appencix @) Prostate or gallbladder Question Explanatior This rare bu: dramatic condition is called pseudomyxoma peritonei, and is produced when a malignant or benign (spread by rupture into the peritoneal cavity rather than true metastasis) mucus producing tumor (mucinous oystadsnoma er musinous cystadenocarcinoma) produces gel ie mucus that fils the perionesl cavity. The usval sites of origin of these tumors are ovary and appendix The condition, even when benign, is difficult to treat because the mucus-producing cells are spread all over the peritoneal lining and cannat he effectively removed. Home parenteral rutition may he necessary hecause the gut (particularly peristalsis) just does not function well in a sea of Tell-C that slowly becomes replaced with fibrous tissue bands Mark this question —& => Question Td : 87133 Question 9 of 30 A 37 year old patient consults a physician because of bloating and increased abdominal size. Exploratory laporatomy is performed, which reveals loculated mastec of semi trancucent mucinous material produced by a mucinous oystadenoma, The most likely sites for a primary tumor in this patieat are? a) Colon or spleen ') Liver or pancreas ©) Tung ot bladder YM © 4) Ovary or appendix @) Prostate or gallbladder Question Explanatior This rare bu: dramatic condition is called pseudomyxoma peritonei, and is produced when a malignant or benign (spread by rupture into the peritoneal cavity rather than true metastasis) mucus producing tumor (mucinous oystadsnoma er musinous cystadenocarcinoma) produces gel ie mucus that fils the perionesl cavity. The usval sites of origin of these tumors are ovary and appendix The condition, even when benign, is difficult to treat because the mucus-producing cells are spread all over the peritoneal lining and cannat he effectively removed. Home parenteral rutition may he necessary hecause the gut (particularly peristalsis) just does not function well in a sea of Tell-C that slowly becomes replaced with fibrous tissue bands Mark this question & => Question Td : 87914 Question 10 of 30 A 36 year cld man who recently retumed fiom a Third World country develops chronic, severe dysentery. Colonoscopy reveals ulceration of the cecum, and a cecal biopsy reveals 15 to 40 micron amoebae with ingested erythrocytes and small nuclei with distinctive tiny central karyosomes. The most likely causative agent is a) Acanthamoeba sp. b) Balantidum coil c) Entamoeba histolytica d) Giardia lamblia ¢) Naegleria fowleri Question Explanation: Entamoeba histolytica is the usual cause of intestinal amebiasis, and has the microscopic features as described above in the question ‘A particularly helpful (but not always present) feature of this orgenism is the presence of mgested red blood cells within the amoebae ‘These amoebae cause flask shaped ulceration of the intestinal mucosa and submucosa, with a particular propensity for involving the cecum and ascending colon. The disease manifestations range from none (asymptomatic carriers) to mild chronic diarrhea, to severe, purging dysentery in symptomatic cases, the liver may develop destructive amoebic liver abscesses that tend to become secondasily (and potentially ie threateningly) infected by bacteria, Acanthamoeba is a free-living amoeba that can cause amoebic meningocephelits. Balantidiam coliis a large ciliated intestinal parasite that can occesionelly cause colonic disease resembling that caused by Entamoeba histolytica Giardia lamblia is a small intestinal protozoon with a distinctive pear shaped merphology that appears to have a "face." Nacgleria fowleriis a fice living amoeba that can cause amocbic meningoencephalitis Mark this question & => Question Td : 87914 Question 10 of 30 A 36 year cld man who recently retumed fiom a Third World country develops chronic, severe dysentery. Colonoscopy reveals ulceration of the cecum, and a cecal biopsy reveals 15 to 40 micron amoebae with ingested erythrocytes and small nuclei with distinctive tiny central karyosomes. The most likely causative agent is a) Acanthamoeba sp. b) Balantidum coil Y © c) Entamoeba histolytica d) Giardia lamblia ¢) Naegleria fowleri Question Explanation: Entamoeba histolytica is the usual cause of intestinal amebiasis, and has the microscopic features as described above in the question ‘A particularly helpful (but not always present) feature of this orgenism is the presence of mgested red blood cells within the amoebae ‘These amoebae cause flask shaped ulceration of the intestinal mucosa and submucosa, with a particular propensity for involving the cecum and ascending colon. The disease manifestations range from none (asymptomatic carriers) to mild chronic diarrhea, to severe, purging dysentery in symptomatic cases, the liver may develop destructive amoebic liver abscesses that tend to become secondasily (and potentially ie threateningly) infected by bacteria, Acanthamoeba is a free-living amoeba that can cause amoebic meningocephelits. Balantidiam coliis a large ciliated intestinal parasite that can occesionelly cause colonic disease resembling that caused by Entamoeba histolytica Giardia lamblia is a small intestinal protozoon with a distinctive pear shaped merphology that appears to have a "face." Nacgleria fowleriis a fice living amoeba that can cause amocbic meningoencephalitis Marl this question = => Question Td : 88077 Question 11 of 30 A.55 year old teacher resents with headaches and fatigue, He is a nonsmoker. however he admits to many yeats of alcohal abuse ‘A smal, nodular iver is revealed on physical exam. AA routine CBC demonstrates a hematocrit of 639%. Vital signe are within normal limits, and oxygen sanuration is 98%. The peripheral smear shows nomo-cellilar erythrocytes, with increased reticulocytes and nucleated red cells. Erythropoietin and bilimbin levels are elevated; carboxyhemoglobin levels are decreased. The most likely dagnosisis a) Carbon monoxide toxicity +6) Cardiac malformation ©) Hepatocellular carcinoma 4) Polycythemia Vera ©) Reactive Polycythemia Question Explanation: Erythrocytosis is an important finding that always wasrants further mvestigetion, Ths patient's past alcohol abuse, nodular cirrhosis on examination, and elevated bilirubin all suggest liver pathology may exist. In fact. hepatocellular carcinoma is one of the five tumors most commonly associated with the overproduction of erythropoietin (1=., paraneoplastic erythrocytosis) with resulant polycythemia ‘The four other tumors associated with paraneoplastic syndromes that are themselves associated with increased erythropoietin include renal cell carcinoma, hemangioblastoma, pheochromocytoma, and uterine myomata, Carbon monoxide toxicity can cause ecythrocstosis with prolonged exposure. This is often suggested by occupational exposures: for exemple, in people who work in underground parking garages, taza drivers with proloaged exposure to car pollution, and industry workers. Cardiac malformations resulting in Eisenmenger syndrome can cause a secondary polycythemia, however physical exattination of this patient is not suggestive of this condition, Polycythemia vera is characteristically associated with low levels of erythropoietin Reactive polycythemia occurs in cases of tissue hypoxia, commonly seenin patients who smoke or have chronic hing concitions (COPD). Decreased oxygen saturation on pulse oximetry and @ history of smoking are suggestive of reactive polycythemia Marl this question = => Question Td : 88077 Question 11 of 30 A 55 year old teacher presents with headaches and fatigue. He is a nonsmoker, however he admits to many years of alcohol abuse. ‘A smal, nodular iver is revealed on physical exam. AA routine CBC demonstrates a hematocrit of 639%. Vital signe are within normal limits, and oxygen sanuration is 98%. The peripheral smear shows nomo-cellilar erythrocytes, with increased reticulocytes and nucleated red cells. Erythropoietin and bilimbin levels are elevated; carboxyhemoglobin levels are decreased. The most likely dagnosisis a) Carbon monoxide toxicity +6) Cardiac malformation YM © oc) Hepatocellular carcinoma 4) Polycythemia Vera ©) Reactive Polycythemia Question Explanation: Erythrocytosis is an important finding that always wasrants further mvestigetion, Ths patient's past alcohol abuse, nodular cirrhosis on examination, and elevated bilirubin all suggest liver pathology may exist. In fact. hepatocellular carcinoma is one of the five tumors most commonly associated with the overproduction of erythropoietin (1=., paraneoplastic erythrocytosis) with resulant polycythemia ‘The four other tumors associated with paraneoplastic syndromes that are themselves associated with increased erythropoietin include renal cell carcinoma, hemangioblastoma, pheochromocytoma, and uterine myomata, Carbon monoxide toxicity can cause ecythrocstosis with prolonged exposure. This is often suggested by occupational exposures: for exemple, in people who work in underground parking garages, taza drivers with proloaged exposure to car pollution, and industry workers. Cardiac malformations resulting in Eisenmenger syndrome can cause a secondary polycythemia, however physical exattination of this patient is not suggestive of this condition, Polycythemia vera is characteristically associated with low levels of erythropoietin Reactive polycythemia occurs in cases of tissue hypoxia, commonly seenin patients who smoke or have chronic hing concitions (COPD). Decreased oxygen saturation on pulse oximetry and @ history of smoking are suggestive of reactive polycythemia “Mark this question <=> Question Id : 88803 Question 12 of 30 A.44 year old jauneiced alcoholic man presents to the emergency room complaining of bright ted blood in his last stool. He denies pain on defecation or change in his bowel habits. The most likely finding on sigmoidoscopic examination would be 2) Colorectal carcinoma +) Diverticulitis ©) Hemorhoids 4) Hyperplastic polyps 6) Ulosrative colitis Question Explanation: ‘The patient's jaundice indicates that he has significant alcoholic liver disease Hepatic Sbrosis and cirrhosis both produce vascular injury and portal hypertension, which leads to the development of portosystemic shunts, most typically causing esophageal varices, caput medusae, and hemorthoids. Brightred Hood in stools is a classic presertation of hemorthoids. Colorectal carcinoma generally evelops in an older population and procuces occult blood loss rather than frankly bloody stools. Although diversiculosis maybe an asymptomatic condition, diverticulitis is associated with significant abdominal distress that may be accomparied by blood loss and iarchea andfor constipation Hyperplastic polyps are asymptomatic, non-neoplastic, polypoid growths, generally less than 5 mm in ameter. They do not prodace bleeding or any increased risk of carcinoma Ulcerative coliis is an inflammatory bowel disease that produces mucoid diarrhea accompanied by lower abdominal pain and cramps. Painless bleeding is unlcely to be ulcerative colitis. Report An Error “Mark this question <=> Question Id : 88803 Question 12 of 30 A.44 year old jauneiced alcoholic man presents to the emergency room complaining of bright ted blood in his last stool. He denies pain on defecation or change in his bowel habits. The most likely finding on sigmoidoscopic examination would be 2) Colorectal carcinoma +) Diverticulitis WV © ¢) Hemorthoids 4) Hyperplastic polyps 6) Ulosrative colitis Question Explanation: ‘The patient's jaundice indicates that he has significant alcoholic liver disease Hepatic Sbrosis and cirrhosis both produce vascular injury and portal hypertension, which leads to the development of portosystemic shunts, most typically causing esophageal varices, caput medusae, and hemorthoids. Brightred Hood in stools is a classic presertation of hemorthoids. Colorectal carcinoma generally evelops in an older population and procuces occult blood loss rather than frankly bloody stools. Although diversiculosis maybe an asymptomatic condition, diverticulitis is associated with significant abdominal distress that may be accomparied by blood loss and iarchea andfor constipation Hyperplastic polyps are asymptomatic, non-neoplastic, polypoid growths, generally less than 5 mm in ameter. They do not prodace bleeding or any increased risk of carcinoma Ulcerative coliis is an inflammatory bowel disease that produces mucoid diarrhea accompanied by lower abdominal pain and cramps. Painless bleeding is unlcely to be ulcerative colitis. Report An Error Marke this question e& => Question Td : 89621 Question 13 of 36 A AA year old man presents with lower gastrointestinal bleeding The patient undergoes colonoscopy that shows smerous polyps ‘The patient is missing several fingernails end is noted to have alopecia. The most appropriate diagnosis is a) Juvenile polyposis ) Turcot syndrome ©) Cronkhite-Canada syndrome 4) Peutz-Jeghers syndrome €) Gardner's syndrome Anower (REBIBPENBIY other User's Expan Question Explanation: Pevts-Jeghers syndrome consists of numerous polyps that are hamertomas (smooth muscle tumors) rather than adenomas and, therefore, have alower malignancy potential. Iris also associated with mucocutaneous pigmentation, especially of the buckle tmucosa. ‘Turcot sendrome refers to the association of colonic adenomas with CNS tumors. Gardner's syndrome is familial polyposis and the presence of soff-tisme tumor and/or osteomas, Malignant potential is relatively high in familial polyposis. Juverile polyposis presents ‘with GI bleeding from polyps in the stornach, and the small and large intestines n Report An Error Marke this question e& => Question Td : 89621 Question 13 of 36 -A.44 yecr eld man presents with lower gastrointestinal bleeding The patient undergoes colonoscopy that shows aumerous polyps ‘The patient is missing several fingernails and is noted to have alopecia, The most appropriate dlagnosis is a) Juvenile polyposis ') Turcot syndrome Y © ©) Cronkhite-Canada syadrome 4) Peutz-Teghers syndrome €) Gardner's syndrome Anower (REBIBPENBIY other User's Expan Question Explanation: Pevts-Jeghers syndrome consists of numerous polyps that are hamertomas (smooth muscle tumors) rather than adenomas and, therefore, have alower malignancy potential. Iris also associated with mucocutaneous pigmentation, especially of the buckle tmucosa. ‘Turcot sendrome refers to the association of colonic adenomas with CNS tumors. Gardner's syndrome is familial polyposis and the presence of soff-tisme tumor and/or osteomas, Malignant potential is relatively high in familial polyposis. Juverile polyposis presents ‘with GI bleeding from polyps in the stornach, and the small and large intestines n Report An Error ‘Marke this question €& => Question Td : 93734 Question 14 of 30 A.54 year old woman presents complaining of fatigue. Her hematocrit is 32%, despite the fact that she went through menopause 5 year ago. The physician suspects gastrointestinal bleeding and refers her to a gactrcenterologist. A thorough evaluation doee net disclose any evidence of malignancy and the gastroenterologist suspects the bleeding may be due to developmental abaomnality. The developmental abaosmality that might accoutt for unexplained small intestinal bleeding is which one of the following’ a) Central nervous eystem heterotopia ) Gastric heterotopia ©) Pancreatic heterotopia 4) Parathyroid heterotopia ©) Thyroid heterotopia newer [RESRIRRNBA) other User's Explanation Report An Esror Question Explanatio: Heterotopic rests are small areas of normal tissue in abnormal sites. These are usually clinically insignificant unless they form a noticeable mass or are misdiagnosed in a biopsy (raising suspicion of metastatic cancer), One exception is a gastric heterotopia, which ‘ypicelly occurs 1 the small intestine and can produce enough acid to cause a peptic ulcer in adjacent mucosa, The ulber may be a source of gastrointestinal bleeding. Heterotopias of the CNS. perathyroid and thyreid can occur but would aot be expected in the small intestine and would not be liely to cance bleeding if present. Pancreatic heterotopia can occur in the small intestine but doss not cause bleeding, ‘Marke this question €& => Question Td : 93734 Question 14 of 30 A.54 year old woman presents complaining of fatigue. Her hematocrit is 32%, despite the fact that she went through menopause 5 year ago. The physician suspects gastrointestinal bleeding and refers her to a gastroenterologist. A thorough evaluation does not disclose any evidence of malignancy and the gastroenterologist suspects the bleeding may be due to developmental abnormality. The developmental abnormality that mght account for unexplained small intestinal bleeding is which one of the following? a) Central nervous system heterotopia VW © b) Gastric heterotopia c) Pancreatic heterotopia 4) Parathyroid heterotopia e) Thyroid heterotopia newer [RESRIRRNBA) other User's Explanation Report An Esror Question Explanatio: Heterotopic rests are small areas of normal tissue in abnormal sites. These are usually clinically insignificant unless they form a noticeable mass or are misdiagnosed in a biopsy (raising suspicion of metastatic cancer), One exception is a gastric heterotopia, which ‘ypicelly occurs 1 the small intestine and can produce enough acid to cause a peptic ulcer in adjacent mucosa, The ulber may be a source of gastrointestinal bleeding. Heterotopias of the CNS. perathyroid and thyreid can occur but would aot be expected in the small intestine and would not be liely to cance bleeding if present. Pancreatic heterotopia can occur in the small intestine but doss not cause bleeding, 3i2/2014 6:24:49 PM ‘Mark this question —& => Question Id : 97589 Question 15 of 30 ‘Which of the following is NOT a risk factor for the development of hepatocellular carcinoma (HCC)? a) Hepatitis A infection. ) Hepatitis B infection ©) Hepattis C infection. 4) Exposure to aflatoxin 8) Schistosomiasis nnfecton. Answer (Boiataion) Other User's Explanation Report An Error Question Explanation: Hepatitis A does not cause chronic infection and, therefore, does not increase the risk of HCC. Risk factors for HCC mnelude cirrhosis (from alcohol, hemochromatosis, Wilson's disease), hepatitis B and C infection, aflatoxin, long term androgen therapy, end parasitic infections, such as schistosomiasis and clonorchiasis, 3i2/2014 6:24:49 PM ‘Mark this question —& => Question Id : 97589 Question 15 of 30 ‘Which of the following is NOT a risk factor for the development of hepatocellular carcinoma (HCC)? V © @ Hepatitis A infection. ) Hepatitis B infection ©) Hepatitis C infection. 6) Exposure to alatoxin 6) Schistosomiasis mnfecton Answer (Boiataion) Other User's Explanation Report An Error Question Explanation: Hepatitis A does not cause chronic infection and, therefore, does not increase the risk of HCC. Risk factors for HCC mnelude cirrhosis (from alcohol, hemochromatosis, Wilson's disease), hepatitis B and C infection, aflatoxin, long term androgen therapy, end parasitic infections, such as schistosomiasis and clonorchiasis, Mark this question e& => Question Td : 98069 Question 16 of 30 Innportant role in defense against peptic ulcer formation is NOT played by which of the following? a) Gastric mucus ») Bicarbonate ions ©) Prostaglandins 4) Blood flow ©) Bile acids Anower (UBINNSEN) oser User's Explanation Report An Eror Question Explanation: “A nearly impenetrable gestric snucosal barrier to back: diffusion of hydrogen ions from the gastric lumen is formed by the gastric epithelial cell laminal surfaces and intercellular tight junctions, Bile salts, ethanol, weak organic acids, and drugs such as salicylates can all disrupt this barrier. Numerous factors are important in maintaining the integrity of the gastric racosa, Th gastric juice, the gastric mmcus forms an insoluble gel layer, which coats the mucosal surface of the stomach. Pepsin molecules secreted into the gastric Inmen are denied reentry by the intact mucus gel, thereby offering mucosal protection against protcolytic damage. Gel thickness is increased by prostaglandins of the E series. Prostaglandins also stimulate secretion of gastric mucus, assist in the maintenance of mucosal blood flow, and stimulate gastric and duodenal bicarbonate secretion Bicarbonate ions are secreted by nonparietal gastric epithelial cells ‘They assistin maintaining a hydrogen ion gradient between the gel and the gastric lumen, Maintenance of blood flow provides the essential oxygen required by the high metabolic activity of the gastric mucosa. Mark this question e& => Question Td : 98069 Question 16 of 30 Innportant role in defense against peptic ulcer formation is NOT played by which of the following? 8) Gastrio mucus ») Bicarbonate ions ¢) Prostaglandins 4) Blood flow oY © ©) Bile acids Anower (UBRIENBEN) oser User's Explanation Report An Eror Question Explanation: “A nearly impenetrable gestric snucosal barrier to back: diffusion of hydrogen ions from the gastric lumen is formed by the gastric epithelial cell laminal surfaces and intercellular tight junctions, Bile salts, ethanol, weak organic acids, and drugs such as salicylates can all disrupt this barrier. Numerous factors are important in maintaining the integrity of the gastric racosa, Th gastric juice, the gastric mmcus forms an insoluble gel layer, which coats the mucosal surface of the stomach. Pepsin molecules secreted into the gastric Inmen are denied reentry by the intact mucus gel, thereby offering mucosal protection against protcolytic damage. Gel thickness is increased by prostaglandins of the E series. Prostaglandins also stimulate secretion of gastric mucus, assist in the maintenance of mucosal blood flow, and stimulate gastric and duodenal bicarbonate secretion Bicarbonate ions are secreted by nonparietal gastric epithelial cells ‘They assistin maintaining a hydrogen ion gradient between the gel and the gastric lumen, Maintenance of blood flow provides the essential oxygen required by the high metabolic activity of the gastric mucosa. Mark this question & => Question Td : 100805 Question 17 of 30 Autopsy of a person shows the typical finely granular surface of a kidney with diffuse arteriolosclerosis. The histopathologic changes consist of hyalinization of the media of arterioles and maltfocal loss and sclerosis of cortical glomeruli, Condltion that would be most likely associated with these findings is a) Acute hypowolemic shock bb) End-stage kidney failure c) Malignant hypertension 4) Subacute infective endocarditis ©) Sysiemic "benign" hypertension Avewor [JEIPIRISHER) otter UeorsExplanation Report An Error Question Explanation: The "finely granular surface" of kidneys reflects the presence of astesiolonephrosclerosis (benign nephrosclerosis). Tkarises from ‘multifocal loss of glomerui in the superficial cortex, which thus acquires this finely puckered appearance. The underlying pathogenetic mechanism is chronic ischemia, resulting from stenosis of the sinall arteries and arterioles. Hypertension is the usual cause, but dabetes mellitus causes simlar pathologic changes in the small vasculature of kidneys and other organs. Acute hypovolemic shock leads to acute tubular necrosis, due to a sudden and severe drop in renal perfusion pressure, Epithetal tubular cells undergo ischemic necrosis and detach from the basement membrane. A rapid decrease in glomerular filtration rate heralds the onset of acute renal failure. The kicheys appear large and edematous at autopsy Th end stage kidney failure the kidneys are usually atrophic and shrunken, The cortex has virtually disappeared and frequently shows cysts (especially £ the patient has been on chalysis for years) ‘Malgnant hypertension may cause acute renal faiure by producing severe arteriolar demage in the forma of necrotizing arteriolitis. The kkidney will have 2 motiled, hemorrhagic appearance. Subacute infective endocarditis leads to septic embolism. Fragments of infective vegetations detach from the valve leaflets and reach the systemic circulation If endocarditis affects the left side, Emboli that reach the renal arteries cause renal infarcts, which appear as wedge shaped areas of necrosis within the cortex. Mark this question & => Question Td : 100805 Question 17 of 30 Autopsy ofa person shows the typical finely granular surface of a kidney with diffuse arteriolosclerosis. The histopathologic changes consist of hyalinization ofthe media of arterioles end multifocal loss and sclerosis of cortical glemerul, Condltica that would be most likely associated with these findings is 8) Acute hypavolemic shock b) Ené-stage kidney failure ©) Malignant hypertencion 4) Subacute infective endocarditis Y © e) Systemic "benign hypertension Avewor [JEIPIRISHER) otter UeorsExplanation Report An Error Question Explanation: The "finely granular surface" of kidneys reflects the presence of astesiolonephrosclerosis (benign nephrosclerosis). Tkarises from ‘multifocal loss of glomerui in the superficial cortex, which thus acquires this finely puckered appearance. The underlying pathogenetic mechanism is chronic ischemia, resulting from stenosis of the sinall arteries and arterioles. Hypertension is the usual cause, but dabetes mellitus causes simlar pathologic changes in the small vasculature of kidneys and other organs. Acute hypovolemic shock leads to acute tubular necrosis, due to a sudden and severe drop in renal perfusion pressure, Epithetal tubular cells undergo ischemic necrosis and detach from the basement membrane. A rapid decrease in glomerular filtration rate heralds the onset of acute renal failure. The kicheys appear large and edematous at autopsy Th end stage kidney failure the kidneys are usually atrophic and shrunken, The cortex has virtually disappeared and frequently shows cysts (especially £ the patient has been on chalysis for years) ‘Malgnant hypertension may cause acute renal faiure by producing severe arteriolar demage in the forma of necrotizing arteriolitis. The kkidney will have 2 motiled, hemorrhagic appearance. Subacute infective endocarditis leads to septic embolism. Fragments of infective vegetations detach from the valve leaflets and reach the systemic circulation If endocarditis affects the left side, Emboli that reach the renal arteries cause renal infarcts, which appear as wedge shaped areas of necrosis within the cortex. Mark this question <& => Question 18 of 30 Hypoglycemia can be associated with which one of the following? a) Gastroparesis. ») Scleroderma. ©) Glucagonoma 4) Rapid gastric emptying. 6) Primary aldosteronistn. Question Explanation: Rapid gastric emptying can produce hypoglycemia. The other disorders are not associated with low serum glucose levels. Mark this question <& => Question 18 of 30 Hypoglycemia can be associated with which one of the following? a) Gastroparesis. ») Scleroderma. ©) Glucagonoma Y © 4) Rapid gastic emptying 6) Primary aldosteronistn. Question Explanation: Rapid gastric emptying can produce hypoglycemia. The other disorders are not associated with low serum glucose levels. ‘Mark this question & => Question Td : 118719 Question 19 of 30 A.40 year old businessman who has long standing cirrhosis fiom chronic alcohol use presents with symptoms of abdominal bloatng On exzemination he was found to have a fever, ascites, testicular atrophy, and a small, nodular liver. The most common cause of infection in this patient is a) Urinary tract infection b) Abdominal abscess ¢} Clostridium dificil coltis ) 4) Spontaneous bacterial peritonitie e) Paenmona Question Explanation: Spontaneous bacterial peritonitis is common in cirhotic petieats with asvites. Paracentesis would yield greater than 500 white blood cells per cubic ml, alow ascites fluid pH, and alow protein level. E. colis the most common orgarism cultured. IFieft untreated for a Jong period of ime, the mortality is high This patient exhibits no symptoms of a urinary tract infection. TFhe had an abdominal abscess from a provious perforatica, he would be in a lot of pain. Clostridium difficile colitis presents with dlarchea and abdominal craraps. This patient has no signs or symptoms of pneumoria. ‘Mark this question & => Question Td : 118719 Question 19 of 30 A.40 year old businessman who has long standing cirrhosis fiom chronic alcohol use presents with symptoms of abdominal bloatng On exzemination he was found to have a fever, ascites, testicular atrophy, and a small, nodular liver. The most common cause of infection in this patient is a) Urinary tract infection b) Abdominal abscess ¢} Clostridium dificil coltis ) Y © 4) Spontaneous bacterial peritonitis e) Paenmona Question Explanation: Spontaneous bacterial peritonitis is common in cirhotic petieats with asvites. Paracentesis would yield greater than 500 white blood cells per cubic ml, alow ascites fluid pH, and alow protein level. E. colis the most common orgarism cultured. IFieft untreated for a Jong period of ime, the mortality is high This patient exhibits no symptoms of a urinary tract infection. TFhe had an abdominal abscess from a provious perforatica, he would be in a lot of pain. Clostridium difficile colitis presents with dlarchea and abdominal craraps. This patient has no signs or symptoms of pneumoria. ‘Mark this question = Question 20 of 30 “Which one of the folowing is NOT a feature of ascending cholangitis? a) Fever. ) Right upper quadrant pain ) Jaundice, ) Hypertension. ©) Mental stats changes Question Explanati Question Td : 131376 Hypotension, not hypertension, is one ofthe classic features in ascending cholangitis. Hypotension is septic shock, ‘Mark this question = Question 20 of 30 “Which one of the folowing is NOT a feature of ascending cholangitis? a) Fever. ) Right upper quadrant pain ) Jaundice, SM © 2 Hypertension. ©) Mental stats changes Question Explanati Question Td : 131376 Hypotension, not hypertension, is one ofthe classic features in ascending cholangitis. Hypotension is septic shock, Mark this question ez Question Id : 136576 Question 21 of 30 “Which one of the following is the best first test in evaluating the cause of iron deficiency anemia? 2) Hemoscult exam of stool b) Sigmoidoscopy ©) Barium study of the esophagus 6) Serum iron. ©) Bone marrow aspirate. Question Explanation: ‘The most common cause ofiron deficiency anemia is occult blood loss in the stool from a gastrointestinal source. Sigmoidoscopy ‘might be the next test performed after a positive hemoccult, but it should not precede the hemoccuft Barium study of the esophagus vwould not be helpful in the early evaluation of ron deficiency. Neither serum iron nor bone marrow aspirate would help with the evaluation of the etiology of the iron loss, though they both would be helpful in evaluation and quantification ofits severity. Report An Error Mark this question ez Question Id : 136576 Question 21 of 30 “Wihich one of the following is the best first testin evaluating the cause of ron deficiency anemia? YM © 2 Hemoccult exam of stool b) Sigmoidoscopy 6) Barium stdy of the esophagus 6) Serum iron, ©) Bone marrow aspirate. Question Explanation: ‘The most common cause ofiron deficiency anemia is occult blood loss in the stool from a gastrointestinal source. Sigmoidoscopy ‘might be the next test performed after a positive hemoccult, but it should not precede the hemoccuft Barium study of the esophagus vwould not be helpful in the early evaluation of ron deficiency. Neither serum iron nor bone marrow aspirate would help with the evaluation of the etiology of the iron loss, though they both would be helpful in evaluation and quantification ofits severity. Report An Error ‘Mark this question Question Td : 139395 Question 22 of 30 A 65 year cld male dies of metastatic liver cancer. At autopsy the iver shows a multinodular vascular tumor. Histologically the turnor is composed of anastomosing channels lined by anaplastic endothelial cells. The liver parenchyma berween tumor nodules anpears normal. The risk factor that is associated with this form of iver cancer is a) Citshosis ) Exposure to vingl chloride ©) Hemochromatosis, ©) Hepatitis B vins infection ©) Oral contraceptive use Question Explanation: The gross and microscopic features of this multinodular tumor are diagnostic of angiosarcoma, a highly agaressive tumor that arises fiom blood vessels. In the liver, one ofthe mostimportant predisposing conditions is chronic professional exposure to vinyl chloride, which is used in the rubber industry, Tn other sites, angiosarcomna has been reported following radiation therapy. Tn the past, exposure to Thorotrast was a risk factor, untl this contrast mech used for radiologic investigations was discontinued Cirthosis is the most important predisposing condition for the development of hepatocellular carcinoma in industrialized courtries regardless ofits underlying etiologic origin, inchuding, for example, the form secondary to hemochromatosis. On the other hand, chronic hepatitis B infection is the most common sisk factor in developing countries, where hepatocellular carcinoma is observed mainly in young hepatitis B virus carriers. ral contraceptive use is a well know risk factor associated with an otherwise rare benign tumor of hepatocelular origin, ie., liver cell adenoma, Occasionally, liver cell adenoma has been shown to regress following discontinuation of steroid treatment. ‘Mark this question Question Td : 139395 Question 22 of 30 A665 year old male dies of metastatic liver cancer. At autopsy the liver shows a multinodular vascular tumor. Histologically the tumor is composed of anastomosing channels lined by anaplastic endothelial cells. The liver parenchyma between tumor nodules appears normal. The risk factor that is associated with this forma of iver cancer is 2) Cirthasis JY © +) Exposure to vinyl chloride ©) Hemochromatosis ©) Hepatitis B vins infection 6) Oral contraceptive use Question Explanation: The gross and microscopic features of this multinodular tumor are diagnostic of angiosarcoma, a highly agaressive tumor that arises fiom blood vessels. In the liver, one ofthe mostimportant predisposing conditions is chronic professional exposure to vinyl chloride, which is used in the rubber industry, Tn other sites, angiosarcomna has been reported following radiation therapy. Tn the past, exposure to Thorotrast was a risk factor, untl this contrast mech used for radiologic investigations was discontinued Cirthosis is the most important predisposing condition for the development of hepatocellular carcinoma in industrialized courtries regardless ofits underlying etiologic origin, inchuding, for example, the form secondary to hemochromatosis. On the other hand, chronic hepatitis B infection is the most common sisk factor in developing countries, where hepatocellular carcinoma is observed mainly in young hepatitis B virus carriers. ral contraceptive use is a well know risk factor associated with an otherwise rare benign tumor of hepatocelular origin, ie., liver cell adenoma, Occasionally, liver cell adenoma has been shown to regress following discontinuation of steroid treatment. ‘Marke this question e& => Question Td = 178943 Question 23 of 30, AAA year old jaundiced woman undergoes endoscopic retroarade cholangiopancreatography. Mukiple common bile duct stones ar imaged and the injected contrast matzral is retained within dilated common bile duct prossimal to the stones. Which nding would be consistent wit this case of jaundice? a) Excessive hemolysis +b) Increased pigmertation of the stool o) Increased urinary Uratilinegen 4) Light brown urine ©) Unconjugated hyperbilirubinermia Anaver [RESRIREHBA) other Users Explanation Report An Esror Question Explanatioy The results of the retrograde cholangopancreatography indicate the presence of an obstruction in the common bile duct, This would ead to an obstructive jaundice, The iver abilty to take up, conjagate, and secrete biliubin is not impaired, However, the conjugated biiubin would "backup" and regurghate into the circulation, produsing e conjugated byperbilimubinemia (not unconjugeted hyperbilirubinemia). Since conjugated bilirubin is water soluble, it can be fitered by the kiduey and show up in the wine, The excessive filtration of conjugated bilirubin can produce a darkening of the urine (tea-colored). Excessive hemolysis produces an unconjugated hyperbiiiubinemia because the ability ofthe liver to take up the increased bilirubin fiom heme metabolism is exceeded. An obstruction of the common bile dust would prevent the delivery of bile to the duodenum. As acensequence, the pigmentation of the stool would be diminished (not increated), Urinary urobilinogen is a reflection of circulating urobilinogen Urobilinogen enters the blood es part ofthe enterchepaiic circulation. Bilirubin secreted in the bile is ordinarily metabolized by gut bacteria to urobilinogen. Most urobiinogen remains in the gut, and after further metabolism, provides the pigmentation to the stool. However, some of the urobilinogen is absorbed by the ileum and enters the enterohepatic circulation Some of this absorbed urobilinogen ends up in the urine. With an obstruction of the common bile duct, less urobilinogen will seach the small intestine, and hence, less will enter the hepatic porial vein, Therefore, urinary urcbilnogen will be decreased (pot increased). ‘Marke this question e& => Question Td = 178943 Question 23 of 30, AAA year old jaundiced woman undergoes endoscopic retroarade cholangiopancreatography. Mukiple common bile duct stones ar imaged and the injected contrast matzral is retained within dilated common bile duct prossimal to the stones. Which nding would be consistent wit this case of jaundice? a) Excessive hemolysis +b) Increased pigmertation of the stool o) Increased urinary Uratilinegen Y © @) Light brown urine ©) Unconjugated hyperbilirubinermia Anaver [RESRIREHBA) other Users Explanation Report An Esror Question Explanatioy The results of the retrograde cholangopancreatography indicate the presence of an obstruction in the common bile duct, This would ead to an obstructive jaundice, The iver abilty to take up, conjagate, and secrete biliubin is not impaired, However, the conjugated biiubin would "backup" and regurghate into the circulation, produsing e conjugated byperbilimubinemia (not unconjugeted hyperbilirubinemia). Since conjugated bilirubin is water soluble, it can be fitered by the kiduey and show up in the wine, The excessive filtration of conjugated bilirubin can produce a darkening of the urine (tea-colored). Excessive hemolysis produces an unconjugated hyperbiiiubinemia because the ability ofthe liver to take up the increased bilirubin fiom heme metabolism is exceeded. An obstruction of the common bile dust would prevent the delivery of bile to the duodenum. As acensequence, the pigmentation of the stool would be diminished (not increated), Urinary urobilinogen is a reflection of circulating urobilinogen Urobilinogen enters the blood es part ofthe enterchepaiic circulation. Bilirubin secreted in the bile is ordinarily metabolized by gut bacteria to urobilinogen. Most urobiinogen remains in the gut, and after further metabolism, provides the pigmentation to the stool. However, some of the urobilinogen is absorbed by the ileum and enters the enterohepatic circulation Some of this absorbed urobilinogen ends up in the urine. With an obstruction of the common bile duct, less urobilinogen will seach the small intestine, and hence, less will enter the hepatic porial vein, Therefore, urinary urcbilnogen will be decreased (pot increased). ‘Mark this question <—& => Question Td ; 194642 Question 24 of 30 Exam of a 57 year old obese woman shows hepatomegaly. 6 years ago she was diagnosed with diabetes melltus and takes metformin and gliclaside. She drinks 15 usits of alcohol weeldy and stopped smoking 10 years ago. No stigmata ofliver disease are evident. Total bilirubin is 11 ymolf., ALP is 145 U/L, AST ic 10OI/L, ALT ie 150UIL, Albumin is 40 g/L, and Fenttin is 434 yp. Ultrascund reveals an echo bright appearence of the liver and gallstones in the gallbladder. The likely cause of her liver disease is a) Alcoholic liver disease +) Drug induced hepatitis c) Gallstone disease 4) Hacmochrometocis ¢) Non-alccholic steatohepatitis (WASH. Question Explanation ‘The patient has a hepatic picture in contrast to Cholestasis. Ferritin level is not too high to be considered for haemockromatosis and is an acute phase reactant being typically incteased in any inflammatory process. NASH is very common and is typically encountered in obese patents, presenting with a hepatitic picture with or without jaundice. Echo bright liver suggests fatty change in the liver seen in NASH. Itwas previously termed idiopathic decompensated hepatitis and ifnot treated in terms of lowering BMI and reducing fat intake cam lead onto irreversible cirrhosis, Gellsiones are a distraction in this history, ‘Mark this question <—& => Question Td ; 194642 Question 24 of 30 ‘Exam of a 57 year old obese woman shows hepatomegaly. 6 years ago she was diagnosed with diabetes mellitus and takes metformin and gliclaside, She drinks 15 units of alcohol weekly and stopped smoking 10 years ago. No stigmata ofliver disease are evident. Total bilirubin is 11 umol/L, ALP is 145 U/L, ASTis 100U/L, ALT is 150U/L, Albumin is 40 g/L, and Ferritin is 434 g/L. ‘Ultrasound reveals an echo bright appearance of the liver and gallstones in the gallbladder. The licely cause ofher liver disease is a) Alcoholic liver disease 'b) Drug induced hepatitis ¢) Gallstone disease d) Haemochromatosis Y © @) Non-alcchoic steatohepatitis (NASH. Question Explanation ‘The patient has a hepatic picture in contrast to Cholestasis. Ferritin level is not too high to be considered for haemockromatosis and is an acute phase reactant being typically incteased in any inflammatory process. NASH is very common and is typically encountered in obese patents, presenting with a hepatitic picture with or without jaundice. Echo bright liver suggests fatty change in the liver seen in NASH. Itwas previously termed idiopathic decompensated hepatitis and ifnot treated in terms of lowering BMI and reducing fat intake cam lead onto irreversible cirrhosis, Gellsiones are a distraction in this history, Mark this question & => Question Td ; 197485 Question 25 of 30 A.55 year old woman has an 18 month history of chest pain and dysphagia for both solids and liquids. She smokes 20 cigarettes per day and drirke 16 units of alcohol per week. Physical ezam was normal. Tae likely diagnosic ie a) Ackalasia +b) Bronchial neoplasma c) Oesophageal neoplasm 4) Oesophageal web ©) Pharyngeal pouch Question Explanation: ‘A longstanding history of dysphagia 10 both solids and Liquids suggest a functional rather than mechanical cause for the dysphagia, Hence a neoplasia or other obstructive lesion is unlikely. Chest pain is not a typical feature of a pharyngeal pouch. Achalasia, in which there is failure of oescphageal peristalsis and of relaxation of the lower oesophageal sphincter, typically causes the symptoms described above. Mark this question & => Question Td ; 197485 Question 25 of 30 A.55 year old woman has an 18 month history of chest pain and dysphagia for both solids and liquids. She smokes 20 cigarettes per day and drirke 16 units of alcohol per week. Physical ezam was normal. Tae likely diagnosic ie WV © a) Achalasia +b) Bronchial neoplasma c) Oesophageal neoplasm 4) Oesophageal web ©) Pharyngeal pouch Question Explanation: ‘A longstanding history of dysphagia 10 both solids and Liquids suggest a functional rather than mechanical cause for the dysphagia, Hence a neoplasia or other obstructive lesion is unlikely. Chest pain is not a typical feature of a pharyngeal pouch. Achalasia, in which there is failure of oescphageal peristalsis and of relaxation of the lower oesophageal sphincter, typically causes the symptoms described above. Mark this question & => Question Td : 197635 Question 26 of 30 A.50 year old male has loose stools. He is dehydrated, weak and in shock. He had previously been complaining oflarge stool volumes for a 1 month period. Stool colour was nomnal, There was no history oflaxative abuse and ne signicant past medical history. What is the lilcely diagnosis? a) Carcinoid syndrome b) Diabetic diarrhoea c) Gastrinoma ) Systemic mastocytosis e) ViPoma Question Explanation: “ViPomas are endocrine timours thet secrete excessive amounts of VIP32, which causes a distinct syndrome characterized by large- volume watery diarthoea, hypokalaemia, and dehydration. This syndrome is also called Verner-Morrison syndrome, pancreatic cholera, or WDHA syndrome for watery diarrhoea, hypokalaemia, and achlorhyciria, which some patients develop. The mean age of patients is 49 years; however, it can occur in children, and when + does is usually caused by a ganglioneuroma or ganglioneuroblastoma, A stool volume of <700 miLid excludes the diagnosis of VIPoma. Mark this question & => Question Td : 197635 Question 26 of 30 A.50 year old male has loose stools. He is dehydrated, weak and in shock. He had previously been complaining oflarge stool volumes for a 1 month petiod. Stool colour was normel. There was no history oflaxative abuse and no significant past medical history. What is the lcely diagnosis? a) Carcinoid syndrome +) Diabetic diarrhoea ) Gastrinoma 4) Systemic mastocytesie Y © e) ViPoma Question Explanation: “ViPomas are endocrine timours thet secrete excessive amounts of VIP32, which causes a distinct syndrome characterized by large- volume watery diarthoea, hypokalaemia, and dehydration. This syndrome is also called Verner-Morrison syndrome, pancreatic cholera, or WDHA syndrome for watery diarrhoea, hypokalaemia, and achlorhyciria, which some patients develop. The mean age of patients is 49 years; however, it can occur in children, and when + does is usually caused by a ganglioneuroma or ganglioneuroblastoma, A stool volume of <700 miLid excludes the diagnosis of VIPoma. ‘Mark this question <—& => Question Id : 197839) Question 27 of 30 Armale aged 20 years presents with acute, profuse, watery diarthoea with some blood after seturting from Tanzania He had been taking oral rehydration salts. The most appropriate treatment is which one of the fellowing? 2) Ciprofoxacin ') Loperamide ©) Metronidazole 4) Predrisclone ©) Vancomycin Question Explanation: ‘The most likely cause of such travellers diarrhoea is E. coli and hence ciprofloxacin is recommended for ft line antbiotic therapy (when needed) before stool culture results are available. Metronidazcle would be suitable tor Giardia infection but its course is usually more insidious. ‘Mark this question <—& => Question Id : 197839) Question 27 of 30 Armale aged 20 years presents with acute, profuse, watery diarthoea with some blood after seturting from Tanzania He had been taking oral rehydration salts. The most appropriate treatment is which one of the fellowing? Y © 2) Ciprofloxacin ') Loperamide ©) Metronidazole 4) Predrisclone ©) Vancomycin Question Explanation: ‘The most likely cause of such travellers diarrhoea is E. coli and hence ciprofloxacin is recommended for ft line antbiotic therapy (when needed) before stool culture results are available. Metronidazcle would be suitable tor Giardia infection but its course is usually more insidious. ‘Marle this question & => Question Ti : 212326 Question 28 of 30 A.5S year old man has abdominal pain, He suffers from chronic pancreatitis and often seeks pain control after drinking alcohol For the last couple of montis he has had chronic, dull pain, which is not as severe and located more over his left upper quadrant Previoudly, the pain wes mainly umbilical. His temperature is 990F, BP is 126/24, pulse is 9Bhnin, and respiratory rate is 1é/min, The spleen tip is palpable. His hematocrit is 32%, WBC countis 7000/mm3, MCV is 70 um3 and platelet court is 490,000/mm3. LFTs are unchanged from his baseline. Upper endoscopy shows profound gastric varices. The most likely diagnosis is 2) Bile duct obstruction ) Cirthosis ©) Hepatic vein thrombosis 6) Pancreatic pseudocyst ©) Splenic vein thrombosis Anewor [UEIRIRHBR) omer sors Explanation Report Question Explanation: Splenic vein thrombosis is rare but well described complication or chronic pancreatitis. The splenic vein runs along the posterior surface of the pancreas andis affected by the adjeceat inflammation, which can eventualyy induce thrombosis. This condition is a cause of upper gastrointestinal bleeding and wil present on endoscopy as the presence of gastric varices without any concomitant esophageal varices. This intitively makes sense, as the gastric mucosa drain into the splenic vein, which is obstructed, Treatment, when necessary consists of splenectomy. Bile duct obstruction (A) from strictures is not uncommon in patients with chronic pancreatiis. Expected findings include postprandial ebdominal pain and abnormal iver function tests. Cirrhosis (B) would result in portal hypertension with typical findings, including esophageal varices and stigmata ofliver disease (palmar erythema, caput medusa, spider angiomata, and ascites). Isolated gastric varices suggest increased pressure solely in venous drainage of the stomach, which can be explained by a splenic vein thrombosis. Hepatic vein thrombosis (C) is also known as Budd Chiari syndrome One would espect right upper quadrant pain, ascites, and hepatemegely. A pencreetic pseudocyst (D) is a common complication of chronic pancreatitis. These cysts are prone to infection or rupture but do not explain the formation of gastric varices. Error ‘Marle this question & => Question Ti : 212326 Question 28 of 30 A.5S year old man has abdominal pain, He suffers from chronic pancreatitis and often seeks pain control after drinking alcohol For the last couple of months he has had chronic, dull pain, which is not as sewere and located more over his left upper quadrant Previoudly, the pain wes mainly umbilical. His temperature is 990F, BP is 126/24, pulse is 9Bhnin, and respiratory rate is 1é/min, The spleen tip is palpable. His hematocrit is 32%, WBC countis 7000/mm3, MCV is 70 um3 and platelet court is 490,000/mm3. LFTs are unchanged from his baseline. Upper endoscopy shows profound gastric varices. The most likely diagnosis is 2) Bile duct obstruction ) Cirthosis ©) Hepatic vein thrombosis 6) Pancreatic pseudocyst Y © 2) Splenic vein thrombosis Anewor (UEIRNRN) omer usors Explanation Report Question Explanation: Splenic vein thrombosis is rare but well described complication or chronic pancreatitis. The splenic vein runs along the posterior surface of the pancreas andis affected by the adjeceat inflammation, which can eventualyy induce thrombosis. This condition is a cause of upper gastrointestinal bleeding and wil present on endoscopy as the presence of gastric varices without any concomitant esophageal varices. This intitively makes sense, as the gastric mucosa drain into the splenic vein, which is obstructed, Treatment, when necessary consists of splenectomy. Bile duct obstruction (A) from strictures is not uncommon in patients with chronic pancreatiis. Expected findings include postprandial ebdominal pain and abnormal iver function tests. Cirrhosis (B) would result in portal hypertension with typical findings, including esophageal varices and stigmata ofliver disease (palmar erythema, caput medusa, spider angiomata, and ascites). Isolated gastric varices suggest increased pressure solely in venous drainage of the stomach, which can be explained by a splenic vein thrombosis. Hepatic vein thrombosis (C) is also known as Budd Chiari syndrome One would espect right upper quadrant pain, ascites, and hepatemegely. A pencreetic pseudocyst (D) is a common complication of chronic pancreatitis. These cysts are prone to infection or rupture but do not explain the formation of gastric varices. Error ‘Marke this question e& => Question Td : 216987 Question 29 of 30 A.4)-year-old woman has intractable diahea. She also has severe heartburn. Her hands tum biue and then red in cold weather. There is retraction of the cof tissues around the mouth, with exposure of the upper teeth There are fine crackles at her hing bases “Which ofthe following would most licely improve this patient's diarrhea? a) Antibiotics b) Avoidance of ghien products ©) Avoidance of lactose 6) Oral loperamide 6) Pancreatic enzyme replacement Answer | Bioanatin | Other User's Explanation Report An Error Question Explanation: ‘The patient descrbed has scleroderma, as indicated by her sclerodactyly, Raynaud symptoms, and facial sclerosis. Additionally, she is compleining of severs heactburn, which is a result of esophagal dysmotiity. The dysmetilty, however, s present throughout the gastrointestinal tract: Dysmotility in the small bowel leads to dilatation and bacterial overgrowth secondary to stasis. Bacterial overgrowth in tum can lead to malabsorption of carbohydrates, which results in an osmotic diamhea as undigested carbohydrates are delivered to the calen, Avoidance of gluten or lactose would be reasonable ifthe patient had celiac sprue or lactose intolerance, respectively, however, gven her clear diagnosis of scleroderma, intestinal dysmotlity and bacterial overgrowth are more likely. Oral loperamide may improve her symptoms slightly; however, her underlying condition of bacterial overgrowth still needs to be addcessed, Pancreatic enzyme replacsment would be reasonable ifthe Patient had chronic pancreatitis or cystic Srosis. ‘Marke this question e& => Question Td : 216987 Question 29 of 30 A.4)-year-old woman has intractable diahea. She also has severe heartburn. Her hands tum biue and then red in cold weather. There is retraction of the cof tissues around the mouth, with exposure of the upper teeth There are fine crackles at her hing bases “Which ofthe following would most licely improve this patient's diarrhea? Y © a Antbioties b) Avoidance of guren products ©) Avoidance of lactose ) Oral loperamide e) Pancreatic enzyme replacement Answer | Bioanatin | Other User's Explanation Report An Error Question Explanation: ‘The patient descrbed has scleroderma, as indicated by her sclerodactyly, Raynaud symptoms, and facial sclerosis. Additionally, she is compleining of severs heactburn, which is a result of esophagal dysmotiity. The dysmetilty, however, s present throughout the gastrointestinal tract: Dysmotility in the small bowel leads to dilatation and bacterial overgrowth secondary to stasis. Bacterial overgrowth in tum can lead to malabsorption of carbohydrates, which results in an osmotic diamhea as undigested carbohydrates are delivered to the calen, Avoidance of gluten or lactose would be reasonable ifthe patient had celiac sprue or lactose intolerance, respectively, however, gven her clear diagnosis of scleroderma, intestinal dysmotlity and bacterial overgrowth are more likely. Oral loperamide may improve her symptoms slightly; however, her underlying condition of bacterial overgrowth still needs to be addcessed, Pancreatic enzyme replacsment would be reasonable ifthe Patient had chronic pancreatitis or cystic Srosis. ‘yew interface \ EB Syl 30 bey * CD www. interface.edu.pk/medical-exarrs/test-analysis, pho 2utid=14759 mac.omran - \ x \ 6? faredai9e4's pr x \ onlymeonly's pr. x ( ? females online. x \__) Lol x #38 Googe A-settngs [isgnin &.. solar dol qo wail buy OFaio ye joie Js [\ Freetotmal —» C3 other bookmar ‘Mark this question Question Td 217163] Total Questions “= Ps Question 30 of 30 2 A.60-year-old woman has abdominal swelling and vague pain. 3 Fhid thill and chifing dullness are positive. Paracentesis is performed. 4 5 2.8 gal. 6 [White blood cells:|100/mm 50% neutrophils| z [Serum a [White blood cells|[5, 700mm a [Hematocnit: 37% 2 [Piateles: 287.000 |AST: 2 9UH1. 9 |ALT: 310/L [Alburain: [B74 ‘Which of the following is the most appropriate next step in management? a) Adninistration of ceftriaxone for 5 days +t) Echocardiography c) Hepatic venography ) Liver biopsy ©) Pelvic ultrasound Question Explanation: ‘This patient has new-onset ascites of unclear etiology, especially given her unremarkable pest medical history and normal liver enzymes, Her serum-to-ascites albumin gradient (SAAG) is 1.1, which places the peritoned! fluid in the exudative category New- onset ascites in a S0-+year-old woman should always raise concem for an ovasian malignancy with peritoneal metastasis. Therefore, pelvic ultrasonography is the next step. Altematively, computed tomography of the abdomen and pelvis could be used. Ceftriaxone would be employed if the patient had spontaneous bacterial peritonitis, with a white blood cell count >500/mm and a neutrophil count >250/mm. This fuid does not meet these criteria Echocardiography would be reasonable ifright heart failure were the etiology of her ascites. However, given the lack of jugular ‘venous distension, this diagnosis is less likely. Aduitionally passive congestion from heart failure usually has SAAG >1.1 Hepatic venography would assess for the presence of hepatic vein thrombosis, Again, however, the SAAG would be >1.1 Liver biopsy is premature, inasmuch as firther imaging of the right upper quadrant would give architectural information about the liver. ‘AAA anathe the narmal leer fearkian tecte rane aoainet hanatin etialnow far the aeritee. BREBBREBBEBBRHKEKERREE xxXxXXXKXXKXKXKXKXKKXKXKKKKKKKKKKKKKKKK ‘yew interface \ EB Syl 30 bey * CD www. interface.edu.pk/medical-exarrs/test-analysis, pho 2utid=14759 mac.omran - \ x \ 6? faredai9e4's pr x \ onlymeonly's pr. x ( ? females online. x \__) Lol x #38 Googe A-settngs [isgnin &.. solar dol qo wail buy OFaio ye joie Js [\ Freetotmal —» C3 other bookmar ‘Mark this question Question Td 217163] Total Questions “= Ps Question 30 of 30 2 A.60-year-old woman has abdominal swelling and vague pain. 3 Fhid thill and chifing dullness are positive. Paracentesis is performed. 4 5 2.8 gal. 6 [White blood cells:|100/mm 50% neutrophils| z [Serum a [White blood cells|[5, 700mm a [Hematocnit: 37% 2 [Piateles: 287.000 |AST: 2 9UH1. 9 |ALT: 310/L [Alburain: [B74 ‘Which of the following is the most appropriate next step in management? a) Adninistration of ceftriaxone for 5 days +t) Echocardiography c) Hepatic venography ) Liver biopsy Y © e)Pelvic ultrasound Question Explanation: ‘This patient has new-onset ascites of unclear etiology, especially given her unremarkable pest medical history and normal liver enzymes, Her serum-to-ascites albumin gradient (SAAG) is 1.1, which places the peritoned! fluid in the exudative category New- onset ascites in a S0-+year-old woman should always raise concem for an ovasian malignancy with peritoneal metastasis. Therefore, pelvic ultrasonography is the next step. Altematively, computed tomography of the abdomen and pelvis could be used. Ceftriaxone would be employed if the patient had spontaneous bacterial peritonitis, with a white blood cell count >500/mm and a neutrophil count >250/mm. This fuid does not meet these criteria Echocardiography would be reasonable ifright heart failure were the etiology of her ascites. However, given the lack of jugular ‘venous distension, this diagnosis is less likely. Aduitionally passive congestion from heart failure usually has SAAG >1.1 Hepatic venography would assess for the presence of hepatic vein thrombosis, Again, however, the SAAG would be >1.1 Liver biopsy is premature, inasmuch as firther imaging of the right upper quadrant would give architectural information about the liver. ‘AAA anathe the narmal leer fearkian tecte rane aoainet hanatin etialnow far the aeritee. BREBBREBBEBBRHKEKERREE xxXxXXXKXXKXKXKXKXKKXKXKKKKKKKKKKKKKKKK Mark this question Question Td > Question 1 of 30 A 61-year-old lady presented with heartburn. She is known to have osteoporosis and has been talking alendronate for a mumber of years. Which of the folowing is the most likely cause of her symptoms? a) Ackalasia ) Esophagtis ©) Calcification of lower ecephageal ephinctor 4) Crush fracture disease ¢) Esophageal adenocarcinoma Question Explanation: Oral bisphosphonates seem to induce serious esophagtis in some patients, may result in gastritis and cause diarrhea When used as recommended, serious esophageal complications are few. Patients with known esophogeal disease (¢.g achalasia, siricture, Barrett's esophagus, severe reflux and scleroderma) should avoid taking oral bisphosphonates. Mark this question Question Td > Question 1 of 30 4 61-year-old lady presented with heartbum. She is known to have osteoporosis and has been talking alendronate for a number of years. Which of the folowing is the most likely cause of her symptoms? a) Achalasia Y © b) Esophagtis ¢) Caleification of lower esophageal sphincter 4) Crush frachwre disease ¢) Esophageal adenocarcinoma Question Explanation: Oral bisphosphonates seem to induce serious esophagtis in some patients, may result in gastritis and cause diarrhea When used as recommended, serious esophageal complications are few. Patients with known esophogeal disease (¢.g achalasia, siricture, Barrett's esophagus, severe reflux and scleroderma) should avoid taking oral bisphosphonates. ‘Marke this question << => Question Td : 49925 Question 2 of 30 A.72 year old man preseats with recurrent bilateral upper lobe pneumonia with mild dysphagia Physical examination is normal. The investigation of choice to attive at the diagnosis is which of the following? a) Barium swellow +) Semum protein electrophoresis o.cBC SESR Answer | Brplanation | Other User's Explanation Report An Error Question Explanation: Swallowing disorders are common, especially in the elderly, and may cause dehydration, weight loss, aspiration and airway obstruction, Aspiration is the passage of food or liquid through the vocal folds. Persons who aspirate are at increased risk for the occurrence of serione reapiratery cequelce, including airway obstruction and aspiration pneumonia. Diagnostic study of choice would be a barium swallow. The barium will show any structural defects such as sticzures, narrowing or tumors ‘Marke this question << => Question Td : 49925 Question 2 of 30 A.72 year old man preseats with recurrent bilateral upper lobe pneumonia with mild dysphagia Physical examination is normal. The investigation of choice to attive at the diagnosis is which of the following? ¥ © @ Barium swallow +) Semum protein electrophoresis oCBC SESR Answer | Brplanation | Other User's Explanation Report An Error Question Explanation: Swallowing disorders are common, especially in the elderly, and may cause dehydration, weight loss, aspiration and airway obstruction, Aspiration is the passage of food or liquid through the vocal folds. Persons who aspirate are at increased risk for the occurrence of serione reapiratery cequelce, including airway obstruction and aspiration pneumonia. Diagnostic study of choice would be a barium swallow. The barium will show any structural defects such as sticzures, narrowing or tumors Mark this question —& => Question Td : 51599 Question 3 of 30 All of the following intestinal complications more common to Crohn's disease than ulcerative colitis, except a) Fistula formation ) Perianal obstruction ©) Intestinal obstruction ad €) Toxic megacolon Question Explanatio Toxic megacolon (megacolon toxicum) isa lift threatening complication of other intestinal conditions. Ibis characterized by a much dilated colon (megacolon), accompanied by abdominal distension (bloating), and sometimes fever, abdomnal pain, or shock. Toxac megacolon is most ususlly a coraplcation of inflammatory bowel disease, such as ulcerative colitis and, more rarely, Crohn's disease, and some infections of the celon. Other forms of megacolon exist and can be congenital (present since birta, such as Hirscheprurg’'s disease) or associated with some forms of constipation, Post-surgical recurrence Mark this question —& => Question Td : 51599 Question 3 of 30 All of the following intestinal complications more common to Crohn's disease than ulcerative colitis, except a) Fistula formation ) Perianal obstruction o) Intestinal obstruction ad v © €) Toxic megacolon Question Explanatio Toxic megacolon (megacolon toxicum) isa lift threatening complication of other intestinal conditions. Ibis characterized by a much dilated colon (megacolon), accompanied by abdominal distension (bloating), and sometimes fever, abdomnal pain, or shock. Toxac megacolon is most ususlly a coraplcation of inflammatory bowel disease, such as ulcerative colitis and, more rarely, Crohn's disease, and some infections of the celon. Other forms of megacolon exist and can be congenital (present since birta, such as Hirscheprurg’'s disease) or associated with some forms of constipation, Post-surgical recurrence 3:2/2014 6:30:07 PM. J % worw.interface. x \ EB Sil sini ids x | Rdemadomran-\ x \ 6} faredat9e4's pr x (Gi onlymeanly’s pr x \ G? females anine x LS Tau] = CG www. interface.edu.k/med pho ?utid=14759 apps Ellcoogle Assetings [Sgn Zon woke dub Ege cmilliyse Gf ..cio gs agiall JS [Free Hotmall |-exarns/test-analy' * 8 Gi other bookmar ‘Matte this question = => (Question Ti : 55472 Question 4 of 30 A.46 year old employed house painter presents to the emergency department with 2 gradual onset of lethargy and weakness. & physical examination is remarkable for 4-tpiting edema of the lower extremities and a prommnent abdomen? Labceatory Finding Serum sodium. 122 mBafL QT 135-145) Serum osmolality.......260 mOsmikg H,O QV 230-296) Urine sodise. 5 mEq(L Urine ostnolality.........250 mOstnvkg HO ‘What is the most likely dicgnosis in this case? a) Syndrome of inappropriate antidiuretic hormone (SIADE) ) Primary polydipsia ©) Adrenel insufficiency 4) Citthosis €) Salt-wasting neptropathy Question Explanation: The individual described in this case has symptomatic hyponatremia Headache, mental confusion, nausea, and malaise are common. Seimires, stupor, and coma generally do not occur until sodium concentrations fall below 120 mEq/L. The presence of signficant peripheral edema in this patient indicates extracellaler fuid volume expansion, and his serum osmolality is low. In this situation hyp onairemia is usually a manifestation of an edematous state, such as hepatic cirthosis, congestive heart falure, or the nephritic syndrome, Although these patients have increase extracelular Suid, their intravascular Quid is depleted, and their body's atternpt 10 conserve sodium at the level ofthe kidney produces unne with a sodium concentration of 20mEq/L. ‘They have approximately increased arginine vasopressin (AVE) levels, resuiting in a urine osmolality thatis less than maximally dilute and often > 100 mOsmike #0. Pefients with the syndrome of inappropriate antidimretic hormone (SLADE) have normal volume status and urine sodium levels which are typically > 20 mEq/L. Patients with primary polydipsia often nave an underlying psychiatric disorder. They have normal volume status, and produce large volumes of very dilute urine (< 50 mOsmlcg 0). Patients with adrenal insufficiency typically have normal volume status, butmay be dehydrated, Patients with salt-wasting nephropathy are typically dehydrated despite producing urine with a sodium concentrate 20mEq/L, Total Questions ith is SBBARBEBBEB tk &E & I XxXxXXXXKXXKXXKKXKXXKKXKRKKKXKKKKKKKKKK 3:2/2014 6:30:07 PM. J % worw.interface. x \ EB Sil sini ids x | Rdemadomran-\ x \ 6} faredat9e4's pr x (Gi onlymeanly’s pr x \ G? females anine x LS Tau] = CG www. interface.edu.k/med pho ?utid=14759 apps Ellcoogle Assetings [Sgn Zon woke dub Ege cmilliyse Gf ..cio gs agiall JS [Free Hotmall |-exarns/test-analy' * 8 Gi other bookmar ‘Matte this question = => (Question Ti : 55472 Question 4 of 30 A.46 year old employed house painter presents to the emergency department with 2 gradual onset of lethargy and weakness. & physical examination is remarkable for 4-tpiting edema of the lower extremities and a prommnent abdomen? Labceatory Finding Serum sodium. 122 mBafL QT 135-145) Serum osmolality.......260 mOsmikg H,O QV 230-296) Urine sodise. 5 mEq(L ‘Urine osmolality. 250 mOsmkg HO ‘What is the most Uely diagnosis in this case? a) Syndrome of inappropriate antidiuretic hormone (SLADE) 'b) Primary polydipsia c) Adrenal insufficiency Y © & Cirthosis e) Salt-wasting nephropathy Question Explanation: The individual described in this case has symptomatic hyponatremia Headache, mental confusion, nausea, and malaise are common. Seimires, stupor, and coma generally do not occur until sodium concentrations fall below 120 mEq/L. The presence of signficant peripheral edema in this patient indicates extracellaler fuid volume expansion, and his serum osmolality is low. In this situation hyp onairemia is usually a manifestation of an edematous state, such as hepatic cirthosis, congestive heart falure, or the nephritic syndrome, Although these patients have increase extracelular Suid, their intravascular Quid is depleted, and their body's atternpt 10 conserve sodium at the level ofthe kidney produces unne with a sodium concentration of 20mEq/L. ‘They have approximately increased arginine vasopressin (AVE) levels, resuiting in a urine osmolality thatis less than maximally dilute and often > 100 mOsmike #0. Pefients with the syndrome of inappropriate antidimretic hormone (SLADE) have normal volume status and urine sodium levels which are typically > 20 mEq/L. Patients with primary polydipsia often nave an underlying psychiatric disorder. They have normal volume status, and produce large volumes of very dilute urine (< 50 mOsmlcg 0). Patients with adrenal insufficiency typically have normal volume status, butmay be dehydrated, Patients with salt-wasting nephropathy are typically dehydrated despite producing urine with a sodium concentrate 20mEq/L, Total Questions ith is SBBARBEBBEB tk &E & I XxXxXXXXKXXKXXKKXKXXKKXKRKKKXKKKKKKKKKK Mark this question & => Question Td Question 5 of 30 A21 year old student consults you because of diarrhea of 10 weeks duration, His ilness started in February, two weeks after a skiing trip in the Rocky Mountains, with nausea, anorexia and abdotninal discomfort. Shortly thereafter, his stools became freanent and watery. Physical examination is unremarkable. Complete blood count, inclading liferentia, is normal. Stool examination shows absence of fecal leukocytes. The mostlikely causative organism involved is a) Salmonella ') Giardia lamblia ©) Yersinia enterocolitica 4) Enterotoxigenic Escherichia coli e) Campylobacter jejuni Answer | Bioianation | Other User's Explanation _Report An Error Question Explanatior Gaardiasis is infection with the flagellated protozoan Giardia lambla. Infection can be asyraptomatic or cause symptoms ranging from intermittent flatalence to chronic malabsorption Waterborne transmission is the major source of giardiasis. Ttis common in campers Diagnesic is by identijing the organism in fresh stool or duodenal contents er by astays of Giardia entigen in stool. Treatment is with metrotidazole, titidazole, or ntazoxanide. Mark this question & => Question Td Question 5 of 30 A.21 year old student consults you because of diarrhea of 10 weeks daration, His ilness started in February, two weeks after a skiing trip in the Rocky Mountains, with nausea, anorexia and abdotninal discomfort. Shortly thereafter, his stools became freanent and watery. Physical examination is unremaricable. Complete blood count, incliding differentia, is normal. Stool examination shows absence of fecal leukocytes. The most likely causative organisia involved is a) Salmonella Y¥ © ») Giardia lamblia ©) Yersinia enterocolitica 4) Enterctoxigenic Escherichia coli e) Campylobacter jejuni Answer | Bioianation | Other User's Explanation _Report An Error Question Explanatior Gaardiasis is infection with the flagellated protozoan Giardia lambla. Infection can be asyraptomatic or cause symptoms ranging from intermittent flatalence to chronic malabsorption Waterborne transmission is the major source of giardiasis. Ttis common in campers Diagnesic is by identijing the organism in fresh stool or duodenal contents er by astays of Giardia entigen in stool. Treatment is with metrotidazole, titidazole, or ntazoxanide. ‘Marke this question & => Question Td : 82361 Question 6 of 30 Incase of which one of the following should the small bowel obstruction be managed with prompt surgical decompression? a) Incomplete obstruction. 'b) Obstruction associated with inflammatory bowel disease. c) Obstruction in a patient with multiple previous operations for obstruction. 4) Post-operative obstruction. ¢) Obstruction wrth strangulation Anewor (JEWIRNIHAN) other Users Explanation Report An Error Question Explanation: (Obstruction with strangulation should be decompressedd as soon as safely possible (after adequate rehydration and preoperative preparation), Incomplete obstruction (A) can be managed expectantly. Patients with inflammatory bowel disease (B) and those with multiple previous operations (C) should be managed conservatively as long as there is no evidence of strangulation. Postoperative patients (D) can often be managed with anzsogastric tube and TV hydration. In all cases, surgery would be indicated ifthe obstruction fails to respond to conservative means. ‘Marke this question & => Question Td : 82361 Question 6 of 30 Incase of which one of the following should the small bowel obstruction be managed with prompt surgical decompression? a) Incomplete obstruction. 'b) Obstruction associated with inflammatory bowel disease. c) Obstruction in a patient with multiple previous operations for obstruction. 4) Post-operative obstruction. Y © 2) Obstruction with strangulation. Anewor [UEWIRNIAN) other tsors Explanation Report An Error Question Explanation: (Obstruction with strangulation should be decompressedd as soon as safely possible (after adequate rehydration and preoperative preparation), Incomplete obstruction (A) can be managed expectantly. Patients with inflammatory bowel disease (B) and those with multiple previous operations (C) should be managed conservatively as long as there is no evidence of strangulation. Postoperative patients (D) can often be managed with anzsogastric tube and TV hydration. In all cases, surgery would be indicated ifthe obstruction fails to respond to conservative means. ‘Marke this question & => Question Td : 82482 Question 7 of 30 “What should be the first step in resuscitating a patient with a GI bleed? a) Place a large-bore intravenous line ) Fass anasogastric tube ©) Type and cross-maich blood. 4) Perform an upper GI endoscopy €) Administer an anoscopy. Answer { Biptanation | Other User's Explanation Report An Error Question Explanatior Before attempting to find the cause of the GI bleeding, its necessary to restore the intravascular volume. This is impossible without a large bore intravenouc line for Bui resuscitation. Once fluid resuscitation begine, a nasogastric tube (B) should be placed and blood typed and cross- matched (2), Eadoscopy (D) and anoscopy (B) should be deferred until after resuscitation ‘Marke this question & => Question Td : 82482 Question 7 of 30 “What should be the first step in resuscitating a patient with a GI bleed? Y © a) Place a large-bore intravenous line. +) Pass anacegastrio tubs c) Type and cross-maich blood 4) Perform an upper GI endoscopy ©) Administer an anoscopy. Answer { Biptanation | Other User's Explanation Report An Error Question Explanatior Before attempting to find the cause of the GI bleeding, its necessary to restore the intravascular volume. This is impossible without a large bore intravenouc line for Bui resuscitation. Once fluid resuscitation begine, a nasogastric tube (B) should be placed and blood typed and cross- matched (2), Eadoscopy (D) and anoscopy (B) should be deferred until after resuscitation Mark this question & => Question Td : 88739 Question 8 of 30 A middle aged woman presents with fatigue, pruritus, and steatorhea. Further evalvation demonstrates the presence of antimitochondial antibodies. Thic patient if not treated ie aleo at increazed risk: of developing which of the fellowing? 2) Markedly decreased serum alanine aminotransferase +t) Markodly decreased serum alkaline phosphatase ©) Markedly decreased serum biirvbin 4) Markedly increased serum alburnin, ©) Markedly increased serum cholesterol Question Explanation: ‘The patient's disease is primary biliary cimthosis, which is an autoimmune disease characterized by sclerosing cholangitis and cholangioltis. The most help clue in the question stem is the antimitochondrial antisody, which is present in over 90% of these paticnts. The course is usually slowly progressive over 5 to 25 years, Patients with advanced disease tend to develop profound hypercholesterolema and xanthomes. Serum alanine aminotransférese and serum aspartate attinotransferase may be elevated but they tend to be less specific for the diagnosis of primary biliary cisthosis. Patients are sometimes diagnosed when routine blood tests demonstrate an elevated alkaline phosphatase. Increased serum bilirubin is seen in late stage disease, and suggests that the patient may be close to needing a liver transplant. Serum albumin levels are often decreased as the disease progresses. Mark this question & => Question Td : 88739 Question 8 of 30 A middle aged woman presents with fatigue, pruritus, and steatorhea. Further evalvation demonstrates the presence of antimitochondial antibodies. Thic patient if not treated ie aleo at increazed risk: of developing which of the fellowing? 2) Markedly decreased serum alanine aminotransferase +t) Markodly decreased serum alkaline phosphatase ©) Markedly decreased serum biirvbin 4) Markedly increased serum alburnin, Y © 2) Markedly increased serum cholesterol Question Explanation: ‘The patient's disease is primary biliary cimthosis, which is an autoimmune disease characterized by sclerosing cholangitis and cholangioltis. The most help clue in the question stem is the antimitochondrial antisody, which is present in over 90% of these paticnts. The course is usually slowly progressive over 5 to 25 years, Patients with advanced disease tend to develop profound hypercholesterolema and xanthomes. Serum alanine aminotransférese and serum aspartate attinotransferase may be elevated but they tend to be less specific for the diagnosis of primary biliary cisthosis. Patients are sometimes diagnosed when routine blood tests demonstrate an elevated alkaline phosphatase. Increased serum bilirubin is seen in late stage disease, and suggests that the patient may be close to needing a liver transplant. Serum albumin levels are often decreased as the disease progresses. Mak this question => Question a: 91208 Question 9 of 30 A.44 year old man presents to his physician for a routine checkup. He was found to have mildly elevated liver transaminase levels No known history of allergy is present and he is taking no medicetions. Liver biopsy reveals an inflammatory process corfined to the portal areas without involvement of the periportal region. He has ao conslititional eymptoms. The most likely lagnesic ie a) Chronic active hepatitis 1b) Chronic persistent hepatitis ©) Hepatitis 4) Lupoid hepatitis ©) Hepatitis D Question Explanation: Chronic persistent hepatitis is usually benign and asymptomatic, Itis most common after hepatitis B. Liver biopsy reveals irflamnmation confined to poral areas only. Chrotic active hepatits is the more serious form of chronic hepatitis. It may progress 10 cirrhosis and liver failure; 20% occurs secondary to chronic hepatitis B infection, Portal areas are involved es well as periportal areas and piecemeal and bridging necrosis, Hepatitis A is usually acute, not chronic. Lupoid hepatitis is an autoimmune entity associzted with positive antinuclear antibodies and extraheptic manifestations. Prognosis is generally poor, but the concition may respond to treainent with steroids, Hepatiis D is associated with chronic hepatitis B and is called the Delta virus. Itis very virulent, on Report An Error Mak this question => Question a: 91208 Question 9 of 30 A.44 year old man presents to his physician for a routine checkup. He was found to have mildly elevated liver transaminase levels No known history of alletay is present and he is taking no medications. Liver biopsy reveals an inflammatory process confined to the portal areas without involvement of the periportal region. He has ao conslititional eymptoms. The most likely lagnesic ie a) Chronic active hepatitis Y © b) Chronic persistent hepatitis ©) Hepatitis 4) Lupoid hepatitis ©) Hepatitis D Question Explanation: Chronic persistent hepatitis is usually benign and asymptomatic, Itis most common after hepatitis B. Liver biopsy reveals irflamnmation confined to poral areas only. Chrotic active hepatits is the more serious form of chronic hepatitis. It may progress 10 cirrhosis and liver failure; 20% occurs secondary to chronic hepatitis B infection, Portal areas are involved es well as periportal areas and piecemeal and bridging necrosis, Hepatitis A is usually acute, not chronic. Lupoid hepatitis is an autoimmune entity associzted with positive antinuclear antibodies and extraheptic manifestations. Prognosis is generally poor, but the concition may respond to treainent with steroids, Hepatiis D is associated with chronic hepatitis B and is called the Delta virus. Itis very virulent, on Report An Error Mark this question <=> Question Id : 91922 Question 10 of 30 A.43 year old Japanese woman presents with mid-epigastric tendemess, which is relieved with ingestion of milk. No associated symptoms of weightloss, jaundice, or change in appetite are present. She is a smoker but does not take any nonsteroidal drugs or aspirin, Urea breath test war postive. The best treatment for this patient ie a) H2 blocker medication 6) Chemotherapy ©) Froton-pump inhibitor plus antibiotics © Bland diet ©) Antacids Question Explanation: This patient has Helicobacter pylori-related duodenal ulcer. In order to prevent recurrence of the ulcer, antibiotics are given to eradicate this Gram-negative organism H2 blockers (A) used to be given alone for the treatment of ulcers, but if H. pylon is documented, then antibiotics are to be given as well, along with a proton-pump inhibitor to prevent acid secretion. Chemotherapy (B) is given for gastric cancer. which this patient does not have. given that there are no symptoms of weight loss or appetite change. A bland diet (D) does not alleviate the symptoms of an ulcer. Antacids (E) alone wil not cure an ulcer. They are heipfil to relieve heartburn and gastric-reflux disease Report An Error Mark this question <=> Question Id : 91922 Question 10 of 30 A.43 year old Japanese woman presents with mid-epigastric tendemess, which is relieved with ingestion of milk. No associated syinotomns of weight loss, jaundice, or change in appetite are present. She is a smoker bur does not take any nonsteroidal drags or aspirin, Urea breath test war postive. The best treatment for this patient ie a) H2 blocker medication 6) Chemotherapy Y © ©) Froton-pump inhibitor plus antibiotics ©) Bland diet ©) Antacids Question Explanation: This patient has Helicobacter pylori-related duodenal ulcer. In order to prevent recurrence of the ulcer, antibiotics are given to eradicate this Gram-negative organism H2 blockers (A) used to be given alone for the treatment of ulcers, but if H. pylon is documented, then antibiotics are to be given as well, along with a proton-pump inhibitor to prevent acid secretion. Chemotherapy (B) is given for gastric cancer. which this patient does not have. given that there are no symptoms of weight loss or appetite change. A bland diet (D) does not alleviate the symptoms of an ulcer. Antacids (E) alone wil not cure an ulcer. They are heipfil to relieve heartburn and gastric-reflux disease Report An Error ‘Marke this question e& => Question Ta : 93097 Question 11 of 30 ‘Which treatment of variceal bleeding is contraindicated when managing an acute bleed? a) Vasopressin ) Balloon tamponade 6) Sslevotherapy 4) Beta-blockers ¢) Portacaval shut Avewor [UBENSRER) ctnor usar Explan Question Explanation: Emergent shunt surgery carries a 50% mortality sate and is contraindicated during the acute management. Beta-blockers (D) have shown some success in preventing recumrent bleeding, The other answer choices are effective management of acute bleeding Report An Error ‘Marke this question e& => Question 11 of 30 ‘Which treatment of variceal bleeding is contrainciceted when managing an acute bleed? a) Vasopressin ) Balloon tamponade 6) Sslevotherapy 4) Beta-blockers Y © ¢) Portacavel shunt Avewor (UBNSHER) ctnor Ussre Explanation Report An Eror Question Explanation: Question Td : 93097 Emergent shunt surgery carries a 50% mortality sate and is contraindicated during the acute management. Beta-blockers (D) have shown some success in preventing recumrent bleeding, The other answer choices are effective management of acute bleeding Mark this cuestion ez Question Id : 94282 Question 12 of 30 A 60 year fernale with numbness in the lower extremities and macrocytosis has a normal serum folate level and a serum B12 level of 200pgimL (150-800). The laboratory finding that would confitm the diagnosis of B12 deficiency is a) Hlevated methylmalonic acid ) Hlevated angiotensin converting enzyme ¢) Hlevated free erythrocyte protoporphyrin 4) Low haptoglobin ¢) Low homocysteine Question Explanation ‘This patient has several cinical features of vitamin B12 deficiency. Some patients with significant vitamin B12 deficiency have levels in the lower range of normal, as this patient does. Vitamin B12 is a cofactor in the synthesis of both methiorine and succinyl coenzyme ‘A, and vitamin B12 deficiency leads to the accumulation of methylmalonic acid and homocysteine, which are the precursors ofthese compounds. An elevated level ofthese substances is therefore more sensitive than alow vita B12 level for vitamin B12 deficiency. Homocysteine is also elevated in folic acid deficiency, however, so a methykmalonic acid level is recommended if vitarnin B12 deficiency is a concern and semum vitamin’B12 levels are 150-400) Pajral ‘A reduced haptoglobin level is usefid to confirm hemolytic anemia. An elevated free erythrocyte protoporphyrin level may oscur in tead poisoning or iron deficiency. An elevated angiotersion convertin enzyme level is found in sarcoidosis Mark this cuestion ez Question Id : 94282 Question 12 of 30 A 60 year female with numbness in the lower extremities and macrocytosis has a normal serum folate level and a serum E12 level of 200pgimL (MT 150-800). The laboratory finding that would confitm the diagnosis of B12 deficiency is V © a) Hevated methylmalonic acid ) Hlevated angiotensin converting enzyme o) Elevated free erythrocyte protoporpyrin 4) Low haptoglobin ¢) Low homocysteine Question Explanation ‘This patient has several cinical features of vitamin B12 deficiency. Some patients with significant vitamin B12 deficiency have levels in the lower range of normal, as this patient does. Vitamin B12 is a cofactor in the synthesis of both methiorine and succinyl coenzyme ‘A, and vitamin B12 deficiency leads to the accumulation of methylmalonic acid and homocysteine, which are the precursors ofthese compounds. An elevated level ofthese substances is therefore more sensitive than alow vita B12 level for vitamin B12 deficiency. Homocysteine is also elevated in folic acid deficiency, however, so a methykmalonic acid level is recommended if vitarnin B12 deficiency is a concern and semum vitamin’B12 levels are 150-400) Pajral ‘A reduced haptoglobin level is usefid to confirm hemolytic anemia. An elevated free erythrocyte protoporphyrin level may oscur in tead poisoning or iron deficiency. An elevated angiotersion convertin enzyme level is found in sarcoidosis Marl this question — => Question Td : 94627 Question 13 of 30 A. 66-year-old men has sudden onset of severe periumblical abdominal pain, diarrhea and vomiting. On exem his abdomen is diffusely tender, pulse rapid irregularly irreguler, pale forearm and no palpable brackial pulse. Urine and stool are positive for blood chemical testing, His Hb is 16 4g/dL and WBC 25000/mm3. What imaging procedure would specifically diagnose his abdominal pain? a) Intravenous pyelozraphy (IVP) b) Sonography ofthe abdominal aorta c) A barium enema 4) Contrast venography €) Celiac and mesenteric arteriography Answer | Bavienaton | Other User's Explanation Report An Error Question Explanation: The sudden onset of severe abdominal pain, vomiting, and diarrhea in a patient with a cardiac source of emboli and evidence of a separate embolic event makes superior mesenteric artery embolization lcely. In this case, evidence of brachial artery embolus and a cardiac rhythm indicating atrial fbilletion suggest the diagnosis. Some patients may heve a surprisingly normal abdominal examination in spite of severe pain. Microscopic hematuria and blood in the stool may both occur with emboliztion. Severe leukocytosis is present in more than two thirds of patients with this problem. Diagnostic confirmation by angiography is recommended, Immediate embolectomy with removal of the propogated clot can then be accomplished and a decision made regarding whether or not the iniestine should be resected, A second procedure may be scheduled to reevaluate intestinal viability. Marl this question — => Question Td : 94627 Question 13 of 30 A.66-year-old men has sudden onset of severe periumblical abdominal pain, diarrhea and vomiting, On exem his abdomen is diffusely teader, pulse rapid irregularly irreguler, pale forearm and no palpable brachial pulse. Urine and stool are positive for blood chemical testing. His Hb is 16.4g/dL and WBC 25000/mm3. What inaging procedure would specifically diagnose his abdominal pain? a) Intravenous pyelozraphy (IVP) b) Sonography of the abdominal aorta c) A barium enema d) Contrast venography Y © e) Celiac and mesenteric arteriography Answer | Bavienaton | Other User's Explanation Report An Error Question Explanation: The sudden onset of severe abdominal pain, vomiting, and diarrhea in a patient with a cardiac source of emboli and evidence of a separate embolic event makes superior mesenteric artery embolization lcely. In this case, evidence of brachial artery embolus and a cardiac rhythm indicating atrial fbilletion suggest the diagnosis. Some patients may heve a surprisingly normal abdominal examination in spite of severe pain. Microscopic hematuria and blood in the stool may both occur with emboliztion. Severe leukocytosis is present in more than two thirds of patients with this problem. Diagnostic confirmation by angiography is recommended, Immediate embolectomy with removal of the propogated clot can then be accomplished and a decision made regarding whether or not the iniestine should be resected, A second procedure may be scheduled to reevaluate intestinal viability. ‘Mark this question = => Question Te : 101502 Question 14 of 30 A.62 year cld man with a history of alcohol abuse presents with the new onset of ascites. All ofthe following statements are true, except 2) Sali restriction is the most important initial wreatment measure +) Bacterial peritonitis may be present in the absence of abdominal pain, tendemess or fever ©) Abdominal paracentesis is indicated for diagnostic purposes 6) Lasixis the diuretic of choice Answer | Explanation Other User's Explanation Report An Error Question Explanation: Ascites is the condition in which there is free fuid in the peritoneal cavity. The most common cause is portal hypertension, Symptoms usually result from abdominal distention, Diagnosis is based on physical examination, ultrasound, or CT. Treatments include bed rest, dictary Na zestriction, dineetics, and therapeutic paracentesis, Asctic Bid can become infected (spontancous bacterial peritonitis), often with pain and fever. Diagnosis of infection involves analysis and cuiture of ascitic fuid, This infection is treated with antbiotics ‘Mark this question = => Question Te : 101502 Question 14 of 30 A. 62 year old man with a history of alcohol abuse presents with the new onset of ascites, All ofthe following statements are true, except 2) Salt restriction is the most important initial reatment measure ¥ © b) Bacterial peritoritis may be present in the absence of abdominal pain, tendemess or fever ¢) Abdominal paracentesis is indicated for diagnostic purposes 6) Lasixis the diuretic of choice Answer | Explanation Other User's Explanation Report An Error Question Explanation: Ascites is the condition in which there is free fuid in the peritoneal cavity. The most common cause is portal hypertension, Symptoms usually result from abdominal distention, Diagnosis is based on physical examination, ultrasound, or CT. Treatments include bed rest, dictary Na zestriction, dineetics, and therapeutic paracentesis, Asctic Bid can become infected (spontancous bacterial peritonitis), often with pain and fever. Diagnosis of infection involves analysis and cuiture of ascitic fuid, This infection is treated with antbiotics “Marke this question & => Question Td : 105522 Question 15 of 30 A.50 year old know TV chug abuser presents with endocarditis, After treatment with antibiotics, he develops watery diarthea associated with crampy abdominal pain. This pationt is ETIV negative. The most likely cause of his diarshea is a) CMV colitis b) Ulcerative colitis ©) Clostridium difficile colitis 4) Shigellosis 8) Viral gastroenteritis Question Explanation CC. difficile colts is commonly associated after treatment with antibiotics. In severe cases the colon shows pseudomembranes, (Clostridium difficile is a Gram-negative organism, and it flourishes in the gut after the body's normal flora is climinated by antibiotics such as cephalosporins and clindamycin. CMV colitis is commonly associated with diaethea in immnocompromised patients Ulcerative colitis is a chronic condition, and it would be associated with mucous and blood in the stool Shigellosis is associated with ingestion of infected food or water with this Gram negative bacteria, It would be unusual for this man to get a viral gastroenteritis, during his hospital stay for weaent of endocarditis “Marke this question & => Question Td : 105522 Question 15 of 30 A.50 year old know TV drug abuser presents with endocarcitis. After treatment with antibiotics, he develops watery diarthea associated with crampy abdominal pain. This patient is HIV negative. The most likely cause of his diarthea is: a) CMV colitis b) Ulecrative colitis ¥ © ©) Clostridium difficile colitis @) Shigellosis ©) Viral gastroenteritis Question Explanation CC. difficile colts is commonly associated after treatment with antibiotics. In severe cases the colon shows pseudomembranes, (Clostridium difficile is a Gram-negative organism, and it flourishes in the gut after the body's normal flora is climinated by antibiotics such as cephalosporins and clindamycin. CMV colitis is commonly associated with diaethea in immnocompromised patients Ulcerative colitis is a chronic condition, and it would be associated with mucous and blood in the stool Shigellosis is associated with ingestion of infected food or water with this Gram negative bacteria, It would be unusual for this man to get a viral gastroenteritis, during his hospital stay for weaent of endocarditis Mark this question & => Question Td : 109716 Question 16 of 30 A 32 year old alcoholic male is noted to have anemia, What is the most probable type of anemia and its associated treatment? a) Tron deficiency anemia oral iron supplementation. b) Tron deficiency anetia-vitamin B12 and folate c) Megaloblastic anemia-oral iron supplementation. 4) Megaloblastic anemia-vitamin B 12 and folate. €) Microcytic anemia secondary to chronic blood less-RBC transfusion. Question Explanation: “The most common anemia affecting alcoholics is megaloblastic anemia ‘This occurs secondarily to their reliance on alcohol for a majority of ther caloric intake and the absence of green. leafy vegetables from their diet. Megaloblastic anemia can be treated with vitamin B12 and folate supplementetion, Tron deficiency anemia, a microcytic anemia, is not as commen in this population. Oral iron supplementation is the initial treatment of thet condition, which is often the resuit of poor diet or inability tp absorb iron from the det Option Eis not a good answer, as there is no evidence to suggest a chronic Gl bieed in this patient, although alecholics with liver carhosis can be susceptible to acute GI bleeds secondary to ruptured varicosities, The anemia that occurs secondary to blood loss is normocytic, not microcytic. Mark this question & => Question Td : 109716 Question 16 of 30 A 32 year old alcoholic male is noted to have anemia, What is the most probable type of anemia and its associated treatment? a) Ikon deficiency anemia oral iron supplementation. b) Tron deficiency anetia-vitamin B12 and folate c) Megaloblastic anemia-oral iron supplementation. JV © 4) Megaloblastic anemia-vitamin B12 and folate. ¢) Microcytic anemia secondary to chronic blood loss-RBC transfusion. Question Explanation: “The most common anemia affecting alcoholics is megaloblastic anemia ‘This occurs secondarily to their reliance on alcohol for a majority of ther caloric intake and the absence of green. leafy vegetables from their diet. Megaloblastic anemia can be treated with vitamin B12 and folate supplementetion, Tron deficiency anemia, a microcytic anemia, is not as commen in this population. Oral iron supplementation is the initial treatment of thet condition, which is often the resuit of poor diet or inability tp absorb iron from the det Option Eis not a good answer, as there is no evidence to suggest a chronic Gl bieed in this patient, although alecholics with liver carhosis can be susceptible to acute GI bleeds secondary to ruptured varicosities, The anemia that occurs secondary to blood loss is normocytic, not microcytic. ‘Marke this question << => Question Td : 131447 Question 17 of 30 All oF the following predispose to gastroesophageal reflux disease, EXCEPT a) Cigarette smoking ) Recumbency ©) Ascites 9) Pregnancy ©) Furdeplcation Answer | Explanation Other User's Explanation Report An Error Question Explanatio Fundoplication, an antireflux procedure in which the gastric fincus is wrapped around the esophagus, increases the lower esophageal sphincter pressure. Gastric contents are prone to refhur when gestric volume is increased. Recumbency and bending predispose to refx because gastric contents are located near the gastroesophageal junction, Conditions that increase the gastric pressure, such as obesity tight binders, or ascites, also predispose to reflux. Redlux occurring during pregnancy may result either from increased intra- abdominal pressure or from the effects of estrogen and progesterone on the lower esophageal sphincter. Incompetence of the lower esophageal sphincter due to cigarette stoking or other smooth snuscle relaxants, also result in refx. ‘Marke this question << => Question Td : 131447 Question 17 of 30 All oF the following predispose to gastroesophageal reflux disease, EXCEPT a) Cigarette smoking ) Recumbency ©) Ascites 9) Pregnancy Y © 2) Fundoplication Answer | Explanation Other User's Explanation Report An Error Question Explanatio Fundoplication, an antireflux procedure in which the gastric fincus is wrapped around the esophagus, increases the lower esophageal sphincter pressure. Gastric contents are prone to refhur when gestric volume is increased. Recumbency and bending predispose to refx because gastric contents are located near the gastroesophageal junction, Conditions that increase the gastric pressure, such as obesity tight binders, or ascites, also predispose to reflux. Redlux occurring during pregnancy may result either from increased intra- abdominal pressure or from the effects of estrogen and progesterone on the lower esophageal sphincter. Incompetence of the lower esophageal sphincter due to cigarette stoking or other smooth snuscle relaxants, also result in refx. Mark this question e& => Question Td : 133338 Question 18 of 30 ‘Which complication of liver failure is effectively managed by water restriction? 2) Hypoglycemia ) Hemorrhage ©) Hyponatremia 4) Hypokalemia ©) Respiratory allcalosis Anower [JERIRIBHRN) otnerveor Exp Question Explanation: ‘Treatment of lver falure consists of supportive therapy and treatment of ts complications, Vitamin K is used to halp control hhemomhage due to decreased syathesis of clotting factors. Water restriction is effective management of the hyponatremia that results from impaired free water clearance. Fresh fozen plasmais necessary f serious bleeding results ftom disseminated intravascular coagulation Hypoglyceriais treated with gucose infusion. Hypakcalemia is treated with potassinm supplementation. Respiratory alkalosis does not reqnire treatment Report An Error Mark this question e& => Question Td : 133338 Question 18 of 30 ‘Which complication of liver failure is effectively managed by water restriction? 2) Hypoglycemia ) Hemorrhage ¥ © ©) Hyponatremia & Hypokalemia «) Respiratory alkalosis Anower [JERIRIBHRN) otnerveor Exp Question Explanation: ‘Treatment of lver falure consists of supportive therapy and treatment of ts complications, Vitamin K is used to halp control hhemomhage due to decreased syathesis of clotting factors. Water restriction is effective management of the hyponatremia that results from impaired free water clearance. Fresh fozen plasmais necessary f serious bleeding results ftom disseminated intravascular coagulation Hypoglyceriais treated with gucose infusion. Hypakcalemia is treated with potassinm supplementation. Respiratory alkalosis does not reqnire treatment Report An Error ‘Marke this question & => Question Td : 136502 Question 19 of 30 A previously heathy 21 years old woman began having nausea and vorniting 24 hours after ingestion of chicken followed by watery clarthea that has lasted three days with a fever of 103°F. She was diagnosed with Salmonella gastroenteritis. What is the most important aspect of management? @) Avoid all treatment, Disease is self limiting and benign. ) Fluid and electroiyte replacement ©) Antbiotic for 14 days. 6) Antimotiity medication. ©) Exploratory laparotomy. Question Explanation: Salmonella gastroenterts is usually self limiting, although it can lead to severe dehydration, The main principal of management consists cf fhid and electrolyte replacement Antitiotc therapy is only indicated for patients at risk for developing metastatic disease Antimotilty medication is contraindicated in the management of Selmonella gastrocntcrits, Exploratory laperotomy is not pert of the management of this disease. Report An Error ‘Marke this question & => Question Td : 136502 Question 19 of 30 A previously healthy 21 years old woman began having nausea and vomiting 24 hours after ingestion of chicken followed by watery diarrhea that has lasted three days with a fever of 103°. She was diagnosed with Salmonella gastroenteritis. What isthe most important aspect of management? 2) Avoid all treatment. Disease is self limiting and benign. Y © b) Fluid and electroiyte replacement ©) Antibiotic for 14 days, 4) Aniimotiliy medication ©) Exploratory laparotomy. Question Explanation: Salmonella gastroenterts is usually self limiting, although it can lead to severe dehydration, The main principal of management consists cf fhid and electrolyte replacement Antitiotc therapy is only indicated for patients at risk for developing metastatic disease Antimotilty medication is contraindicated in the management of Selmonella gastrocntcrits, Exploratory laperotomy is not pert of the management of this disease. Report An Error ‘Marke this question & => Question Td : 140040 Question 20 of 30 A patient with hepatitis is undergoing evaluation for progressive renal failure, Liver biopsy shows inflammation of vessel walls with nodular areas of fbrinoid necrosis. Investigations are negetive for p-ANCA and c-ANCA. Hepatitis type that is associated with his current condition is which one of the following? a) Hepatiis A b) Hepatitis B ©) Hepatitis 4) Hepatitis D ©) Hepatiis E Anewor [UEIINSHIR) omer sors Explanation Repost An Error Question Explanatios Hepatitis B is associated with polyarteritis nodosa (PAN), anecrotizing vasculitis of small and medium sized muscular arteries involving all organ systems, these patients often progress to renal failure. A significant percentage of patients with PAN have hepatitis B antigenema. They also have circulating immune complexes containing hepatitis B antigen. Hepatitis B antigen, IgM, and complement can be demonstrated! in blood vessel walls. p-ANCA (perinuclear antineutrophil cytoplasmic antibody) is negative in classic PAN, but is positive in microscopic polyangitis, which is viewed by some to be a vasiant of PAN. c-ANCA (cytoplasmic artinewrophil cytoplasmic antibody) is a marker for Wegener granulomatosis, Hepatitis A is transmied by the fecal oral route and is not associated with a vascultis, carrier stete, or increased risk for hepatitis C. Hepatitis C accounts for 50 to 70% of chronis hepatitis. Chronic hepatitis C can he associated with immnne complex mediated extrahepatic complications, but is less common than hepatitis B. Hepatitis C docs have a significent association with essential mixed cryoglobulinemia, which presents with glomnerulonephritis, arthralgias, hepatosplenomegely, and lymphadenopathy, in addition to a vasculitis. However, there is no association with p-ANCA. Hepatitis D requires that the patient be co-infected with hepatitis B. As such, hepatitis D does not independently cause disease but can produce a worsening of the liver disease. Hepatitis E resembles hepatits A due to its primarly enteric mode of spread, It is not associated with chronic hepatitis and does not have a predisposition for vasculitis. ‘Marke this question & => Question Td : 140040 Question 20 of 30 A patient with hepatitis is undergoing evaluation for progressive renal failure, Liver biopsy shows inflammation of vessel walls with nedular areas of Bbrinoid necrosis, Investigations are negetive for p-ANCA and c-ANCA. Hopattis type that is associated with his current condition is which one of the following? a) Hepatiis A Y © b) Hepatitis B ©) Hepatiis @ 4) Hepattis D ©) Hepatic E Anewor [UEIINSHIR) omer sors Explanation Repost An Error Question Explanatios Hepatitis B is associated with polyarteritis nodosa (PAN), anecrotizing vasculitis of small and medium sized muscular arteries involving all organ systems, these patients often progress to renal failure. A significant percentage of patients with PAN have hepatitis B antigenema. They also have circulating immune complexes containing hepatitis B antigen. Hepatitis B antigen, IgM, and complement can be demonstrated! in blood vessel walls. p-ANCA (perinuclear antineutrophil cytoplasmic antibody) is negative in classic PAN, but is positive in microscopic polyangitis, which is viewed by some to be a vasiant of PAN. c-ANCA (cytoplasmic artinewrophil cytoplasmic antibody) is a marker for Wegener granulomatosis, Hepatitis A is transmied by the fecal oral route and is not associated with a vascultis, carrier stete, or increased risk for hepatitis C. Hepatitis C accounts for 50 to 70% of chronis hepatitis. Chronic hepatitis C can he associated with immnne complex mediated extrahepatic complications, but is less common than hepatitis B. Hepatitis C docs have a significent association with essential mixed cryoglobulinemia, which presents with glomnerulonephritis, arthralgias, hepatosplenomegely, and lymphadenopathy, in addition to a vasculitis. However, there is no association with p-ANCA. Hepatitis D requires that the patient be co-infected with hepatitis B. As such, hepatitis D does not independently cause disease but can produce a worsening of the liver disease. Hepatitis E resembles hepatits A due to its primarly enteric mode of spread, It is not associated with chronic hepatitis and does not have a predisposition for vasculitis. 3i2/2014 6:33:56 PM J % worw.interface. x \ EB Sil sip ils X | Wd (1 unread) en x \ 6) faredat964's pr x (Gi onlymennlys pr x \ 6? females cnine x LS Tau = Di www. interface. edu.pk /medicel-exarns/test-analysis pho ®ubd=14759 apps EJ cocgle \ seungs Pisonin uu. wbbe dul © yo umilléaye Olutio ue fosoil JS _[) Free Hotmall * 8 Gi other bookmar ‘Matte this question <= => Question Td : 142046 Question 21 of 30 An autopsy is performed on a male who suddealy began vomiting voluminous quantities of blood and bled to death, His stomach’s photograph is shown below. The major pathogenic attribute ofthe most licely infectous agent in this condition is also shared by which ongenisin? a) Bacillus anthracis )Protous mirabilis ©) Staphylococcus aureus d) Uropathogeaic Escherichia coil @) Vibrio cholerae Answer (Banister) Other User's Explanation Report An Error Question Explanation: ‘The photograph shows a stomach with a perforated peptic ulcer. Perforation of @ peptic ulcer is potertially fata, because of either peritonitis with sepsis or sudden exsanguination (ifthe perforation damages one of the many arteries of the stomach). Peptic ulcer disease gastitic, and pescibly gastric carcinoma and gectric lymphoma have been strongly ascociated with Heloobacter pylori colonization of the mucus layer covering the gastric mucosa The most important pathogenic atribute of H. pyloriis its production of urease, which neutralizes the acidic pH ofthe stomach and allows the organism to invade the mucosal surface. Bacillus anthracis shares the atritute of production of superoxide dismutase with Helicobacter pylori. This prevents phagocytic killing by neutralizing oxygen metabolites, but is net the most imep ortant pethogenio attribute of sither orgeniam. Staphylococcus aureus shares the attribute of catalase, production with Helicobacter pylori, This prevents phagocytosis by neutralizing peroxides, butis not the most imp ortant pathogenic attribute of either organism. Uropathogenic Escherichia coli share the attribute of motlity via flagella with Helicobacter pylori Flagella in E.coli allow the organism to move against the flow of urine into the urinary bladder, andiin H. pylori allow the orgeniam to penetrate the gastric mucus layer and be protected from the acid environmeat These are not the mostimportant pathogenic attributes of either organism Vibsio cholerae shares the attribute of mucinase production with HL pylon. This disrupts the mucus layer covering the stomach or intestine, but is not the major pathogenic attribute for either Total Questions SBRBSBBREREBBSBKRRKEGERREE xxxXKXKXKXKKXKKKKKKKKKKKKKKKKKKKKKK 3i2/2014 6:33:56 PM J % worw.interface. x \ EB Sil sip ils X | Wd (1 unread) en x \ 6) faredat964's pr x (Gi onlymennlys pr x \ 6? females cnine x LS Tau = Di www. interface. edu.pk /medicel-exarns/test-analysis pho ®ubd=14759 apps EJ cocgle \ seungs Pisonin uu. wbbe dul © yo umilléaye Olutio ue fosoil JS _[) Free Hotmall * 8 Gi other bookmar ‘Matte this question <= => Question Td : 142046 Question 21 of 30 An autopsy is performed on a male who suddealy began vomiting voluminous quantities of blood and bled to death, His stomach’s photograph is shown below. The major pathogenic attribute ofthe most licely infectous agent in this condition is also shared by which ongenisin? a) Bacillus anthracis Y¥ © b) Proteus mirabilis ©) Staphylococcus aureus d) Uropathogeaic Escherichia coil @) Vibrio cholerae Answer (Banister) Other User's Explanation Report An Error Question Explanation: ‘The photograph shows a stomach with a perforated peptic ulcer. Perforation of @ peptic ulcer is potertially fata, because of either peritonitis with sepsis or sudden exsanguination (ifthe perforation damages one of the many arteries of the stomach). Peptic ulcer disease gastitic, and pescibly gastric carcinoma and gectric lymphoma have been strongly ascociated with Heloobacter pylori colonization of the mucus layer covering the gastric mucosa The most important pathogenic atribute of H. pyloriis its production of urease, which neutralizes the acidic pH ofthe stomach and allows the organism to invade the mucosal surface. Bacillus anthracis shares the atritute of production of superoxide dismutase with Helicobacter pylori. This prevents phagocytic killing by neutralizing oxygen metabolites, but is net the most imep ortant pethogenio attribute of sither orgeniam. Staphylococcus aureus shares the attribute of catalase, production with Helicobacter pylori, This prevents phagocytosis by neutralizing peroxides, butis not the most imp ortant pathogenic attribute of either organism. Uropathogenic Escherichia coli share the attribute of motlity via flagella with Helicobacter pylori Flagella in E.coli allow the organism to move against the flow of urine into the urinary bladder, andiin H. pylori allow the orgeniam to penetrate the gastric mucus layer and be protected from the acid environmeat These are not the mostimportant pathogenic attributes of either organism Vibsio cholerae shares the attribute of mucinase production with HL pylon. This disrupts the mucus layer covering the stomach or intestine, but is not the major pathogenic attribute for either Total Questions SBRBSBBREREBBSBKRRKEGERREE xxxXKXKXKXKKXKKKKKKKKKKKKKKKKKKKKKK Mark this question & => Question Td : 151382 Question 22 of 30 Examination of a 40 year old female with systemic sclerosis who presents with malaise is significant for jaundice, and serum chemistry studies show moderately elevated serum allaline phosphatase, whie AST and ALT are only mirimally elevated. Which autoantibodies would be most helpfil in elucidating the Ukcely ctiology of her lirer disease? a) Anti-double-stranded DNA. +) Anti-mitochondlrial antibodies ©) Antiphospholipid antibodies ) Anti-selfIgG ©) Ant-smooth muscle antibodies Question Explanation: Primary biliary citrhosis is strongly associated with systemic sclerosis (scleroderma). This disorder characteristically affects intrahepatic bile ucts more than the hepatic parenchyma, at least in the earlier siages, and consequently causes a disproportionate increace in serum alkaline phosphatase compared with AST and ALT. The most distinctive markers for primary biliary cirhosic are anti-mitochonria antibodies, especially the M2 subtype. Primary biliary cirthosis is also associated with a vatiety of other diseases such as Sjogren's syndrome, rheumatoxd arthrits, thyrosdits, celiac disease, and glomerulonephntis Anti-double-stranded DNA and anti-phospholipid artibodees are markers for systemic lupus erythematosus, Anti-self 1gG (also known as rheumatoid factor) isa marker for theumateid arthritic. Anti-cmooth muscle antibodies are seen in autoimmune hepatic, which i not ac strongly astociated with scleroderma as primary biliary cirthosis. Report An Error Mark this question & => Question Td : 151382 Question 22 of 30 Examination of a 40 year old female with systemic sclerosis who presents with malaise is significant for jaundice, and serum chemistry studies show moderately elevated serum allaline phosphatase, whie AST and ALT are only mirimally elevated. Which autoantibodies would be most helpfil in elucidating the Ukcely ctiology of her lirer disease? a) Anti-double-stranded DNA Y © © Ant-mitochondrial antibodies ©) Antiphospholipid antibodies ) Anti-selfIgG @) Anti-smooth muscle antibocies Question Explanation: Primary biliary citrhosis is strongly associated with systemic sclerosis (scleroderma). This disorder characteristically affects intrahepatic bile ucts more than the hepatic parenchyma, at least in the earlier siages, and consequently causes a disproportionate increace in serum alkaline phosphatase compared with AST and ALT. The most distinctive markers for primary biliary cirhosic are anti-mitochonria antibodies, especially the M2 subtype. Primary biliary cirthosis is also associated with a vatiety of other diseases such as Sjogren's syndrome, rheumatoxd arthrits, thyrosdits, celiac disease, and glomerulonephntis Anti-double-stranded DNA and anti-phospholipid artibodees are markers for systemic lupus erythematosus, Anti-self 1gG (also known as rheumatoid factor) isa marker for theumateid arthritic. Anti-cmooth muscle antibodies are seen in autoimmune hepatic, which i not ac strongly astociated with scleroderma as primary biliary cirthosis. Report An Error Mark this question & => Question Td : 158546 Question 23 of 30 A.35 year old man has abdominal pain and cramping. He has frequertly observed blood in his stools, and has occasionally had fevers, Examination reveals abdominal tenderness, and gusiac positive stool is found in the rectal vanit. Colonoscopy reveals ulceration, which biopsy shows to be mucosal and submucosal. The most appropriate pharmacotherapy ic which one of the following? 2) Lansoprazole ) Methylcellulose ©) Metotlopramide @) Octreotide ©) Sulfasalazine Answer | Explanation Other User's Explanation Report An Error Question Explanatio ‘This patient hes ulcerative colitis (UC), which is associated with the development of bloody diarrhea, lower abdominal cramps, fecal wegency. anemia, and low serum albumin in the presence of negative stool culhures. Sulfaselazine, which contains sulfapyridine and 5- aminosalicylic acid (5-AS.A) moieties, is used for mild to moderate inflammatory bowel disease. Side effects include bone marrow suppression, nausea, vomiting, headaches, and general malaise. Other agents that can be effective for mild to moderate ulcerative colitis include mesalamine (5-amnnosalieylic atid, 5-ASA) and olselazine (a dimer of) ASA). Lansoprazole is a proton pump inhibitor similar to the prototype omeprazole, which acts to decrease gastric acid secretion. Methylcellulose is a bulk forming laxative used for coastpation. Metoclopramide is an antiemetic and prokinetic agent. Octreotide, an analog of somatostatin, is used to control diarrhea fiom a variety of endocrine disorders and to control variceal bleeding. Mark this question & => Question Td : 158546 Question 23 of 30 2.35 year old man fas abdominal pain and cramping. He has frequently observed blood in his stools, and has occasionally had fevers, Examination reveals abdominal tenderness, and gusiac positive stool is found in the rectal vanit. Colonoscopy reveals ulceration, which biopsy shows to be mucosal and submucosal. The most appropriate pharmacotherapy is which one of the following? 2) Lansoprazole +b) Methylcellulose ©) Metotlopramide ¢) Octreotide: ¥ © © Sulfasalazine Answer | Explanation Other User's Explanation Report An Error Question Explanatio ‘This patient hes ulcerative colitis (UC), which is associated with the development of bloody diarrhea, lower abdominal cramps, fecal wegency. anemia, and low serum albumin in the presence of negative stool culhures. Sulfaselazine, which contains sulfapyridine and 5- aminosalicylic acid (5-AS.A) moieties, is used for mild to moderate inflammatory bowel disease. Side effects include bone marrow suppression, nausea, vomiting, headaches, and general malaise. Other agents that can be effective for mild to moderate ulcerative colitis include mesalamine (5-amnnosalieylic atid, 5-ASA) and olselazine (a dimer of) ASA). Lansoprazole is a proton pump inhibitor similar to the prototype omeprazole, which acts to decrease gastric acid secretion. Methylcellulose is a bulk forming laxative used for coastpation. Metoclopramide is an antiemetic and prokinetic agent. Octreotide, an analog of somatostatin, is used to control diarrhea fiom a variety of endocrine disorders and to control variceal bleeding. Mark thie question & => Question Td : 165424 Question 24 of 30 A 64 year old man has 2 months of worsening watery diarrhea, dehydration, and weight loss. Diatthea is unrelated to food and that he offen also experiences cramping abdominal pain and flushing of the acck and face. The onset is usuelly sudden and is preceded by siress or alcohol intake, Exem shows a blood pressure of 85/68 mm Hg and a pulse of 102 min, Hepatomegaly is seen. Ultrasound shows lesions in the liver consistent with metastases, Which test would likely confirm the diagnosis? 2) Serum aspartate aminotransferase (AST) ) Serum estradiol ©) Serum metanephrine 4) Urinary ghicose ©) Urinary S-hydrozyindoleacetic (J-HTAA) Question Explanatic This nan hes the cardinal features of the carcinoid syndrome: diamthea, fushing, and abdominal pain. Patients are typically in their 50s, but the range of presentation spans adulthood to old age. Carcinoid tumors may secrete a vatiety of bioactive substances, inching gastrointestinal (I) peptides (gastrin, insulin, vasoactive intestinal peptide [VTP], ghucagon), other peptides (ACTH, calcitonin), and bioactive amines (5-HT). Cardiac involvementis present in up to 40% of cases daring the course of disease. Tae carcinoid syndrome occurs when a sufficient concentration of secreted substance reaches the systemic circulation, usually once there are metastases to the liver. Since the major product of carcinoid tumors is S-HT, many of these patients have incteased amounts of the serotonin metabolite 5-HIAA excreted in the urine. Therefore, urinary 5-HIAA is the most frequently used diagnostic test. Measurement of iver enzymes (such as AST) is a nonspecific test of liver status and would not elucidate this patient's condition. “Although decreased estradiol secretion in postmenopausal women causes hot flushes, it would not be something to measure in this patient Seram metanephrine is ametabolte of epinephrine that would be elevated in pheochromocytoma Urinary ghicose is frequently used to evaluate patients with hyperglycemia but would not be usefil in diagnosing carcinoid syndeome Mark thie question & => Question Td : 165424 Question 24 of 30 A 64 year old man has 2 months of worsening watery diarrhea, dehydration, and weight loss. Diatthea is unrelated to food and that he offen also experiences cramping abdominal pain and flushing of the aeck and face. The onset is usuelly sudden and is preceded by siress or alcohol intake, Exem shows a blood pressure of 85/68 mm Hg and a pulse of 102:min. Hepatomegaly is seen. Ultrasound shows lesions in the liver consistent with metastases. Which test would likely confirm the diagnosis? 2) Scrum aspartate aminotransferase (AST) ) Serum estradiol ©) Serum metanepbrine 4) Urinary ghicose Y © © Urinary 5-hydroxyindoleacetic (5-HIAA) Question Explanatic This nan hes the cardinal features of the carcinoid syndrome: diamthea, fushing, and abdominal pain. Patients are typically in their 50s, but the range of presentation spans adulthood to old age. Carcinoid tumors may secrete a vatiety of bioactive substances, inching gastrointestinal (I) peptides (gastrin, insulin, vasoactive intestinal peptide [VTP], ghucagon), other peptides (ACTH, calcitonin), and bioactive amines (5-HT). Cardiac involvementis present in up to 40% of cases daring the course of disease. Tae carcinoid syndrome occurs when a sufficient concentration of secreted substance reaches the systemic circulation, usually once there are metastases to the liver. Since the major product of carcinoid tumors is S-HT, many of these patients have incteased amounts of the serotonin metabolite 5-HIAA excreted in the urine. Therefore, urinary 5-HIAA is the most frequently used diagnostic test. Measurement of iver enzymes (such as AST) is a nonspecific test of liver status and would not elucidate this patient's condition. “Although decreased estradiol secretion in postmenopausal women causes hot flushes, it would not be something to measure in this patient Seram metanephrine is ametabolte of epinephrine that would be elevated in pheochromocytoma Urinary ghicose is frequently used to evaluate patients with hyperglycemia but would not be usefil in diagnosing carcinoid syndeome ‘Marke this question e& => Question Td : 178543 Question 25 of 30 Tn anormal person a tube with transducer at its endlis swallowed and passed an unknown distance down the esophagus and records pressure of 25 mm Hg between swallows. Within 2 seconds the pressure falls to 5 mm Hg after small amount of water intake, where itremains until returning to its resting pressure 6 seconds later. In esophagedl disease patisnt itrecords a pressure of 39 mm Hg at samne location, After swallowing, the pressure fails to decsease. In which site is the transducer lcely located? 8) Esophageal body distal to the diaphragm. b) Esophageal body proximal to the diaphragm ©) Lower esophageal sphincter 6) Pharynx ©) Upper esophageal sphincter Question Explanatic Achalasia is an acquired esophageal motity disorder that is characterized by loss of enteric inhibitory neurons. The lower esophageal sphincter may exhibit increased tone in between swallows aad tai to relax normally with a swallow. Derisalsis in the esophageal body is also abnormal. A swallow may not induce any peristalsis in the esophageal body or may produce simultaneous contractions along, its entice length, Diagnosis is made with a barium swallow (shows esophageal narrowing into a "bird's beak’ atthe distal end) and esophageal manometry (most accurate test overall), which was done in this patient. The esophageal body distal to the diaphragm is relaxed in between swallows. The intraesophageal pressure at this point reflects the intra-abdominal pressure which is slightly positive (G mm Hg). During inspiration the pressure inside the distal esophagus rises along with the intra-abdominal pressure, during expiration this pressure falls. The pressure in the esophageal body proximal to the diaphragm reflects the intrathoracic pressure. Iris slightly negative at the end of inspiration and sightly positive at the end of expiration. Since the rrouth and pharyras are open to the atmosphere, in between ewellews the pressure within the pharm is atmcspheric (0 mm Hg). The precoure sises abruptly to a ‘maximum of 100 min Hg at the start of a swallow and remamns to baseline within 0.5 seconds. At rest, the pressure in the upper esophageal sphincter can be as high as 60 mmm Hg. It is maintained by the normal elasticity of the sphincteric structures, and by active contraction of the cricopharyngeal rmascle which composes most ofthe sphincter. Shortly after the pharyngeal mmiscles contract during a swallow, the upper esophageal aphincter relaxes as the tonic newal input to the cricopharyngeal muscle (skeletal zmiscle) is inhibited as part of the swallowing program, Function of this sphincter is unaffected by achalasia, ‘Marke this question e& => Question Td : 178543 Question 25 of 30 Tn anormal person a tube with transducer at its endlis swallowed and passed an unknown distance down the esophagus and records pressure of 25 mm Hg between swallows, Within 2 ceconds the prescure falls to 5 mma Hg after small amount of water intake, where itremains until returning to its resting pressure 6 seconds later. In esophageal disease patient it records a pressure of 39 mm Hyg at same location. After swallowing, the pressure fais to decrease. In which site is the transdacer likely located? 8) Ecophageal body distal to the diaphragm +b) Esophageal body proximal to the diaphragm SM © ©) Lower esophageal sphincter 6) Pharynx 6) Upper esophageal sphincter Question Explanatic Achalasia is an acquired esophageal motity disorder that is characterized by loss of enteric inhibitory neurons. The lower esophageal sphincter may exhibit increased tone in between swallows aad tai to relax normally with a swallow. Derisalsis in the esophageal body is also abnormal. A swallow may not induce any peristalsis in the esophageal body or may produce simultaneous contractions along, its entice length, Diagnosis is made with a barium swallow (shows esophageal narrowing into a "bird's beak’ atthe distal end) and esophageal manometry (most accurate test overall), which was done in this patient. The esophageal body distal to the diaphragm is relaxed in between swallows. The intraesophageal pressure at this point reflects the intra-abdominal pressure which is slightly positive (G mm Hg). During inspiration the pressure inside the distal esophagus rises along with the intra-abdominal pressure, during expiration this pressure falls. The pressure in the esophageal body proximal to the diaphragm reflects the intrathoracic pressure. Iris slightly negative at the end of inspiration and sightly positive at the end of expiration. Since the rrouth and pharyras are open to the atmosphere, in between ewellews the pressure within the pharm is atmcspheric (0 mm Hg). The precoure sises abruptly to a ‘maximum of 100 min Hg at the start of a swallow and remamns to baseline within 0.5 seconds. At rest, the pressure in the upper esophageal sphincter can be as high as 60 mmm Hg. It is maintained by the normal elasticity of the sphincteric structures, and by active contraction of the cricopharyngeal rmascle which composes most ofthe sphincter. Shortly after the pharyngeal mmiscles contract during a swallow, the upper esophageal aphincter relaxes as the tonic newal input to the cricopharyngeal muscle (skeletal zmiscle) is inhibited as part of the swallowing program, Function of this sphincter is unaffected by achalasia, ‘Mark this question & => Question Ta : 197425 Question 26 of 30 A 37 year old woman presents with a recent history of prurits, fatigue and jaundice. Liver biopsy shows periportal fbrosis with periportal inflammation and prominent enlargement of the portal tracts. The antibodies most likely to be found in the blood include which one of the following? 2) Anticardiolipin b) Anticentromere ©) Antimitochondtial & Antimyeloperoxidase ©) Antinuclear Question Explanation: This patient has primary biliary cirrhosis which is associated with antimitochondrial antibo dies found in 90-95% of cases with a specificity of 98% for this disease. ‘Mark this question & => Question Ta : 197425 Question 26 of 30 A 37 year old woman presents with a recent history of prurits, fatigue and jaundice. Liver biopsy shows periportal fbrosis with periportal inflammation and prominent enlargement of the portal tracts. The antibodies most likely to be found in the blood include which one of the following? a) Anticardiolipin 6) Anticentromere © ©) Antimitochondrial & Antimyeloperoxidase ©) Antinuclear Question Explanation: This patient has primary biliary cirrhosis which is associated with antimitochondrial antibo dies found in 90-95% of cases with a specificity of 98% for this disease. Mark this question & => Question Td : 197% Question 27 of 30 A.54 year old male presents with dysphagia, retrosternal discomfort and weight loss, Studies reveal achalasia, What is likely ta provide symptomatic relief? a) Buscopan ') Diazepam c) Nifedipine 4) Omeprazole e) Surgical cardiomyotomy Question Explanation: 75% care may be expected by cardiomyotomy in achalasia, Nitrates and hydralazine can help in the short term. None of other choices help the symptoms. Oesophageal dilatation is usually attempted before lavaroscopic surgical myatomy. jon Report An Error Mark this question & => Question Td : 197% Question 27 of 30 A.54 year old male presents with dysphagia, retrosternal discomfort and weight loss, Studies reveal achalasia, What is likely ta provide symptomatic relief? a) Buscopan 6) Diazepara ©) Nifedipine &) Omeprazole v © ©) Surgical cardiomyotomy Question Explanation: 75% care may be expected by cardiomyotomy in achalasia, Nitrates and hydralazine can help in the short term. None of other choices help the symptoms. Oesophageal dilatation is usually attempted before lavaroscopic surgical myatomy. jon Report An Error Marl this question = => Question Ta : 197455 Question 28 of 30 A.woman had lunch ata Chinese restaurant. She presented with diarrhea and vomiting in the evening. There was no fever. The likely cause of food poisoning in her case is which one of the following? a) Bacillus cereus +b) Clostridium perftingens ) Escherichia coli 4) Staphylococcus aureus ©) Yersinia enterocolitica Question Explanatior Bacillus cereus food poisoning is the general description, although two recognized types of ilness art caused by two distinct metabolites. The diatheal type ofillaess is caused by 2 large molecular weight protein, while the vomitng (emetic) type of illness :s believed to be caused by a low molecular weight, heat-stable peptide. The onset of watery diahoea, abdominal cramps, and pan occurs 6-15 hours after consumption of contaminated food. Syraptems usually persist for 24 hours. The emetic type of food poisoning is characterized by nausea and vorniing within 0.5 to 6 hours after consumption of contaminated foods. Occasionally, abdominal cramps and/or diarthoea may also occur. Duration of symptoms is generally less than 24 hours. A wide vanety of foods inchiding meats, milk, vegetables, and fish have been associated with the diarrhoeal type focd poisoning. The vomiting type outbreaks hhave generally been associated with rice products. Staphylososcue aureus and Clostridium perfringens are associated with meat and ‘Yersinia enterocolitica with milk. Marl this question = => Question Ta : 197455 Question 28 of 30 A.woman had lunch ata Chinese restaurant. She presented with diarrhea and vomiting in the evening. There was no fever. The likely cause of food poisoning in her case is which one of the following? V © a) Bacillus cereus +b) Clostridium perftingens ) Escherichia coli 4) Staphylococcus aureus ©) Yersinia enterocolitica Question Explanatior Bacillus cereus food poisoning is the general description, although two recognized types of ilness art caused by two distinct metabolites. The diatheal type ofillaess is caused by 2 large molecular weight protein, while the vomitng (emetic) type of illness :s believed to be caused by a low molecular weight, heat-stable peptide. The onset of watery diahoea, abdominal cramps, and pan occurs 6-15 hours after consumption of contaminated food. Syraptems usually persist for 24 hours. The emetic type of food poisoning is characterized by nausea and vorniing within 0.5 to 6 hours after consumption of contaminated foods. Occasionally, abdominal cramps and/or diarthoea may also occur. Duration of symptoms is generally less than 24 hours. A wide vanety of foods inchiding meats, milk, vegetables, and fish have been associated with the diarrhoeal type focd poisoning. The vomiting type outbreaks hhave generally been associated with rice products. Staphylososcue aureus and Clostridium perfringens are associated with meat and ‘Yersinia enterocolitica with milk. ‘Marke this question & => Question Td : 197829 Question 29 of 30 A 42 year old asymptomatic female underwent an abdominal uitrascund scan as part of a clinical trial and was found to have gallstones but entirely normal liver fiction tests. The most appropriate management is a) Chenodeoxycholic acid b) Laparoscopic cholecystectomy: ¢) Lithotripsy dD) Observation ¢) Ursodeoxychelic acid Answer (Bepiznation) Other User's Explanation Report An Error Question Explanation ‘This patientis asymptomatic and does not require any treatment at present. There no proven role for the use of oral drugs to try and reduce the formation of gallstones. The only definitive treatment would be a cholecystectomy but that ir not generally offered for asymptomatic gallstones ‘Marke this question & => Question Td : 197829 Question 29 of 30 A 42 year old asymptomatic female underwent an abdominal uitrascund scan as part of a clinical trial and was found to have gallstones but entirely normal liver fimctioa tests. The most appropriate management is a) Chenodeoxycholic acid +) Laparoscopic cholecystectomy ¢) Lithotripsy o¥ © AD Observation €) Ursodeoxychelic acid Answer (Bepiznation) Other User's Explanation Report An Error Question Explanation ‘This patientis asymptomatic and does not require any treatment at present. There no proven role for the use of oral drugs to try and reduce the formation of gallstones. The only definitive treatment would be a cholecystectomy but that ir not generally offered for asymptomatic gallstones Mark this question e& estion Td : 217020 Question 30 of 30 A. 65-year-old has longstanding citthosis and portal hypertension due to previous alcohol abuse. Hle has no history of gastrcintestinal bleeding. He kad a few inappropriately preccribed drugs. Coadministration of which of the following i likely to cause thic patient the greatest han if not corrected? 4) Lactulose and oral potassium ) Prednisone and inhaled albuterol ©) Propranolol and isasorbide dinitrate 4) Spironolactone and hydochlorothiazide ©) Spironolactone and oral potassium Question Explanation: Pharmacology is one of the mostitrportent applied basic science topics in clinical mectcine. The possibilty of drug interactions is so important to recegrize that entire organizations (phermacies) and computer cross-checking (Micro Medex) have been created to address this issue. These conveniences, however, in no way absclve a physician from knowing the consequences and potential interactions of each and every medicine that he or she prescribes Spironolactone, triamterene, and arriloride are potassium- sparing diuretics, The mistake of calling these drugs a ‘diuretic and cueing one’s brain to think "replenish electrolytes" could canse this patient to dic from hyperkalemic cardiac arrest with coadministration of potassium supplementation Lactulose and oral potassium are a usefil combination for the treatment of hepatic encephalopathy. Lactulose 1s a non- metabolizable sugar thet acidifies the stool ard thus traps ammonia in the GI men for excretion Prednisone and inhaled albuterolis the combination of glucocorticoid and an inhaled beta adrenergic agonist for asthma Propranolol and isosorbide dnitrate is a combination ofa nonspecific beta-blocker and a nonspecific vasodilator. This combination is usefid in the treatment of angina, because the beta- blocker prevents reflex tachycardia secondary to the isosorbide dinitrats-induced decrease in preload, Spironolactone and hydrochlorothiazide is a combination of a potassium-sparing diuretic and a thiazide diuretic that is very effective in the treatment of eirthotic edema and ascites, Report An Error Mark this question e& estion Td : 217020 Question 30 of 30 A. 65-year-old has longstanding citthosis and portal hypertension due to previous alcohol abuse. Hle has no history of gastrcintestinal bleeding. He kad a few inappropriately preccribed drugs. Coadministration of which of the following i likely to cause thic patient the greatest han if not corrected? 4) Lactulose and oral potassium ) Prednisone and inhaled albuterol ©) Propranolol and isasorbide dinitrate 4) Spironolactone and hydochlorothiazide V © 2) Spironolactone and oral potassitm Question Explanation: Pharmacology is one of the mostitrportent applied basic science topics in clinical mectcine. The possibilty of drug interactions is so important to recegrize that entire organizations (phermacies) and computer cross-checking (Micro Medex) have been created to address this issue. These conveniences, however, in no way absclve a physician from knowing the consequences and potential interactions of each and every medicine that he or she prescribes Spironolactone, triamterene, and arriloride are potassium- sparing diuretics, The mistake of calling these drugs a ‘diuretic and cueing one’s brain to think "replenish electrolytes" could canse this patient to dic from hyperkalemic cardiac arrest with coadministration of potassium supplementation Lactulose and oral potassium are a usefil combination for the treatment of hepatic encephalopathy. Lactulose 1s a non- metabolizable sugar thet acidifies the stool ard thus traps ammonia in the GI men for excretion Prednisone and inhaled albuterolis the combination of glucocorticoid and an inhaled beta adrenergic agonist for asthma Propranolol and isosorbide dnitrate is a combination ofa nonspecific beta-blocker and a nonspecific vasodilator. This combination is usefid in the treatment of angina, because the beta- blocker prevents reflex tachycardia secondary to the isosorbide dinitrats-induced decrease in preload, Spironolactone and hydrochlorothiazide is a combination of a potassium-sparing diuretic and a thiazide diuretic that is very effective in the treatment of eirthotic edema and ascites, Report An Error ‘Marke this question => Question Ta : 59350 Question 1 of 30 A.44 year ald obese woman presents to the Emergency Room with a history of right upper quadrant pain lasting two hours. She states that tis is her third attacks of similar pain in the past two months. The pain is sudden in onset and is sharp, constant and radiates to her right shoulder. Ox examination she is afebrile end has mild sight upper quadrant tendemess. Tae most likely chagnosis is 2) Dondenal ulcer +b) Myocardial infarction 6) Acute cholecystitis 6) Biliary colie ©) Hepatitis Avewor [UERASNEREN) oer voor Explanation Report An Error Question Explanation: Biliary colic is a condition characterized by extreme cramping painin the right upper abdomen. Gellstones in the gallbladder or in the bile ducts are the cause of the severe pain, About one third of patients with gallstones develop biliary colic or other complications People who are overweight or have a high level of blood cholesterol increase their risk of developing gallstones and bilary colic Pregnant women of those on birth control pills or estrogen replacement therapy also have a greater risk of developing biliary colic Other risk factors include rapid weight loss, diabetes, certain gastrointestinal conditions and certain medications. Biliary colic is caused by gallstones that form in the gallbladder or bile ducts. Gallstones form when bile, a substance made in the liver to help digest fats, contains too much cholesterol A solid particle forms and becomes a gallstone Biliary colic can canse extreme pain in the upper right abdomen and nausea, especially after cating amcal high in fat The condition can also cause jaundice in the skin and eyes. The ppain can last up to three hours and sometimes spreads to the right shoulder or through the center of the back. Fatty food intolerance, dyspepsia, indigestion, heartburn, flatulence and erictation are other symptoms associated with gallstone disease. Attacks of bllary colic are more commen at aight, possibly because the gallbladder shifts to a horizontal position, making it easier for stones to enter into the eysiie duct. ‘Marke this question => Question Ta : 59350 Question 1 of 30 A.44 year ald obese woman presents to the Emergency Room with a history of right upper quadrant pain lasting two hours. She states that tis is her third attacks of similar pain in the past two months. The pain is sudden in onset and is sharp, constant and radiates to her right shoulder. Ox examination she is afebrile end has mild sight upper quadrant tendemess. Tae most likely chagnosis is 2) Dondenal ulcer +b) Myocardial infarction 6) Acute cholecystitis Y © 6) Biliary cotie ©) Hepatitis Avewor [UERASNEREN) oer voor Explanation Report An Error Question Explanation: Biliary colic is a condition characterized by extreme cramping painin the right upper abdomen. Gellstones in the gallbladder or in the bile ducts are the cause of the severe pain, About one third of patients with gallstones develop biliary colic or other complications People who are overweight or have a high level of blood cholesterol increase their risk of developing gallstones and bilary colic Pregnant women of those on birth control pills or estrogen replacement therapy also have a greater risk of developing biliary colic Other risk factors include rapid weight loss, diabetes, certain gastrointestinal conditions and certain medications. Biliary colic is caused by gallstones that form in the gallbladder or bile ducts. Gallstones form when bile, a substance made in the liver to help digest fats, contains too much cholesterol A solid particle forms and becomes a gallstone Biliary colic can canse extreme pain in the upper right abdomen and nausea, especially after cating amcal high in fat The condition can also cause jaundice in the skin and eyes. The ppain can last up to three hours and sometimes spreads to the right shoulder or through the center of the back. Fatty food intolerance, dyspepsia, indigestion, heartburn, flatulence and erictation are other symptoms associated with gallstone disease. Attacks of bllary colic are more commen at aight, possibly because the gallbladder shifts to a horizontal position, making it easier for stones to enter into the eysiie duct. ‘Marle this question ez Question Ta : 59913 Question 2 of 30 A.46 year old with a 20 year history of severe Crokn's disease requiring in patient hospitalization complains ofthe recent orset of le kenee pain on walking What is the most licely cause of the pair’? a) Osteomalacia ') Seronegative spendyloarthrapathy ©) Osteoporosis © Avascnler necrosis ©) Osteoarthis Question Explanation: ‘The extra-ietostinal menifestations of flammatory bowel disease are mumerous. Crohn's disease is associated with a type of theumatologic disease known as seronegative spondyloarthropathy. This group of diseases is characterized by inflammation of one or ‘more joints (arthritis) or ruscle insertions (enthesitis). The acthrtis can affect larger joints such as the knee or shoulder or may exclusively involve the stall joints of the hand and feet The arthritis may also involve the spine, leading te ankslosing spondylitis ifthe entire spine ic involved or simply sacrolltis # only the lower spine ic involved. The symptom of ertbritic inclade painful, warm, swollen, stiff joints and loss of joint mobilty or function. ‘Marle this question ez Question Ta : 59913 Question 2 of 30 A.46 year old with a 20 year history of severe Crokn's disease requiring in patient hospitalization ccmplains of the recent onset of left kenee pain on walking What is the most licely cause of the pair’? a) Osteomalacia JY © b) Seronegative spondyloarthropathy: ©) Osteoporosis ©) Avasculer necrosis, ©) Osteoarthis Question Explanation: ‘The extra-ietostinal menifestations of flammatory bowel disease are mumerous. Crohn's disease is associated with a type of theumatologic disease known as seronegative spondyloarthropathy. This group of diseases is characterized by inflammation of one or ‘more joints (arthritis) or ruscle insertions (enthesitis). The acthrtis can affect larger joints such as the knee or shoulder or may exclusively involve the stall joints of the hand and feet The arthritis may also involve the spine, leading te ankslosing spondylitis ifthe entire spine ic involved or simply sacrolltis # only the lower spine ic involved. The symptom of ertbritic inclade painful, warm, swollen, stiff joints and loss of joint mobilty or function. Marl this question <= => Question Td : 67275 Question 3 of 30 A 52 year old men with a history of Hemochromatosis presents to the ER vomiting up bright red blood. He had his most recent phlebotomy yesterday His blood pressure is 110/85 mm, his pulse 11Sfmin; his face is flushed, and he is diaphoretic. During the physical examination splenomegaly and a venous pattern on his chest and abdomen are noted. He seems somewhat drowsy and confused but has no focal neurologic signs. The probable source of this patient’s confusion is 8) Severe anemia ) Hepatic encephalopathy ©) Subarachnoid hemorrhage 6) Vitamin B12 deficiency 6) Serum iron overload Question Explanation: Hemochromatosis is the abnormal accumulation ofiron in parenchymal organs, leading to organ toxicity. Ibis the most comamon icherited liver dicease in whiter and the most common autosomal rececsive genetic disorder. Hemockromatosic can cause liver damage and lead to hepatic encephalopathy. Hepatic encephalopathy is a neuropsychiatric syndrome. It most often resuits from high gut protein or acute metaboke stress (eg, GI bleeding, infection, electrolyte abnormality) in a patient with portal systemic shunting. Symptoms are mainly neuropsychiatric (e.g. confusion, Bapping rence, and coma). Diagnosis is baced on clinical ndings. Treatment ueually ie correction of the acute cause, restriction of dietary protein, and administration of oral lacvalose. Marl this question <= => Question Td : 67275 Question 3 of 30 A.52 year old man with a history of Hemochromatosis presents to the ER vomiting up bright red blood. He kad his most recent phlebotomy yesterday. His blood pressure is 110/85 mmHg, his pulse 11S/min; his face is flushed, and he is diephorstic. During the paysical examination splenomegely ancl a venous pattern on his chest and abdomen are noted. He seonss somewhat drowsy and confused but has no focal neurologic signs. The probable source of this patients confusion is 8) Severe anemia VW © b) Hepatic encephalopathy 6) Subarachnoid hemorrhage 6) Vitamin B12 deficiency 6) Serum iron overload Question Explanation: Hemochromatosis is the abnormal accumulation ofiron in parenchymal organs, leading to organ toxicity. Ibis the most comamon icherited liver dicease in whiter and the most common autosomal rececsive genetic disorder. Hemockromatosic can cause liver damage and lead to hepatic encephalopathy. Hepatic encephalopathy is a neuropsychiatric syndrome. It most often resuits from high gut protein or acute metaboke stress (eg, GI bleeding, infection, electrolyte abnormality) in a patient with portal systemic shunting. Symptoms are mainly neuropsychiatric (e.g. confusion, Bapping rence, and coma). Diagnosis is baced on clinical ndings. Treatment ueually ie correction of the acute cause, restriction of dietary protein, and administration of oral lacvalose. Mark this question & => Question Id : 84737 Question 4 of 30 A patient complains of epigastric pain that fails to respond to antacids. Endoscopy demonstrates an ulcerated mass on the greater curvature of the stomach. Genetic studies on the tumor demonstrate an abered DCC gene. The other tumor suppressor genes found on the same chromosome as DCC is a ERCAL +) DPC ONE @ NF2 ©) p53 ‘newer (JENSEN) OtnorUeorsExplanation Report An Error Question Explanatic The timer is gastric carcinoma, which is assaciated with the DCC oncogene located on the long arm of chromosome 18 (18q). DCC is also associated with carcinomas of colon. 18q also has the DPC gene, associated with pancreatic cancer. The BRCA-1 gene (choice A), associated with breast cancer and ovarian cancer, is on 174, ‘The NE-1 gene (choice C) associated with neurofibromatosis type [, is on 17q, ‘The NE-2 gene (choice D). associated with neurofibromatosis type IL is on 224, ‘The p53 gene (choice E), associated with many cancers, is on 17p. [Genel[Associated Condition|| Chromosome Location) [Dec] [Gasizic cancer 184 [DPC |[Pancreatic cancer [18a [NE-][Newofbromatoss 1 |[I74 rE-2|fivewoibromatosis 2 |[22q P53 |fMfany carcinomas [17 Mark this question & => Question Id : 84737 Question 4 of 30 A patient complans of epigastric pain that fails to respond to antacids. Endoscopy demonstrates an ulcerated mass onthe greater curvature of the stomach, Genetic studies on the tumor demonstrate an atered DCC gene. The other tumor suppressor genes found on the same chromosome as DCC is a) ERCAL Y © &) DPC o)NF1 @ NF2 ©) p53 ‘newer (JENSEN) OtnorUeorsExplanation Report An Error Question Explanatic The timer is gastric carcinoma, which is assaciated with the DCC oncogene located on the long arm of chromosome 18 (18q). DCC is also associated with carcinomas of colon. 18q also has the DPC gene, associated with pancreatic cancer. The BRCA-1 gene (choice A), associated with breast cancer and ovarian cancer, is on 174, ‘The NE-1 gene (choice C) associated with neurofibromatosis type [, is on 17q, ‘The NE-2 gene (choice D). associated with neurofibromatosis type IL is on 224, ‘The p53 gene (choice E), associated with many cancers, is on 17p. [Genel[Associated Condition|| Chromosome Location) [Dec] [Gasizic cancer 184 [DPC |[Pancreatic cancer [18a [NE-][Newofbromatoss 1 |[I74 rE-2|fivewoibromatosis 2 |[22q P53 |fMfany carcinomas [17 J % wow. interface.: * € BBD sty! sail los x ‘L unread) - en x |G? fareds1984's pr x |G onlymeony'spr x |G females online x lel CL www. interface.edu.pk/medi xams/test-analysis.php 2utd=14759 apps El.coogle Asetings [)Sgnin Zou woe duly oo wsdl dips Ofte ge sgsoll JS) Free Hotmall * 8 Gi other bookmar ‘Mark this question >= Question Id : 84778 Question 5 of 30 A.62 year old woman is admitted to the hospital for increased abdominal growth. She has noted that her abdomen has rapidly expanded over the past month, She is not aware of any medical problems and has not seen a physician in many years. On physical exarrination, her heast rate is 90/min, and her blood pressure ie 105/65 rem Hg, She looks cachectis and her abdomen is markedly distended, with a positive Duid wave. Ultrasound demonstrates a large amount of ascites. The quantity of 500 mL of aid is aspirated from the peritoneal cavity, and the fluid has a specific gravity of 1.010, very low protein, and few cells. The history that would be ‘most consistent with this patient's presentation is a) Dialysis through a peritoneal membrane ) History of alcohol abuse ©) History of filariasis d) Penetrating stab wound to the abdomen ©) Peptic ulcer disease Question Explanation: Edema (accummlation of fuid within the interstitium or in body cavities) may develop by different mechanisms. Generally edema secondary to increased hydrostatic pressure, reduced plastna osmotic pressure, or sodium setention is a hypocelular, protein poor trensadate with a specific aravity less than 1.012. Th contrast, edema due to inflammation ot lymphatic obstruction is typically a celular, protein-rich exmdate with a specific gravity over 1.020. Distinguishing between the two may have significant implications on diagnosis and treaiment of the underlying disease, Liver cihosis causes accumulation of a transudaie in the peritoneal cavity (ascites), by two concurrent mechanisms: increased hydrostatic pressure due to portal hypertension and decreased plasma osmotic pressure due to hypoalbuminemia A history of alcohol abuse would be consistent with this patient's history and laboratory findings. Acute pertonitis often results in accumlation of an inflammatory exudate within the peritoneal cavity. Peritoneal dielysis (choice A) is often complicated by acute peritonitis. Examination of this exudate may provids important clinical information as to the nature of the disease (acute versus chronic inflammatory process) and etiologic agents involved (for example, bacteria or neoplastic cell) Acute inflammation of the peritoneum may result from different types of injurious stimuli, Among these are contaminating bacteria from a penetrating stab wound to the abdomen (choice D) or initating gastric juices escaping through an ulcerated peptic ulver of the stomach (choice E) Lymphatic obstruction leads to accumulation of protein and lymphocyte-rich fhid and may result from infections (lariasis [choice Cl)’ neoplastic invasion, surgical procedures (axillary dissection of radical mastectomy), and radiation fibrosis Accumulation of lymph within the peritoneal cavity is called chylous ascites. Total Questions SBBARBEBBEBBERBERERKEBBEBY PR eee ene XxxXXXKXXKXKXKKXKXKXKKKKXKKXKKXKKXKKXKKKKXKK J % wow. interface.: * € BBD sty! sail los x ‘L unread) - en x |G? fareds1984's pr x |G onlymeony'spr x |G females online x lel CL www. interface.edu.pk/medi xams/test-analysis.php 2utd=14759 apps El.coogle Asetings [)Sgnin Zou woe duly oo wsdl dips Ofte ge sgsoll JS) Free Hotmall * 8 Gi other bookmar ‘Mark this question >= Question Id : 84778 Question 5 of 30 4.62 year old woman is admitted to the hospital for increased abdominal growth. She has noted that her abdomen has rapidly expanded over the past month. She is not aware of any medical problems and has not seen a physician in many yeers. On physical examination, her heart rate is 90/min, and her blood pressure is 105/65 mm Hg, She looks cachectic and her abdomen is markedly distended, with a positive Quid wave. Ultrasound demonstrates a large amount of ascites. The quantity of 500 mL of faidis aspirated from the pertoneal cavity, and the fuid has a specific gravity of 1.010, verylow protein, and few cells. ‘The history thet would be ‘most consistent with this patient's presentation is a) Dialysis through a peritoneal membrane ¥ © b) History of alcohol abuse c) History of filariasis 4) Penetrating stab wound to the abdomen e) Peptic ulcer disease Question Explanation: Edema (accummlation of fuid within the interstitium or in body cavities) may develop by different mechanisms. Generally edema secondary to increased hydrostatic pressure, reduced plastna osmotic pressure, or sodium setention is a hypocelular, protein poor trensadate with a specific aravity less than 1.012. Th contrast, edema due to inflammation ot lymphatic obstruction is typically a celular, protein-rich exmdate with a specific gravity over 1.020. Distinguishing between the two may have significant implications on diagnosis and treaiment of the underlying disease, Liver cihosis causes accumulation of a transudaie in the peritoneal cavity (ascites), by two concurrent mechanisms: increased hydrostatic pressure due to portal hypertension and decreased plasma osmotic pressure due to hypoalbuminemia A history of alcohol abuse would be consistent with this patient's history and laboratory findings. Acute pertonitis often results in accumlation of an inflammatory exudate within the peritoneal cavity. Peritoneal dielysis (choice A) is often complicated by acute peritonitis. Examination of this exudate may provids important clinical information as to the nature of the disease (acute versus chronic inflammatory process) and etiologic agents involved (for example, bacteria or neoplastic cell) Acute inflammation of the peritoneum may result from different types of injurious stimuli, Among these are contaminating bacteria from a penetrating stab wound to the abdomen (choice D) or initating gastric juices escaping through an ulcerated peptic ulver of the stomach (choice E) Lymphatic obstruction leads to accumulation of protein and lymphocyte-rich fhid and may result from infections (lariasis [choice Cl)’ neoplastic invasion, surgical procedures (axillary dissection of radical mastectomy), and radiation fibrosis Accumulation of lymph within the peritoneal cavity is called chylous ascites. Total Questions SBBARBEBBEBBERBERERKEBBEBY PR eee ene XxxXXXKXXKXKXKKXKXKXKKKKXKKXKKXKKXKKXKKKKXKK Mark this question = => Question Ti : 86303 Question 6 of 30 A.42 year old men with diarchea, weight loss, and epigastic discomfort undergoes endoscopic examination, Examination reveals diffisely thickened mgal folds, suggestive of the lititis plastic type adenocarcinoma. Biopsy reveals no cancer but dramatic hyperplasia of superficial mancous cell with can gastric glands. This man may be losing excessive amounts of which of the following fiom his digestive tract due to this condition? 2) Carbohydrates +) Chloride ©) Fat 3) Water 6) Frovein Answer | Explanation Other User's Explanation Report An Error Question Explanation: ‘This patient has Menetrier disease, also kaown as protein losing gastroenteropathy. This disease is a hyperplasic gastropathy characterized by enlarged nugal folds because of the increased proliferation of the mucus-producing cells of the stomach. Increased ‘mucus production in Menetrier disease leads to protein losses from excessive gastric secretions, and may cause hypoalbummemia and peripheral edema, Absorption of carbohydrates, which occurs in the small intestine, is unimpaired in Menstrier disease ‘Menetrier disease is associated with deficient production of gastric; acid, so chloride less would be unlikely. This is in merked contrast to hypertrophic hypersecretory gastropatiy and Zolinger Elison syndrome, in which there is hyperplasia of parietal cells (Gather than atrophy as in Menetrier disease). Dietary fet loss is generally due to enteric malabsorption. such as that which occurs in celiac disease or cystic fibrosis. Gastric mucus is poor in fats, and no appreciable fat lose ascompanies Menetrie: dicease. Fluid fosses due to mucus overproduction in Menettier disease are reabsorbed in the gut. Menetrier dsease does not cause dehydration. Mark this question = => Question Ti : 86303 Question 6 of 30 £42 year old men with darchea, weight loss, and epigastric discomfort undergoes endoscopic examination. Examinaton reveals diffusely thickened ragal folds, suggestive ofthe liitis plastic type adenocarcinoma. Biopsy reveals no cancer but dramatic hyperplasia of superficial mucous cell with scan gastric glands. This man may be losing excessive amounts of which of the following from his digestive tract due to this condition? 2) Carbohydrates b) Chloride ©) Fat ¢) Water Y © ®) Protein Answer | Explanation Other User's Explanation Report An Error Question Explanation: ‘This patient has Menetrier disease, also kaown as protein losing gastroenteropathy. This disease is a hyperplasic gastropathy characterized by enlarged nugal folds because of the increased proliferation of the mucus-producing cells of the stomach. Increased ‘mucus production in Menetrier disease leads to protein losses from excessive gastric secretions, and may cause hypoalbummemia and peripheral edema, Absorption of carbohydrates, which occurs in the small intestine, is unimpaired in Menstrier disease ‘Menetrier disease is associated with deficient production of gastric; acid, so chloride less would be unlikely. This is in merked contrast to hypertrophic hypersecretory gastropatiy and Zolinger Elison syndrome, in which there is hyperplasia of parietal cells (Gather than atrophy as in Menetrier disease). Dietary fet loss is generally due to enteric malabsorption. such as that which occurs in celiac disease or cystic fibrosis. Gastric mucus is poor in fats, and no appreciable fat lose ascompanies Menetrie: dicease. Fluid fosses due to mucus overproduction in Menettier disease are reabsorbed in the gut. Menetrier dsease does not cause dehydration. ‘Mark this question & => Question Td : 86935 Question 7 of 30 A 53 year old man presents with epigastric pain that improves with meals. Endoscopy reveals a2 cm ulcerated area located 3 cm dista to the pyloric junction, Basal acid outputs within normal limits. The strongest contribution to the development ofthis disease is most likely to be made by a) Helicobacter pylori infection ) Aspirin use ©) Chronic antacid use 4) Cytomegalovinus infection ¢) Drinking alcohol £) Herpes virus infection Question Explanation The patient has a duodenal peptic ulcer. The strongest risk facior for duodenal peptic ulcer is Helicobacter pylor! infection, which is found in almost 100% ofthese cases (mn contrast to 70% infection rate in gastnc peptic ulcer). The basal acid output is normalin many patients with duodenal ulbes. Aspirin use and drinking alcohol are more strongly implicated in gastric ulcer disease than in dhodenal ulcer disease. Chronic antacid use is seen as a result of peptis ulcer disease, not as a cause of it Cytomegalovinis and herpes virus infections can invelve the stomach but are usually clinically signiicant only in profoundly immunosuppressed (e.g, AIDS) patients, ‘Mark this question & => Question Td : 86935 Question 7 of 30 A 53 year old man presents with epigastric pain that improves with meals. Endoscopy reveals a2 cm ulcerated area located 3 cm dista to the pyloric junction, Basal acid outputs within normal limits. The strongest contribution to the development ofthis disease is most likely to be made by Y © a) Helicobacter pylori infection ) Aspirin use ©) Chronic antacid use 4) Cytomegalovinus infection ¢) Drinking alcohol £) Herpes virus infection Question Explanation The patient has a duodenal peptic ulcer. The strongest risk facior for duodenal peptic ulcer is Helicobacter pylor! infection, which is found in almost 100% ofthese cases (mn contrast to 70% infection rate in gastnc peptic ulcer). The basal acid output is normalin many patients with duodenal ulbes. Aspirin use and drinking alcohol are more strongly implicated in gastric ulcer disease than in dhodenal ulcer disease. Chronic antacid use is seen as a result of peptis ulcer disease, not as a cause of it Cytomegalovinis and herpes virus infections can invelve the stomach but are usually clinically signiicant only in profoundly immunosuppressed (e.g, AIDS) patients, Mark ths question = => Question a: 93 Question 8 of 30 A 62 year old man preseats with complain of progressive dysphagia, fst for solids, then for liquids. A large, fimgating mass 2 em above the gastroesophageal junction is found on endoscopy. Biopsy of the mass demonstrates glands that extend into the muscular layer and contain celle with large, hyperchromatic nuclei. The condition most likely ascociated with development of this macs ic 2) Esophageal rings +) Ecephageal webs ) Refux esophagitis 4) Scleroderma 6) Slicing hiatal hernia Question Explanation: The biopsy is consistent with esophageal adenocarcinoma, which usually arises in areas of gastric or intestinal metaplasia Barrett esophagus) in the lower or middle third of the esophagus. Reflux esophagitis is a major predisposing factor for Barrett esophagus so reflux esophastis is also considered to be arrisk factor for adenocarcinoma of the esophagus. The prognosis for adenocascinoma is ppoce, unless caught very easly, which is why clinicians usually monitor patients with Barrett ecophagus yew carefilly. A major reason ‘why esophageal carcinoma has such poos prognosis is that the esophagus is only about 3 mun thick and ittakes very litle invasion either to reach the submucosal lymphatics or to breach the esophageal wall and spread directly into the cntical adjacent chest structures, Esophageal rings are sims of fibrovasculer tissue found in the lower esophagus Esophageal webs are raucosal ledges in the upper esophagus. Sclerodenna can cance fibrosis and impaired motility of the esophagus. Ix cling hiatal hernias, part ofthe stomach protrudes above the daphragm, Mark ths question = => Question a: 93 Question 8 of 30 A 62 year old man preseats with complain of progressive dysphagia, fst for solids, then for liquids. A large, fimgating mass 2 em above the gastroesophageal junction is found on endoscopy. Biopsy of the mass demonstrates glands that extend into the muscular layer and contain celle with large, hyperchromatic nuclei. The condition most likely ascociated with development of this macs ic 2) Esophageal rings b) Esophageal webs Y © ©) Refux esophagitis 4) Scleroderma €) Sliding hiatal hernia Question Explanation: The biopsy is consistent with esophageal adenocarcinoma, which usually arises in areas of gastric or intestinal metaplasia Barrett esophagus) in the lower or middle third of the esophagus. Reflux esophagitis is a major predisposing factor for Barrett esophagus so reflux esophastis is also considered to be arrisk factor for adenocarcinoma of the esophagus. The prognosis for adenocascinoma is ppoce, unless caught very easly, which is why clinicians usually monitor patients with Barrett ecophagus yew carefilly. A major reason ‘why esophageal carcinoma has such poos prognosis is that the esophagus is only about 3 mun thick and ittakes very litle invasion either to reach the submucosal lymphatics or to breach the esophageal wall and spread directly into the cntical adjacent chest structures, Esophageal rings are sims of fibrovasculer tissue found in the lower esophagus Esophageal webs are raucosal ledges in the upper esophagus. Sclerodenna can cance fibrosis and impaired motility of the esophagus. Ix cling hiatal hernias, part ofthe stomach protrudes above the daphragm, Mark this question —&= => Question Td : 87924 Question 9 of 30 A.46 vear old businessman presents to his physician with complaints of feeling tied and several months of abdominal pain and “dark- colored" urine. On physical examination the patient is slightly jaundiced and his gelblacder is palpable but not tender. The disorder thatis mostlikely associated with the given presentation is a) Acute cholecystitis b) Amyloidosis ) Hepatic cirthesis 4d) Hepatoma «) Pancreatic cancer Question Explanation: Carcinoma ofthe pancreas is often associated with vague, nonspecific symptoms such as abdominal pain, but it may be initially asymptomatic. Jaundice is found in the majority of patients with cancers of the head ofthe pancreas, along with an enlarged, palpable gallbladder (Courvoisicr’s sign) without significant tendermess or other signs of cholecystiis. Acute cholecysitis typically produces tendemess on palpation of the gallbladder. Amyloidosis can involve the liver, but renal problems and cardiomyopathy are more common presenting complaints. Hepatic cirrhosis most commonly presents with features of portal hypertension, including ascites. Hepatoma causes amass that may be palpable on physical examination or may be picked up as an incidental finding on CT or other n Report An Error Mark this question —&= => Question Td : 87924 Question 9 of 30 A.46 year old businessman presents to his rhysician with complaints of feeling tired and several months of abdominal pain and "dark- colored" urine. On physical examination the patient is slightly jaundiced and his gallbladder is palpable but not tender. The disorder that is most likely associated with the given presentation is 2) cute cholecystitis b) Amyloidosis ¢) Hepatic cirthosis 4) Hepatoma Y © 6) Pancreatic cancer Question Explanation: Carcinoma ofthe pancreas is often associated with vague, nonspecific symptoms such as abdominal pain, but it may be initially asymptomatic. Jaundice is found in the majority of patients with cancers of the head ofthe pancreas, along with an enlarged, palpable gallbladder (Courvoisicr’s sign) without significant tendermess or other signs of cholecystiis. Acute cholecysitis typically produces tendemess on palpation of the gallbladder. Amyloidosis can involve the liver, but renal problems and cardiomyopathy are more common presenting complaints. Hepatic cirrhosis most commonly presents with features of portal hypertension, including ascites. Hepatoma causes amass that may be palpable on physical examination or may be picked up as an incidental finding on CT or other n Report An Error Mark this question & => Question 1d: 91263 Question 10 of 30 Jn which of the following scenarios of a petieat with GI bleedis a red cell radionuclide scanis indicated as the next step in evaluation? a) Patient with a positive NG aspirate +) Patient with minimal bleeding on anoscopy from site proximal to sigmoid. o) Patient with negative NG aspirate 6) Negative anoscopy with brisk blesding from site proximal to sigmoid 6) Patient with brisk bleeding fiom a site prowimal to sigmoid with negative upper GI endoscooy. Question Explanation: In a patient with brisk bleeding proximal to the sigmoid, an upper Gl endoscopy is indicated. If this fails to identity the source of bleeding, a red cell scan or arteriography is necessary to locate the site of bleeding A patient with a positive NG aspirate should initially have an upper GL scope. The patient with a negative NG aspirate should have proctosiamoidoscopy. Patients with minimal bleeding on anoccopy thould have elective colonoscopy. Patients with bridle bleeding and negative anoscopy chould have an upper GI endoscopy. Report An Error Mark this question & => Question 1d: 91263 Question 10 of 30 Jn which of the following scenarios of a petieat with GI bleedis a red cell radionuclide scanis indicated as the next step in evaluation? a) Patient with a positive NG aspirate +) Patient with minimal bleeding on anoscopy from site proximal to sigmoid. o) Patient with negative NG aspirate 6) Negative anoscopy with brisk blesding from site proximal to sigmoid Y © 6) Patient with brisk bleeding fiom a site prowimal to sigmoid with negative upper GI endoscooy. Question Explanation: In a patient with brisk bleeding proximal to the sigmoid, an upper Gl endoscopy is indicated. If this fails to identity the source of bleeding, a red cell scan or arteriography is necessary to locate the site of bleeding A patient with a positive NG aspirate should initially have an upper GL scope. The patient with a negative NG aspirate should have proctosiamoidoscopy. Patients with minimal bleeding on anoccopy thould have elective colonoscopy. Patients with bridle bleeding and negative anoscopy chould have an upper GI endoscopy. Report An Error Mark this question & => Question Td : 91953 Question 11 of 30 ASI year old woman has been diagnosed with primary biliary cimhosis. She has portal hypertension and ascitis. Her physician placed her on a waiting list for iver transplantation. Which of the following is NOT an absolute contraindication for liver transplantation? a) HIV seropositivity +) Extrahepatic malignancy, c) Advanced cardiac disease. © Futminant fiver falure €) Severe emphysema Question Explanation: “Alcoholic liver disease and hepatitis C are two of the most commen indications for liver transplantation. The one year survival rats has risen from around 50% to greater than 80%. The development of prognostic criteria for various patients such as age, prothrombin time, and jaundice can predict the success ofthe transplantation, Being HIV-positive (A) is a contraindication for liver transplantation becanse of the relatively shortened life span. Life span would be shortened dramatically as a resut of the Report An Error immunosuppressing drugs required to prevent host rejection of the liver transplant, as it would exacerbate the immunosuppression induced by HIV extrahepatic malignancies (B), such as breast and lung cancer, are absolute contraincications for transplantation because of the shortened life span and the lack of available donors for liver transplantation, Advanced cardiac disease (C) such as, triple vessel coronary artery disease and end stage congestive heart failure are contraindlcations for transplantation because of the very high risk of mostalty during the surgery. Empiaysema (E) and pumonary fibrosis are also diseases associated with a poor prognosis during liver transplantation surgery and thus are absolute contvaindications. Mark this question & => Question Td : 91953 Question 11 of 30 A.51 year old woman has been diagnosed with primary biiary cirrhosis. She has portal hypertension and accitis. Her physician placed her on a waiting list for iver transplantation. Which of the following is NOT an absolute contraindication for liver transplantation? a) HIV seropositivity. +) Extrahepatic malignancy, ©) Advanced cardiac disease, Y © & Futninant fiver faihare ©) Severe emphysema. Question Explanation: “Alcoholic liver disease and hepatitis C are two of the most commen indications for liver transplantation. The one year survival rats has risen from around 50% to greater than 80%. The development of prognostic criteria for various patients such as age, prothrombin time, and jaundice can predict the success ofthe transplantation, Being HIV-positive (A) is a contraindication for liver transplantation becanse of the relatively shortened life span. Life span would be shortened dramatically as a resut of the Report An Error immunosuppressing drugs required to prevent host rejection of the liver transplant, as it would exacerbate the immunosuppression induced by HIV extrahepatic malignancies (B), such as breast and lung cancer, are absolute contraincications for transplantation because of the shortened life span and the lack of available donors for liver transplantation, Advanced cardiac disease (C) such as, triple vessel coronary artery disease and end stage congestive heart failure are contraindlcations for transplantation because of the very high risk of mostalty during the surgery. Empiaysema (E) and pumonary fibrosis are also diseases associated with a poor prognosis during liver transplantation surgery and thus are absolute contvaindications. ‘Mark this question = => Question Td : 92902 Question 12 of 30 Several members of a family develop abdominal pain, nausea, and vomiting, followed in a few hours by different degrees of paralysis, about 24 hous after dinner of home canned beans. The most likely etiology of ther illness is which one of the following? a) Dysentery. ) Trichinosis. c) Botulism. 4) Guilain-Barre syndrome. ¢) Poliomyelitis. Anower [REISER] other Users Explanation Report An Esror Question Explanation: This is a typical presentation for food borne botulism, which results from the ingestion ofhome canned food. Symptoms usually develop within 24-36 Lours after consumption and can include gastrointestinal involvement, A descending paralysis, often syinmetrical, can result and can cause respiratory failure and death in severe cases. Dysentery cen produce severe diamhea but is not related to canned food and is not associated with paralysis. Trichinosis can resut from consumption of unccoked ot poorly cooked meat, Guillain-Barre syndrome is an ascending paralysis not associated with food or gastrointestinal symptoms. Polio is not associated with canned foods and does not usually produce symmetrical paralysis, ‘Mark this question = => Question Td : 92902 Question 12 of 30 Several members of a family develop abdominal pain, nausea, and vomiting, followed in a few hours by different degrees of paralysis, about 24 hours after dinner of home canned beans. The most likely etiology of their illtess is which one of the following? a) Dysentery. 6) Trichinosis. ¥ © c) Botulism. 4) Guilain-Barre syndrome. «) Poliomyelitis. Anower [REISER] other Users Explanation Report An Esror Question Explanation: This is a typical presentation for food borne botulism, which results from the ingestion ofhome canned food. Symptoms usually develop within 24-36 Lours after consumption and can include gastrointestinal involvement, A descending paralysis, often syinmetrical, can result and can cause respiratory failure and death in severe cases. Dysentery cen produce severe diamhea but is not related to canned food and is not associated with paralysis. Trichinosis can resut from consumption of unccoked ot poorly cooked meat, Guillain-Barre syndrome is an ascending paralysis not associated with food or gastrointestinal symptoms. Polio is not associated with canned foods and does not usually produce symmetrical paralysis, ‘Marke this question <= Question 13 of 30 Question Td : 96235 A53 year old man develops esophageal cancer. Which of the following is NOT a risk factor for the development of esophageal cancer? a) Caucasian ancestry. b) Tobacco use ©) Alochol use. ) 4) Plummer-Viasen syndrome ¢) Chronic ingestion of very hot beverages (e.g,, coffee. tea). Anowor (UBRIENSHER) ctner User's Explon Question Explanation: Report An Error “Also increasing the chance of development of esophageal cancer is previous existence of oral and pharyngeal cancer. ‘Marke this question <= Question 13 of 30 Question Td : 96235 A53 year old man develops esophageal cancer. Which of the following is NOT a risk factor for the development of esophageal cancer? WV © a) Caucasian ancestry. b) Tobacco use. ©) Alochol use. ) 4) Plummer-Viasen syndrome ¢) Chronic ingestion of very hot beverages (e.g,, coffee. tea). Anowor (UBRIENSHER) ctner User's Explon Question Explanation: Report An Error “Also increasing the chance of development of esophageal cancer is previous existence of oral and pharyngeal cancer. ‘Marke this question <= => Question Td : 97107 Question 14 of 30 A.A6 year old men presents with symptoms of severe epigastric pain, diarrhea, and weight loss. H12 blockers provide no relief His serum gastrin level is elevated to over 800 ng/L. Upper endoscopy was performed that showed @ lesion located in the postbulbar duodenum, The most lcely diagnosis is a) Pancreatitis ) Zolinger Bison syndrome ©) IL pyloriinduced duodenal ulcer MEN tre I ©) Celiac sprue Question Explanatic Zolinger Elison syndrome is associated with muhiple endocrine neoplasia syndrome type T in 20% of patients. Symptoms of abdominal pain and diarrhea are secondary to an elevated basal acid output and recurrence of disease, Serum gestrin, measured by radisimmnoassay, is the most reliable diagnostic test. A calcium or secretin simulation test can confirm the diagnosis by helping to elevate the gastrin and basal acid output, The ulcers in this syndrome are located in unusual locations such as postbulbar jejunum or duodenum, Pancreatitis would present wath severe abdominal pain and nausea or vorniting. Serum amylase and lipase levels would be elevated, not gastrin levels. H. pylori induced duodenal ulcer would present with epigastric pain, but usually H2 blockers would temporarily relieve the pein. The serum gastrin level would not be elevated to such a degree Mubiple endocrine neoplasia type Tis, associated with medullary thyroid carcinoma, pheochromocytoma, and parathyrcid hypesplasia. Zollinger Ellison syndrome is affliated with MEN type I, which is more likely to be characterized by parathyroid and pitutary adenomas, and pancreatic islet cell tumor. Celiac sprue would present with malabsorption symptoms secondary to cletary intake of wheat gluten, Serum gastrin levels, ace usually normed in this disorder, ‘Marke this question <= => Question Td : 97107 Question 14 of 30 A.46 year old man presents with symptoms of severe epigastric pain, diarrhea, and weight loss. H2 blockers provide no relief His scrum gastrin level is clevated to over 800 ng/L. Upper cadoscopy was performed that showed a lesion located in the postbulbar duodenum, The most licely diagnosis is a) Pancreattis Y © b) Zollinger Elison syndrome ©) H. pyloriinduced duodenal ulcer ) MEN type IZ ©) Celiac sprue Question Explanatic Zolinger Elison syndrome is associated with muhiple endocrine neoplasia syndrome type T in 20% of patients. Symptoms of abdominal pain and diarrhea are secondary to an elevated basal acid output and recurrence of disease, Serum gestrin, measured by radisimmnoassay, is the most reliable diagnostic test. A calcium or secretin simulation test can confirm the diagnosis by helping to elevate the gastrin and basal acid output, The ulcers in this syndrome are located in unusual locations such as postbulbar jejunum or duodenum, Pancreatitis would present wath severe abdominal pain and nausea or vorniting. Serum amylase and lipase levels would be elevated, not gastrin levels. H. pylori induced duodenal ulcer would present with epigastric pain, but usually H2 blockers would temporarily relieve the pein. The serum gastrin level would not be elevated to such a degree Mubiple endocrine neoplasia type Tis, associated with medullary thyroid carcinoma, pheochromocytoma, and parathyrcid hypesplasia. Zollinger Ellison syndrome is affliated with MEN type I, which is more likely to be characterized by parathyroid and pitutary adenomas, and pancreatic islet cell tumor. Celiac sprue would present with malabsorption symptoms secondary to cletary intake of wheat gluten, Serum gastrin levels, ace usually normed in this disorder, ‘Mark this question & => Question Td : 104455 Question 15 of 30 Achalasia is NOT characterized by a) The lower escphageal sphincter is hypertensive 'b) There is defective innervation of the smoot muscle portion of the lower esophageal sphincter. ©) Normal peristalsis of the esophageal body is replaced by abnormal contractions 4) The lower esophageal sphincter does not relax properly with swallowing #) High amplitude contractions are present in the classic form of the disease Question Explanation: Tn the classic form of achalasia, small amplinde contractions occur Achalasia is a motor disorder of the esophageal smooth muscle ‘The hypertensive lower esophageal sphincter does not rela properly with swallowing. Normal peristalsis of the esophageal body is replaced by abnormal contractions. There is defective inervetion of the smooth muscle portion of both the esophageal bedy and the lower esophageal sphincter. ‘Mark this question & => Question Td : 104455 Question 15 of 30 Achalasia is NOT characterized by a) The lower esophageal sphincter is hypertensive 'b) There is defective innervation of the smooth muscle portion of the lower esophageal sphincter. c) Normal peristalsis of the esophageal body is replaced by abnormal contractions ) The lower esophageal sphincter does not relax properly with swallowing. Y © #) High amplitude contractions are present in the classic form of the disease Question Explanation: Tn the classic form of achalasia, small amplinde contractions occur Achalasia is a motor disorder of the esophageal smooth muscle ‘The hypertensive lower esophageal sphincter does not rela properly with swallowing. Normal peristalsis of the esophageal body is replaced by abnormal contractions. There is defective inervetion of the smooth muscle portion of both the esophageal bedy and the lower esophageal sphincter. Marle this question & => Question Td : 121178 Question 16 of 30 A.25 yese eld man presente with eymptome of facial fiching which spreads to the trunk and upper extremities ascociated with Giamhea and wheezng Exan showed erythematous facial Dishing which eventually turned purplish. Hepatomegaly was also present ‘The blood pressure was low during this episode. 5-hydroxyindoleacetic acid level inthe urine was elevated. The most likely diagnosis is 2) Pheochromocytema +) Carcinoid syndrome ©) Mastocytosis 4) Thyrotoxicosie «) Urticaria Answer | Explanation Other User's Explanation Report An Error Question Explanation: Carcinoid syndrome is characterized by valvular heart disease, fushing, ciarthea and, less frequently, wheezing and paroxysmal typotension. Liver involvement suggests that carcinoid tumors are laccted in the foreant, in which case ishing is intense and leng lasting These tumors release serotonin, which triggers diarrhea and cardiac abnormalities. Serotonin is metabolized to 5- hydrozyindoleacetic acid, which is excreted excessively in urine and, in that way serves as a diagnostic marker for this disorder. Pheochromocytoma is associated with hypertension, not hypotension. It usvally presents with headaches, diaphoresis. and palpitations secondary to excess catacholamine production by the adrenel glands. Mastocytosis is associated with urticaria pigmentosa There is an abnormal increase in mast cells in the bone marrow, liver, spleen, iymph nodes, skin, and gastrointestinal tract. Patients who have mastocytosis do not excrete excessive amounts of 5-hydroxyindoleacetic acid. Thyrotoxic patients usualy ave kypertension and palpitations, Flushing is urmeual 5 hyckosyindeleacetic acid production ie normal. Urticaria i ascociated with symptomatic pruitic wheals and hives on the skin secondary to an allergic event. Liver enlargementis not commen. Marle this question & => Question Td : 121178 Question 16 of 30 A.25 yese eld man presente with eymptome of facial fiching which spreads to the trunk and upper extremities ascociated with Giamhea and wheezng Exan showed erythematous facial Dishing which eventually turned purplish. Hepatomegaly was also present ‘The blood pressure was low during this episode. 5-hydroxyindoleacetic acid level inthe urine was elevated. The most likely diagnosis is 2) Pheochromocytema Y © b) Carcinoid syndrome ©) Mastocytosis 4) Thyrotoxicosie «) Urticaria Answer | Explanation Other User's Explanation Report An Error Question Explanation: Carcinoid syndrome is characterized by valvular heart disease, fushing, ciarthea and, less frequently, wheezing and paroxysmal typotension. Liver involvement suggests that carcinoid tumors are laccted in the foreant, in which case ishing is intense and leng lasting These tumors release serotonin, which triggers diarrhea and cardiac abnormalities. Serotonin is metabolized to 5- hydrozyindoleacetic acid, which is excreted excessively in urine and, in that way serves as a diagnostic marker for this disorder. Pheochromocytoma is associated with hypertension, not hypotension. It usvally presents with headaches, diaphoresis. and palpitations secondary to excess catacholamine production by the adrenel glands. Mastocytosis is associated with urticaria pigmentosa There is an abnormal increase in mast cells in the bone marrow, liver, spleen, iymph nodes, skin, and gastrointestinal tract. Patients who have mastocytosis do not excrete excessive amounts of 5-hydroxyindoleacetic acid. Thyrotoxic patients usualy ave kypertension and palpitations, Flushing is urmeual 5 hyckosyindeleacetic acid production ie normal. Urticaria i ascociated with symptomatic pruitic wheals and hives on the skin secondary to an allergic event. Liver enlargementis not commen. Merle this question = => Question 17 of 30 A665 year old woman presented with lower gestrointestinal bleeding, not associated with abdominal pain, anoresia, or weight loss Examination showed tachycardia with a blood pressure of 90/70 min Hg and pallor. Rectal examninetion revealed bright red blood Her hematocrit was 22%, The most commen cause for thie patients bleeding ic which one of the following? a) Duodenal ulcer +) Neoplastic disease ofthe colon ©) Meckel’s ctverticulum with retained gastric mucosa 4) Gastric ulcer 6) Diverticulosis Question Explanation: Diverticulosis is the most common cause of lower GI bleeding in this age group. Diverticular hemorthage is cheracteristically painless and associated with large volume hematochezia, Inflammation of the diverticuli causes diverticulitis. A high fiber cietis recommended, Ducdenal ulcer is more common in cigarette smokers, and melena is usuelly the presenting sign, along with epigastric pain. Neoplastic disease of the colon is common in this age group, but this patient did not have weight loss of anotexia to suggest mmot. Meckel’s diverticulum also causes lower GI bleeding, but in adults it is often accompanied by abdominal pain and vomiting Patients with gastric uicers would present with upper Gl bleeding which would be darkcer in color than blood irom the lower Gl wract and epigastric pain. Merle this question = => Question 17 of 30 A665 year old woman presented with lower gestrointestinal bleeding, not associated with abdominal pain, anoresia, or weight loss Examination showed tachycardia with a blood pressure of 90/70 min Hg and pallor. Rectal examninetion revealed bright red blood Her hematocrit was 22%, The most commen cause for thie patients bleeding ic which one of the following? a) Duodenal ulcer +) Neoplastic disease ofthe colon ©) Meckel’s ctverticulum with retained gastric mucosa 4) Gastric ulcer Y © 2) Diverticulosis Question Explanation: Diverticulosis is the most common cause of lower GI bleeding in this age group. Diverticular hemorthage is cheracteristically painless and associated with large volume hematochezia, Inflammation of the diverticuli causes diverticulitis. A high fiber cietis recommended, Ducdenal ulcer is more common in cigarette smokers, and melena is usuelly the presenting sign, along with epigastric pain. Neoplastic disease of the colon is common in this age group, but this patient did not have weight loss of anotexia to suggest mmot. Meckel’s diverticulum also causes lower GI bleeding, but in adults it is often accompanied by abdominal pain and vomiting Patients with gastric uicers would present with upper Gl bleeding which would be darkcer in color than blood irom the lower Gl wract and epigastric pain. Marke this question <> Question Td : 139589 Question 18 of 30 A 33 year man has been chewing tobacco since childhood, He does not smoke but occasionally uses alcohol. He presents with thiclcened white regon in his mouth. Biopsy shows hyperieratotic skin with some mild dysplasia. The most icely location of the lesion on the basis of history and biopsy is a) Floor of the mouth ) Lower lip ©) Tongue 4) Tonsiis ©) Upper lip Question Explanation: Oral cancer is strongly related to tobacco chewing, and is also related to cigarette smoking pipe smoking, and alcohol use Cancers of the lower lip are also inked to sun exposure, Oral cancers tend to occur on the lower lip, floor of the mouth, and tongue, with other oral sites being less common, Note that the most common locations within the oral cavity proper are the floor of the routh tongue (inchding the base) and hard palate bur ifthe lower ip is included in the choices it will generally be the comrect answer for this type of question, These cancers are usally squamous cell carcinomas and unlice their skin counterparts, frequently cause both extensive morbidity and mortality. Marke this question <> Question Td : 139589 Question 18 of 30 A 33 year man has been chewing tobacco since childhood, He does not smoke but occasionally uses alcohol. He presents with thiclcened white regon in his mouth. Biopsy shows hyperieratotic skin with some mild dysplasia. The most icely location of the lesion on the basis of history and biopsy is a) Floor of the mouth Y © b) Lower lip ©) Tongue 4) Tonsiis ©) Upper lip Question Explanation: Oral cancer is strongly related to tobacco chewing, and is also related to cigarette smoking pipe smoking, and alcohol use Cancers of the lower lip are also inked to sun exposure, Oral cancers tend to occur on the lower lip, floor of the mouth, and tongue, with other oral sites being less common, Note that the most common locations within the oral cavity proper are the floor of the routh tongue (inchding the base) and hard palate bur ifthe lower ip is included in the choices it will generally be the comrect answer for this type of question, These cancers are usally squamous cell carcinomas and unlice their skin counterparts, frequently cause both extensive morbidity and mortality. 3i2/2014 6:40:53 PM Mark this question & => Question Td : 197229 Question 19 of 30 34 year old woman with alcoholic citthosis is admitted with deteriorating encephalopathy and abdominal discomfort. An ascitic tap reveals a pelymorphomuclear cell count of 350 cells per mm3. The appropriate therapy is a) Intravenous amoxicillin ) Intravenous Cefotaaiine ©) Intravenous Metronidazole 46) Oral neomycin 6) Oral norfloxacin Answer [FEIRIRRRHBR) Other Users Explanaion Repost An Exar Question Explanation: ‘hic lady hac epentaneous bacterial peritontic ac cuggested by the typically history, ascites and raiced polymorphonuclear count ‘within the ascitic tap. Ibis most commonly seen in alcoholic cicrhosis and the causative organist: is usually Esherichia coli, ebsiela, $ pneumoniae or Enterococci, (Compare this with the mixed growth seen in other forms of peritonitis). Sending some ascitis fai in ‘blood cuiture bottles increases the yield. Intiel treatment ic with broad spectrum antibiotics suck as cefotaxime, Norflozacin is recommenced for sho:t term prophylazis 3i2/2014 6:40:53 PM Mark this question & => Question Td : 197229 Question 19 of 30 34 year old woman with alcoholic citthosis is admitted with deteriorating encephalopathy and abdominal discomfort. An ascitic tap reveals a pelymorphonuclear cell count of 350 cells per mun. The appropriate therapy is a) Intravenous amoxicillin © b) Intavenous Cefotexime ©) Intravenous Metronidazole 6) Oral neomycin 6) Oral nosozacin Answer [FEIRIRRRHBR) Other Users Explanaion Repost An Exar Question Explanation: ‘hic lady hac epentaneous bacterial peritontic ac cuggested by the typically history, ascites and raiced polymorphonuclear count ‘within the ascitic tap. Ibis most commonly seen in alcoholic cicrhosis and the causative organist: is usually Esherichia coli, ebsiela, $ pneumoniae or Enterococci, (Compare this with the mixed growth seen in other forms of peritonitis). Sending some ascitis fai in ‘blood cuiture bottles increases the yield. Intiel treatment ic with broad spectrum antibiotics suck as cefotaxime, Norflozacin is recommenced for sho:t term prophylazis Mark this question <= >> Question Td ; 197270 Question 20 of 30 A.48 year old male has confision and drowsiness. A diagnosis of hepatic encephalopathy is suspected and treatment with lactulose is begun. True statement regarding lactulose is which one of the following? a) Absorbed from the gut 'b) Causes Hypermagnesaemia c) Contraindlcated in diabetes melitus ) Inhibits proliferation of ammonia forming organisms in the gut ¢) Reduces absorption of spironolactone Answer (Berisnenon) Other User's Explanation Report An Error Question Explanatioy Lactulose, an osmotic curetic causes hypomagneszemia associated with diarrhoea, is not absorbed, does not affect the absorption of spiranclactone and may he uted in diabetics. Tt is used in patients with circhasisthepetic encephalopathy to lit the proliferation of ammoria forming gut organisms and increase the clearance of preteia load in the gut Mark this question <= >> Question Td ; 197270 Question 20 of 30 A48 year old male has confusion anc drowsiness. A diagnosis of hepatic encephalopathy is suspected and treatment with lactulose is ‘begun, True statement regarding lactulose is which one of the following? a) Absorbed from the gut 'b) Causes Hypermagnesaceia c) Contraindlcated in diabetes melitus V © A) Inhibits proliferation of ammonia forming organisms in the gut ¢) Reduces absorption of spironolactone Answer (Berisnenon) Other User's Explanation Report An Error Question Explanatioy Lactulose, an osmotic curetic causes hypomagneszemia associated with diarrhoea, is not absorbed, does not affect the absorption of spiranclactone and may he uted in diabetics. Tt is used in patients with circhasisthepetic encephalopathy to lit the proliferation of ammoria forming gut organisms and increase the clearance of preteia load in the gut Question Td : 197394 Merle this question ¢<>=> Question 21 of 30 A 34 year old with a history of recurrent anemia was noted have target cells and Howell Joly bodies on a bloed film examination His Eb is 7.0 g/dL, MCV is 77 £L, MCH is 26.2pg, serum B12, red cell folate and serum ferritn are low. The disease specific antibody thatis most likely to be present is a) Anti-endomysial ) Anti-gasttic parietal cell ©) Anti-gutamic acid decarboxylase 6) Anti-intrinsic factor ©) Antimitochondrial Anewor [UEQIRNSAN) Other teers Explanation Report An Error Question Explanatic ‘The patient has hyposplenism as suggested by the blocd film and a mised anaemia Ciceliac disease could therefore fit the above pictare with anti-endomysial antibodies being the most appropriate selection from the above list. Antimitochonsial antibodies are seen in PBC, ant- gastric and ant intrinsic Abs are seen in pemicious anaemia Ant-GAD antbodies are found in auto-immune DM Screening for coeliac disease should include high-risk groups such es anaertia (iron or folate deficiency), hyposplenism, reduced, bone density and inferility. Anti-endomysial TeA antibodias are extremely specific markers for CD and for dermatitis herpetiformis, ‘These antibodies are drected to a component of the gut endomysium (connective tissue surrounding smooth muscle fibers of the gu). Question Td : 197394 Merle this question ¢<>=> Question 21 of 30 A.34 year old with a history of recurrent anemia was noted have target cells and Howell Joly bodies on a blood film examination His Eb is 7.0 g/dL, MCVis 77 £L, MCH is 26.2pe, serum B12, red cell folate and serum ferritin are low. The disease specific antibody that is most likely to be present is Y © a) Ant-endomysial +) Anti-gastric parietal cell c) Ant-ghutamic acid decarboxylase ) Anti-intrinsic factor 2) Antimitochondrial Anewor [UEQIRNSAN) Other teers Explanation Report An Error Question Explanatic ‘The patient has hyposplenism as suggested by the blocd film and a mised anaemia Ciceliac disease could therefore fit the above pictare with anti-endomysial antibodies being the most appropriate selection from the above list. Antimitochonsial antibodies are seen in PBC, ant- gastric and ant intrinsic Abs are seen in pemicious anaemia Ant-GAD antbodies are found in auto-immune DM Screening for coeliac disease should include high-risk groups such es anaertia (iron or folate deficiency), hyposplenism, reduced, bone density and inferility. Anti-endomysial TeA antibodias are extremely specific markers for CD and for dermatitis herpetiformis, ‘These antibodies are drected to a component of the gut endomysium (connective tissue surrounding smooth muscle fibers of the gu). ‘Mark this question = => Question Id : 197788 Question 22 of 30 AT? year old woman with hip osteoarthritis presents with altered bowel habit. Rectal biopsy demonstrates normal epithelium and pigment laden macrophages in the lamina propria, The most likely cause of these findings is a) Diverticular disease ') Laxative abuse c) mesenteric ischaemia 4) Non-steroidal antiinflammatory drugs €) Ulcerative colitis Question Explanation: She has ‘melanosis coli’ as a result of prolonged laxative use. Often the bowel mucosa locks dark and ‘stained’ during colonoscopy. She may be predisposed to constipation due to immobility from her arthritis and/or use of constipating pain killers. ‘Mark this question = => Question Id : 197788 Question 22 of 30 ATT year old woman with hip osteoartivtis presents with altered bowel habit. Rectal biopsy demonstrates normal epithelium and pigment laden macrophages in the lamina propria, The most likely cause of these findings is a) Diverticular disease Y © b) Laxative abuse c) mesenteric ischaemia 4) Non-steroidal antiinflammatory drugs €) Ulcerative colitis Question Explanation: She has ‘melanosis coli’ as a result of prolonged laxative use. Often the bowel mucosa locks dark and ‘stained’ during colonoscopy. She may be predisposed to constipation due to immobility from her arthritis and/or use of constipating pain killers. Mark this question Question Td : 197849 ¢-= Question 23 of 30 A57 year old type 2 diabetic female has diarthoea and voniting, She has noticed small amounts of blood in her stools. The vomiting started one day after ameal of chicken and chips. Her Type 2 diabetes is treated with diet alone. Stool cultures reveal Campylobacter jejuni, The most appropriate therapy is a) Amoxicilin ) Cefaclor c) IV Fhids d) Meironidazole e) Trimethoprin Answer (Birisnaton) Other User's Explanation Report An Error Question Explanatior Campslobacter is aleading cause of diarrhoea ilhess, offen caused by ingestion of undercooked meat harbouring the pathogen. Its also a major cause of traveller's diarrhoea. The use of antibiotic therapy for the management of Campylobacter infection in adults is controversial. Antibiotic of cheice in this infection is erythromycin, though ciproflexacin and tetracycline may also be appropriate ‘However, appropriate fluid replacement and antiemetics are intially indicated-most units advocate no antibiotic treatment. Mark this question Question Td : 197849 ¢-= Question 23 of 30 A.57 year old type 2 diabetic female has diarrhoea and vomiting, She has noticed small ancunts of blood in her stools. The vorniing started one day after ameal of chicken and chips. Her Type 2 diabetes is treated with diet alone. Stool cultures reveal Campylobacter jejuni. The most appropnate therapy is a) Amoxicilin b) Cefaclor ¥ © IV Fluids oD) Metronidazole ) Trimethoprim Answer (Birisnaton) Other User's Explanation Report An Error Question Explanatior Campslobacter is aleading cause of diarrhoea ilhess, offen caused by ingestion of undercooked meat harbouring the pathogen. Its also a major cause of traveller's diarrhoea. The use of antibiotic therapy for the management of Campylobacter infection in adults is controversial. Antibiotic of cheice in this infection is erythromycin, though ciproflexacin and tetracycline may also be appropriate ‘However, appropriate fluid replacement and antiemetics are intially indicated-most units advocate no antibiotic treatment. Mark this question ez Question 24 of 30 A man is referred for possible treatment of Hepatitis B. He has stigmata of chronic Lver disease. There is portal hypertension and ascites, His INR is 2.2 (<1.4) and albumin 25 g/L. 37-49). HBsAg and HBeAg are positive. Hepatitis C screen is negative. Which cone of the following would he suggested far treatment? 2) Beta interferon ') Lumivudine alone ©) Lurivudine pls interferon 4) Ribavarin done ¢) Ribavarin plus interferon newer [FERRBIRNNY Other Users Expan Question Explanation: Pibavirinis used for Hepatitis C infection Tis combination with interferon confers more success in treating HCV infection. Interferon camot be used in this case as it can initially worsen hepatic decompensation, Lumivadine alone is safe in decompensated HBV infection. n Report An Error Mark this question ez Question 24 of 30 A man is referred for possible treatment of Hepatitis B. He has stigmata of chronic lwer cisease. There is portal hypertension and. ascites, His INR is 2.2 (<1.4) and albumin 25 g/L 37-49), HBsAg and EBeAg are positive Hepatitis C screen is negative. Which cone of the following would be suggested for treatment? a) Beta interferon Y © b) Lumivudine alone ¢) Lumivudine plus interferon 4) Ribavarin done ¢) Ribavarin plus interferon newer [FERRBIRNNY Other Users Expan Question Explanation: Pibavirinis used for Hepatitis C infection Tis combination with interferon confers more success in treating HCV infection. Interferon camot be used in this case as it can initially worsen hepatic decompensation, Lumivadine alone is safe in decompensated HBV infection. n Report An Error Marke this question << => Question Td : 199497 Question 25 of 30 Conceming iron the trie statement is which one of the fellowing? 2) Absorption is prevented by ascorbic acid ') Iron absorption is mainly in the distal jejunum. ¢) Parenteral ion is indicated ifthe haemoglobin level is net raiced within three days by oral iron. 4) Sustained release preparations are usefil iflarger doses are required €) 200 mg iron culphate has more elemental iron than an equal dose of ron gluconate newer (UBNREREN) oer users Explanation Report An Error Question Explanation: Tis important for the structure of haemoglobin and myoglobin, for O2 and CO2 transport: oxidative enzymes: Cytochrome C and catalase, Absorbed in ferrous form in small bowel according to body need, sided by gestric juice and ascorbic acid; hindered by fibre, phytic acid a steatorrhoea, Transported in plasma in ferric state bound to transferrin; stored in iver, spleen, bone marrow and kkiduey as ferritin and haemosiderin, conserved end reused; minimal losses in urine and sweat, about 90% offintake excreted in stool Ferrous sulphate contains about twice the amount of elemental iron as the ghiconate. Levels raise Hb levels about 0.5a/100mnl per week. Sustained release preparations should not be used, as they delay release beyond the early small bowel, where most iron absorption occurs. Marke this question << => Question Td : 199497 Question 25 of 30 Conceming iron the trie statement is which one of the fellowing? 2) Absorption is prevented by ascorbic acid ') Iron absorption is mainly in the distal jejunum. ¢) Parenteral ion is indicated ifthe haemoglobin level is net raiced within three days by oral iron. 4) Sustained release preparations are usefil iflarger doses are required Y © 6) 200 mg iron culphate has more elemental iron than an equal dose of ron gluconate newer (UBNREREN) oer users Explanation Report An Error Question Explanation: Tis important for the structure of haemoglobin and myoglobin, for O2 and CO2 transport: oxidative enzymes: Cytochrome C and catalase, Absorbed in ferrous form in small bowel according to body need, sided by gestric juice and ascorbic acid; hindered by fibre, phytic acid a steatorrhoea, Transported in plasma in ferric state bound to transferrin; stored in iver, spleen, bone marrow and kkiduey as ferritin and haemosiderin, conserved end reused; minimal losses in urine and sweat, about 90% offintake excreted in stool Ferrous sulphate contains about twice the amount of elemental iron as the ghiconate. Levels raise Hb levels about 0.5a/100mnl per week. Sustained release preparations should not be used, as they delay release beyond the early small bowel, where most iron absorption occurs. Mark this question e¢-= Question 26 of 30 Question Td : 199517 ‘A 32 year old man with chronic hepatitis C is admitted with general deterioration, He has missed many of his previous outpatient appointments and currently is not recexving any treatment His temperature is 37.8°C, BP is 110/72 mmElg and appears jaundiced. Serum urea, crestinine, bilirubin, AST, and ALP are elevated. Albumin is 30 g/L and wrine sodium is 1Smmol/L. The liceiy diagnosis a) Abdominal tuberculosis ) Hepatocellular carcinoma ¢) Hepatorenal syndrome 4) Mized essential Cryoglobulinaemia €) Spontaneous bacterial peritonitis Question Explanation: The abnormal urca and creatinine plus the low urine sodinm suggest a diggnosis of hepatorenal syncrome. Fluid balance is very difficuit in these patients but some respond so wreatment with intravenous (IV) Glypressin which improves kidney perfusion. Mark this question e¢-= Question 26 of 30 Question Td : 199517 ‘A 32 year old man with chronic hepatitis C is admitted with general deterioration, He has missed many of his previous outpatient appointments and currently is not recexving any treatment His temperature is 37.8°C, BP is 110/72 mmElg and appears jaundiced. Serum urea, crestinine, bilirubin, AST, and ALP are elevated. Albumin is 30 g/L and wrine sodium is 1Smmol/L. The liceiy diagnosis a) Abdominal tuberculosis ) Hepatocellular carcinoma Y © ¢) Hepatorenal syndrome 4) Mized essential Cryoglobulinaemia €) Spontaneous bacterial peritonitis Question Explanation: The abnormal urca and creatinine plus the low urine sodinm suggest a diggnosis of hepatorenal syncrome. Fluid balance is very difficuit in these patients but some respond so wreatment with intravenous (IV) Glypressin which improves kidney perfusion. ‘Mark this question e& => Question Td : 199549 Question 27 of 30 Afemale aged 64 years presents with symptoms suggestive of intitable bowel syndrome, Which finding would represent a ‘sed flag’ indicator and prompt further investigation? a) Abdominal pain +) Bloating c) Change in bowel habit for last twa years 4) More frequent stoollast two months ©) Weight gain Answer | Bolanation | Other User's Explanation Report An Error Question Explanation: The ‘red flag’ indicators are listed in the NICE guidelines as -unintentional and unintended weightloss - rectal bleeding - a family history of bowel or ovarian cancer - a change in bowel habit to looser and/ or more frequent stools persisting for more than six weeks in a person aged ever 60 years. Also on clinical examination the other ‘red flag’ indicators are: - anaeatnia -abdomninal mass - rectal mass “inflammatory markers for inflammatory bowel dissase. ‘Mark this question e& => Question Td : 199549 Question 27 of 30 Afemale aged 64 years presents with symptoms suggestive of intitable bowel syndrome, Which finding would represent a ‘sed flag’ indicator and prompt further investigation? a) Abdominal pain +) Bloating c) Change in bowel habit for last twa years Y © d) More frequent stool last two months e) Weight gain Answer | Bolanation | Other User's Explanation Report An Error Question Explanation: The ‘red flag’ indicators are listed in the NICE guidelines as -unintentional and unintended weightloss - rectal bleeding - a family history of bowel or ovarian cancer - a change in bowel habit to looser and/ or more frequent stools persisting for more than six weeks in a person aged ever 60 years. Also on clinical examination the other ‘red flag’ indicators are: - anaeatnia -abdomninal mass - rectal mass “inflammatory markers for inflammatory bowel dissase. Mark this question & => Question Ta : 199653 Question 28 of 30 ‘A student aged 19 years hes a 15 week history of dicrrhea He has lost 2 kgin weight, and has no recent travel abroad. Duodenal biopsy sinear reveals many trophozoites. The best treatment option is which one of the following? a) Ciprofloxacin +) Giutea free diet c) Metronidazole 4) Prednisolone ©) Quinine newer (UBINSRREN) oer usar Explanation Report An Error Question Explanation: The diagnosis here is giardiasis, caused by Giardia lamblia. Giardia has been reported as a cause of chronic diarrhoea, Most patient respond to oral metronidazol 250-400 mg three timed daily for five days Mark this question & => Question Ta : 199653 Question 28 of 30 ‘A student aged 19 years has a 15 week hictory of dicerkea. He has lost 2 kgin weight, and has ne recent travel abroad, Duodenal biopsy smear reveals many trophozoites. The best treatment option is which one of the following? 2) Ciprofloxacin ) Ghitea ftee diet Y © ¢) Metronidazole 4) Predrisolone ©) Quinine newer (UBINSRREN) oer usar Explanation Report An Error Question Explanation: The diagnosis here is giardiasis, caused by Giardia lamblia. Giardia has been reported as a cause of chronic diarrhoea, Most patient respond to oral metronidazol 250-400 mg three timed daily for five days 3122014 &% worw.interface, x \ FE Styl s8all alos % (2 unread) err \ 6? faredai9@4's pix \ Gil onlymeonly's pr_ x \ &? females online. x \\_) Lele] 2 ie ¢ www. interface edu.pk/medical-exams/test-analysis pho tutid=14759 x 8 Apps EJ Google A Settings [Signin 2 isle dul En yo uedlldaye GF..aio yt Boiol JS |) Free Hotmail » (0) Other bookmar ‘Marke this question & => Question Td Total Questions t Question 29 of 30 4 A.56-year-old woman had en accident 1 week ago. Her abdomen was struck by the steering wheel. She localizes severe mid- 3 abdominal pain ta the umbiliens with radiation ta the back. She is in mild distress secondary to pain with no ather findings. No 4 masses, rebound tendemess, or guarding exists, CTis as follows: 5 é 2 g a 10 Localize the lesion a) Aorta ) Kidney ©) Liver d) Pancreas ©) Spleen 8) Stomach Question Explanation: The patient had a motor vehicle accident that resulted in damage ofthe pancreas, as seen on the CT scan, This can occur with direct ‘blunt injury to the abdomen, when the pancreas becomes compressed toward the spine, resulting in traumatic pancreatitis Approximately 60% of patients with acute pancreattis due to trauma require surgical exploration Tnnury to the abdominal acrta would result in urstable hemodynamics in the patient and would show evidence of bleeding into the abdomen, A pulsetile mass is often found in patients with an abdominal aneurysm, but this is unlikely considering this patient's age and uncoatibutory history. Addiionally, the CT acan clearly chows anormal dismetor of the aorta The kichey liver and spleen appear nommal on this CT scan, with no evidence of blood (white) or ischemia (dark grey). Contrast, seen in the stomach (anteriorly), fails to show any abnormalities, and is not typically the diagnostic tool used for suspected stomach BREBRBREBBEBKBEHBBE is Ek xxxXxXXKXXKXXKXKXKKXKKXKKKKKKKKKKKKKKR 3122014 &% worw.interface, x \ FE Styl s8all alos % (2 unread) err \ 6? faredai9@4's pix \ Gil onlymeonly's pr_ x \ &? females online. x \\_) Lele] 2 ie ¢ www. interface edu.pk/medical-exams/test-analysis pho tutid=14759 x 8 Apps EJ Google A Settings [Signin 2 isle dul En yo uedlldaye GF..aio yt Boiol JS |) Free Hotmail » (0) Other bookmar ‘Marke this question & => Question Td Total Questions t Question 29 of 30 4 A.56-year-old woman had en accident 1 week ago. Her abdomen was struck by the steering wheel. She localizes severe mid- 3 abdominal pain ta the umbiliens with radiation ta the back. She is in mild distress secondary to pain with no ather findings. No 4 masses, rebound tendemess, or guarding exists, CTis as follows: 5 é 2 g a 10 Localize the lesion: a) Aorta ) Kidney c) Liver ¥ © @) Pancreas e) Spleen, £) Stomach Question Explanation: The patient had a motor vehicle accident that resulted in damage ofthe pancreas, as seen on the CT scan, This can occur with direct ‘blunt injury to the abdomen, when the pancreas becomes compressed toward the spine, resulting in traumatic pancreatitis Approximately 60% of patients with acute pancreattis due to trauma require surgical exploration Tnnury to the abdominal acrta would result in urstable hemodynamics in the patient and would show evidence of bleeding into the abdomen, A pulsetile mass is often found in patients with an abdominal aneurysm, but this is unlikely considering this patient's age and uncoatibutory history. Addiionally, the CT acan clearly chows anormal dismetor of the aorta The kichey liver and spleen appear nommal on this CT scan, with no evidence of blood (white) or ischemia (dark grey). Contrast, seen in the stomach (anteriorly), fails to show any abnormalities, and is not typically the diagnostic tool used for suspected stomach BREBRBREBBEBKBEHBBE is Ek xxxXxXXKXXKXXKXKXKKXKKXKKKKKKKKKKKKKKR € CD www. interface. edu pk/medical-exarrs/test-analysis, pho ?utid=14759 EI cosgle Aseungs [spin Su. wlohe dul Eun ge uediléye Of,.ci0 G2 giall JS [) Free Hotmall ‘Matte this question e Question Ta : 213778 Question 30 of 30 A.4B-year-old alcoholic has severe persistent pain located in the epigastzic area along anorexia, fever, and chills. She had a receat bout of pancreatitis 3 weeks ago. She is febrile and tachycardic. Examination suggests a tender, il-defined epigastric mass, with no rebound or guarding. There is leukocytosis and increased lipase and amaylase. LFTs are normal. CT abdomen is as follows ‘What is the most appropriate next step? a) Endoscopic retrograde cholangiopancreatography (ERCP) b) Bsophagogesttoduodeaoscapy EEGD) ) Gastric emptying study 4) Mesentene angiogrephy ©) Percutaneous drainage £) Supportive management Question Explanation: This patient has presented with an infected pancreatic pseudocyst (pancreatic abscess) as complication of her recent bout of pancreatitis. Typically, acute pancreatitis causes pseudocyst formation in 50% of cases with the frst symptoms presenting 4 weeks after the episode, Complications inchde enlargement (pain, biliary, or pyloric abstruction), infection, abscess formation, rupture into the perkonewn, or erosion into a vessel, causing Lemorhage. The history and clinical presentation in this patient is highly suggestive, and is also confirmed by the CT scan. Because her condition has worsened since initial symptoms a week ago, surgical therapy is indicated scan of the abdomen shows dilated portion of the distal pancreatic duct and small fid collection, as outlined below. Endescapic retrograde cholangiopancreatography ERCP) is used for sphincteratomy ard gallstone remaval ia patients with panccestitis secondary to choledocholihiesis. This patient's history suggests alechol abuse as the likely cause of her intial bout of pancreatitis, thus making ERCP ineffective Esophagogastro duodenoscopy is indicated in the workup of hematemesis, melena, peptic ulcer disease, upper gastrointestinal carcinoma exclusion, and dysphagia, Tris alsa necessary in cases of suspected gasric or duodenal ulcers However, this patient's clinical presentation is not consistent with gastric or duodenal ulcers. Gastric emptying study can reveal delayed gastric emptying, which may produce symptoms like nausea and vorntting, but usually not abdominal pain. Because this patient's clinical presentation and laboratory and CT findings are consistent with pancreatits, a delayed gastric ematying is not llcely. Mesenteric angiography is a gold standard test for mesenteric ischemia, Its net used for diagnosis of pancreatitis. Supportive management is appropriate in the intial management of a pancreatic pseudocyst. However, this patient's symptoms have persisted, with subsequent infection ofthe cyst material. There is further risk of rupture, erosion, and obstruction of surrounding structures without percutaneous drainage of infected cect material #38 other bookrnar Total Questions bl He BEBE i6 th ie ee ee BEB BREBRE xxXxXKXKXKXKXKKKXKKKKKKKKKKKKKKKKKKKK € CD www. interface. edu pk/medical-exarrs/test-analysis, pho ?utid=14759 EI cosgle Aseungs [spin Su. wlohe dul Eun ge uediléye Of,.ci0 G2 giall JS [) Free Hotmall ‘Matte this question e Question Ta : 213778 Question 30 of 30 A.4B-year-old alcoholic has severe persistent pain located in the epigastzic area along anorexia, fever, and chills. She had a receat bout of pancreatitis 3 weeks ago. She is febrile and tachycardic. Examination suggests a tender, il-defined epigastric mass, with no rebound or guarding. There is leukocytosis and increased lipase and amaylase. LFTs are normal. CT abdomen is as follows ‘What is the most appropriate next step? a) Endoscopic retrograde cholangiopancreatography (ERCP) b) Bsophagogesttoduodeaoscapy EEGD) ) Gastric emptying study 4) Mesentene angiogrephy JY © ¢)Percutaneous drainage £) Supportive management Question Explanation: This patient has presented with an infected pancreatic pseudocyst (pancreatic abscess) as complication of her recent bout of pancreatitis. Typically, acute pancreatitis causes pseudocyst formation in 50% of cases with the frst symptoms presenting 4 weeks after the episode, Complications inchde enlargement (pain, biliary, or pyloric abstruction), infection, abscess formation, rupture into the perkonewn, or erosion into a vessel, causing Lemorhage. The history and clinical presentation in this patient is highly suggestive, and is also confirmed by the CT scan. Because her condition has worsened since initial symptoms a week ago, surgical therapy is indicated scan of the abdomen shows dilated portion of the distal pancreatic duct and small fid collection, as outlined below. Endescapic retrograde cholangiopancreatography ERCP) is used for sphincteratomy ard gallstone remaval ia patients with panccestitis secondary to choledocholihiesis. This patient's history suggests alechol abuse as the likely cause of her intial bout of pancreatitis, thus making ERCP ineffective Esophagogastro duodenoscopy is indicated in the workup of hematemesis, melena, peptic ulcer disease, upper gastrointestinal carcinoma exclusion, and dysphagia, Tris alsa necessary in cases of suspected gasric or duodenal ulcers However, this patient's clinical presentation is not consistent with gastric or duodenal ulcers. Gastric emptying study can reveal delayed gastric emptying, which may produce symptoms like nausea and vorntting, but usually not abdominal pain. Because this patient's clinical presentation and laboratory and CT findings are consistent with pancreatits, a delayed gastric ematying is not llcely. Mesenteric angiography is a gold standard test for mesenteric ischemia, Its net used for diagnosis of pancreatitis. Supportive management is appropriate in the intial management of a pancreatic pseudocyst. However, this patient's symptoms have persisted, with subsequent infection ofthe cyst material. There is further risk of rupture, erosion, and obstruction of surrounding structures without percutaneous drainage of infected cect material #38 other bookrnar Total Questions bl He BEBE i6 th ie ee ee BEB BREBRE xxXxXKXKXKXKXKKKXKKKKKKKKKKKKKKKKKKKK ‘Mark this question => Question Td : 14179 Question 1 of 30 “Vhhich one of the fellowing requices urgent referral for upper endoscopy? 2) A 45-year-old male with a one month history of persistent dyspepsia +) A 56-year-old male with a one month history of dyspepsia and a pulsatile central abclominal mass ©) A 73-year-old male with a three month history of dyspepsia which has failed to respond to a course of proton pump inhibitors 4) A 35-year-old male who has a history of waterbrash and dyspepsia which has responded to a course of ranitidine but after stopping, has recurred 6) A 62-year-old male with a three month history of unexplained weight locs, tenesnauc and sight abdomiral macs newer (UBINREREN) ner Users Pxplanation Report An Error Question Explanation: Criteria for referral for urgent endoscopy include + dysphagia (at any age) + dyspepsia at any age combines with any one of weight loss, anemia or vomiting + dyspepsia in a patient aged 55 or above with onset of dyspepsia within one year and persistent symptoms + dyspepsia with one of Barrett's esophagus. famiial hyper cholestrolemia (FH) or upper gastrointestinal (GI) carcinoma, + -pemicious anemia ce upper GI curgery more thaa 20 years ago + jaundice * abdominal mass. ‘With regard to the preseated cases, the second case has dyspepsia with what seems to be an aortic aneurysm, This requires an uitrascund and vascular opinion. In the last case of unexplained weightloss, tenesmus and upper right mass the problem is likely to be a colonic carcinoma, ‘Mark this question => Question Td : 14179 Question 1 of 30 “Vhhich one of the fellowing requices urgent referral for upper endoscopy? 2) A 45-year-old male with a one month history of persistent dyspepsia +) A 56-year-old male with a one month history of dyspepsia and a pulsatile central abclominal mass Y © 0) 73-year-old male with a three month history of dyspepsia which has failed to respond to a course of proton pump inhibitors 4) A 35-year-old male who has a history of waterbrash and dyspepsia which has responded to a course of ranitidine but after stopping, has recurred 6) A 62-year-old male with a three month history of unexplained weight locs, tenesnauc and sight abdomiral macs newer (UBINREREN) ner Users Pxplanation Report An Error Question Explanation: Criteria for referral for urgent endoscopy include + dysphagia (at any age) + dyspepsia at any age combines with any one of weight loss, anemia or vomiting + dyspepsia in a patient aged 55 or above with onset of dyspepsia within one year and persistent symptoms + dyspepsia with one of Barrett's esophagus. famiial hyper cholestrolemia (FH) or upper gastrointestinal (GI) carcinoma, + -pemicious anemia ce upper GI curgery more thaa 20 years ago + jaundice * abdominal mass. ‘With regard to the preseated cases, the second case has dyspepsia with what seems to be an aortic aneurysm, This requires an uitrascund and vascular opinion. In the last case of unexplained weightloss, tenesmus and upper right mass the problem is likely to be a colonic carcinoma, Question Td Mark this question <= => Question 2 of 30 A 29 year old male presents with abdominal pain radiating to the back, after aweekend of heavy drinking Whatis the investigative and prognostic modality of choice for the cuspected diagnosic? a) Abdorninal ultrasound +) Abdominal plain fa c) Endoscopic retrograde cholangiopancrestogrephy ) Abdomixal CT e) Laparoscopy Question Explanation: Acute pancteatitis is inflammation of the pancreas (end, sometimes, adjacent tissues) caused by the release of activated pancreatic enzymes. The most common triggers are biliary tract disease and chronic heavy alohel intake. The condition ranges from rrild (abdominal pain and vomiting) to severe (pencreatic necrosis end a systemic inflammatory process with shock and multiorgan failare). ‘Diagnosis is based on clinical presentation and serum amylase and lioase levels. CT'with IV contrast is generally done to identify nectosis, uid colections, or pseudocysts once pancreatitis has been diagnosed. It is particularly recommended for severe pancreatiis or if a complication ensues (eg, hypotension or progressive leukocytosis and elevation of temperature). Treatmentiis supportive, with IV fluids, analgesics, and fasting. Question Td Mark this question <= => Question 2 of 30 A 29 year old male presents with abdominal pain radiating to the back, after aweekend of heavy drinking Whatis the investigative and prognostic modality of choice for the cuspected diagnosic? a) Abdorninal ultrasound +) Abdominal plain fa c) Endoscopic retrograde cholangiopancrestogrephy Y @ & Abdominal CT e) Laparoscopy Question Explanation: Acute pancteatitis is inflammation of the pancreas (end, sometimes, adjacent tissues) caused by the release of activated pancreatic enzymes. The most common triggers are biliary tract disease and chronic heavy alohel intake. The condition ranges from rrild (abdominal pain and vomiting) to severe (pencreatic necrosis end a systemic inflammatory process with shock and multiorgan failare). ‘Diagnosis is based on clinical presentation and serum amylase and lioase levels. CT'with IV contrast is generally done to identify nectosis, uid colections, or pseudocysts once pancreatitis has been diagnosed. It is particularly recommended for severe pancreatiis or if a complication ensues (eg, hypotension or progressive leukocytosis and elevation of temperature). Treatmentiis supportive, with IV fluids, analgesics, and fasting. ‘Marke this question & => Question Ti : 46820 Question 3 of 30 4.69 year old woman comes for flexible sigmoidoscopy as part of a yearly screening A 3 cm polyp is found in the sigmoid colon and is removed, She retuned 6 hours later complaining of left lower quacrant pain, fever, nausea and vomriting. Vital signs arc: temperature 38.1°C (100 6°F), pulse 110/min respirations 26/min and blood pressure 120/60 mm Hg. Abdominal examination discloses bowel sounds, tendemess and guarding in the lef lower quadrant. Rectal examination shows no stool and orly tenderness superiorly. What is the most appropriate next step? a) Obtain immediate consultation with a surgeon ) Obtain an angiogram to rule ovt intestinal ischemia ©) Pass a soft rubber rectal tube under fuorescopy ©) Repeat the dezible sigmoidoscopy in order to evaluate the operative site 8) Start hydrocortisone, intravenously, to decrease any inflammatory response Question Explanation: ‘The physical findings described clearly suggest a probable colon perforation at the site ofthe polypectomy, and therefore surgical consultation ie werranted, There is no reason to cuspect ischemia, and any further instrumentation of the colons contraindicated. The patient shouid be prepped for probable surgery with antibiotics and not hydrocortisone since she is already at risk for pecitonils Report An Error ‘Marke this question & => Question Ti : 46820 Question 3 of 30 A 69 year old woman comes for flesible sigmoidoscopy as part of a yearly screening A 3 cm polyp is found in the sigmoid colon and is removed. She returned 6 hours later complaining of left lower quacrant pain, fever, nausea and vomiting. Vital signs are: temperature 38.1°C (100.6°F), pulse 110/min respirations 26/min and blood pressure 120/60 mm Hg. Abdominal examination discloses bowel sounds, tendemess and guarding in the lef lower quadrant. Rectal exarrinction shows no stool aad orly tendemess superiorly. What is the most appropriate next step? Y © a) Obtain immediate consultation with a surgeon. b) Obtain an angiogram to rule out intestinal ischemia ¢) Pass a soft rubber rectal tbe under Buorescopy D Repeat the exible sigmoidoscopy in order to evaluate the operative site €) Start hydovoriisone, intravenously, to decrease any inflammatory response Question Explanation: ‘The physical findings described clearly suggest a probable colon perforation at the site ofthe polypectomy, and therefore surgical consultation ie werranted, There is no reason to cuspect ischemia, and any further instrumentation of the colons contraindicated. The patient shouid be prepped for probable surgery with antibiotics and not hydrocortisone since she is already at risk for pecitonils Report An Error Di www. interface. edu pk /medical-exarns/test-analysis pho ®ubd=14759 apps El coogle A settings Fisonin 2. ube duties © yo umilléaye Ol ..tio ye jgioll JS [Free Hotmall #38 Other bookrner 1216 ‘Marte this question = => Question Ta: Question 4 of 30 AAS ycar old housewife presents with heartsum, stomach pain and satiety for several weeks. You order an EGD and a gastric ulesr is present, You treatthe patient for 12 weeks with ranitidine (HZ blocker). She retuns and is asymptomatic now. But a repeat gastroscopy shows that the ulcer is still present. Her biopsies are negative for malignancy and H. Pylori, The appropriate next step is which of the following? a) Lifelong treatment with E2 receptor blocker b) Treat for 8 mose weeks with H2 receptor blocker c) Follow up only 4) Partial gastrectomy with excision ofulcer Answer (Bona) Other User's Explanation Report An Error Question Explanation: ‘A gastric or peptic ulcer essentially damages the inner ining ofthe stomach, causing a ‘crater’. This can be caused by H. Pylori infections, excessive acid production in the stomach and long term use ofthe NSAID class of druge which inciude aspirin, ouprofen and naproxen. soprague Peptic Ulcer Disease Smee ctcaion Symptoms of gastric ulcers that patients will present with are pain, heartburn; early satiety and sometimes blocting and vomiting. Diagnosis is made by doing an EGD (esophagogestrocuodenoscopy) to visualize the ulcer. A biopsy can be taken at the time also ‘Treatment options include medicines that seduce the amount of acid that your stomech makes. H2 blockers and proton purap inkibtors (PPI) are commonly used for this. They usually help people start to feel better within 3 days Itss advised to take such medicines for up to 8 weeks. Most ulcers heal within ths time, [Pnot then tis recommended to continue with either an H2 blocker or PPI for the long term to heal the ulcer and prevent recurrence. She should also be counseled to not use those NSAIDS. Total Questions a. fs KO eo i kn Bh Bide hla io te is Fk SR BEREERBE xx KKKKKKKKKKKKKKKKKXKKXKKXKKKKKKRK Di www. interface. edu pk /medical-exarns/test-analysis pho ®ubd=14759 apps El coogle A settings Fisonin 2. ube duties © yo umilléaye Ol ..tio ye jgioll JS [Free Hotmall #38 Other bookrner 1216 ‘Marte this question = => Question Ta: Question 4 of 30 A.45 year oll housewife presents with heartburn, stomach pain aad satiety for several weeks. You order an EGD and a gastric ulcer is present. You treat the patient for 12 weeks with ranitidine (H2 blocker). She reums and is asymptomatic now. Eut a sepeat gastroscopy shows that the ulcer is still present. Her biopsies are negative for malignancy and H. Pylori. The appropriate next step is ‘which of the following? Y © a) Lifelong treatment with H2 receptor blocker b) Treat for 8 more weeks with H2 receptor blocker ©) Follow up oaly 4) Partial gastrectomy with excision ofulcer Answer (Bona) Other User's Explanation Report An Error Question Explanation: ‘A gastric or peptic ulcer essentially damages the inner ining ofthe stomach, causing a ‘crater’. This can be caused by H. Pylori infections, excessive acid production in the stomach and long term use ofthe NSAID class of druge which inciude aspirin, ouprofen and naproxen. soprague Peptic Ulcer Disease Smee ctcaion Symptoms of gastric ulcers that patients will present with are pain, heartburn; early satiety and sometimes blocting and vomiting. Diagnosis is made by doing an EGD (esophagogestrocuodenoscopy) to visualize the ulcer. A biopsy can be taken at the time also ‘Treatment options include medicines that seduce the amount of acid that your stomech makes. H2 blockers and proton purap inkibtors (PPI) are commonly used for this. They usually help people start to feel better within 3 days Itss advised to take such medicines for up to 8 weeks. Most ulcers heal within ths time, [Pnot then tis recommended to continue with either an H2 blocker or PPI for the long term to heal the ulcer and prevent recurrence. She should also be counseled to not use those NSAIDS. Total Questions a. fs KO eo i kn Bh Bide hla io te is Fk SR BEREERBE xx KKKKKKKKKKKKKKKKKXKKXKKXKKKKKKRK 1377 ‘Marke this question <= => Question Td Question 5 of 30 4.39 year old alcoholic woman presents with complaints of epigastric pain radiating to her back with nausea and vomiing, Epigastric tenderness is present on physical examination. The most bely diagnosis is a) Pancreatitis b) Cholangitis c} Choledocholithiasis 4) Hepatitis ©} Cholecystitis Avewor [UEIPNSAR) other UroreExplanation Report An Error Question Explanat “Acute pancreatitis is inflammation of the pancreas (end, sometimes, adjacent tissues) caused by the release of activated pancreatic enzymes. The most common triggers are biliary tract disease and chronic heavy alcohol intake. The condition ranges from mile (abdominal pain and vorniting) to severe (pancreatic necrosis end a systemic inflammatory process with shock and multiorgan failure). Diagnosis is based on clinical presentation and serum amylase and lipaze levels. Treatment is supportive, with IV fluids, analgesics, and fasting, 1377 ‘Marke this question <= => Question Td Question 5 of 30 A.39 year old alcoholic woman presents with complaints of epigastric pain radiating to her bacle with nausea and vomiting. Epigastric tenderness is present on physical examination. The most likely diggnosis is Y © a) Pancreatitis, +b) Cholangitis c) Cheledocholithiasis 4) Hepattis ©) Cholecystitis Avewor [UEIPNSAR) other UroreExplanation Report An Error Question Explanat “Acute pancreatitis is inflammation of the pancreas (end, sometimes, adjacent tissues) caused by the release of activated pancreatic enzymes. The most common triggers are biliary tract disease and chronic heavy alcohol intake. The condition ranges from mile (abdominal pain and vorniting) to severe (pancreatic necrosis end a systemic inflammatory process with shock and multiorgan failure). Diagnosis is based on clinical presentation and serum amylase and lipaze levels. Treatment is supportive, with IV fluids, analgesics, and fasting, 3i2/2014 7:46:01 PM ‘Mark this question & => Question Td : 55442 Question 6 of 30 A.25 year old man presents for evaluation of a 4 month history of postprandial diarrhea, weight loss 9 pounds, and lower abdominal pain He denies recent travel on antibiotic use. His tcmpereture is 38.0 C (100.4), and he has several oral aphthous ulcers. On. abdotrinal exernination, there ss tendemess and mild voluntary guarding inthe right lower quadrant. A rectal examination reveals brown stool that is strongly guaiac positive. The most likely causing this patient's symptoms is a) A gram-negative organism ) Folate deficiency ©) Transmural inflammation in the region of the terminal deurn ¢) Mucosal ulceration with no transmural involvement in the ascending colon 8) Toxin-producing organism Answer | Brpianaton | Other User's Explanation Report An Error Question Explanatic ‘This patient, postprandial diarrhea, weight loss, low-grade fever, and right lower quadrant findings on physical examinetion, has the typical presentation of Crohn disease, which most commonly involves the terminal ileam. Inflammation in this disease is tranemural, as opposed to the inflammation in ulcerative colitis that is limited to the mucosa of the large inbestine 3i2/2014 7:46:01 PM ‘Mark this question & => Question Td : 55442 Question 6 of 30 A 25 year old man presente for eveluction af a 4 month history of postprandial diarrhea, weight loss 9 pounds, and lawer abdorinal pain He deaies recent travel on antibiotic use. His temperature is 28.0 C (100.4), and he has several oral aphthous ulcers. On abdominal examination, there is tendemess and mild voluntary guarding in the right lower quadrant. A rectal examination reveals ‘brown stool that is strongly guaiac positive. The most likely causing this patient's symptoms is 2) A gram-negative organism +) Folate deficiency V © ©) Transmural inflammation in the region of the terminal deum 4) Mucosal ulveration with no trancnaural invelvement in the ascendirg colon 6) Toxin-producing organism. Answer | Brpianaton | Other User's Explanation Report An Error Question Explanatic ‘This patient, postprandial diarrhea, weight loss, low-grade fever, and right lower quadrant findings on physical examinetion, has the typical presentation of Crohn disease, which most commonly involves the terminal ileam. Inflammation in this disease is tranemural, as opposed to the inflammation in ulcerative colitis that is limited to the mucosa of the large inbestine Merk this question —& => Question Td : 59360 Question 7 of 30 ‘Which of the following is not a isk factor for colon cancer? 2) Low fiber dict 6) Severe diverticular disease 6) Familial adenomatous polyposis 6) Uicerative colitis 6) High fat diet Avewor (UEQSNEWER) tno: UsorsExplanation Repost An Error Question Explanatiot People with a family history of colorectal cancer have a higher rick of develeping the cancer themselves. A family hictory of polyps also increases the risk of colorectal cancer. People with ulcerative colis or Crohn's disease are at greater risk as well. This risk is related to the person's age when the disease developed and the length of time the person has had the disease. People at highest risk tend to consume a high fat. low fiber diet. Greater exposure to air and weter pollution, particularly to industrial cancer causing substances (carcinogens), may play a role. Merk this question —& => Question Td : 59360 Question 7 of 30 ‘Which of the following is not a isk factor for colon cancer? 2) Low fiber dict Y © 8) Severe diverticular disease 6) Familial adenomatous polyposis 6) Uicerative colitis 6) High fat diet Avewor (UEQSNEWER) tno: UsorsExplanation Repost An Error Question Explanatiot People with a family history of colorectal cancer have a higher rick of develeping the cancer themselves. A family hictory of polyps also increases the risk of colorectal cancer. People with ulcerative colis or Crohn's disease are at greater risk as well. This risk is related to the person's age when the disease developed and the length of time the person has had the disease. People at highest risk tend to consume a high fat. low fiber diet. Greater exposure to air and weter pollution, particularly to industrial cancer causing substances (carcinogens), may play a role. “Mark this question & => Question Id : 63728 Question 8 of 30 ‘The foods that are ok for a patient with Celiac disease to eat are a) Wheat b) Rice and com ©) Barley 4) Oats Anewor [FEIRIRIRNBN) othe: User's Explanation Report An Evor Question Explanation: Celiac disease (nontropical spre, guiten enteropathy, celiac sprue) is a hereditary intolerance to ghuten, a protein found in wheats, barley, and oats, which causes characteristic changes in the lining of the small intestine, resubing in malabsorption, People with celiac disease must exclude all ghuten from their diet, since eating even smell amounts may cause symptoms. The response to a gluten-free detis usually rapid. Once giuten is avoided, the brushlice surface of the small intestine and its absorptive function rem t> normal Gluten is 50 widely used in food products that people with celiac disease need detailed lists of foods to be avoided and expert advice fiom a dietitian, Gluten is found, for example, in commercial soups, sauces, ice cream, and hot degs. Several grains and starch. sources are considered acceptable for a gluien fice dict. The most Bequently used are com, potatoes and rice. “Mark this question & => Question Id : 63728 Question 8 of 30 ‘Tue foods that are ok for a patient with Celiac disease to eat are a) Wheat Y © b) Rice andcom ©) Barley ) Oats Anewor (Berianation Other User's Explanation Report An Error Question Explanat Celiac disease (nontropical spre, guiten enteropathy, celiac sprue) is a hereditary intolerance to ghuten, a protein found in wheats, barley, and oats, which causes characteristic changes in the lining of the small intestine, resubing in malabsorption, People with celiac disease must exclude all ghuten from their diet, since eating even smell amounts may cause symptoms. The response to a gluten-free detis usually rapid. Once giuten is avoided, the brushlice surface of the small intestine and its absorptive function rem t> normal Gluten is 50 widely used in food products that people with celiac disease need detailed lists of foods to be avoided and expert advice fiom a dietitian, Gluten is found, for example, in commercial soups, sauces, ice cream, and hot degs. Several grains and starch. sources are considered acceptable for a gluien fice dict. The most Bequently used are com, potatoes and rice. Mark this question & => Question Td : 84574 Question 9 of 30 A patient is aciited with severe upper abdominal pain that radiates to the back. He describes the pain as steady, but aotes that sis more sever when he lies down on his back. His blood pressure starts to drop, but no source of bleedingis identified. When questioned about what he had been doing before the pain started, ne reports having been at a fiiend’s cabin, and spending the weekend "smoking cigarettes, using allitle cocaine, and drinking beer." The patients white blood cell couat is within normal limits. Of the following serum laboratory tests, which wil most ikely be diagnostic? 2) Alkaline phosphatase ) Armylace ©) Bilirubin 4) Glucose ©) Hematocrit Answer | Explanation | Other User's Explanation Report An Error Question Explanation: This isa classic presentation of acute pancreatitis. The other classic presentation is as a complication of gallstones occluding the pancreatic duct. Many cases of acute pancreatitis are comparatively mild, but severe cases can cause Mfe-threctening saocle secondary to release of vasoactive substances fiom the dameged pancreas. Acute pancreatitis is diagnosed by demonstrating elevated serum amslase and/or lpase levels ‘Alkaline phosphatase (choice A) may increase in liver and bilary tract pathology. but do not generally increese in pancreatic disease Serum bilirubin (choice C) mey increase if the bout of acute paxcreatitis is sesondary to gallstone impaction, It may also be elevated if the stone is impacted at other sites. Iris therefore, not specific. Serum glacose (choice D) levels may rise (nyperglycemia) secondary to impaired insulin production and release in the damaged pancreas, but the finding is nonspecific. The heraatocsit (choice E) may rice to 50 to 55% cecondary to third space fluid locses in acuie pancreatitis, but the Ending is nonspecific and will not specifically suggest the diagnosis

You might also like